Вы находитесь на странице: 1из 557

Cracker Book for Bank (IBPS | SBI | RRB PO | Clerk) Mains Exams

1 Adda247 Publications For any detail, mail us at


Publications@adda247.com
Cracker Book for Bank (IBPS | SBI | RRB PO | Clerk) Mains Exams

Chapter
Simplification, Approximation and
1 Number Series

BEST APPROACH TO SOLVE THE QUESTIONS

Simplification: Simplification simply means simplifying the complex


expressions. There are ways available always which are simpler, easier and
faster to calculate a particular complex expression. Questions related to
simplification can be solved in variety of ways. But in examination where
one needs to calculate as fast as possible, applying good simplification
techniques may serve a good purpose. The most interesting about numbers
is that they can be expressed in an innumerable number of ways and thus
comes the concept of breaking the expression in such a way that one
reaches the answer in less time.

Following are some examples citing the ways to simplifying an


equation or expression.

9 11 17 12 16
1. + + =? + + + 27× 33
2 3 6 5 10
27+22+17 24+16
⇒ =? + + (30-3)(30+3)
6 10
66
⇒ =? +4 + 900 − 9
6

⇒ ?=11+9-900-4= –884

2. ? × 65 ÷ 72 = 195 × 312 + 260 × 6


?×65
⇒ =65× 6(3× 52 + 4 × 1)
72

⇒ ? = 72× 6 × 160 = 432 × 160 = 69120

2 Adda247 Publications For any detail, mail us at


Publications@adda247.com
Cracker Book for Bank (IBPS | SBI | RRB PO | Clerk) Mains Exams

Approximation Questions based on approximation are usually easy to


solve but turn out to be a bit difficult when approximate values are not
clearly visible. In case where a value like 17.97 is given, it is easy to replace
120 101
it with 18. But cases where values like or are given, then
13 11
approximate values can be used by making appropriate adjustments to
other terms of the expression.

Let’s, for example, try to find out the approximate value of expression given
below:
66 50
+ + 41% of 97 =?
16 17

66 50
The value of is a bit more than 4 and that of is a bit less than 3. So we
16 17
66 50
may take value of and to be equal to 4 and 3 respectively.
16 17

We know 40% of 100 is equal to 40. As 41 near to and greater than 40 and
97 is near to and smaller than 100, we may take the approximate value of
41% 𝑜𝑓 97 to be equal to 40 itself.
This way, the approximate value of the above expression= 4+3+40=47

A few sample problems:

178 217
− + 33 × 41 =? −32% 𝑜𝑓 207
12 18
⇒ 15 + 12 + 33(40+1) = ? – 33% of 200
⇒ 27 + 1355 = ? – 66 ⇒ ? = 1450
19.95 68.12
+ − 12.5% 𝑜𝑓 127.9 =?
4 17
20 68 1
⇒ + − × 128 =?
4 17 8

⇒ 5 + 4 − 16 =? ⇒ ? = −7

3 Adda247 Publications For any detail, mail us at


Publications@adda247.com
Cracker Book for Bank (IBPS | SBI | RRB PO | Clerk) Mains Exams

Number Series When different numbers are arranged in any mathematical


and logical pattern, a number series is said to be formed. There may be
infinite such patterns. So it’s really difficult to find out one out of them. But
there is one element that gives us the idea of any pattern.

That element is the difference of the terms of a number series. By finding


the difference of consecutive terms of a series we can make out the pattern
on which the number series is based. Some patterns are clearly visible but
others can be found out by cracking the pattern of the differences.

A few examples follow:

1. 45, 44, 48, 39, 55, 30, 66

2. 3, 9, 23, 97, 479, 2881, 20159

3. 10, 12, 15, 22, 38, 70, 127

4 Adda247 Publications For any detail, mail us at


Publications@adda247.com
Cracker Book for Bank (IBPS | SBI | RRB PO | Clerk) Mains Exams

Practice Exercise Based on new Pattern

Directions (1-15): What will come in place of the question mark (?) in the
following questions ?

1. (4 × 4)3 ÷ (512 ÷ 8)4 × (32 × 8)4 = (2 × 2)?+4


(a) 8 (b) 12 (c) 6
(d) 14 (e) None of these

2
2. (2√392 − 21) + (√8 − 7) = (? )2
(a) 4 (b) –4 (c) 12
(d) 2 (e) 6

1 1 1 1
3. 1 + 1 − 1 =? +1
4 6 8 12
5 7 5
(a) (b) (c)
24 24 12
7
(d) (e) None of these
12

3
4. of 30% of 3420 = (? )2 × 2
19
(a) (81)2 (b) 7 (c) 9
(d) 81 (e) 49

5. 65% of √3136 × 5 =? +154


(a) 56 (b) 28 (c) 35
(d) 32 (e) None of these

1
6. 29% of 193 – 20% of 204.85 = (3375)3 − (? )2
(a) 1 (b) 4 (c) 16
(d) 64 (e) None of these

5 Adda247 Publications For any detail, mail us at


Publications@adda247.com
Cracker Book for Bank (IBPS | SBI | RRB PO | Clerk) Mains Exams

1 3 1 1 1
7. 3 ÷ 6 × 1 × 3 = (? )2
3 7 2 7
22 484 22
(a) (b) (c)
3 3 9
484
(d) (e) None of these
81

8. 60% of 68.05 × 15% of 8 × 1.02 = ?


(a) 49.97592 (b) 49.73482 (c) 54.77442
(d) 41.84822 (e) None of these

(112.6 ×114.4 )
9. = (? )2
117.5 ÷115 ×√11
(a)√11 (b) 11 (c) 121
(d) 1331 (e) 14641

1
10. √3481 ÷ 7 × (441)2 = 12.5% of 1200 + ?
(a) 3 (b) 9 (c) 27
(d) 81 (e) None of these

19 1
11. 3 % of 675 + 25 % of 184 = x
27 4
(a) 70.64 (b) 71.64 (c) 71.46
(d) 70.46 (e) None of these

2
12. [2.5 ÷ 0.1)3 ÷ (6.25)2 ] × [(1.25)2 ÷ (5)2 ] = (25)3+x
(a) 1 (b) 2/3 (c) 1/3
(d) 2 (e) None of these

161 1595 7 1
13. × ÷ = (x)3
377 253 13

(a) 343 (b) 216 (c) 64


(d) 125 (e) None of these
6 Adda247 Publications For any detail, mail us at
Publications@adda247.com
Cracker Book for Bank (IBPS | SBI | RRB PO | Clerk) Mains Exams

2 4 7 2145 169
14. of of of ÷√ =x
3 5 11 1288 529
(a) 1 (b) 0.5 (c) 2
(d) 4 (e) None of these

3
√8649 √15625
15. × 3 = x2
√961 √1728
(a) 0.5 (b) 1.5 (c) 2.5
(d) 3.5 (e) None of these

APPROXIMATION

Directions (1-20): What approximate value will come in place of (x or ?) in


the following questions ?

𝑥
1. 294.01 × − 19.99% 𝑜𝑓 119.99𝑥 = 254.9 ÷ 1.9
8.01
(a) 10 (b) 16 (c) 8
(d) 5 (e) 12

410
2. 55.2 × + 499.9 – 127.9 =? % of 5600
69
(a) 17.5 (b) 12.5 (c) 10
(d) 15 (e) 8

3. 630 × ? + 599.85 – 55% of 12000 = 37 ½ % of 9200


(a) 12 (b) 18 (c) 15
(d) 20 (e) 8

11999.87 1
4. + 54.9% of 1800 – 389.9 = 11 % of 9900
? 9

(a) 24 (b) 28 (c) 20


(d) 18 (e) 32

7 Adda247 Publications For any detail, mail us at


Publications@adda247.com
Cracker Book for Bank (IBPS | SBI | RRB PO | Clerk) Mains Exams
31 5 1
5. 8099.9 × + 2699.8 × – 1799.8 = 62 % of ?
27 3 2
(a) 22500 (b) 15200 (c) 16200
(d) 18200 (e) 19200

6. 119.89% of 2000.23 + (35.96)² – 59.49% of 2999.89 = (49.89)² – ?


(a) 559 (b) 569 (c) 589
(d) 549 (e) 539

7. (11.87)² + 12.493 × 15.89 – √13224.98 – (?)² = (14.96)²


(a) 9 (b) 2 (c) 8
(d) 5 (e) 10

359.93
8. = (8.89)3 – 14.5 × 39.89 + (1.95)2 – 34
?
(a) 8 (b) 2 (c) 3
(d) 9 (e) 5

9. √429.87 + 520.23 + √120.97 = (? )2 + √35.98


(a) 2 (b) 3 (c) 8
(d) 5 (e) 9

10. (23.89)² + √3598.97 – 171.93 + (?)² = (27.98)² + 4


(a) 18 (b) 24 (c) 28
(d) 22 (e) 26

11. (13.97)² – 39.87% of 239.97 + (29.87)² + 330.97 = (?)³


(a) 18 (b) 19 (c) 17
(d) 11 (e) 15

?+134.5
12. + 209.87 + (69.87)² – 1999.83 = (54.87)² + 99.85
24
(a) 235.5 (b) 245.5 (c) 225.5
(d) 215.5 (e) 205.5
8 Adda247 Publications For any detail, mail us at
Publications@adda247.com
Cracker Book for Bank (IBPS | SBI | RRB PO | Clerk) Mains Exams

13. ? × 12.97 + (24.97)² – (19.89)² + 363.83 = (27.93)²


(a) 10 (b) 20 (c) 18
(d) 12 (e) 15

593.89 1259.81
14. + 14.87 × 35.88 + = (25.89)2
? 17.93
(a) 12 (b) 15 (c) 13
(d) 9 (e) 11

15. 48.01% of 249.99 + 𝑥% 499.99 = (11.01)2 + (2.11)2


(a) 2 (b) 3 (c) 5
(d) 1 (e) 6

2
16. 49.11 × ÷ 14.09 + √𝑥 = 1025 ÷ 257
7
(a) 9 (b) 8 (c) 27
(d) 16 (e) 25

3
17. × (15.99) + 31.9% 𝑜𝑓 3199 + 294.9 = 𝑥 3
4
(a) 10 (b) 11 (c) 12
(d) 15 (e) 8

18. ? % of 6519.97 + (21.97)2 – 29.98% 𝑜𝑓 9139.93 = (25.91)2


(a) 25 (b) 15 (c) 30
(d) 45 (e) 40

577.93+?
19. + 61.87% of 1449.87= –√2024.87 + 13.89 % 𝑜𝑓 7099.97
(15.97)
(a) 222 (b) 282 (c) 262
(d) 200 (e) 308

20. 32.87% of ? – 17.87 × 44.86 – √399.81 = (24.98)2 + 29.89


(a) 4450 (b) 4500 (c) 4350
(d) 4600 (e) 4900

9 Adda247 Publications For any detail, mail us at


Publications@adda247.com
Cracker Book for Bank (IBPS | SBI | RRB PO | Clerk) Mains Exams

NUMBER SERIES

Wrong Series

Directions (1-15): Find the wrong number in the given series that does not
follow the pattern?

1. 480, 960, 320, 1280, 272 , 1536


(a) 960 (b) 272 (c) 1280
(d) 320 (e) 1536

2. 210, 197, 171, 135, 80, 15


(a) 197 (b) 15 (c) 80
(d) 171 (e) 135

3. 4, 3, 5, 14, 60, 528


(a) 60 (b) 4 (c) 5
(d) 14 (e) 528

4. 9, 63, 25, 216, 49, 512


(a) 25 (b) 216 (c) 63
(d) 512 (e) 49

5. 4498, 4888, 5336, 5846, 6422, 7070


(a) 5846 (b) 4498 (c) 5336
(d) 7070 (e) 6422

6. 122, 125.2, 131.6, 141.2, 152, 170


(a) 125.2 (b) 131.6 (c) 141.2
(d) 170 (e) 152
10 Adda247 Publications For any detail, mail us at
Publications@adda247.com
Cracker Book for Bank (IBPS | SBI | RRB PO | Clerk) Mains Exams

7. 3671, 2341, 4537, 1163, 6072, − 783


(a) 6072 (b) 2341 (c) − 783
(d) 3671 (e) 1163

8. 15, 91, 457, 1831, 5497, 10997


(a) 457 (b) 91 (c) 5497
(d) 15 (e) 10997

9. 10, 17, 45, 108, 220, 390


(a) 220 (b) 390 (c) 10
(d) 108 (e) 45

10. 10, 30, 90, 450, 3150, 34650


(a) 10 (b) 34650 (c) 3150
(d) 3150 (e) 30

11. 325, 546, 754, 936, 1078, 1170


(a) 936 (b) 546 (c) 325
(d) 1078 (e) 1170

12. 192, 202, 210, 216, 225, 243, 288


(a) 210 (b) 202 (c) 288
(d) 192 (e) 243

13. 3, 11, 49, 191, 569, 1135, 1134


(a) 1135 (b) 1134 (c) 3
(d) 49 (e) 11

14. 23, 30, 42, 63, 95, 140, 200


(a) 42 (b) 140 (c) 30
(d) 200 (e) 23

15. 8, 30, 60, 120, 180, 180, 90


(a) 30 (b) 120 (c) 90
(d) 8 (e) 60

11 Adda247 Publications For any detail, mail us at


Publications@adda247.com
Cracker Book for Bank (IBPS | SBI | RRB PO | Clerk) Mains Exams

MISSING SERIES

Directions (1-15): What should come in place of the question mark (?) in
the following series ?

1. 10, 15, 45, 270, 3240, ?


(a) 72250 (b) 77760 (c) 75260
(d) 85280 (e) 762560

2. 12, 26, 81, 328, 1645, ?


(a) 9981 (b) 8281 (c) 7321
(d) 9876 (e) 9978

3. 1024, 512, ?, 16, 1, 0.03125


(a) 64 (b) 128 (c) 256
(d) 216 (e) 324

4. 193, 97, 49, 25, ?, 7


(a) 15 (b) 12 (c) 17
(d) 13 (e) 11

5. 17, 24, 13, 26, 9, ?


(a) 30 (b) 32 (c) 29
(d) 28 (e) 26

6. 18, 6, 2, −2 , − 10, ?
(a) −36 (b) – 48 (c) − 40
(d) −32 (e) − 52

7. 56, 1144, 2104, 2944, 3672, ?


(a) 4556 (b) 4456 (c) 4256
(d) 4296 (e) 4552

12 Adda247 Publications For any detail, mail us at


Publications@adda247.com
Cracker Book for Bank (IBPS | SBI | RRB PO | Clerk) Mains Exams

8. 21, 1750, 2752, 3267, 3487, ?


(a) 3556 (b) 3456 (c) 3536
(d) 3544 (e) 3558

9. 9, 15, 23, 33, 45, ?


(a) 57 (b) 59 (c) 53
(d) 51 (e) 49

10. 141, 197, 269, 359, 469, ?


(a) 607 (b) 609 (c) 601
(d) 603 (e) 605

11. 25, 33, 46, 69, 112, ?


(a) 175 (b) 180 (c) 185
(d) 190 (e) 195

12. ? , 5, 12, 39, 160, 805


(a) 2 (b) 4 (c) 3
(d) 5 (e) 8

13. ?, 10, 20, 46, 96, 178


(a) 6 (b) 7 (c) 8
(d) 9 (e) 5

14. 2160, ? , 360, 180, 60, 30


(a) 1080 (b) 720 (c) 900
(d) 810 (e) 1440

15. 2, 35, 156, 477, ?, 959


(a) 858 (b) 564 (c) 958
(d) 727 (e) 676

13 Adda247 Publications For any detail, mail us at


Publications@adda247.com
Cracker Book for Bank (IBPS | SBI | RRB PO | Clerk) Mains Exams

Solutions

SIMPLIFICATION

1. (c); (4 × 4)3 ÷ (512 ÷ 8)4 × (32 × 8)4 = (2 × 2)?+4


3 4
(42 ) ×(44 )
or, (43 )4
= (4)?+4
4 ×416
6
or, = (4)?+4
412
or, 4 = 4?+4
10

or, ? = 6

2. (e); ?2 = 2√14 × 14 × 2 − 21 + 8 + 49 − 28√2


= 28√2 − 21 + 57 − 28√2 = 36 = 62
∴?=6

1 1 1 1
3. (a); 1 + + 1 + − 1 − =? +1
4 6 8 12
1 1 1 1
⇒? = 1 + + 1 + − 1 − − 1 −
4 6 8 12
1 1 1 1
= + − −
4 6 8 12
6+4−3−2 5
= =
24 24

30 3
4. (c); 3420 × × = (? )2 × 2
100 19
2
⇒ 162 = (? × 2 )
162
⇒ (? )2 = = 81
2
∴ ? = √81 = 9

65
5. (b); √3136 × × 5 = ? +154
100
65
⇒ 56 × × 5 =? +154
100
⇒ 182 = ? +154
⇒ ? = 182 − 154 = 28

14 Adda247 Publications For any detail, mail us at


Publications@adda247.com
Cracker Book for Bank (IBPS | SBI | RRB PO | Clerk) Mains Exams

6. (e); (55.97 – 40.97) = 15 – (?)2


(?)2 = 0
?=0

10 7 3 22 1
7. (d); × × × = (? )2
3 45 2 7
1 22
(? ) = 2
9
484
?=
81

8. (a); 40.83 × 1.02 ×1.2 = ?


? = 49.97592

117
9. (c); = (? )2
113
(? )2 = 14641
? = 121

10. (c); 59 × 3 = 150 + ?


? = 27

100 101
11. (c); % of 675 + of 184
27 100×4
675
+ 46.46
27
25 + 46.46 = 71.46

2
+𝑥
12. (c); [(25 )3 ÷ (6.25 )2 ] × ((1.25) ÷ 5 2 2)
= (25)
3

253 1252 1 2
+𝑥
[ × 1002 ] × [ × ] = (25) 3
254 1002 52
1 100×100 125×125 2
[ × × ] = (25)3+𝑥
25 100×100 5×5
2
+𝑥
25 = (25) 3
2 2
⇒ +𝑥 = 1 ⇒𝑥 = 1−
3 3
1
𝑥=
3

15 Adda247 Publications For any detail, mail us at


Publications@adda247.com
Cracker Book for Bank (IBPS | SBI | RRB PO | Clerk) Mains Exams

161 1595 13 1
13. (d); × × = (𝑥 )3
377 253 7
Explanation
161 = 23 × 7
377 = 29 × 13
1595 = 29 × 55
253 = 23 × 11
23×7 29×55 13 1
× × = (𝑥 )3
29×13 23×11 7
1
5 = (𝑥 ) ⇒ 𝑥 = 125
3

2 4 7 2145 529
14. (a); × × × ×√
3 5 11 1288 169
13 23
× =1
23 13

3
√8649 √15625
15. (c); × 3 = 𝑥2
√961 √1728
93 25
× = 6.25 = 𝑥 2
31 12
𝑥 = 2.5

APPROXIMATION

𝑥 1 255
1. (a); ≈ 294 × − × 120𝑥 =
8 5 2
≈ 36.75𝑥 − 24𝑥 = 127.5
≈ 12.75𝑥 = 127.5
≈ 𝑥 = 10

X
2. (b); × 5600 = 8 × 41 + 500 – 128
100
700
X=
56
X = 12.5

16 Adda247 Publications For any detail, mail us at


Publications@adda247.com
Cracker Book for Bank (IBPS | SBI | RRB PO | Clerk) Mains Exams
9200 55
3. (c); 630X + 600 = 37.5 × + × 12000
100 100
630X + 600 = 3450 + 6600
630X = 10050 – 600
9450
X=
360
X = 15

12000 55 100 1
4. (a); = + × 1800 = × × 9900 + 390
X 100 9 100
12000
= = 1490 – 990
X
12000
X=
500
X = 24

62.5X 31 5
5. (e); = 8100 × + 2700 × – 1800
100 27 3
5x
= 9300 + 4500 – 1800
8
12000 × 8
X=
5
X = 19200

120 59.5
6. (c); × 2000 + (36)2 – × 3000 = (50)2 – ?
100 100
2400 + 1296 – 1785 = 2500 – ?
2500 – 1911 = ?
? = 589

7. (b); (12)² + 12.5 × 16 – √13225 – (?)²


= (15)²
144 + 200 – 115 – (?)² = 225
(?)² = 229 – 225
?=2

17 Adda247 Publications For any detail, mail us at


Publications@adda247.com
Cracker Book for Bank (IBPS | SBI | RRB PO | Clerk) Mains Exams
360
8. (e); = (9)3 – 14.5 × 40 + (2)2 – 81
?
360
= 729 – 580 + 4 – 81
?
360
?=
72
?=5

9. (d); √430 + 520 + √121 = (? )2 + √36


√430 + 520 + 11 = (? )2 + 6
31 – 6 = (?)²
?=5

10. (a); (23.89)² + √3600 – 172 + (?)² = (28)² + 4


576 + 60 – 172 + (?)² = 784 + 4
464 + (?)² = 788
(?)² = 788 – 464
? = 18

11. (d); (14)² – 40% of 240 + (30)² + 331 = (?)³


40×240
196 – + 900 + 331 = (? )3
100

1000 + 331 = (?)³


? = 11

?+134.5
12. (c); + 210 + (70)2 – 2000 = (55)2 + 100
24
?+134.5
+ 3110 = 3125
24

? + 134.5 = 15 × 24
? = 360 – 134.5
? = 225.5

18 Adda247 Publications For any detail, mail us at


Publications@adda247.com
Cracker Book for Bank (IBPS | SBI | RRB PO | Clerk) Mains Exams

13. (e); ? × 13 + (25)² – (20)² + 364 = (28)²


? × 13 + 225 + 364 = 784
? × 13 = 784 – 589
195
?=
13

? = 15

594 1260
14. (d); + 15 × 36 + = (26)2
? 18
594
+ 540 + 70 = 676
?

594 = ? (676 – 610)


594
?=
66

?=9

48 𝑥
15. (d); × 250 + × 500 ≈ 112 + 22
100 100
120 + 5𝑥 ≈ 121 + 4
5𝑥 ≈ 5
𝑥≈1

2 1 1024
16. (a); 49 × × + √𝑥 ≈
7 14 256
1 + √𝑥 ≈ 4
√𝑥 ≈ 3
𝑥≈9

3 32
17. (b); × 16 + × 3200 + 295 ≈ 𝑥 3
4 100
12 + 1024 + 295 ≈ 𝑥 3
𝑥 3 ≈ 1331
𝑥 ≈ 11
19 Adda247 Publications For any detail, mail us at
Publications@adda247.com
Cracker Book for Bank (IBPS | SBI | RRB PO | Clerk) Mains Exams
? 30
18. (d); × 6520 + (22)2 – × 9140 = (26)2
100 100
?
× 6520 = 676 + 2741 – 484
100
2934×100
?=
6520
? = 45

578+? 62 14
19. (a); + × 1450 =– √2025 + × 7100
16 100 100
578+?
+ 899 =– 45 + 994
16
578+?
= 949 – 899
16
578 + ? = 50 × 16
? = 800 – 578
? = 222

33
20. (b); ×? = 625 + 30 + 810 + 20
100
1485×100
?=
33
? = 4500

WRONG SERIES

1. (b);

So wrong number is 272

2. (e);

So wrong number is 135

20 Adda247 Publications For any detail, mail us at


Publications@adda247.com
Cracker Book for Bank (IBPS | SBI | RRB PO | Clerk) Mains Exams

3. (a);

So wrong number is 60

4. (c);

So wrong number is 63

5. (d); Wrong number = 7070

5. (e); Wrong number = 152

6. (a); Wrong number = 6072


Pattern of series —

21 Adda247 Publications For any detail, mail us at


Publications@adda247.com
Cracker Book for Bank (IBPS | SBI | RRB PO | Clerk) Mains Exams

8. (e);

So wrong number is 10997

9. (b);

So wrong number is 390

10. (a);

So wrong number is 10

11. (d);

So wrong number is 1078

12. (b);

So wrong number is 202


22 Adda247 Publications For any detail, mail us at
Publications@adda247.com
Cracker Book for Bank (IBPS | SBI | RRB PO | Clerk) Mains Exams

13. (b);

So wrong number is 1134

14. (e);

So wrong number is 23

15. (a);

So wrong number is 30

MISSING SERIES

1. (b); Series is ×1.5, ×3, ×6, ×12, ×24


So, 3240 × 24 = 77760

2. (d); Series is ×2+2, ×3+3, ×4+4, ×5+5, ×6+6


So, 1645 × 6 + 6 = 9876

3. (b); Series is
÷2, ÷4, ÷8, ÷16, ÷32
So, ? = 512 ÷ 4
= 128

4. (d); −96, −48, − 24, − 12, −6


So, 25 – 12 = 13
23 Adda247 Publications For any detail, mail us at
Publications@adda247.com
Cracker Book for Bank (IBPS | SBI | RRB PO | Clerk) Mains Exams

5. (d); Series
+7, −11, +13, −17, +19
So, 9 + 19 = 28

6. (d); Pattern of series –


18 × 0.5 − 3 = 6
6×1−4=2
2 × 1.5 − 5 = −2
− 2 × 2 − 6 = − 10
? = − 10 × 2.5 −7 = − 32

7. (d);

8. (a); 21+ (123 + 1) = 1750


1750 + (103 + 2) = 2752
2752 + (83 + 3)= 3267
3267 + (63 + 4) = 3487
? = 3487 + (43 + 5) = 3556

9. (b);

24 Adda247 Publications For any detail, mail us at


Publications@adda247.com
Cracker Book for Bank (IBPS | SBI | RRB PO | Clerk) Mains Exams

10. (c); 141 + 7 × 8 = 197


197 + 8 × 9 = 269
269 + 9 × 10 = 359
359 + 10 × 11 = 469
? = 469 + 11 × 12 = 601

11. (e);

12. (b);

13. (c);

14. (a);

15. (c); (2 + 5) × 5 = 35
(35 + 4) × 4 = 156
(156 + 3) × 3 = 477
(477 + 2) × 2 = 958
(958 + 1) × 1 = 959

25 Adda247 Publications For any detail, mail us at


Publications@adda247.com
Cracker Book for Bank (IBPS | SBI | RRB PO | Clerk) Mains Exams

1 Adda247 Publications For any detail, mail us at


Publications@adda247.com
Cracker Book for Bank (IBPS | SBI | RRB PO | Clerk) Mains Exams

Chapter
Ratio Proportion and Mixture
2 & Alligation

BEST APPROACH TO SOLVE THE QUESTIONS

Alligation is a method used to solve problems concerned with weightage


average. Or, in other words, alligation is just a way of writing equations of
weightage average for the easy calculation. Problems involving mixtures or
any form of weightage average can be solved easily using the method of
alligation.

Questions related to replacement, addition or omission of components of


mixture may be easily solved using fractions. Be it in form of ratio or by
taking volume into consideration. A very useful formula for problems based
on mixtures is given below:
Final ratio= Initial ratio (fraction of component remained)
Final volume= initial volume (fraction of component remained)
A few sample problems explaining the concept of alligation are given below.

Example 1: ‘An alloy containing 50% gold is formed by melting two metals
2
A and B. Metal A contains 66 % gold while metal B contains 40% gold. Find
3

the ratio in which metal A and B are mixed to form the alloy.’

Solution: Let’s take the fractional equivalents of given percentages and


calculate using alligation.

2 Adda247 Publications For any detail, mail us at


Publications@adda247.com
Cracker Book for Bank (IBPS | SBI | RRB PO | Clerk) Mains Exams

Hence, required ratio is 3:5.

Example 2: ‘Average weight of boys in a class is 55kg while average age of


girls in the same class is 35. If the average of all the students in the class is
40 kg, find the ratio of boys and girls in the class.’

Solutions:

Required ratio is 1:3.

Example 3: ‘A can contains mixture of two liquids A and B in the ratio 7 : 5.


When 9 litres of mixture are drawn off and the can is filled with B, the ratio
of A and B becomes 7 : 9. How many litres of liquid A was contained by the
can initially?’

Solution: Since B is filled by replacing some amount of mixture, we should


take A into consideration
Initial ratio of A to the whole mixture = 7/12
7
Final ratio of A to the whole mixture =
16
7 7 12 3
Using formula, = ×𝑦 Or, 𝑦 = =
16 12 16 4

3 Adda247 Publications For any detail, mail us at


Publications@adda247.com
Cracker Book for Bank (IBPS | SBI | RRB PO | Clerk) Mains Exams
3 1th
This shows the mixture remained th of itself which means part was
4 4

drawn off.
1
ATQ, of total = 9
4

Or, total mixture = 9 × 4 = 36 litres


7
Initial amount of liquid A = × 36 = 21 litres
12

Practice Exercise Based on new Pattern

1. 150 kg of alloy containing copper and zinc in the ratio 3 : 2 mixed with
‘X’ kg of alloy containing copper and zinc in the ratio 2 : 3. If the
overall alloy should contain copper between 45% to 55%, what can be
minimum value of X?
(a) 450 kg (b) 100 kg (c) 50 kg
(d) Cannot be determined (e) None of these

2. Three different liquids which have 10% water, 20% water and x% of
water are mixed in the ratio of their quantity 2 : 3 : 4 respectively. If
12% of water is present in final mixture. Calculate value of x.
(a) 9% (b) 20% (c) 7%
(d) 15% (e) 17%

3. ‘x’ liters of a 30% alcohol solution is mixed with 40 liters of 60%


alcohol solution & a resultant of 50% alcohol solution is formed. Now
‘3x’ liters of y% alcohol solution is added to 30 liters of 50% alcohol
solution which resulted in 45% alcohol solution. The ratio of y : x is
(a) 17 : 6 (b) 16 : 15 (c) 7 : 15
(d) 14 : 5 (e) 17 : 8

4 Adda247 Publications For any detail, mail us at


Publications@adda247.com
Cracker Book for Bank (IBPS | SBI | RRB PO | Clerk) Mains Exams

4. There are three varities of sugar with their quantity in the ratio of 3 :
4 : 5. If 9 kg of first variety and 4 kg of second variety are added to
their respective quantity and x kg of 3rd type is removed from it, then
final ratio becomes 9 : 10 : 10. Find the sum of initial quantities of
these varieties.
(a) 120 kg (b) 96 kg (c) 84 kg
(d) 108 kg (e) None of these

5. A jeweler mixed gold and copper in 2 proportion. In type ‘A’ alloy, 6


gm gold is mixed with 5 gm copper and in type ‘B’ alloy, 5 gm gold in
mixed with 3 gm copper. If jeweler have 122 gm gold and 90 gm
copper, then find the weight of type ‘B’ alloy.
(a) 60 gm (b) 80 gm (c) 70 gm
(d) 100 gm (e) 90 gm

6. Two liquid containing (X and Y) mixture mixed, in the ratio 2 : 3. X


contain 40% sugar in water, while Y contain alcohol in water. After
adding 20 gram sugar in the final mixture ratio of Sugar, alcohol and
water becomes 8 : 5 : 16. Given that all ratio is calculate on weight.
Find the weight alcohol in mixture ‘Y’.
(a) 25 grams (b) 50 grams (c) 60 grams
(d) 80 grams (e) 75 grams

7. Two vessels A and B of equal capacities contain mixtures of milk and


water in the ratio 4:1 and 3:1, respectively. 25% of the mixture from A
is taken out and added to B. After mixing it thoroughly, an equal
amount is taken out from B and added back to A. The ratio of milk to
water in vessel A after the second operation is
(a) 79: 21 (b) 83: 17 (c) 77: 23
(d) 81: 19 (e) None of these

5 Adda247 Publications For any detail, mail us at


Publications@adda247.com
Cracker Book for Bank (IBPS | SBI | RRB PO | Clerk) Mains Exams

8. A 280 ml mix real juice pack contains mixture of Mango juice and
orange juice in the ratio of 9 : 5. If x ml juice taken out from pack and
20 ml of orange juice mixed in pack so the new ratio of Mango juice
and orange juice become 7 : 5, then find what quantity of mango juice
taken out from pack initially?
(a) 48 ml (b) 42 ml (c) 54 ml
(d) 60 ml (e) 64 ml

9. There are two vessels A and B which contains mixture of sulphuric


acid and nitrous oxide in the ratio of 7 : 2 and 3 : 4 respectively.
Mixture of both vessels are mixed to obtain a mixture of 390 ml, in
which quantity of nitrous oxide is 160 ml. Find ratio of quantity of
mixture in vessel A quantity of mixture in vessel B?
(a) 7 : 6 (b) 6 : 7 (c) 5 : 7
(d) 7 : 9 (e) 4 : 7

10. There is two vessels A and B. Vessel A & B contains the mixture of
milk, mango juice and water in the ratio of 8 : 5 : 3 & 6 : 5 : 2
respectively. If both vessels are mixed respectively in the ratio of 16 :
13 into another vessel C then the total quantities of mango juice in
vessel C will be 20 liters. Find the difference between the quantity of
milk and water in vessel C?
(a) 15 liters (b) 10 liters (c) 18 liters
(d) 25 liters (e) 20 liters

11. Two container P and Q have mixture. In P three liquid A, B and C are
mixed in a ratio of 2 : 1 : 1 and in Q two liquid B and C are mixed in
ratio of 3 : 2. Mixture of both container P and Q poured into third
container X which show the quantity ratio of A, B and C is 8 : 13 : 10.
Find the quantity in Q is what % less or more than the quantity in P.
(a) 3.25% (b) 12.5% (c) 6.25%
(d) 11% (e) 25%

6 Adda247 Publications For any detail, mail us at


Publications@adda247.com
Cracker Book for Bank (IBPS | SBI | RRB PO | Clerk) Mains Exams

12. In vessel A 80% is milk and rest is water. Vessel B also contain same
1
mixture in which milk is 62 % of total volume. 25ℓ of mixture taken
2
out from vessel A and poured in vessel B so that milk becomes 100%
more than water in vessel B. If new quantity of mixture in vessel B is
16% less than initial quantity of mixture in vessel A then find initial
quantity of milk in vessel A?
(a) 100l (b) 125l (c) 115l
(d) 105l (e) 112l

Directions (13-14): Three milkman P, Q and R have mixture of milk and


water in the quantity of (X + 24) liters, (X+54) liters and (X + 84) liters and
milk and water in the ratio of 2 : 3, 8 : 7 and 13 : 5 respectively.

13. If P sold 50 liters of his mixture and 8 liters of milk added in


remaining mixture, new ratio of milk and water becomes 6 : 7 . What
quantity of water should be added by Q and R, so new ratio of their
mixture becomes 5 : 7 & 5 : 4 respectively ?(1 marks)
(a) 56 liters& 40 liters (b) 54 liters& 42 liters
(c) 50 liters& 30 liters (d) 60 liters& 50 liters
(e) 64 liters& 30 liters

14. Q sold 40% of his mixture and R sold 50% of his mixture, remaining
mixture of both became equal. If all three charge Rs. 2 for one liter of
water and production cost of milk for P, Q and R in the ratio of 5 : 6 : 7
and total mixture, which all three milkman have cost Rs. 3644. Find
production cost of milk per liter for P, Q and R respectively? (1 marks)
(a) 5 Rs. 6 Rs & 7 Rs. (b) 25 Rs. 30 Rs & 36 Rs.
(c) 10 Rs. 12 Rs & 16 Rs. (d) 10 Rs. 12 Rs & 14 Rs.
(e)10 Rs. 14 Rs& 15s.

7 Adda247 Publications For any detail, mail us at


Publications@adda247.com
Cracker Book for Bank (IBPS | SBI | RRB PO | Clerk) Mains Exams

15. Vessel A & B contains mixture of orange juice and mango juice in the
ratio of 5 : 4 and 5 : 2 respectively. Some quantity of mixture taken out
from vessel A and B and mixed in another vessel C, if vessel C contains
mixture of orange juice and mango juice in the ratio of 5 : 3 and total
quantity of mixture in vessel C is 96 l. then find the difference between
mixture taken out from vessel A & B?
(a) 16 l (b) 12 l (c) 8 l
(d) 10 l (e) 18 l

16. Mixture of petrol and water in vessel A are in the ratio of 4:1 and in
vessel B are in the ratio of 5:3. If 20 l of mixture taken out from vessel
A and proud in vessel B so new ratio of petrol to water in vessel B
becomes 19:10, then find initially quantity of petrol in vessel A, if new
quantity of mixture in vessel B is 26 l more than initial quantity of
mixture in vessel A?
(a) 64l (b) 72l (c) 68l
(d) 56l (e) 48L

17. Two container P and Q have mixture of different liquids. In P three


liquid A, B and C is mixed in a ratio of 2 : 1 : 1 and in Q two liquid B
and C is mixed in ratio of 3 : 2. Mixture of both container P and Q
poured into third container X which show the quantity ratio of A, B
and C is 8 : 13 : 10. Find the quantity in Q is what % less or more than
the quantity in P.
(a) 3.25% (b) 12.5% (c) 6.25%
(d) 11% (e) 25%

18. Ratio of Vodka and Wine in vessel A is 5 : 3 and same mixture in vessel
B in the ratio of 3 : 2, 16ℓ of mixture from vessel A taken out and
poured in vessel B new ratio of Vodka to wine becomes 29 : 19. If new
quantity of mixture in vessel B is equal to initial quantity of mixture in
vessel A, then find quantity of Vodka after 16 ℓ of mixture has been
taken out from vessel A ?
(a) 50𝑙 (b) 48𝑙 (c) 54𝑙
(d) 80𝑙 (e) 84𝑙

8 Adda247 Publications For any detail, mail us at


Publications@adda247.com
Cracker Book for Bank (IBPS | SBI | RRB PO | Clerk) Mains Exams

19. Sameer and Veer entered into partnership business with the capital of
Rs. x and Rs. (x + 12000) respectively. After One year Divyaraj joined
them with capital of Rs. (x + 8000). At the end of 2 nd year Sameer and
Divyaraj withdraw their capital and Veer invested for one more year.
If Sameer, Divyaraj and Veer gets profit in the ratio of 8 : 6 : 21
respectively. Find sum of capital invested by all three ?
(a) 66000 Rs. (b) 54000 Rs. (c) 64000 Rs.
(d) 68000 Rs. (e) 70000 Rs.

20. Wheat of x Rs./kg is mixed with wheat of y Rs./kg in the ratio 2 : 3. On


2
selling the mixture at 49 Rs./kg there is a gain of 16 %. If the quantity
3
in which they were mixed is reversed and selling price remains same,
550
then gain percent becomes %. What is the volume of ‘x’.
19
(a) 25 Rs./kg (b) 30 Rs./kg (c) 28 Rs./kg
(d) 32 Rs./kg (e) 20 Rs./kg

21. Three vessels A, B and C of same capacity are all full. Vessel A and B
contain mixture of milk and water in ratios 4 : 1 and 7 : 3 while vessel
C contains only pure milk. The contents of all the vessels are poured
into a larger vessel D. After drawing 30 liters of the mixture from
vessel D, the final quantity of water in Vessel D is 40 liters. Find the
capacity of each vessel
(a) 80 (b) 85 (c) 95
(d) 70 (e) 90

22. Two container P and Q have mixture. In P three liquid A, B and C is


mixed in a ratio of 2 : 1 : 1 and in Q two liquid B and C is mixed in ratio
of 3 : 2. Mixture of both container P and Q poured into third container
X which show the quantity ratio of A, B and C is 8 : 13 : 10. Find the
quantity in Q is what % less or more than the quantity in P.
(a) 3.25% (b) 12.5% (c) 6.25%
(d) 11% (e) 25%

9 Adda247 Publications For any detail, mail us at


Publications@adda247.com
Cracker Book for Bank (IBPS | SBI | RRB PO | Clerk) Mains Exams

23. Vessel A contains (X + 24) 𝑙 of mixture of milk and water in the ratio of
7 : 8, while vessel B contains (X + 54) 𝑙 of same mixture of milk and
1
water in the ratio of 3 : 2. If 37 % of mixture from vessel A and 40%
2
of mixture from vessel B taken out and mixed in vessel C, then the
remaining mixture in vessel B is 15𝑙 more than that of in vessel A. find
quantity of milk in vessel C?
(a) 49 litre (b) 57 litre (c) 55 litre
(d) 53 litre (e) 51 litre

24. Three container (A, B and C) have capacity in the ratio of 6 : 8 : 7


respectively. These three container are pour with three type of liquid
X, Y and Z in the ratio 2 : 3 : 1 (in A), 1 : 2 : 5 (in B) and 3 : 1 : 3 (in C)
respectively. Equal quantity of mixture is taken out from these
containers and poured in container D. Quantity of liquid Y is 1/8 liter
more than quantity of liquid X in container D. Find out the quantity
taken out from each container.
(a) 24 (b) 35 (c) 21
(d) 20 (e) None of these

25. A shopkeeper has mixtures of nitrous oxide and water in two vessels
A and B. Vessel A and vessel B contains mixtures of nitrous oxide and
water in the ratio of 7 : 2 & 5 : 3 respectively. The shopkeeper has
taken out the mixtures from vessels A and B in the ratio of 9 : 8 and
mixed it in vessel C. If shopkeeper sold 68 gm of mixture from vessel C
on the cost price of nitrous oxide, which is Rs. 80 per gm, find profit of
shopkeeper?
2 2 2
(a) 39 % (b) 41 % (c) 45 %
3 3 3
2 2
(d) 37 % (e) 35 %
3 3

10 Adda247 Publications For any detail, mail us at


Publications@adda247.com
Cracker Book for Bank (IBPS | SBI | RRB PO | Clerk) Mains Exams

26. Amount invested in business respectively by Rana, Nikhil and Yash,


was in AP respectively, and Yash invested thrice amount as that of
Rana. Rana was manager and Nikhil was a programmer, and they both
get salary of 8% and 5% of total profit respectively. If Rana invested
for a month, Nikhil for 8 months and Yash invested for 4 months and
Nikhil got Rs.6000 more than both of them together, find their total
profit.
(a) Rs. 100000/- (b) Rs. 150000/- (c) Rs. 174000/-
(d) Rs. 180000/- (e) Rs. 20000/-

27. There are three vessels A, B & C having Milk to water ratio as 3 : 5, 1 :
3 and 7 : y. After adding mixture of vessel B to A, the milk to water
ratio becomes 1 : 2 in A . Now 60% of mixture is taken out from vessel
A and mixture of vessel C is mixed with remaining mixture of vessel A
which results in final Milk to Water ratio of 3 : 4. Find the value of y
given that volume of B is 20 ltrs and volume of C is (7 + y) ltrs.
(a) 1 (b) 4 (c) 5
(d) 3 (e) 2

28. There are three mixtures A, B and C. Mixture A contains 10% of water
and B contains 15% of water. If A and B are mixed together than
resultant mixture contains 12% water. while If B is mixed with C,
which contains 22% water so the resultant mixture contains of 18%
water. All these mixture are mixed in a jar and then 30 ml water is
also added. Find total volume of water preset in jar, if initially mixture
C contains 66 ml water.
(a) 144 ml (b) 216 ml (c) 184 ml
(d) 196 ml (e) 200 ml
11 Adda247 Publications For any detail, mail us at
Publications@adda247.com
Cracker Book for Bank (IBPS | SBI | RRB PO | Clerk) Mains Exams

29. There are two containers A and B filled with oil with different prices
and their volumes are 140 litres and 60 litres respectively. Equal
quantities are drawn from both A and B in such a manner that the oil
drawn from A is poured into B and the oil drawn from B is poured into
A. The price per litre becomes equal in both A and B. How much oil is
drawn from each of A and B:
(a) 40 litre (b) 80 litre (c) 30 litre
(d) 21 litre (e) 42 litre

30. A shopkeeper has two types Wheat i.e Type A & Type B and cost price
per kg of Type A wheat is 10 times of cost price per kg of type B
wheat. Shopkeeper cheated a costumer by saying that he will give him
200 kg of Type A wheat but shopkeeper gives 200 kg of mixture of
Type A & Type B wheat on 10% above the cost price per kg of Type A
7
wheat. If shopkeeper made a total profit of 71 % in this transaction,
8
then find ratio between quantity of Type B wheat to Type A wheat in
sold mixture?
(a) 2 : 5 (b) 2 : 3 (c) 1 : 3
(d) 2 : 7 (e) 1: 5

31. Two farmers A & B have mixture of urea & Potassium in the quantity
1
of 120 kg and 80 kg respectively. The mixture of farmer A has 37 %
2
potassium and the mixture of farmer B has 40% potassium. If farmer
A used 60% of his mixture & B used 50% of his mixture and both gives
their remaining mixture to farmer C, who have already an equal
quantity of urea & Potassium. If ratio of urea & Potassium in resulting
mixture of farmer C is 7 : 5, find total initial quantity of mixture farmer
C had?
(a) 24 kg (b) 36 kg (c) 32 kg
(d) 42 kg (e) 48 kg

12 Adda247 Publications For any detail, mail us at


Publications@adda247.com
Cracker Book for Bank (IBPS | SBI | RRB PO | Clerk) Mains Exams

32. Vessel A, B and C contains mixture of milk and water. Ratio of the
quantity of A, B and C is 25 : 20 : 18 respectively. In vessel A milk to
water ratio is 2 : 3 and in vessel B water to milk ratio is 2 : 3. If vessel
C contain milk to water ratio is 4 : 5 and all the mixture of 3 vessel
poured to another vessel D then in vessel D quantity of water exceeds
by 6 litres than the quantity of milk in that vessel. Find out the
quantity of mixture contained in vessel A, B and C.
(a) 50 Ltr., 40 Ltr., 36 Ltr. (b) 25 Ltr., 20 Ltr., 18 Ltr.
(c) 75 Ltr., 60 Ltr., 54 Ltr. (d) 100 Ltr., 80 Ltr., 72 Ltr.
(e) None of these

33. In a party a tank is filled with some quantities of juice. Three vessels
of different volume are used to serve the juice. Volume of the smallest
vessel is 25% less than volume of second largest vessel and the
1
volume of second largest vessel is 33 % less than volume of the
3
largest vessel. If 5 times the juice is served by largest vessel, 8 times
1
juice is served by second largest vessel and finally when 3 times
3
juice is served by the smallest vessel, the tank gets emptied
completely. Find the total quantity of juice served by second largest
vessel is what percent of volume of the tank?
4 4 4
(a) 42 % (b) 44 % (c) 48 %
9 9 9
4 4
(d) 40 % (e) 36 %
9 9

34. There are three fruit sellers Ramesh, Suresh and Vikash, all of them
have a mixture of two types of apple. Ramesh has (x+18) kg of apple,
Suresh has 12 kg of apple more than that of Ramesh while Vikash has
3 kg of apple more than that of Suresh. The two types of Apple are
mixed in the ratio of 5 : 3, 2 : 3 and 4 : 3 for them respectively. If
Ramesh sold 16 kg of mixed apple and added 12 kg of second type of
apple then the new ratio becomes 5 : 6 for him. Vikash added y kg of
second type of apple to his mixture and thus the new ratio of both
types for him is 1 : 1. Then find the value of y.
(a) 8 kg (b) 9 kg (c) 10 kg
(d) 6 kg (e) 7 kg
13 Adda247 Publications For any detail, mail us at
Publications@adda247.com
Cracker Book for Bank (IBPS | SBI | RRB PO | Clerk) Mains Exams

35. There are two vessels, vessel A and vessel B. Vessel A contains milk
and water in the ratio 5 : 4 and vessel B contains Y litre of wine. 18
litre of mixture from vessel A is taken out and poured in vessel B.
Now, 10 litre of mixture from vessel B is taken out and poured in
vessel A. If quantity of wine in vessel A is found to be 4 litre then, find
the initial quantity of wine in vessel B.
(a) 8 L (b) 9 L (c) 10 L
(d) 12 L (e) 6 L

Solutions

1. (c); If overall alloy contain copper as 45%


3 2x 45
(150 × + ) = (150 + x)
5 5 100
2x 9
(90 + )= (150 + x)
5 20
1800 + 8x = 1350 + 9x
x = 450 kg
If overall alloy contain copper as 55%
2x 55
(90 + )= (150 + x)
5 100
1800 + 8x = 1650 + 11x
3x = 150 ⇒ x = 50 kg
Minimum value of X is 50 kg

2. (c); Let the quantity of three liquids is 200a, 300a and 400a
10% of water in first type means 20a water
20% of water in second type means 60a water
x% of water in third type means 4xa water
∴ ATQ,
20a+60a+4xa 12
=
900a 100
28a
⇒ 4xa = 108a – 80a ⇒ x= =7
4a

14 Adda247 Publications For any detail, mail us at


Publications@adda247.com
Cracker Book for Bank (IBPS | SBI | RRB PO | Clerk) Mains Exams

3. (e); From statement I


30 60
x×100+40×100 1 (2400+30x) 1
(x+40)
= ⇒ =
2 100(x+40) 2

⇒ x = 20
Now from statement II
y 50
(3x×100+30×100) 45
=
3x+30 100

Here 3X = 3 × 20 = 60 litres
60y
( )+15 45 60y 81
100
⇒ = ⇒ = – 15
90 100 100 2

⇒ y = 42.5
∴ y ∶ x = 42.5 : 20 = 17 : 8

4. (d); Let the initial quantity is 3y, 4y & 5y of these varities


According to condition
3y + 9 ∶ 4y + 4 ∶ 5y – x = 9 ∶ 10 ∶ 10
From 1st 2 ratios
3y+9 9
⇒ =
4y+4 10
⇒ y = 9, Hence sum of initial quantities is (3 + 4 + 5) × 9
= 108 kg
NOTE: No need calculate Value of x

5. (b); Let gold and copper in Type A alloy be 6a and 5a


Let gold and copper in type B alloy 5b and 3b
⇒ 6a + 5b = 122 … (i)
5a + 3b = 90 … (ii)
Solving equation (i) and (ii) we get
a = 12 ⇒ b = 10
Weight of type ‘B’ alloy = (5 + 3) × 10
= 80 gm
15 Adda247 Publications For any detail, mail us at
Publications@adda247.com
Cracker Book for Bank (IBPS | SBI | RRB PO | Clerk) Mains Exams

6. (a); Let weight of alcohol in Y = a gram


Let weight of mixture X = 100x gram
So, sugar weight in X = 40x gram
Water weight in X = 60x gram
So, weight of Y = 150x gram
Weight of water in Y = (150x – a) gram
ATQ,
40x+20 : a : 150x–a+60x = 8 : 5 : 16… (i)
a 5
⇒ =
210x–a 16
⇒ a = 50x … (ii)
Put value of ‘a’ in eqn. (i)
40x+20 8 40x+20 8 1
= ⇒ = ⇒ x=
a 5 50x 5 2
1
weight of alcohol in Y = 50 × = 25 gram.
2

7. (a); Assume there is 20litres of the mixture in both the vessels.


In vessel A, milk = 16litres and water
= 4litres
If 25% mixture from A added to B,
milk in B = 15 + 4 = 19litres
water in B = 5 + 1 = 6litres
Ratio = 19: 6
Equal amount from vessel B to vessel A
19 79
milk in A = 12 + =
5 5
6 21
= water in A = 3 + =
5 5
Hence, the ratio is 79: 21

8. (c); Mango juice in mixture


9
= 280 × (9+5) = 180 ml
5
Orange juice in mixture = 280 × (9+5)
= 100 ml
ATQ—
9x
180–14 7
5x =
100–14+20 5

16 Adda247 Publications For any detail, mail us at


Publications@adda247.com
Cracker Book for Bank (IBPS | SBI | RRB PO | Clerk) Mains Exams

5(2520 – 9x) = 7(1400 – 5x + 280)


45x – 35x = 12600 – 11760
10x = 840 ⇒ x = 84 ml
Quantity of mango juice taken out from pack initially
9
= 84 × = 54ml
14

9. (b); Let total mixture of sulphuric acid and nitrous oxide in vessel A
and B be P mℓ and Q mℓ respectively
ATQ,
P + Q = 390 … (i)
2P 4Q
+ = 160
9 7
14P + 36Q = 10080 … (ii)
From 14 × (i) – (ii)
Q = 210 mℓ
P = 390 – 210 = 180 mℓ
180
Required ratio = =6:7
210

10 (c); Let quantity of mixture mixed in vessel C from vessel A & B be


16x and 13x respectively.
ATQ—
5 5
16x × + 13x × (6+5+2) = 20 liters
(8+5+3)
5x + 5x = 20 liters
x = 2 liter
Required difference =
8 6 3 2
[(16 × 2) × + (13 × 2) × ] – [(16 × 2) × + (13 × 2) × ]
16 13 16 13
= (16 + 12) – (6 + 4) = 18 liters

11. (c); Let in P, Quantity of A, B and C is 2x, x and x respectively


In Q → B and C is 3y and 2y
Now in final mixture → (X) A : B : C
2x : x+3y : x+2y = 8 : 13 : 10
Solving →
x = 4 unit
y = 3 unit
Quantity in P → (2 + 1 + 1) × 4 = 16 unit
Quantity in Q → (3 + 2) × 3 = 15unit
1
Required % = × 100 = 6.25%
16
17 Adda247 Publications For any detail, mail us at
Publications@adda247.com
Cracker Book for Bank (IBPS | SBI | RRB PO | Clerk) Mains Exams

12. (a); Let milk and water in vessel A


4a and a respectively
Let Milk and water in vessel B
5b and 3b respectively
ATQ—
4
5b+25×5 2
1 =
3b+25×5 1
5b+20 2
=
3b+5 1
6b – 5b = 20 – 10
b = 10
New quantity of mixture in vessel B
= (5 × 10 + 20) + (3 × 10 + 5) = 105ℓ
Initial quantity of mixture in vessel A
100
= 105 × = 125ℓ
84
Quantity of milk in vessel A
4
= 125 × = 100ℓ
5

13. (b); In 50 liters sold mixture of P


2
Milk = 50 × = 20liters
5
3
Water = 50 × = 30 liters
5
ATQ -
2(P+24)
5
−20+8 6 2P+48−60 6
3(P+24) = ⇒ =
−30 7 3P+72−150 7
5
7P – 42 = 9P = 234
2P = 234 – 42
192
P= ⇒ P = 96 liters
2
Mixture, which Q have
= (96 + 54) = 150 liter
Mixture which R have = (96 + 84)
= 180 liters
For Q –
8(96+54)
Milk = = 80 liters
15
(96+54)
Water = = 70 liters
15
18 Adda247 Publications For any detail, mail us at
Publications@adda247.com
Cracker Book for Bank (IBPS | SBI | RRB PO | Clerk) Mains Exams

Lets Q added x liters of water


80 5
=
70+x 7
5x = 560 – 350
5x = 210 ⇒ x = 42 liters
for R-
13(96+84)
Milk = = 130 liters
18
5(96+84)
Water = = 50 liters
18
Lets R added y liters of water
130 5
=
50+y 4
5y = 520 – 250
270
y= ⇒ y= 54 liters
5

14. (d); ATQ-


(100−40) (100−50)
(P + 54) × = (P + 84) ×
100 100
60P + 3240 = 50P + 4200
10P = 960 ⇒ P = 96 liters
Let production cost be Rs 5x, Rs 6x and Rs 7x
ATQ_
(48 × 5x + 72 × 2) + (80 × 6x + 70 × 2) + (130 × 7x + 50 × 2) =
3644
240x + 480x + 910x = 3644 – (144 + 140 + 100)
1630x = 3260
x = 3260 ⇒ x = 2 Rs
production cost for P = 5 ×2 = 10 Rs
production cost for Q = 6 × 2 = 12 Rs
production cost for R = 7 × 2 = 14 Rs
5
15. (b); Part of Orange juice in vessel A =
9
5
Part of Orange juice in vessel B =
7
5
Part of Orange juice in vessel C =
8
By allegation —

19 Adda247 Publications For any detail, mail us at


Publications@adda247.com
Cracker Book for Bank (IBPS | SBI | RRB PO | Clerk) Mains Exams

=9:7
(9–7)
Required difference = 96 × = 12ℓ
16

16. (b); Lets petrol and water in vessel A be 4x and x


Petrol and water in vessel B by 5y and 3y
ATQ—
4
5y+20×5 19 5y+16 19
1 = ⇒ =
3y+20× 10 3y+4 10
5
50y + 160 = 57y + 76
7y = 84 ⇒ y = 12
New mixture in vessel B
= (5 × 12 + 16) + (3 × 12 + 4)
= 76 + 40 = 116 ℓ
Initial quantity of vessel A
= (116 – 26)
= 90ℓ
4
Quantity of petrol in vessel A = 90 ×
5
= 72 ℓ

17. (c); Let in P, Quantity of A, B and C is 2x, x and x respectively


In Q → B and C is 3y and 2y
Now in final mixture → (X) A : B : C
2x : x+3y : x+2y = 8 : 13 : 10
Saving →
x = 4 unit ⇒ y = 3 unit
Quantity in P → (2 + 1 + 1) × 4 = 16 unit
Quantity in Q → (3 + 2) × 3 = 15unit
1
Required % = × 100 = 6.25%
16
20 Adda247 Publications For any detail, mail us at
Publications@adda247.com
Cracker Book for Bank (IBPS | SBI | RRB PO | Clerk) Mains Exams

18. (a); Let ratio of Vodka and wine in vessel A be 5x and 3x


and ratio of Vodka and wine in vessel B be 3y and 2y
ATQ—
5
3y+16× 29 3y+10 29
8
3 = ⇒ =
2y+16× 19 2y+6 19
8
58y – 57y = 190 – 174
y = 16
New quantity of mixture in vessel B
= (16 × 3 + 10) + (16 × 2 + 6)
= 96 ℓ
= initial quantity of mixture in vessel A
Quantity of Vodka remaining in vessel A
5 16×5
= 96 × – = 60– 10 = 50ℓ
8 8

19. (d); Investment Ratio of Sameer : Divyaraj : Veer


= x × 2 : Rs. (x + 8000) × 1 : Rs. (x + 12000) × 3
= 8 : 6 : 21
ATQ—
2x 8
(x+8000)
=
6
6x – 4x = 32000
x = 16000 Rs.
Required sum of capital (Sameer + Divyaraj + Veer)
= 16000 + (16000 + 8000) + (16000 + 12000)
= 68000 Rs.
6
20. (b); C.P. of mixture = 49 × = 42Rs/kg
7
2 y –42
=
3 42 –x
84 – 2x = 3y – 126
3y + 2x = 210 … (i)
New cost price of mixture after reverting the quantity in which
they are mixed.
11
(1 + )  new CP = 49
38
38
New CP = 49 × = 38 Rs./kg
49
21 Adda247 Publications For any detail, mail us at
Publications@adda247.com
Cracker Book for Bank (IBPS | SBI | RRB PO | Clerk) Mains Exams

So,

3 y –38
=
2 38 –x
114 – 3x = 2y – 76
190 = 2y + 3x ... (ii)
Solving (i) and (ii)
x = 30 Rs/kg

21. (e); Let the capacity of each vessel be 10x liters.


Then, vessel A contains 8x liters of milk and 2x liters of water.
Vessel B contains 7x liters of milk and 3x liters of water.
Vessel C contains 10x liters of milk.
Hence, Vessel D contains 25x liters of milk and 5x liters of water
i.e. in the ratio 5 : 1.
According to the question,
30 liters of the mixture is drawn from vessel D.
Final quantity of water in Vessel D = 40 liters
1
⟹ 5x − × 30 = 40
6
⟹ 5x = 45
⟹x=9
Capacity of each vessel = 10x = 90 liters

22. (c); Let in P, Quantity of A, B and C is 2x, x and x respectively


In Q → B and C is 3y and 2y
Now in final mixture → (X) A : B : C
2x : x+3y : x+2y = 8 : 13 : 10
Saving →
x = 4 unit
y = 3 unit
Quantity in P → (2 + 1 + 1) × 4 = 16 unit
Quantity in Q → (3 + 2) × 3 = 15unit
1
Required % = × 100 = 6.25%
16
22 Adda247 Publications For any detail, mail us at
Publications@adda247.com
Cracker Book for Bank (IBPS | SBI | RRB PO | Clerk) Mains Exams

23. (b); ATQ,


(100−37.5) (100−40)
(x + 24) × + 15 = (x + 54) ×
100 100
5 3
(x + 24) × + 15 = (x + 54) ×
8 5
5(5x + 120 + 120) = 8(3x + 162)
25x – 24x = 1296 – 1200
x = 96 liters
total mixture take out from vessel
3
A = (96 + 24) × = 45 litre
8
Total mixture taken out from vessel B
2
= (96 + 54) × = 96 liter
5
7 3
Milk in vessel C = 45 × + 60 ×
15 5
= (21 + 36) = 57 litre

24. (c); Let P ltr. of mixture are taken out from all 3 containers.
Quantity of X, Y and Z from A container
2P 3P P
→ , ,
6 6 6
From B container
P 2P 5P
→ , ,
8 8 8
From C container
3P P 3P
→ , ,
7 7 7
3P 2P P
Quantity of Y → + +
6 8 7
84P+42P+24P 150P
= =
168 168
2P P 3P
Quantity of X → + +
6 8 7
56P+21P+72P 149P
= =
168 168
150P 149P 1
Diff. = – = ⇒ P = 21 liter
168 168 8

25. (b); In vessel A


Nitrous oxide : Water = 7 : 2
In vessel B
Nitrous oxide : Water = 5 : 3
In 68 gm of mixture
23 Adda247 Publications For any detail, mail us at
Publications@adda247.com
Cracker Book for Bank (IBPS | SBI | RRB PO | Clerk) Mains Exams

Mixture taken from vessel A


9
= 68 × = 36 gm
17
8
Mixture taken from vessel B = 68 ×
17
= 32 gm
Total nitrous oxide in 68gm of mixture
7 5
= 36 × + 32 × = 28 + 20 = 48 gm
9 8
Total cost of 68 gm mixture = 68 × 80
= 5440 Rs
Cost of nitrous oxide = 48 × 80 = 3840
5440−3840 2
Required profit = × 100 = 41 %
3840 3

26. (a); Let Rana invested Y Rs.


∴ Yash invested 3Y Rs.
And these are in AP.
Y+3Y
∴ middle term is = 2Y.
2
2Y is amount invested by Nikhil.
Let total profit be 100x.
Then as a salary Rana got 8X.
And Nikhil got 5X.
Remaining 87X was divided according to share.
Y × 1 : 2Y × 8 : 3Y × 4
Y : 16Y : 12Y
1 : 16 : 12
Out of 87X Nikhil got.
16
× 87X = 48X.
29
Total money Nikhil got is 53X
Hence ATQ,
53X – 47X = 6000 ⇒ X = 1000
∴ 100X = 100000/- Which is total profit.

24 Adda247 Publications For any detail, mail us at


Publications@adda247.com
Cracker Book for Bank (IBPS | SBI | RRB PO | Clerk) Mains Exams

27. (b); ATQ, Volume of B is 20 liters


1
Milk (vessel B)= × 20 = 5 liters.
4
3
Water (vessel B) = × 20 = 15 liters.
4
When this is added to vessel A.
3x+5 1
= ⇒ 6x + 10 = 5x + 15
5x+15 2
x= 5 liters
Hence, milk in vessel A = 3x + 5 = 3 × 5 + 5
= 20 liters
Water in vessal A = 5x + 15 = 5 × 5 + 15
= 40 liters
60
60% of mixture of A = × (40 + 20)
100
= 36 liters.
Remaining mixture = 24 liters.
Milk remaining = 8 liters.
Water remaining = 16 liters.
After adding the content of Vessel C.
8+7 3 12
= ⇒ 60 = 48 + 3x ⇒ x = = 4
16+x 4 3

28. (b); ATQ –

Therefore,
Ratio of mixture, A to B is 3:2.
And ratio of mixture B to C is 4:3.
Hence, ratio of volume of mixtures A, B & C is
A:B:C=6:4:3
It is given that contains 22% water, which is 66 ml. Hence total
volume of mixture C is
100
66 × = 300 ml
22
B is 400 ml
A is 600 ml

25 Adda247 Publications For any detail, mail us at


Publications@adda247.com
Cracker Book for Bank (IBPS | SBI | RRB PO | Clerk) Mains Exams
15
Water in B = × 400 = 60 ml
100
10
Water in A = ×600=60 ml
100
Total water in jar after adding 30 ml water
= 60 + 66 + 60 + 30 = 216 ml.

29. (e); Let the price of oil per litre in container 'A' be 'x' and that in 'B'
be 'y' and the litres mixed be 'a' Initial price of A is 140x.
When 'a' litres are removed,it should be 140x-ax.
When 'a' litres from B are added,it is 140x-ax+ay , similarly for B
is 60y-ay+ax.
Now, the overall litres of both mixtures do not change as ‘a’
litres are taken away and are replaced, So price per litre after
mixing for :-
1. Solution A :- (140x-ax+ay)/140
2. Solution B :- (60y-ay+ax)/60.
Equating them,
8400(x − y) = 200a(x − y)
Or, a = 8400/200
Or, a = 42 litres

30. (b); Let cost price per kg of Type B wheat = x Rs.


And, Cost price per kg of type B wheat
= 10x Rs.
Selling price of mixture of Type A & Type B wheat = 11x Rs.
Cost price per kg of mixture of Type A & Type B wheat = 11x
32
× = 6.4x Rs.
55
ATQ –
Let N kg of type B wheat in mixture of 200 kg of Type A & Type B
of wheat
6.4x × 200 = N × x + 10x (200 − N)
1280 = N + 2000 – 10N
9N = 920 ⇒ N = 80 kg
80 2
Required ratio = =
120 3

26 Adda247 Publications For any detail, mail us at


Publications@adda247.com
Cracker Book for Bank (IBPS | SBI | RRB PO | Clerk) Mains Exams

31. (c); Farmer A has ratio of urea & Potassium in mixture = 5 : 3


Farmer B has ratio of urea & Potassium in mixture = 3 : 2
Let farmer C have x kg of urea & x kg of potassium
Farmer C has Urea in resulting mixture
40 5 50 3
= 120 × × + 80 × × +x
100 8 100 5
= (54 + x ) kg
Farmer C has Potassium in resulting mixture
40 3 50 2
= 120 × × + 80 × × +x
100 8 100 5
= (34 + x) kg
ATQ –
(54 + X ) 7
(34 + x)
=
5
270 + 5x = 238 + 7x
2x = 32 ⇒ x = 16 kg
initial quantity of mixture farmer C had = 16 × 2 = 32 kg

32. (a); Let quantity of vessels A, B and C is 25x, 20x and 18x
respectively
Now in vessel A → Milk : Water
2 : 3
10x milk and 15x water
In vessel B → Milk : Water
3 : 2
12x milk and 8x water
In vessel C → Milk : Water
4 : 5
8x milk and 10x water
ATQ→
33x– 30x = 6 ⇒ 3x = 6 ⇒ x = 2
Quantity in A, B and C
50 liter, 40 liter and 36 liter

27 Adda247 Publications For any detail, mail us at


Publications@adda247.com
Cracker Book for Bank (IBPS | SBI | RRB PO | Clerk) Mains Exams

33. (b); Let volume of largest vessel = 3x liter


2
Volume of second largest vessel = 3x ×
3
= 2x liter
75
Volume of smallest vessel = 2x ×
100
= 1.5x liter
ATQ—
10
Q = 5 × 3x + 8 × 2x + × 1.5x
3
Q = 15x + 16x + 5x
Q = 36x liter
16x 4
Required percentage = × 100 = 44 %
36x 9

34. (b); Ramesh has = (x + 18) kg


Suresh = (x + 18) + 12 = (x + 30) kg
Vikash = (x + 30) + 3 = (x + 33) kg
For Ramesh,
5
First type of Apple = (x + 18) × kg
8
3
2nd type of Apple = (x + 18) × kg
18
ATQ,
5 5 5
(x+18)× −16× 5 (x+18)× −10 5
8 8 8
3 3 = ⇒ 3 =
(x+18)× −16× +12 6 (x+18)× +6 6
8 8 8
30 15
⇒ (x + 18) × − 60 = (x + 18) × + 30
8 8
15
⇒ (x + 18) × = 90 ⇒ x = 30 kg
8
For Vikash,
Total quantity = 63 kg
4
First type = 63 × = 36 kg
7
3
Second type = 63 × = 27 kg
7
ATQ,
36 1
= ⇒ y = 9 kg
27+y 1

28 Adda247 Publications For any detail, mail us at


Publications@adda247.com
Cracker Book for Bank (IBPS | SBI | RRB PO | Clerk) Mains Exams

35. (d); Let vessel A has 5x L and 4x L of milk and water


After first process ratio of wine, milk and water in vessel B will
be
5 4
= Y ∶ 18 × ∶ 18 × = Y : 10 : 8
9 9

Now 10 litre of mixture from B is poured in A


y
Then, wine will be = × 10
y+10+8
y
× 10 = 4
y+18

10y = 4y + 72 ⇒ 6y = 72
y = 12 litre

29 Adda247 Publications For any detail, mail us at


Publications@adda247.com
Cracker Book for Bank (IBPS | SBI | RRB PO | Clerk) Mains Exams

1 Adda247 Publications For any detail, mail us at


Publications@adda247.com
Cracker Book for Bank (IBPS | SBI | RRB PO | Clerk) Mains Exams

Chapter

3 Partnership

BEST APPROACH TO SOLVE THE QUESTIONS

Any partnership in business is based on two components. One is the money


or capital that is invested and the other is the time for which the money is
invested.
Mathematically,

Profit ∝ investment
Profit ∝ time
Profit ∝ investment × time
Profit share of an individual is directly proportional to investment and time.
Hence, any profit share of a person out of total profit is decided by the net
effect of these two components.
If ratio of investment by two persons is X:Y and ratio of time for which they
invested is A:B respectively, then their profit share is in the ratio of 𝑋 × 𝐴 ∶
𝑌 × 𝐵= XA : YB

Example 1: A, B and C entered into a business by investing their intial sum


of Rs. 12600, Rs. 14400 and Rs. 13200 respectively. After 6 months C left
the partnership and B also left the partnership after 8 months. If after a year
total profit was Rs. 69600, then find profit share of C.’

Solution. Here, instead of taking absolute values of money invested, we may


use their simplest values i.e. ratio.
Ratio of investment of A, B and C is 21:24:22 respectively. And ratio of time
for which they invested is 12: 8 : 6 = 6 : 4 : 3
Ratio of their profit share = (21× 6): (24 × 4): (22 × 3)= 21:16:11
11
Now, profit share of C= × 69600 = 15950.
48
2 Adda247 Publications For any detail, mail us at
Publications@adda247.com
Cracker Book for Bank (IBPS | SBI | RRB PO | Clerk) Mains Exams

Example: ‘Aman started a business investing Rs. 70,000. Rakhi Joined him
after six months with an amount of Rs. 1,05,000 and Sagar Joined them with
Rs. 1.4 lakhs after another six months. The amount of profit earned should
be distributed in what ratio among Aman, Rakhi and Sagar respectively, 3
years after Aman started the business?’

Solution. Ratio of profit share is given by the ratio of (investment×time)


Or ratio of their investment is 70000 : 105000 : 140000 = 2 : 3 : 4
Ratio of the time for which they invested = 36 : 30 : 24 = 6 : 5 : 4.
Hence, ratio of profit share among them = (2× 6): (3 × 5) ∶ (4 × 4)
= 12:15:16

Practice Exercise Based on New Pattern

1. A and B started a business with a capital of Rs. 32,000 and Rs. 56,000.
C join the business on a condition that they all will share the profit
equally. For the loss of A and B, C gives Rs. 19800 to A and B to
compensate their loss. How much amount A get out of Rs. 19800?
(a) 1200 (b) 1600 (c) 1800
(d) Data inadequate (e) None of these

Direction (2-3): A, B and C start a business. Ratio between investment of A


and B is 5 : x while ratio between investment of B and C is 9 : 11. A, B and C
invested for 8 month, 6 month and 12 month respectively.

2. If ratio between A’s share in profit to B’s share in profit is 5 : 6 and


difference between C’s share in profit to B’s share in profit is 2340,
then find the total profit earned by all three together? (2 Marks)
(a) 7700 (b) 5390 (c) 6160
(d) 6930 (e) Cannot be determined

3. Find the value of ‘x’. If out of total profit of Rs 2460, A’s share in profit
is Rs 600. (1 Mark)
(a) 8 (b) 7 (c) 6
(d) 5 (e) 4
3 Adda247 Publications For any detail, mail us at
Publications@adda247.com
Cracker Book for Bank (IBPS | SBI | RRB PO | Clerk) Mains Exams

4. Rahul and Rohit start a business by investing Rs.4900 and Rs.3600


2 1
respectively. After 4 months, Rahul and Rohit withdraw th and of
7 3
their initial investment and after 6 months of starting of business they
invested 60% of what they withdraw. Find the share of Rohit in total
profit of Rs. 8490
(a) Rs.3555 (b) Rs.4935 (c) Rs.1855
(d) Rs.4550 (e) Rs.4850

5. ‘P’ and ‘Q’ entered into a partnership with 50,000 and 60,000 for 8
and 6 months respectively. ‘P’ buy a car from its whole profit and ‘Q’
invested his whole profit in a scheme which offer 10% interest on S.I.
1
P’s car value depreciated 11 % every year. If after 2-year difference
9
between their money (excluding initial investment) is 23,480 then
find P’s profit.
(a) Rs. 72000 (b) Rs. 35000 (c) Rs. 64000
(d) Rs. 54000 (e) Rs. 81000

6. A and B started a business in partnership by investing in the ratio of


7 : 9 . After 3 months A withdraw 2/3 of its investment and after 4
1
months from the beginning B withdraw 33 % of its investment. If a
3
total earned profit is Rs. 10201 at the end of 9 months, find the share
of each in profit.
(a) Rs. 3535, Rs.6666 (b) Rs. 3055, Rs. 5555
(c) Rs. 4503, Rs. 1345 (d) Rs. 3545, Rs. 3333
(e) None of these

7. Satish and Saurbh entered into a partnership business for eight


months. Satish invested Rs. 40x and Saurbh invested 40% more than
Satish, after four months from starting of business Rituraj joined the
both with the equal capital of Satish and Saurbh invested together
initially. If they got a total profit of Rs. 7200, then find profit share of
Rituraj?
(a) 1600 Rs. (b) 1800 Rs. (c) 2000 Rs.
(d) 2200 Rs. (e) 2400 Rs.
4 Adda247 Publications For any detail, mail us at
Publications@adda247.com
Cracker Book for Bank (IBPS | SBI | RRB PO | Clerk) Mains Exams

8. Veer and Sameer enter into a business by making the investment in


the ratio of 3 : 4. After six months Veer added Rs. 2000 more in his
initial investment, while Sameer withdraw Rs. 4000 from his initial
investment. If at the end of one year Veer and Sameer shared profit in
the ratio of 7 : 9, then find initial investment of both?
(a) 69000 Rs. 92000 Rs (b) 36000 Rs. 48000 Rs.
(c) 27000 Rs. 36000 Rs. (d) 45000 Rs. 60000 Rs.
(e) 24000 Rs. 32000 Rs.

9. P and R entered into partnership business with the capital of Rs. x and
Rs. (x + 12000), after One year Q joined them with capital of Rs. (x +
8000) at the end of 2nd year P and Q with draw their capital and R
invest for one more year, if P, Q and R gets profit in the ratio of 8 : 6 :
21 respectively. Find sum of capital invested by all three?
(a) 66000 Rs. (b) 54000 Rs. (c) 64000 Rs
(d) 68000 Rs. (e) 70000 Rs.

10. Three partners A, B and C invested their amounts in ratio of 2 : 5 : 8.


At the end of four months, A invests some amount such that, his total
investment will be equal to C’s initial investment and C withdraw
some amount that its total investment will be equal to B’s initial
investment. If 10% of B’s share in profit of one year is Rs 2550. Then
share of A and B in total annual profit will be?
(a) 81500 (b) 61200 (c) 56100
(d) 75000 (e) None of these

11. A and B enter into a partnership business and decided to distribute


60% of profit as per (investment× 𝑡𝑖𝑚𝑒) ratio and rest in the ratio of
7:10. If the amount invested by A is 25 % more than B and B withdraw
half of his total investment after 9 months. Find the profit distribution
ratio of A and B according to given condition at the end of a year.
(a) 43:41 (b) 44:41 (c) 48:43
(d) 49:47 (e) None of these

5 Adda247 Publications For any detail, mail us at


Publications@adda247.com
Cracker Book for Bank (IBPS | SBI | RRB PO | Clerk) Mains Exams

Directions (12-13): Three partners Abhishek, Neeraj and Aniket enter into
a partnership business with the capitals in the ratio of 3 : 5 : 8. After four
month Abhishek, Neeraj and Aniket added Rs. 6000, Rs. 9000 and Rs. 12000
respectively, after next four months Abhishek and Aniket withdraw Rs.
5000 and Rs. 8000 respectively and Neeraj added additional Rs. 6000.

12. Two persons Ankur and Saurabh enter a business. Ankur invested
capital 6000 more than what Abhishek invested for first four month
and Saurabh invested what Aniket invested for last four months. If
Ankur and Saurabh got profit share after one year in the ratio of
15 :28 then find theinvestment of Neeraj for last four months?(1
marks)
(a) 24000 Rs. (b) 18000 Rs. (c) 16000 Rs.
(d) 30000 Rs. (e) 36000 Rs.

13. Amir and Mayank started a business in partnership. Amir


investmentfor six months isRs. 6000 more thanwhat Neeraj invested
for first fourmonthsand Mayank invested for eight months, Rs. 2000
lessthan whatAniket invested for first four monthsIf Mayank got profit
share of Rs. 15400, out of total profit Rs. 22750 thenfind the
investment of Amir is how much less than investment of Mayank?
(2 marks)
(a) 218000 Rs (b) 216000 Rs. (c) 210000 Rs.
(d) 220000 Rs. (e) 232000 Rs.

14. Veer, Sameer and Gopal enter into a business by making investment in
the ratio of 5 : 6 : 7 respectively. After six months Veer and Sameer
1
withdraw th and 25% of his initial investment respectively. After
5
1
eight months from the starting of business Gopal added 𝑡ℎ of his
7
initial investment. If at the end of one year profit share of Gopal is Rs.
2000 more than Sameer, then find profit share of Veer?
(a) 4420 Rs. (b) 4520 Rs. (c) 4820 Rs.
(d) 4320 Rs. (e) 4720 Rs.
6 Adda247 Publications For any detail, mail us at
Publications@adda247.com
Cracker Book for Bank (IBPS | SBI | RRB PO | Clerk) Mains Exams

15. Manish and Rituraj invested Rs. 12000 and Rs. 16000 in a business.
After four months Manish and Rituraj both added Rs. 4000 in their
initial investment. At the end of one year the total profit was Rs.
172500, if Manish and Rituraj invested their profit share on
compound interest at the rate of 20% and 10% respectively then find
difference between interests got by both at the end of two years?
(a) Rs.10250 (b) Rs.11520 (c) Rs.12210
(d) Rs.13110 (e) Rs.12660

Direction (16 – 17): Data given below about investment of three partners
in a business. Read the data carefully and answer the questions:
Swati, Monika and Anshika three business partners enter into a business by
making investment Rs. 4000, Rs. 6000 and Rs. 8000 respectively. After first
quarter Swati, Monika and Anshika added some amount in the ratio of 2 : 3 :
2 1
5. After second quarter Swati, Monika and Anshika withdrew 𝑡ℎ , 𝑟𝑑 and
5 3
3
𝑡ℎ respectively of capital invested in second quarter. At the end of nine
7
month profit share divided among Swati, Monika and Anshika in the ratio of
24 : 37 : 49 respectively.

16. Satish and Veer enter into a business, Satish invested 200% more than
Swati capital for second quarter, while Veer invested 300% more than
Anshika capital for third quarter. And profit share of Satish at end of
two year is Rs. 7250 out of total profit of Rs. 15950. Then find for how
many months Veer invested his capital?
(a) 14 months (b) 12 months (c) 18 months
(d) 10 months (e) 8 months

17. Monika and Anshika enter into a business with another partner
Shikha. Monika invested 144% of what she invested in second
quarter, while Anshika invested 160% of what she invested in second
quarter. If all three invested their capital for a year and Shikha gets a
profit of Rs. 9600 out of total profit of Rs. 28000, then find investment
of Shikha?
(a) 12,800 Rs. (b) 11,600 Rs. (c) 14,400 Rs.
(d) 14,200 Rs. (e) 13,600 Rs.

7 Adda247 Publications For any detail, mail us at


Publications@adda247.com
Cracker Book for Bank (IBPS | SBI | RRB PO | Clerk) Mains Exams

18. Satish, Veer and Neeraj entered into a business by investing in the
ratio of 4 : 6 : 3. After two years, Satish and Veer withdrew 25% and
1
33 % of their respective initial investment, while Neeraj added 50%
3
of his initial investment more. If at the end of three years they
received total profit of Rs. 93750, out of which 22.5% are used for
maintaining business and the remaining profit was divided among
them according to their investment ratio. Find difference between
profit share of Satish and Neeraj?
(a) 978.75 Rs. (b) 976.75 Rs. (c) 974.25 Rs.
(d) 968.75 Rs. (e) 966.75 Rs.

19. A, B and C enter into a partnership and invested some amount. After
1
one year A double its investment, B increase its investment by 33 %
3
and C increase its investment by 20%. In the third year A and B
withdraw their investments and D joins the partnership with C. After
three year they got profit in the ratio of 12 : 14 : 17 : 8 (A : B : C : D). If
difference between initial investment of B and C is 1150. Then Find
out the total initial investment made by A and D together?
(a) 12100 (b) 14400 (c) 13800
(d) 15000 (e) None of these

5 4 6
20. Three partners A, B and C invested in the ratio of ∶ ∶ in a business.
4 5 5
After 3 months A increased his capital by 40%, after 3 months more A
again increased his capital by 20%. B increased his capital by 25%
after every 4 months. C increased his capital by 25% after 3 months
and after 4 months more C increased his capital by 20%. If the total
profit of Rs. 30,130 is earned at the end of the year, find what was the
A’s share of profit?
(a) Rs. 12,420 (b) Rs. 14,420 (c) Rs. 13,240
(d) Rs. 15,340 (e) Rs. 12,340

8 Adda247 Publications For any detail, mail us at


Publications@adda247.com
Cracker Book for Bank (IBPS | SBI | RRB PO | Clerk) Mains Exams

21. A, B and C entered into a partnership business, A invested Rs. x, B


invested 25% more than A and C invested 20% more than B for first 6
months. After that A left the business and B withdraw 50% of his
investment. B left the business after 3 more months whereas C
2
increased his investment by 16 % . If at the end of year difference
3
between profit share of C and (A + B) together is Rs. 12375, then find
profit share of C ?
(a) 58500 Rs. (b) 56500 Rs. (c) 55680 Rs
(d) 55580 Rs. (e) 52680 Rs.

22. Veer, Sameer and Satish entered into a business by investing equal
amount each. After four months Sameer added half of his initial
investment, while Veer and Satish withdrew half of their initial
investment. At end of one year the total profit is Rs. 8000 and Veer
and Sameer invested their profit share at the rate of 10%
compounded annually for some time in a scheme. If the difference of
the amount received by Veer and Sameer from the scheme at the end
is Rs. 2420, find for how much time Veer and Sameer invested in the
scheme?
1
(a) 4 yr (b) 3 yr (c) 3 yr
2
(d) 2 yr (e) 1yr

23. P, Q and R entered into a partnership with their initial capital in the
1
ratio of 3 : 5 : 8 respectively. After third month P withdrew rd of his
3
initial capital and Q withdraw half of his initial capital. After next six
months P increased his capital by 100% more, Q increased by 300%
more and R increased by 25% more. If at the end of an year, difference
between profit share of Q and R was Rs. 7000, then find profit share of
P?
(a) Rs. 6600 (b) Rs. 7700 (c) Rs. 5500
(d) Rs. 5750 (e) Rs. 5950

9 Adda247 Publications For any detail, mail us at


Publications@adda247.com
Cracker Book for Bank (IBPS | SBI | RRB PO | Clerk) Mains Exams

24. Veer, Sameer and Divyaraj enter into a business by making an


1 1 1
investment in the ratio of ∶ ∶ respectively. After six months
129 301 602
1
from starting of business Veer withdrew 33 % of his initial
3
investment, while Sameer added 50% of his initial investment and
Divyaraj added Rs. 1800 respectively. If after one year profit share of
Veer is Rs. 10500 out of total profit of Rs. 20850, then find difference
between initial investment of Veer and Sameer?
(a) 6400 Rs. (b) 7200 Rs. (c) 7800 Rs.
(d) 5600 Rs. (e) 8400 Rs.

25. A, B & C, three friends started the business in which A invested for 6
months, B initially didn’t invest & started as working partner, while C
invested for 4 months. They decided to donate 1% of total profit and
to give 22% of total profit to B as salary. Find the ratio of their profit
shares (A:B:C) if A and C invested in the ratio of 2 : 3 and B also
invested an amount which is 80% of total amount invested by A and C
together for a single month.
(a) 1 : 3 : 1 (b) 1 : 1 : 3 (c) 3 : 1 : 3
(d) 1 : 1 : 1 (e) None of these

10 Adda247 Publications For any detail, mail us at


Publications@adda247.com
Cracker Book for Bank (IBPS | SBI | RRB PO | Clerk) Mains Exams

Solutions

A B
1. (c); Ratio of share of profit = 32 : 56
= 4 : 7 = 11
Let, total profit = Rs. 11
Now, A, B and C will distribute Rs. 11 equally
i.e.
A : B : C
11 11 11
3
: 3
: 3
11 1
Loss of A = 4 – =
3 3
11 10
Loss of B = 7 – =
3 3
Ratio of Loss = 1 : 10
Amount that got to compensate the loss
1
= × 19800 = 1800
11

Solution (2-3): Ratio between A’s, B and C’s profit share = 45 × 8 : 9x × 6 :


11x × 12
= 60 : 9x : 22x

2. (d); Let profit sharing of A, B and C be 60z, 9xz and 22xz


respectively.
ATQ,
22xz – 9xz = 2340
⇒ 13xz =2340
⇒ xz = 180
B’s and C’s share in profit is 9 × 180 and 22 × 180 respectively.
9×180
A’s share in profit = × 5 = 1350
6
Total profit earned by all three together
= 1350 + 1620 + 3960 = 6930
11 Adda247 Publications For any detail, mail us at
Publications@adda247.com
Cracker Book for Bank (IBPS | SBI | RRB PO | Clerk) Mains Exams

3. (c); ATQ,
60 600
=
60+9𝑥+22𝑥 2460
⇒ 246 = 60 + 31𝑥
186
⇒𝑥 = =6
31

4. (a); Investment of Rahul


For first four month = 4900
For fifth and sixth month
5
= × 4900 = 3500
7
3 2
For Last 6 month = 3500 + × × 4900
5 7
= 4340 Rs.
Investment of Rohit
For first 4 month = 3600 Rs.
2
Next 2 month = × 3600 = 2400 Rs.
3
3 1
Last 6 month = 2400 + × × 5600 = 3120
5 3
Profit ratio
Rahul Rohit
4900 × 4 + 3500 × 2 + 4340 × 6 : 3600 × 4 + 2400 × 2 + 3120 × 6
329 : 237
Total profit = 8490 Rs.
Share of Rohit out of total profit
8490
= × 237 = 3555 Rs.
566

5. (e); Profit ratio of P and Q


50,000 × 8 : 60,000 × 6
10 : 9
Let P’s and Q’s profit → 10𝑥 and 9𝑥
Value of P’s money after 2 years
8 8 640𝑥
= 10𝑥 × × =
9 9 81
Value of Q’s money after 2 years
12 Adda247 Publications For any detail, mail us at
Publications@adda247.com
Cracker Book for Bank (IBPS | SBI | RRB PO | Clerk) Mains Exams
9𝑥×2×10 120 108
=9𝑥 + = 9𝑥 × = 𝑥
100 100 10
108𝑥 640𝑥
Difference = – = 23,480
10 81
𝑥 = 8100
P’s profit = Rs. 81,000

6. (a);
A : B
Capital → 7×3 : 9×3
New Ratio → A : B
21x : 27x
Total capital invested by A in 9 months
= 21x × 3 + 7x × 6 = 105x
Total capital of B invested in 9 months
= 27x × 4 + 18x × 5
= 108x + 90x = 198x
A : B
Capital → 105x : 198x
According to question,
(105x + 198x) = Rs. 10201
303x = 10201
10201
x = Rs.
303
Hence,
10201
Share of A = × 105 = Rs. 3535
303
10201
Share of B = 198 × = Rs. 6666
303

7. (e); Satish capital = 40x Rs.


40
Saurbh capital = 40x + 40x ×
100
= 56x Rs.
Rituraj capital = 40x + 56x = 96x Rs.

13 Adda247 Publications For any detail, mail us at


Publications@adda247.com
Cracker Book for Bank (IBPS | SBI | RRB PO | Clerk) Mains Exams

Ratio of profit (Satish : Saurbh : Rituraj)


= 8 × 40x : 8 × 56x : 96x × 4
= 320x : 448x : 384x
=5:7:6
6
Rituraj profit share = 7200 × (5+7+6)
= 2400 Rs.

8. (a); Lets Veer and Sameer invested Rs. 3x and Rs. 4x respectively.
Ratio of investment of Veer and Sameer
= [3x × 6 + (3x + 2000) × 6] : [4x × 6 + (4x – 4000) × 6]
= (36x + 12000) : (48x – 24000)
ATQ—
36𝑥+12000 7
=
48𝑥 –24000 9
36𝑥 × 9 + 12000 × 9 = 7 × 48𝑥 − 24000 × 7
12000(9 + 14) = 12(7 × 4𝑥 − 3 × 9𝑥)
1000 × 23 = 28𝑥 − 27𝑥
x = 23000 Rs.
Investment of Veer = 3 × 23000 = 69000 Rs.
Investment of Sameer = 4 × 23000
= 92000 Rs.

9. (d); P : Q : R
Rs. x × 2 : Rs. (x + 8000) × 1 : Rs. (x + 12000) × 2
= 8 : 6 : 21
ATQ—
2𝑥 8
(𝑥+8000)
=
6
6x – 4x = 32000
x = 16000
Required sum of capital (P + Q + R)
= 16000 + (16000 + 8000) + (16000 + 12000)
= 68000 Rs.
14 Adda247 Publications For any detail, mail us at
Publications@adda247.com
Cracker Book for Bank (IBPS | SBI | RRB PO | Clerk) Mains Exams

10. (c); Let amount invested by A, B and C are 2x, 5x and 8x respectively.
⇒ At the end of 4th month A’s investment 2x + 6x = 8x
= c’s initial invested
⇒ C’s investment = 8x – 3x = 5x = B’s initial investment
Ratio of investment
A : B : C
= (2x × 4 + 8x × 8) : (5x × 12) : (8x × 4 + 5x × 8)
6 : 5 : 6
Let B’s profit = 5y
Now ATQ
5𝑦×10
= = 2550
100

y = 5100
share of A and B = 5y + 6y = 11y
= 11 × 5100 = 56100

11. (b); Let the total investment of B = 100


Investment of A = 100×125/100
Ratio of 60% profit
= 125×12 : 100×9+50×3= 10:7
Let the total profit = 170
Profit for A for only 60 % of profit
= 170×0.6×(10/17)=60
Profit of B for only 60 % of profit
= 102-60 = 42
Total profit of A = 60+ 68×7/17= 88
Total Profit of B = 42+68×10/17 = 82
Required ratio = 88:82 = 44:41
15 Adda247 Publications For any detail, mail us at
Publications@adda247.com
Cracker Book for Bank (IBPS | SBI | RRB PO | Clerk) Mains Exams

Solutions (12-13); Let investment of Abhishek, Neeraj and Ankit is


Rs. 3x, Rs. 5x and Rs. 8x respectively.
ATQ—Investment ratio of Abhishek, Neeraj and Aniket
= [3x × 4 + (3x + 6000) × 4 + (3x + 6000 – 5000)
× 4] : [5x × 4 + (5x + 9000) × 4 + (5x + 9000 + 6000) × 4] : [8x × 4 + (8x +
12000) × 4 + (8x + 12000 – 8000)
× 4]
= (36x + 28000) : (60x + 96000) : (96x + 64000)

12. (d); Investment of Ankur = Investment of Abhishek for first four


months and next four months
= (3x + 6000) Rs.
Investment of Saurabh = Investment of Aniket for last four
months
= (8x + 12000 – 8000) Rs.
ATQ—
(3𝑥+6000)×12 15
(8𝑥+4000)×12
=
28

120x - 84x = 168000 – 60000


36x = 108000
x = 3000 Rs.
Neeraj invested for last four months
= (5x + 9000 + 6000) Rs.
= (5×3000 + 9000 + 6000)
= 30000 Rs

16 Adda247 Publications For any detail, mail us at


Publications@adda247.com
Cracker Book for Bank (IBPS | SBI | RRB PO | Clerk) Mains Exams

13. (e); Investment of Amir = Investment of Neeraj for first four months
+6000
= 5x + 6000
Investment of Mayank = Investment of Aniket for first four
months – 2000
= 8x -2000
(5x+6000)×6 22750 –15400
(8x−2000)×8
=
15400
(5𝑥+6000) 7
(8𝑥−2000)
=
11

56x – 55x = 80000


x = 80000
required difference = 8x-2000-5x-6000
3x – 8000
240000 – 8000= 232000

14. (d); Let Veer, Sameer and Gopal invested Rs. 5x, Rs. 6x and Rs. 7x
respectively
Ratio between profit share of Veer, Sameer and Gopal
4 3
= (5𝑥 × 6 + 5𝑥 × × 6) : (6𝑥 × 6 + 6𝑥 × × 6):
5 4
8
(7𝑥 × 8 + 7𝑥 × × 4)
7

= 54𝑥 ∶ 63𝑥 ∶ 88𝑥


= 54 : 63 : 88
2000
Profit share of Veer = 54 × (88
–63)

= 4320 Rs.

17 Adda247 Publications For any detail, mail us at


Publications@adda247.com
Cracker Book for Bank (IBPS | SBI | RRB PO | Clerk) Mains Exams

15. (d); Ratio of profit of Manish and Rituraj


= [(12000 × 4)+ (12000 + 4000) × 8] : [(16000 × 4) + (16000 +
4000)× 8]
= 17600 : 224000 = 11 : 14
11
Profit share of manish = 172500 ×
25

= Rs.75900
14
Profit share of Rituraj = 172500 ×
25

= 96600 Rs
Equivalent CI of two year at the rate of 20%
20×20
= 20 + 20 + = 44%
100

Equivalent CI of two year at 10%


10×10
= 10 + 10 + = 21%
100

Required difference between interest


44 21
= 75900 × − 96600 ×
100 100

= 13110

Solutions (16-17): Let total amount added by Sawti and Monika after first
3
4000 ×3+(4000+2𝑥)×3+(4000+2𝑥)×5×3 24
quarter be Rs. 2x and Rs. 3x 2 =
6000×3+(6000+3𝑥)×3+(6000+3𝑥)×3×3 37

1
12000+12000+6𝑥+(36000+18𝑥)×5 24
=
18000+18000+9𝑥+12000+6𝑥 37
156000+48𝑥 24
=
240000+75𝑥 37

240500 + 74x = 240000 + 75x


x = 500 Rs.
18 Adda247 Publications For any detail, mail us at
Publications@adda247.com
Cracker Book for Bank (IBPS | SBI | RRB PO | Clerk) Mains Exams

16. (c); Total amount invested by Sawti in second quarter = (4000 +


2× 500) = 5000 Rs.
Total amount invested by Anshika in third quarter = (8000 +
500 × 5 − 4500)
= 6000 Rs.
300
Satish investment = 5000 ×
100
= 15000 𝑅𝑠.
400
Veer investment = 6000 × = 24000 𝑅𝑠.
100

Let Veer invested for T months


ATQ –
15000 ×24 7250
=
24000×𝑇 (15950−7250)
120 145
=
8×𝑇 174

T = 18 months

17. (c); Total amount invested by Monika in second quarter = 6000 +


3 × 500 = 7500 𝑅𝑠.
144
Monika investment = 7500 ×
100
= 10800 𝑅𝑠.
Total amount invested by Anshika in second quarter = 8000
+ 5 × 500 = 10500 𝑅𝑠.
160
Anshika investment = 10500 ×
100
= 16800 𝑅𝑠.
Let Shikha invested = P Rs.
ATQ –
𝑃×12 9600
=
(10800+16800)×12 (28000−9600)
𝑃 9600
=
27600 18400
P = 14,400 Rs.

19 Adda247 Publications For any detail, mail us at


Publications@adda247.com
Cracker Book for Bank (IBPS | SBI | RRB PO | Clerk) Mains Exams

18. (d); Let the amount invested by Satish, Veer and Neeraj initially be
4x, 6x and 3x respectively.
Ratio of profit share of Satish, Veer and Neeraj
3 2
= 4x × 24 + 4x × × 12) ∶ (6x × 24 + 6x × × 12) : (3x × 24 +
4 3
3
3x × × 12)
2
= 22 : 32 : 21
Total remaining profit after excluding the amount spend on
maintaining business
(100–225)
= 93750 × = 72656.25 Rs.
100
(22–21)
Required difference = 72656.25 ×
75

= 968.75 Rs.

19. (c); Let investment of A, B, C and D is a, b, c and d respectively.


A B C 𝐷

Now in firt year → a × 12 : b × 12 : c × 12

4b 6c
In 2nd year → 2a × 12 :
× 12 : × 12
3 5
6c
In 3rd year × 12 : 𝑑 × 12
5
4
A : B : C : D ⇒ (a × 12 + 2a × 12) : (b × 12 + b × 12) : c × 12 +
3
6
2 c × 12 : d × 12
5
7𝑏 17
3a : : 𝑐 : d = 12 : 14 : 17 : 8
3 5
⇒a:b:c:d=4:6:5:8
Difference between B and C initial investment = 1150
Total Investment of A and D together
1150
= × 12 = 13800
1

20 Adda247 Publications For any detail, mail us at


Publications@adda247.com
Cracker Book for Bank (IBPS | SBI | RRB PO | Clerk) Mains Exams

20. (a); Ratio of investment of A, B and C


5 4 6
= ∶ ∶ = 25 ∶ 16 ∶ 24
4 5 5
Ratio of profit after one year
A : B : C
25  3  35  3  42  6 : 16  4  20  4  25  4 : 24  3  30  4  36  5
= 432 : 244 : 372
= 108 : 61 : 93
30130
A’s share in profit = × 108 = 12,420
262

21. (a); Let investment of A, B and C


= x : 1.25x : 1.5x
= 4x : 5x : 6x
ATQ—
Profit share of A : B : C = 4x × 6 : (5x × 6 + 2.5x × 3) : (6x × 6 +7x
× 6)
= 24x : 37.5x : 78x
Given→ C – (A + B) = 12375
78x – (24x + 37.5x) = 12375
x = 750
Profit share of C = 750 × 78 = 58500 Rs.

22. (d); Let Veer, Sameer and Satish invested Rs. x initially
Ratio of profit of Veer, Sameer and Satish
x 3x x
= (x × 4 + × 8) : (x × 4 + × 8) : (x × 4 + × 8)
2 2 2
= 8x ∶ 16x ∶ 8x
=1:2:1
1
Profit share of Veer = 8000 × = 2000 Rs.
4
Profit share of Sameer
2
= 8000 × = 4000 Rs.
4
Let both invested for n years
10 n 10 n
4000 (1 + ) – 2000 (1 + ) = 2420
100 100
10 n
2000 (1 + ) = 2420
100
10 n
(1 + ) = 1.21
100
n = 2 years
21 Adda247 Publications For any detail, mail us at
Publications@adda247.com
Cracker Book for Bank (IBPS | SBI | RRB PO | Clerk) Mains Exams

23. (c); Lets P, Q and Q investment be 3x, 5x and 8x profit share of


P:Q:R
= (3x × 3 + 2x × 6 + 4x × 3) : (5x × 3 + 2.5x × 6 + 10x × 3) :
(8x × 9 + 10x × 3)
= 33x : 60x : 102x
= 11 : 20 : 34
ATQ—
34 – 20 => 7000
14 => 7000
1 => 500
Profit share of P = 11 × 500
= Rs. 5500

24. (b); Initial investment of Veer, Sameer and Divyaraj


1806 1806 1806
= ∶ ∶
129 301 602
= 14 : 6 : 3
Let Veer, Sameer and Divyaraj initial investment be 14x Rs., 6x
Rs. & 3x Rs. respectivelyProfit ratio of Veer, Sameer and Divyaraj
2
= (14x × 6 + 14𝑥 × × 6 ) : (6x × 6 + 9𝑥 × 6) ∶ [3𝑥 × 6 + (3𝑥 +
3
1800) × 6]
= (84x + 56x) : (36x + 54x) : (18x + 18x + 10800)
ATQ –
140𝑥 10500
=
90𝑥+36𝑥+10800 20850−10500
𝑥 1
(21𝑥+1800)
=
23

2x = 1800
x = 900 Rs.
Required difference = 14× 900 − 6 × 900
= 7200 Rs.

22 Adda247 Publications For any detail, mail us at


Publications@adda247.com
Cracker Book for Bank (IBPS | SBI | RRB PO | Clerk) Mains Exams

25. (d); Let A invested 2x and C invested 3x.


80
Then B invested = × (3x + 2x)
100
80
= × 5x = 4x
100
Let 100Y be their total profit.
According to question,
1% is given as donation and B gets 22% as salary of total profits.
Now ratio of their shares is
2x × 6 : 4x × 1 : 3x × 4
12x : 4x : 12x
3 : 1 : 3
3
A gets × [100Y– 23Y]
7
3
= × 77Y = 33Y
7
3
C gets = × 77Y = 33Y
7
1
B gets = × 77Y + 22Y = 33Y
7
Hence, ratio of their profit shares is
33Y : 33Y : 33Y
=1:1:1

23 Adda247 Publications For any detail, mail us at


Publications@adda247.com
Cracker Book for Bank (IBPS | SBI | RRB PO | Clerk) Mains Exams

1 Adda247 Publications For any detail, mail us at


Publications@adda247.com
Cracker Book for Bank (IBPS | SBI | RRB PO | Clerk) Mains Exams

Chapter

4 Average and Ages

BEST APPROACH TO SOLVE THE QUESTIONS

INTRODUCTION:
We all know that definition of average is sum of magnitude (weight, age,
marks or any measurable quantity) of all quantities divided by total number
of all quantities. Always remember, if you are calculating average you
are making all the quantities equal.

For eg. three brothers have number of candies with them is 9, 11 and 16.
But their mom took all the candies and distribute 12 to each. Average in
some questions referred as ‘Arithmetic mean’.
Questions from this topic are extremally easy and doesn’t require as much
time or sharp concept, mostly questions are asked in the form of average of
ages.

Example: Marks obtained by seven students are 70, 85, 95, 86, 74, 83 and
67. What is average marks obtained by them.

Solution: Sum of magnitude of all the observations is 70 + 85 + 95 + 86 +


74 + 83 + 67 = 560.
560
Required average = = 80.
7
NOTE: Please avoid short cut tricks mentioned in many books for such
small calculations. Those tricks are more time consumable. There is other
very simple and only trick that you must apply. Look at next question.

Example: Average age of 4 members of a family is 28 years and average


increases by 2 years when one of their relative joins them. Calculate the age
of relative.
2 Adda247 Publications For any detail, mail us at
Publications@adda247.com
Cracker Book for Bank (IBPS | SBI | RRB PO | Clerk) Mains Exams

Solution: Easy one! Let age of relative is x years. Total age of family before
joining the relative = 4×28=112
112+x
After joining the relative = 28+2, which gives us x = 38 years.
5
Right approach? Or wrong?
Try this approach, Imagine, they all have 28 rupees/candies/marbles
before their relative joins them. When he comes, they all have Rs 30 each.
What does it mean? It means their relative gave Rs 2 to all 4 of them and
even then, he was left with Rs 30, as average means amount is distributed
equally. How much he had given to them? Rs. 8, Right? Therefore, he
initially had total of Rs 38 with him.
Why to waste paper and time if we can do it mentally. Try next question.

Example: Average marks scored by a student in 4 subjects is 75. But when


marks of English are added to it, overall average became 70. How much did
he score in English?

Solution: Let 4 men have 75 gold coins each and when 5th one joins them,
each of those 4 men/exams gave 5 coins/marks to 5th one. And after taking
5 coins from 4 persons, 5th have total 70 coins. It means he initially had 50
coins and that the answer.

Example: Average marks scored by a student in 5 subjects was 79. But


when average is calculated for 4 subjects (without Hindi) it was 4 lesser
than the average of 5 subjects. What was the marks scored by him in Hindi?

Solution: Again let 5 men have 79 candies each and when one of them
leave, he took all candies belonging to him. It means each of these 4 had
borrowed 4 candies from him. He took 79 candies that was with him, plus
these 4 persons returned 16 of his. Therefore, he had 95 candies or marks
whatever you assume.

3 Adda247 Publications For any detail, mail us at


Publications@adda247.com
Cracker Book for Bank (IBPS | SBI | RRB PO | Clerk) Mains Exams

Example: Average run scored by Sachin in 19 innings of year 2007 is 49


runs while total run scored run scored by Afridi in 19 matches were 760. In
20th match, average of Afridi decreased by 2 runs/ match and average
run/match increased by 1 for Sachin. What is the difference b/w the scored
runs in 20th match by them?

760
Solution: Average runs by Afridi in 19 innings = 40 runs.
19
His average decreased by 2, which means all the 19 innings/persons have
given 2 runs/coins to 20th inning/person. New inning/person borrowed
19 × 2 = 38 runs/coins to become equal with others. Therefore, he himself
have no money and we can conclude Afridi scored 0 in that match.
While for Sachin, his 19 previous innings got 1 run each and even then, his
20th inning remains with 50 runs. Therefore, total runs by Sachin in 20th
inning are 69 runs.
Required difference is 69 runs.

This may look longer when you read it, but once you get the concept
you can solve these questions within 30 seconds.

AGES
The questions on ages are of two types either the average age of a group of
people is asked or we are given the ratios of ages of friends and we are
asked to calculate their individual age or average age. See it through a
question.

Example: Ratio of present age of Rahul and his father is 5:8 and 14 years
earlier this ratio was 1:2. What is present age of Rahul

Solution: 1 way to solve this is assume present age of Rahul and his father
5x and 8x. According to question 14 years earlier ratio of their age was
5x−14 1
=
8x−14 2

4 Adda247 Publications For any detail, mail us at


Publications@adda247.com
Cracker Book for Bank (IBPS | SBI | RRB PO | Clerk) Mains Exams

 (5x − 14) × 2 = 8x − 14
 2x = 14
 X=7
 Present age of Rahul = 7× 5= 35 years.
Or
If you want to solve it mentally, or in a shorter time, observe carefully
Present age ratio 5 : 8
14 years earlier 1 : 2
Note that as difference b/w ages of two persons is always same, so what we
do? We will make difference b/w two ratio’s same.
Multiply first ratio by difference of second and second ratio bye difference
of first.
First ratio when multiplied by difference of other i.e. (2-1=1) it becomes
5:8
When second ratio is multiplied by difference of first i.e. (8 - 5 = 3) we will
get
1×3 : 2× 3
3 : 6

Now think mentally, that 14 years ago age of Rahul was 3 units and now it is
5 unit.
Therefore 2 units are equal to 14 years, which means 5 unit (age of Rahul)
is 35 years.
NOTE: Concept behind multiplying by difference is that, we must make both
differences (b/w the ratios) equal. To make two numbers equal what we
do? We take their LCM as common. If difference b/w two ratios are 4 and 6,
we will multiply first ratio by 3 and second ratio by 2 to make their
difference equal. Cross check it by your self for some quantities for better
confidence.

Example: Ratio of age of two friends is 7:9 while 10 years later it will be
19:23. Calculate their average age after 7 years from present.

5 Adda247 Publications For any detail, mail us at


Publications@adda247.com
Cracker Book for Bank (IBPS | SBI | RRB PO | Clerk) Mains Exams

Solution: Difference b/w the ratios are 2 and 4. So to make them equal
multiply first ratio by 2. After multiplying we will get
14 : 18.
And second ratio we have is 19 : 23.
Hence conclude mentally, that 14 units is present age of younger friend and
19 units is his 10 years later age. It means that
19 – 1 4 =5 units = 10 year
1 unit = 2 years.
And their present average age is (28 + 36)/2 = 32 years.
And 7 years later mean 7 is added to both of their age, which means average
age is 32 + 7 = 39 years.

Key Points to Remember for New Pattern Questions


➢ Calculating average means making each quantity equal.
➢ To make difference of two ratios equal, try to think through
‘LCM of difference’ approach.
➢ If the average age of a group of people is X years, Y years
later it will be X + Y years.

6 Adda247 Publications For any detail, mail us at


Publications@adda247.com
Cracker Book for Bank (IBPS | SBI | RRB PO | Clerk) Mains Exams

Practice Exercise Based on new Pattern

1. In a group of 4 friends, ratio of present age of A and D is 4:5 and that B


to C is 3:4. Calculate the present average age of A, B and C, if 4 years
ago, A was 20 years younger than C and at present C is twice of age of
A.
(a) 20 years (b) 50 years (c) 40 years
(d) 25 years (e) 30 years

2. Ratio of age of Vikas to age of Rohit 2: 3 and average age of Rohit,


Vikas and Rahul is 23. When Arjun joins the group average age of
these 4 persons is 25 years. Calculate sum of age Arjun and Rahul, if
Rahul is 13 years older than Vikas?
(a) 40 years (b) 47 years (c) 60 years
(d) can’t be determined (e) None of these

3. S₁ is a series of five consecutive multiple of three, whose sum is 180


and S₂ is the series of four consecutive multiple of four whose second
smallest number is 13 more than second highest number of S₁ series.
Find the average of smallest number of S₁ series and highest number
of S₂ series.
(a) 51 (b) 49 (c) 47
(d) 45 (e) 43

4. A student finds the average of five two digits numbers. If One number
is reversed and the average is taken again then the average increase
by 5.4. If all five digits are consecutive multiple of four, then find the
number which is reversed?
(a) 58 (b) 36 (c) 74
(d) 48 (e) None of these

7 Adda247 Publications For any detail, mail us at


Publications@adda247.com
Cracker Book for Bank (IBPS | SBI | RRB PO | Clerk) Mains Exams

5. Average weight of a group is 20 kg. When 2 people include in the


group average weight becomes 22 kg after that when 4 another
people removed from the group, having weight half of the weight of 2
person included previously, average weight become 25. Find the
strength of group initially
(a) 18 (b) 20 (c) 22
(d) 16 (e) 14

6. Four years ago, Ratio of Ram’s to Shyam’s age is 9 : 11 and Shyam’s


age four years ago is same as Ram’s present age. If Rahul’s present age
is average of present age of Ram and Shyam, then find Rahul’s age 2
year ago.
(a) 20 (b) 18 (c) 24
(d) 22 (e) 26

7. Ratio of Ayush Age to his first son’s age is 20 : 9 and Ayush wife’s age
to Ayush 2nd son’s age is 3 : 1. If elder son of Ayush is 6 year elder than
another son (ayush has only two son and no daughter) and average
age of family is 26.5 year then find the age of Ayush’s wife.
(a) 24 (b) 36 (c) 40
(d) 38 (e) 28

8. Ritu take four, two-digit numbers and take average of these number
which is 52.5. If she reversed all the digits of 4 numbers she found
that their average is now 22.5 less than the previous one and they
formed A.P having common difference 4. Find the largest number.
(a) 82 (b) 80 (c) 36
(d) cannot be determined (e) 64

8 Adda247 Publications For any detail, mail us at


Publications@adda247.com
Cracker Book for Bank (IBPS | SBI | RRB PO | Clerk) Mains Exams

9. The average age of a group of six children is 15 years. From the group,
two children, whose ages were 3 years more and 5 years more than
the average age, left. 4 new children, whose average age is 4 years
more than the given average age, join the group. Find the new average
age.
(a) 15 years (b) 16 years (c) 17 years
(d) 18 years (e) 12 years

10. If age of P and R are added to twice the age of Q, the total becomes 59. If
the ages of Q and R are added to thrice the age of P, the total become
68. And if the age of P is added to thrice the age of Q and thrice age of
R, the total becomes 108, what is the total of ages of P, Q and R?
(a) 45 years (b) 47 years (c) 49 years
(d) 42 years (e) 44 years

11. Sweta and Neha profess to tell their present ages as 25 and 20 years
respectively. (Not original age). Ratio of their original ages 5 year ago
400
is 5 : 4. Sum of ages of both 5 years hence is % more than the sum
9
of present ages of both professed by them. Find the ratio of their
present original age.
(a) 4:7 (b) 5:6 (c) 6:5
(d) 4:5 (e) 5:7

12. Average age of a man, woman and their son is 30 years. Man’s age is
1
two year more than his wife and age of son is th the sum of age of his
4
mother and father.When two other family members were added, new
average becomes 27 years. If difference between age of two new
member are one year then find difference between son and the new
member who is elder.
(a) 7 years (b) 8 years (c) 4 years
(d) 2 years (e) 5 years

9 Adda247 Publications For any detail, mail us at


Publications@adda247.com
Cracker Book for Bank (IBPS | SBI | RRB PO | Clerk) Mains Exams

13. Present average age of A, B, C and D is 25 years. Sum of age of A and B


is 150% of sum of age of C and D. Ratio of age of B to age of C is 3 : 5.
Calculate 10 years later age of A, if B and D are of same age.
(a) 45 years (b) 40 years (c) 35 years
(d) 55 years (e) 50 years

14. Ratio of present age of A and B is 4 : 5, while that of C and D is 6 : 7. If


10 years ago Ratio of age of A and C is 1 : 2 and that of B & D is 3 : 5,
Find average of present age of A, B and C together.
(a) 27.5 (b) 25 (c) 27
(d) 26.5 (e) 24.5

1 1 1
15. The ratio of ages of three friends Veer, Sandeep and Satish is : : .
3 2 4
The average age of Veer and his wife is 33 years whereas the average
3
age of Sandeep and his wife is 39 years. The age of Satish’s wife is th
8
of the sum of age of Veer’s wife and Sandeep’s wife and the ratio of
Veer’s age to that of age of Satish’s wife is 4 : 3 then find the sum of
ages of all the three friends.
(a) 104 yr (b) 112 yr (c) 117 yr
(d) 91 yr (e) 120 yr

16. There are some teachers in a college with average age of 45 years. Sum
of maximum age and minimum age is 108 years and difference of
maximum age and minimum age is 27 years. If fourteen teachers left
the college with the average age of 32.5 years and six new teachers
joined college. Out of six new teachers four ages are equal to
maximum age teacher and two teachers age equal to minimum age
4
teacher. If new average becomes 49 years, then find the number of
7
teachers initially?
(a) 54 (b) 60 (c) 64
(d) 68 (e) 72

10 Adda247 Publications For any detail, mail us at


Publications@adda247.com
Cracker Book for Bank (IBPS | SBI | RRB PO | Clerk) Mains Exams

17. The ratio between present age of Veer & Saurbh is 3: 4 and between
present age of Divyaraj & Gopal is 2: 3. If sum of present age of Veer,
Saurbh, Divyaraj & Gopal is 128 years and ratio between sum of age of
Veer & Saurbh to sum of age of Divyaraj & Gopal is 63: 65, then find
the sum of age of Veer & Gopal after eight years.
(a) 90 years (b) 78 years (c) 86 years
(d) 84 years (e) 82 years

18. 2n years ago, the age of Raju was four times that of his son and n years
ago, the age of Raju was thrice that of his son. If n years later, the sum
of the ages of Raju and his son will be 80 years, then the difference in
the ages of Raju and his son is
(a) 20 years (b) 40 years (c) 24 years
(d) 30 years (e) 34 years

19. ‘2n’ years ago ratio of Amit’s age to Inder’s age is 5 : 4. ‘n’ years ago
ratio of Inder’s age to Satish’s age is 9 : 7. Difference between present
age of Amit to Satish’s present age is 12 years. Find the sum of present
ages of all three if ratio of Amit’s age to Satish’s age after ‘n’ year will
be 13 : 9.
(a) 81 (b) 84 (c) 87
(d) 90 (e) Cannot be determined

20. P1 is the series of four consecutive numbers divisible by 4 with an


average of 34, while P2 is the series of five consecutive numbers
divisible by 9, and whose second lowest number is equal to second
highest number of P1 series. If P3 is the series of seven consecutive
1
numbers, whose lowest number is 𝑟𝑑 𝑜𝑓 lowest number of P2 series,
3
then find the sum of average of P2 & P3 series?
(a)53 (b)51 (c)57
(d)55 (e)49

11 Adda247 Publications For any detail, mail us at


Publications@adda247.com
Cracker Book for Bank (IBPS | SBI | RRB PO | Clerk) Mains Exams

21. Average of five two digit numbers, in which 2 numbers say A and B
are reverse of each other, is 25.4. If all three numbers except A and B
multiplied by (–2) then the average of five numbers decreased by
43.2. What could be the greatest value among A and B.
(a) 32 (b) 23 (c) 50
(d) 41 (e) 14

22. Average of any 200 consecutive natural numbers is 499.5. If next 1000
numbers more add in it then find the new average.
(a) 1035.5 (b) 1299.5 (c) 1199.5
(d) 1099.5 (e) 999.5

23. Average age of a group of people is four times of the number of people
in the group. Sakshi leaves the group and the average age is still four
times of the number of people in the group. After that Sheetal leaves
the group and the average age is still four times of the number of
people in the group. If ratio between Sakshi’s age to Sheetal’s age is 21
: 19, then find the average age of the group if Ritu leaves the group
whose age is 20 years
(a) 36 years (b) 37 years (c) 38 years
(d) 39 years (e) 40 years

Directions (24-25): Rakesh distributed Rs.50000 into (X, Y, Z) 3 groups


having 50 people in total. Number of people in group ‘X’ is equal to the
number of people in group ‘Z’ and total money got by group ‘Y’ is equal to
the total money got by group ‘Z’. Average money got by group X and Y
3500
together is Rs. and average money got by group Y and Z is Rs.1000
3

24. Find the number of people in group Y?


(a) 20 (b) 16 (c) 10
(d) 18 (e) 24

25. Find the amount got by each person in group Z.


(a) 800 Rs. (b) 600 Rs. (c) 1000 Rs.
(d) 750 Rs. (e) 500 Rs.
12 Adda247 Publications For any detail, mail us at
Publications@adda247.com
Cracker Book for Bank (IBPS | SBI | RRB PO | Clerk) Mains Exams

Solutions

11. (e); Let the present age of B is 3K and that of C is 4K.


Also assume present age of A is 4m and D is 5m.
Note that age difference between A and C will always be 20
years.
4K – 4m = 20
K – m = 5 …(i)
4K
Also =2
4m
K = 2m …(ii)
m = 5 years
K = 10 years
20+30+40
∴ Average of A, B and C = = 30 years
3

2. (c); Let age of Arjun, Vikas, Rohit and Rahul is a, b, c and d


respectively
Now,
b : c = 2 : 3 …(i)
b + c + d = 23 × 3 = 69 …(ii)
a + b + c + d = 25 × 4 = 100
∴ a = 31 years.
Also d – b = 13
d=b+13
Put this value in eq. (ii)
2b + c = 56
From (i) b = 16 years, d = 29 years
c = 24 years
∴ a + d = 31 + 29 = 60 years.

13 Adda247 Publications For any detail, mail us at


Publications@adda247.com
Cracker Book for Bank (IBPS | SBI | RRB PO | Clerk) Mains Exams

3. (d); Let S₁ is a series consists 3x – 6, 3x – 3, 3x, 3x + 3, 3x + 6


ATQ,
3x – 6 + 3x – 3 + 3x + 3x + 3 + 3x + 6 = 180
⇒ x = 12
S₁ series = 30, 33, 36, 39, 42
Second smallest no. of S₂ = 39 + 13 = 52
S₂ series = 48, 52, 56, 60
30+60 90
Required average = = = 45
2 2

4. (b); Let the number which is reversed is “10a+b”


If it is reversed then the number becomes “10b+a”
ATQ,
10𝑏 + 𝑎 − 10𝑎 − 𝑏 = 5 × 5.4
⇒ 9𝑏 − 9𝑎 = 27
⇒𝑏−𝑎 =3
Numbers can be 14, 25, 36, 47, 58 and 69 but all the five digits
are multiple of 4
⇒ The number should be 36

5. (a); Let initially strength = x


ATQ,
Total weight → 20x
When two persons include
Total weight = 22 (x + 2)
Weight of two persons included = 2x + 44
4 persons excluded weight
2x+44
= = x + 22
2

ATQ,
22 (x + 2) – (x + 22) = (x + 2 – 4) × 25
x = 18
14 Adda247 Publications For any detail, mail us at
Publications@adda247.com
Cracker Book for Bank (IBPS | SBI | RRB PO | Clerk) Mains Exams

6. (d); Let age of Ram and Shyam 4 years age 9x and 11x respectively
So ATQ,
11x = 9x + 4
x=2
their present age is 22 and 26 years
Rahul’s age 2 year ago
22+26
= – 2 = 22
2

7. (b); Let Ayush Age, his wife age, his first son’s age and his second
son’s age is 20x, 3y, 9x and y respectively.
ATQ,
9x – y = 6 …(i)
20x + 3y + 9x + y = 26.5 × 4 …(ii)
Solving (i) & (ii)
x=2
y = 12
his wife’s age = 12 × 3 = 36

8. (a); Let four number is w, x, y and z after reversing they formed A.P.
Sum of all number = (52.5 – 22.5) × 4 = 120
n=4
d=4
n
Sn = [2a + (n– 1)d]
2
4
120 = [2a + (4 – 1)4]
2
a = 24
Reversed numbers
24, 28, 32, 36
Original number
42, 82, 23, 63
Largest number → 82

15 Adda247 Publications For any detail, mail us at


Publications@adda247.com
Cracker Book for Bank (IBPS | SBI | RRB PO | Clerk) Mains Exams

9. (b); Sum of the ages of 6 children


= 15 × 6 = 90
When two children left, sum of the ages of 4 children = 90 −
(18 + 20) = 52
Sum of the ages of New children
= (15 + 4) × 4 = 76
76+52
∴ Required average =
8
128
= = 16 years
8

10. (e); P + 2Q + R = 59
3P + Q + R = 68
P + 3Q + 3R = 108
Solving the equation, P = 12 years, Q = 15 years, R = 17 years.
Sum of their ages = 44 years.

11. (c); Let age of Sweta 5 years ago = 5x


Let age of Neha 5 years ago = 4x
According to question
400
(5x + 10) + (4x + 10) = (100% + %) (25 + 20)
9
13
9x + 20 = × 45
9
x=5
Sum of their present age = (5+ 4)×5+10
= 45 year + 10 years = 55 years
Required ratio = 6 : 5

12. (e); Let age of man is x years


Age of woman = (x – 2) years
(x+x–2) (x–1)
Age of son = = years
4 2
ATQ—

16 Adda247 Publications For any detail, mail us at


Publications@adda247.com
Cracker Book for Bank (IBPS | SBI | RRB PO | Clerk) Mains Exams
(x–1)
x+(x –2)+ 2
= 30
3
2x+2x–4+x –1
= 90
2
5x = 185
x = 37 years
(37–1)
son age = = 18 years
2
let age of two new members be y years and (y – 1) years
ATQ—
90+(y+y–1)
= 27
5
2y – 1 = 135 – 90
46
y=
2
y = 23
Required difference = 23 – 18 = 5 years

13. (d); If average age of A, B, C and D is 25 years, then total age of A, B, C


and D is 100 years.
Also if C + D = 2x
Then A + B = 3x [A + B is 150% of C + D]
5x = 100
x = 20
A + B = 60 …(i)
C + D = 40 …(ii)
Now, in question ratio of B : C is given as 3 : 5.
This is also ratio of D : C, as B and D are of same age. From (ii)
We can calculate
C = 25 years
D = 15 years
& Hence B = 15 years
∴ A = 45 years
10 years later age of A is 45 + 10 = 55 years.

17 Adda247 Publications For any detail, mail us at


Publications@adda247.com
Cracker Book for Bank (IBPS | SBI | RRB PO | Clerk) Mains Exams

14. (b); Let present age of A, B, C and D is 4x, 5x, 6y and 7y respectively.
Then, ATQ,
4𝑥–10 1
= ⇒ 8x – 20 = 6y – 10
6𝑦–10 2
⇒ 8x – 6y = 10 …(i)
Similarly
5x–10 3
= ⇒ 25x – 50 = 21y – 30
7y–10 5
⇒ 25x – 21y = 20 …(ii)
Solving (i) and (ii)
25𝑥 – 21𝑦 = 20
16𝑥 – 12𝑦 = 20
– + –
9𝑥 – 9𝑦 = 0
𝑥=𝑦
Put this value in equation (i), we will get x=y=5.
15 ×5 75
Therefore, average age of A, B and C together is = =
3 3
25 𝑦𝑒𝑎𝑟𝑠.

15. (a); Ratio of ages of Veer, Sandeep and Satish be 4 : 6 : 3


Let their age be 4x years, 6x years and 3x years respectively
Age of veer’s wife = (66 – 4x) years
Age of Sandeep’s wife = (78 – 6x) years
Atq,
Age of Satish’s wife
3
= × (66 − 4𝑥 + 78 − 6𝑥)
8
3
= × (144 − 10𝑥)𝑦𝑒𝑎𝑟s
8
Now,
4𝑥 4
3 =
(144−10𝑥) 3
8
⇒ 8x = 144 – 10x
⇒ 18x = 144
⇒ x = 8 years
Required sum = 8 × (4 +6 + 3) = 104 years
18 Adda247 Publications For any detail, mail us at
Publications@adda247.com
Cracker Book for Bank (IBPS | SBI | RRB PO | Clerk) Mains Exams

16. (c); Let total teachers in a college = N


Let maximum age = a year
Minimum age = b years
ATQ –
a + b = 108 ------- (i)
a – b = 27 --------(ii)
From (i) & (ii)
2a = 135 year
a = 67.5 years
Ans, b = 40.5 years
347
N× 45 − 14 × 32.5 + 4 × 67.5 + 2 × 40.5 + = (N – 14 + 6)
7
7(45N – 455 + 270 + 81) = 347N – 2776
315N − 728 = 347N – 2776
32N = 2048
2048
N=
32
N = 64

17. (e); Let present age of Veer & Saurbh be 3x and 4x respectively and
present age of Divyaraj & Gopal be 2y and 3y respectively
3𝑥+4𝑥 63
=
2𝑦+3𝑦 65
x : y = 9: 13=9k:13k
ATQ –
3x+4x+2y+3y=128
3× 9𝑘+4× 9𝑘+2×13k+3×13k=128k=128
K=1.
Therefore, age of Veer is 27 years.
Similarly, age of Gopal is 39 years.
Sum of their present ages=66years
After 8 years, sum of their ages = 66+16 = 82years.

19 Adda247 Publications For any detail, mail us at


Publications@adda247.com
Cracker Book for Bank (IBPS | SBI | RRB PO | Clerk) Mains Exams

18. (d); Let, the present ages of Raju and his son be 𝑥 and 𝑦 respectively.
2n years ago,
𝑥 − 2𝑛 = 4(𝑦 − 2𝑛)
𝑥 = 4𝑦 − 6𝑛 ……….(i)
𝑛 years ago,
𝑥 − 𝑛 = 3(𝑦 − 𝑛)
⇒ 𝑥 = 3𝑦 − 2𝑛 …………..(ii)
Solving (i) and (ii),
𝑦 = 4𝑛
And, 𝑥 = 4 × 4𝑛 − 6𝑛 = 10n
N years later,
𝑥 + 𝑛 + 𝑦 + 𝑛 = 80
⇒ 4𝑛 + 𝑛 + 10𝑛 + 𝑛 = 80
⇒ 16𝑛 = 80
⇒𝑛=5
Difference in their ages = 10𝑛 − 4𝑛 = 50 − 20 = 30

19. (d); Let, Present age of Amit, Inder and Satish be x, y and z
respectively.
ATQ,
x –2n 5
=
y –2n 4
⇒ 4x – 8n = 5y – 10n
⇒ 5y – 4x = 2n …(i)
y –n 9
=
z –n 7
⇒ 7y – 7n = 9z – 9n
⇒ 2n = 9z – 7y …(ii)
Equating (i) & (ii)
5y – 4x = 9z – 7y
12y = 9z + 4x
And, x – z = 12
x+n 13
=
z+n 9

20 Adda247 Publications For any detail, mail us at


Publications@adda247.com
Cracker Book for Bank (IBPS | SBI | RRB PO | Clerk) Mains Exams

⇒ 9x + 9n = 13z + 13n
⇒ 4n = 9x – 13z
9x – 13z = 18z – 14y
9x + 14y = 31z
Now x = 12 + z
9 (12 + z) + 14y = 31z
⇒ 22z – 14y = 108
or 11z – 7y = 54 …(iii)
and 10y – 8x = 9x – 13z
17x = 10y + 13z
17 (12 + z) = 10y + 13z
204 + 17z = 10y + 13z
4z + 204 = 10y …(iv)
On solving (iii) and (iv)
y = 30, z = 24, x =36
Required sum = 30 + 24 + 36 = 90

20. (c); Let four consecutive numbers of P₁


Series be a, (a + 4), (a + 8) and (a + 12)
ATQ—
𝑎+(𝑎+4)+(𝑎+8)+(𝑎+12)
= 34
4
4a + 24 = 136
4a = 112
𝑎 = 28
P₁ series = 28, 32, 36, 40
Let P₂ series be b, (b + 9), (b + 18), (b + 27), (b + 36)
Given,
𝑏 + 9 = 36
𝑏 = 27
P₂ series = 27, 36, 45, 54, 63
Let P₃ series be, 𝑐, (𝑐 + 1), (𝑐 + 2), (𝑐 + 3), (𝑐 + 4), (𝑐 + 5) and
(𝑐 + 6)
21 Adda247 Publications For any detail, mail us at
Publications@adda247.com
Cracker Book for Bank (IBPS | SBI | RRB PO | Clerk) Mains Exams

Given,
27
𝑐=
3
𝑐=9
P₃ series
9, 10, 11, 12, 13, 14, 15
Required average
(27+36+45+54+63) (9+10+11+12+13+14+15)
= +
5 7
225 84
= + = 45 + 12 = 57
5 7

21. (d); Let A = 10a + b


and B = 10b + a
Let sum of other three number is = X
X + 10a + b + 10b + a = 5 × (25.4) = 127 …(i)
And
–2X+10a+b+10b+a= 127 – 5 (43.2) …(ii)
Solving (i) and (ii)
We get a + b = 5
If 𝑎 would be → 0, 1, 2, 3, 4, 5
Then b would be → 5, 4, 3, 2, 1, 0
as 10a + b can’t be 50 because its reverse is 05 which is not a
two digit number.
Greatest 10a + b = 41
10a + b = 41

22. (e); Let 200 numbers are → a₁, a₂, a₅……, a₂₀₀
ATQ,
a1 +a2 +a3 +...+a200
499.5 =
200
⇒ 99900 = a1 + a2 + a3 +. . . +a200
Now,
This form an A.P.
a = a1
22 Adda247 Publications For any detail, mail us at
Publications@adda247.com
Cracker Book for Bank (IBPS | SBI | RRB PO | Clerk) Mains Exams

d=1
n = 200
200
⇒ 99900 = [2a1 + (200– 1)1]
2
a1 = 400
Required sum for 1200 terms whose a1 = 400
1200
= [2 × 400 + (1200– 1)1]
2
⇒ 1199400
1199400
Required Average = = 999.5
1200

23. (c); Let, initial Number of people in the group be ‘n’.


Let 21𝑥 𝑎𝑛𝑑 19𝑥 be ages of Sakshi and Sheetal respectively,
A.T.Q.,
4𝑛2 − 21𝑥 = 4(𝑛 − 1)2 ……..(i)
And 4𝑛2 − 21𝑥 − 19𝑥 = 4(𝑛 − 2)2
4𝑛2 − 40𝑥 = 4(𝑛 − 2)2 ………….(ii)
Solving (i) and (ii),
𝑥 = 4, 𝑛 = 11
Average age of group after Ritu leaves the group
4×112 −21×4−19×4−20 304
= = = 38
11−3 8

Solution (24-25): Let number of people in X, Y and Z group is a 𝑎, 𝑏 and 𝑎


respectively.
Let total money got by group X, Y and Z is h, k and k respectively.
Average amount got by X and Y together
ℎ+𝑘 3500
= …(i)
𝑎+𝑏 3
Average amount got by Y and Z together
𝑘+𝑘
= 1000
𝑎+𝑏
k = 500 (𝑎 + 𝑏)…(ii)
Put value of (𝑎 + 𝑏)from eq (ii) in eq (i)

23 Adda247 Publications For any detail, mail us at


Publications@adda247.com
Cracker Book for Bank (IBPS | SBI | RRB PO | Clerk) Mains Exams
ℎ+𝑘 7
=
𝑘 3
ℎ 7
+1=
𝑘 3
ℎ 4
= …(iii)
𝑘 3
We know
ℎ + 𝑘 + 𝑘 = 50,000
so, h = 20,000
and k = 15000
now ratio (i) becomes
𝑎 + 𝑏 = 30
And we know
𝑎 + 𝑏 + 𝑎 = 50
So, 𝑎 = 20
𝑏 = 10

24. (c); Number of people in Y group = 10

25. (d); Amount got by each person in group Z


15000
= = Rs. 750
20

24 Adda247 Publications For any detail, mail us at


Publications@adda247.com
Cracker Book for Bank (IBPS | SBI | RRB PO | Clerk) Mains Exams

1 Adda247 Publications For any detail, mail us at


Publications@adda247.com
Cracker Book for Bank (IBPS | SBI | RRB PO | Clerk) Mains Exams

Chapter
Percentage and Profit & Loss
5
BEST APPROACH TO SOLVE THE QUESTIONS

1
A percentage is a number or ratio that represents a fraction of 100 i.e. .
100
For faster and easy calculation one should use fractional equivalent of
2 2
percentage. For example, it’s always easier to use instead of 66 %.
3 3

It becomes easy to understand a question when one talks in terms of


percentage, but while calculating, it is always the fractional equivalent that
comes in hand.

While solving the question one must try to stay away from unnecessary
calculation. This happens in many of the questions related to profit and loss.
Many times one starts calculating the absolute values of CP, SP etc. even
when it’s not required.

Let’s take a couple of sample questions.

Example 1 : 'The CP of an item is Rs. 270. After giving a discount of Rs. 27 a


shopkeeper earns a profit of 20%. What will be the profit percentage if he
does not give the discount?’

Sol. Here, instead of finding out SP or MP, one should look at it the other
way round. If discount of Rs. 27 is not given, then Rs. 27 will become further
profit for the shopkeeper.
27 is 10% percent of 270. Profit of 20% is already there, so new profit%
would become 20% + 10% = 30%.

2 Adda247 Publications For any detail, mail us at


Publications@adda247.com
Cracker Book for Bank (IBPS | SBI | RRB PO | Clerk) Mains Exams

Example 2: ‘A man while buying cloth from a wholesaler uses a scale that
measures 10% more than the actual length and while selling the same cloth
he uses a scale that measures 10% less than the actual length. Find his
overall profit or loss in this transaction.’

Sol. Here, one just needs to deal with the percentage the shopkeeper gains.
While buying he gets 110 metre cloth on the cost of 100 metre coth and
while selling he gains the cost of 100metre just by selling 90 metre cloth.
10
Multiplication factor while buying = 1+
100
11
=
10
100−90
Multiplication factor while selling = 1+
90
10
=
9
11 10
The final multiplying factor becomes = ×
10 9
11 2
= =1+
9 9
2
which clearly shows a profit of 22 %.
9

Practice Exercise Based on New Pattern

1. Nishant bought an article at 20% discount on MRP, and claims to sell


it at profit of 10% of MRP. When Nikhil offered him Rs. 500 banknote,
he cheated again by giving him Rs. 125 instead of Rs. 225. Find overall
profit% of Nishant.
(a) 87.5% (b) 37.5% (c) 100%
(d) 62.5% (e) None of these

2. A shopkeeper marked up the price of a mobile phone by 40% of its


cost price, if he increases the discount from 5% to 10%, the profit
would decrease by 1400 Rs. How much profit shopkeeper would earn
if he gives a discount of 20% on the marked price ? Assume that, he
calculates discount only on MRP.
(a) 1800 Rs. (b) 1200 Rs. (c) 2800 Rs.
(d) 2200 Rs. (e) 2400 Rs.
3 Adda247 Publications For any detail, mail us at
Publications@adda247.com
Cracker Book for Bank (IBPS | SBI | RRB PO | Clerk) Mains Exams

3. A manufacturer of cricket balls wants to earn 25% profit on


manufacturing cost after giving a discount of 23 ⅓% on MRP marked
by him. But due to some reasons he lost 25% of balls & he decided to
9
offer discount of 7 % on MRP of remaining balls. Find his overall
13
profit% or loss%.
(a) 8% Profit (b) 12% Profit (c) 12% Loss
(d) 8% Loss (e) 5% Profit

4. Rahul bought a cycle at a discount of 16⅔% on MRP. He earned half


the amount of his CP by renting it for 200 days. After that he resells it
at half of MRP. In this transaction he earned Rs. 200, find MRP of cycle
(in Rs).
(a) 1860 (b) 2490 (c) 2400
(d) 2280 (e) 2310

5. A man purchased two items A & B and invested Rs 50 & Rs 75 on their


repairing respectively. If he earns profit of 10% on A and 12% on B,
overall profit earned by him is is Rs 84. But if he earns 20% on A &
10% on B, overall profit earned by him is 14% of total price of items.
Find initial total purchasing price of both items.
(a) Rs 675 (b) Rs 725 (c) Rs 750
(d) Rs 625 (e) Rs 775

6. A shopkeeper sold two articles, if he marked up second article at 11


1
% above first article’s selling price and gives a discount of 20% on
9
1
that, then a loss of 20% occur on first one and 33 % profit on second
3
article. Find total selling price of both articles, if he made total loss of
Rs.75 on both articles.
(a) 4000 Rs (b) 4500 Rs (c) 5100Rs.
(d) 4800 Rs (e) 5800Rs

4 Adda247 Publications For any detail, mail us at


Publications@adda247.com
Cracker Book for Bank (IBPS | SBI | RRB PO | Clerk) Mains Exams

7. Satish buy two articles i.e. type A at Rs 500 and type B at Rs 1500. He
sold type A article at x% profit and mark up type B article 2x% above
the cost price and gave x% discount at the time of the sale. By this
Satish earn (x -6) % profit. Find the value of ‘x’.
(a) 15% (b) 18% (c) 25%
(d) 20% (e) 30%

Directions (8-10): A article is mark up above cost price such that markup
percent is double of the profit percent. If discount is 12.5%, then profit
1
percent increased by 33 %.
3

8. On selling 20 such article at 12.5% discount, profit is Rs.300. Find the


M.P. of each article.
(a) Rs.60 (b) Rs.160 (c) Rs.80
(d) Rs.240 (e) Rs.72

9. If shopkeeper cheat his customer by giving 20% less quantity and


reducing value of discount percentage by 20% then find the new
profit percent.
(a) 60% (b) 75% (c) 62.5%
(d) 80% (e) 70%

10. Initially shopkeeper have 20 articles. Out of 20, 7 articles damaged


and remains unsold. Marked Price should he labeled by how much
percent more than cost price so that his overall profit does not change
neither his discount percentage.
(a) 156% (b) 146% (c) 136%
(d) 120% (e) 125%

5 Adda247 Publications For any detail, mail us at


Publications@adda247.com
Cracker Book for Bank (IBPS | SBI | RRB PO | Clerk) Mains Exams

11. Thirty percent of total employees of Bankersadda are females, and


sixty percent of female employees earn more than Rs. 17000 as
monthly salary. If fifty five percent of total employees of Bankersadda
earn more than Rs. 17000 as monthly salary. What fraction of total
male employees of Bankersadda earn Rs. 17000 or less as monthly
salary ?
13 11 51
(a) (b) (c)
20 20 100
33 69
(d) (e)
100 100

12. A shopkeeper raised the marked price of an article by 60% and allow
1 2
three successive discounts of 12 %, 14 % and 20% on new MP and
2 7
make a profit of 20%. If shopkeeper would allow only two discounts
1
of 12 % and 20% on new MP, he made a profit of Rs 800. Find the
2
cost price of article?
(a) 1800 Rs. (b) 2000 Rs. (c) 1600 Rs.
(d) 2400 Rs. (e) 2800 Rs.

13. Bankersadda started its publication with the 1500 DI books. Printing
cost per book is 125 Rs, packing cost per book is 15 Rs and shipping
cost per book is 40 Rs. if 50% of books sold on half of total cost price,
then find on how much percent above should be remaining book sold
to gets 20% profit on total outlay?
(a) 50% (b)60% (c) 80%
(d) 90% (e)70%

14. Adda247 publication sold a puzzle book in Rs. 475 and made profit of
25% on CP and sold a DI book in Rs. 575 and made a profit of 15% on
CP. If publication sold puzzle book in Rs. 360, then find what price
should DI book will be sold to made a profit of 30% on both the books
?
(a) Rs.784 (b) Rs.874 (c) Rs.847
(d) Rs.748 (e) Rs.478

6 Adda247 Publications For any detail, mail us at


Publications@adda247.com
Cracker Book for Bank (IBPS | SBI | RRB PO | Clerk) Mains Exams

Directions (15-16): Abhishek bought some chairs and tables from a


shopkeeper. The marked price of a chair and a table were in the ratio 5 : 8.
The shopkeeper gave discounts of 20% and 25% on the chair & the table
respectively. The ratio of number of chairs and tables bought by Abhishek
is 6 : 5.

15. If Abhishek sells each chair and table bought by him at discounts of
25% and 20% respectively after marking up the prices of both by 50%
and gives one table free for every four chairs bought by a customer
2
and only rd of the total chairs are sold in bunch of four chairs, then
3
what is the net profit /loss % made by Abhishek after selling all of the
items which he bought from the shopkeeper?
2 1 1
(a) 6 % (b) 3 % (c) 2 %
3 3 2
1
(d) 4 % (e) 5 %
4

16. If the marked price of a table set by the shopkeeper was Rs.300 more
than that of a chair and the total expenditure made by Abhishek in
purchasing the chairs and table from the shopkeeper was Rs.108000,
then how many chairs were purchased by Abhishek?
(a) 150 (b) 60 (c) 120
(d) 90 (e) 80

Directions (17-18): A car ‘X’ is manufactured in a company ‘A’. Cost of raw


material on a car is 25% of total cost and cost of labor is 20% of rest. 30% of
total cost occur due to four engineers working there and remaining cost is
due to machines and taxes which is in the ratio 7 : 3. Company sold the car
‘X’ at 28% profit.

17. If cost of raw material is increased by 30% and all their 4 engineers
leave the company, so they hire 2 new engineers by giving 50% more
salary as they give to previous engineer each. Find the % change in
selling price so overall profit must remain same.
(a) 0% (b) 100% (c) 2%
(d) 50% (e) 3%

7 Adda247 Publications For any detail, mail us at


Publications@adda247.com
Cracker Book for Bank (IBPS | SBI | RRB PO | Clerk) Mains Exams

18. Company A, started making car ‘Y’ in which same raw material used,
labor cost is increased by 20%. If all other cost is same then find the
ratio of selling price of car ‘X’ to car ‘Y’, given that car ‘Y’ sold at 20%
profit.
(a) 331 : 309 (b) 320 : 314 (c) 313 : 309
(d) 320 : 309 (e) 309 : 320

19. Marked price of two articles A and B are in the ratio of 7 : 9, at the
time of selling shopkeeper gives discount of d% on article A and (d +
5)% on article B and made a profit of 25% on each article, if cost price
of article A and B are in ratio 112 : 135. Then find percent of discount
given by shopkeeper on both articles?
(a) 12.5%, 17.5% (b) 25%, 30% (c) 10%, 15%
(d) 15 %, 20% (e) 20%, 25%

20. Neeraj purchased two mobile from a shop. He sold first mobile at the
price of Rs. 18750 and second mobile at the price of Rs. 14250. If the
profit percent on first mobile is five times of the loss percent on
second mobile, then find the overall profit made by Neeraj after
selling both the mobile phones. (C.P of both mobile is same)?
(a) 15% (b) 12% (c) 14%
(d) 10% (e) 18%

21. There are three societies A, B and C. Ratio of total population of A, B


and C is 2 : 3 : 5. In society A, 35% of total population is female. In
2
society B, number of females is 66 % less than number of males of
3
society B. In society C, ratio of male to female is 27 : 23. Find total
number of females is what percent less than total number of males in
all societies together ?
1
(a) 33 % (b) 40% (c) 60%
3
2
(d) 66 % (e) 75%
3

8 Adda247 Publications For any detail, mail us at


Publications@adda247.com
Cracker Book for Bank (IBPS | SBI | RRB PO | Clerk) Mains Exams

22. The marked price of a bike in two stores A and B is same.In store A
bike is available at 30% discount and in store B same bike is available
at two successive discounts of d% and 8%. A man bought bike from
store A at Rs. 49000. If man would have paid Rs. 7672 more, he could
have bought the same bike from store B. Find the discount ‘d’ allowed
by store B on bike ?
(a) 8% (b) 10% (c) 12%
(d) 16% (e) 15%

23. A shopkeeper gives a discount of 19% on marked price of article A


1
and selling price of article B is 11 % more than selling price at article
9
A. If shopkeeper made 20% profit on article B and selling price of
article A was Rs. 840 more than cost price of article B, then find the
cost price of article A. Shopkeeper sold article A at profit of 25%?
(a) 9070 Rs. (b) 9060 Rs. (c) 9040 Rs.
(d) 9020 Rs. (e) 9072 Rs.

Directions (24-25): A sale conducted on Samsung mobiles store where


Cost price of three types Samsung mobiles A, B and C in the ratio of 5 : 7 : 9.
Store owner made mark price of mobile A and C, 30% above cost price and
mobile B, 40% above cost price. Discounts allowed by store on mobiles A, B
5 3 1
and C are 15 %, 21 % 𝑎𝑛𝑑 11 % respectively.
13 7 9

24. A man purchased fourteen type A mobile, twenty type B mobile and
fifteen type C mobile in sale. If store owner made a total profit of Rs.
17500 on all type A mobile, purchased by man than find total profit
made by store owner on type B and type C mobiles, which was
purchased by man?
(a) 82500 Rs. (b) 84500 Rs. (c) 78500 Rs.
(d) 87500 Rs. (e) 88500 Rs.

9 Adda247 Publications For any detail, mail us at


Publications@adda247.com
Cracker Book for Bank (IBPS | SBI | RRB PO | Clerk) Mains Exams

25. Store decided to give no discount on purchase of any type of mobile it


will be given two mobiles free on purchase of six type A mobiles, one
mobile free on purchase of five type B mobiles and three mobiles free
on purchase of ten type C mobiles. A retailer come to purchase mobile
and take away48 type A mobiles, 36 type B mobiles and 39 type C
mobiles. Find overall loss percentage of store owner in this
transaction?(2 marks)
28 28 76
(a) 10 % (b) 8 % (c)4 %
281 281 281
28 28
(d) 14 % (e) 9 %
281 281

26. Ratio between marked price of article A to article B is 4 : 5.


Shopkeeper allowed d% discount on article ‘A’ and (d + 18)%
discount on article ‘B’, so selling price of both articles become equal. If
shopkeeper made a profit of 20% on article A and 25% on article B
and profit made on article B is Rs. 384 more than that of article A, then
find the cost price of article ‘A’ and article ‘B’ respectively?
(a) 9000 Rs. 8400 Rs (b) 9600 Rs. 9216 Rs.
(c) 9800 Rs. 9012 Rs. (d) 9600 Rs. 8488 Rs.
(e) 9200 Rs. 9216 Rs.

27. Aman started a shoes manufacturing company, manufacturing cost of


each shoes is Rs. 500 , Raw material cost is Rs 300 and transport cost
of Rs 100 If company manufactured 150 Shoes in first order and sold
50% of shoes on half of its cost price. Then find at what price of
should the company sold the remaining shoes to gets 50% profit on its
total cost?
(a) 100% (b) 150% (c) 75%
(d) 125% (e) 120%

28. The ratio between marked price of a same watch in two shops P and Q
is 7 : 5 . In shop P watch available at two successive discounts of 10%
& 25% , while in shop Q watch available at two successive discounts of
d% and 10%. If selling price of watch at shop Q is less than that of
44
selling price of watch at shop P by 𝑡ℎ of the selling price of watch at
105
shop P, then find the value of ‘d’ ?
(a) 39% (b) 29% (c) 31%
(d) 35% (e) 37%
10 Adda247 Publications For any detail, mail us at
Publications@adda247.com
Cracker Book for Bank (IBPS | SBI | RRB PO | Clerk) Mains Exams

29. Cost price of a Samsung mobile is 20% more than cost price of a Mi
mobile. Shopkeeper marked up Samsung mobile at 25% above its cost
price and Mi mobile at 40% above cost price. If shopkeeper allowed
discount of 20% on Mi mobile and 25% on Samsung mobile and
difference between selling price of Samsung mobile and Mi mobile
was 160 Rs. then find sum of marked price of one Samsung mobile
and one Mi mobile?
(a) 92840 Rs (b) 92260 Rs (c) 98460 Rs
(d) 92800 Rs (e) 98530 Rs

30. In a city 60 percent are male and remaining are females. Out of total
males, 20% of male are government employees. Out of remaining
males, ratio between private employees to unemployed male is 5 :7.
Out of total females, 40% of female are in government jobs and out of
remaining females 40% of female work for private firms and
remaining are unemployed. Find the total number of male and female,
who are government employees, if difference between male and
female who are unemployed is 3400?
(a) 6000 (b) 5000 (c) 7000
(d) 4000 (e) 3000

31. A person ‘A’, invested 37½% of his savings in scheme XYZ, while B
invested 62½% of his savings in the same scheme. If B invested 50%
of his remaining savings in scheme PQR, and A thought to invest
double of the amount invested by B in PQR scheme then find what
percent of A’s remaining savings after investment in XYZ, A should
invest in PQR. Given that amount invested by B in XYZ is twice the
amount invested by A in same scheme.
(a) 80% (b) 68% (c) 64%
(d) 72% (e) 60%

11 Adda247 Publications For any detail, mail us at


Publications@adda247.com
Cracker Book for Bank (IBPS | SBI | RRB PO | Clerk) Mains Exams

32. A man bought few kg’s of sugar of 3 variety (X, Y, Z) in the ratio of
their quantity 2 : 3 : 5 respectively and started to sell them at profit of
20%, loss of 10% and profit of 32% respectively. He managed to sell
only 80% of total sugar and rest was return at the same price as he
bought. The price of these sugar is Rs. 30/kg, Rs. 40/kg and Rs. 50/kg
respectively. Calculate his total profit amount, if total sugar bought
was 50 kg and he sold all the sugar of X and Y types.
(a) 320 (b) 300 (c) 0
(d) 180 (e) None of these

33. In an election survey, 83 ⅓% of total voters took part in survey, 50%


then claims to votes for candidate A, 10% are uncertain & rest says to
vote for B. If all of them voted according to their commitments on the
day of election and those who were not part of survey, voted to A and
B in ratio of 2 : 1. All the people who are uncertain in survey, voted to
A : B in the ratio of 1 : 4 in election. If A won by 640 votes then find
total number of votes in election.
(a) 6000 (b) 5000 (c) 9000
(d) 7200 (e) 6750

2
34. In a village 60% are males in which 6 % males are above 60 years, in
3
2
remaining 14 % males are below 18 years, out of remaining
7
1
62 % are above 18 years but below 50 years and remaining males are
2
above 50 years but below 60 years. If difference between males above
60 years and males above 18 years but below 50 years are 7800, then
find total female population of village?
(a) 16000 (b) 18000 (c) 20000
(d) 12000 (e) 24000

12 Adda247 Publications For any detail, mail us at


Publications@adda247.com
Cracker Book for Bank (IBPS | SBI | RRB PO | Clerk) Mains Exams

35. Divyaraj purchased jeans and shirt from a seller. Marked price of
2
jeans and shirt are in the ratio of 9 : 7 and seller offered 14 %
7
1
discount on shirt and 11 % on jeans and number of jeans and shirt
9
purchased by Divyaraj are in the ratio of 5 : 8. If Divyaraj marked up
jeans and shirt 50% above their cost price and offered 25% discount
1
on jeans and 11 % on shirt, Find overall profit of Divyaraj in this
9
transaction.
19 19 19
(a) 22 % (b) 21 % (c) 19 %
22 22 22
19 19
(d) 23 % (e) 25 %
22 22

36. In 2016 a store sold some mobile phone at certain price. In 2017 store
increase selling of mobile phone by reducing the price of mobile
phone by 25%. The total revenue generated by selling mobile phone
in 2017 is increased by 5% compared to previous year. The total
revenue generated by store in 2017 is 94.5 lakh and number of mobile
phones sold by store in the year 2017 is 90 more than that in 2016.
Find initial cost of one mobile.
(a) 32000 Rs. (b) 40000 Rs. (c) 36000 Rs.
(d) 24000 Rs. (e) 20000 Rs.

37. Veer bought some Shirts & some Jeans from a store. The marked price
of a shirt and a jeans is in the ratio 5 : 7 and store allows discounts of
20% and 25% on the shirt & the jeans respectively. The ratio between
number of shirt and jeans bought by Veer is 9 : 8. Veer marking up the
prices of both by 50% on the price at which he bought and sells each
shirt and jeans bought by him at discounts of 25% and 20%
respectively. If Veer gives one 1 shirt free for every 4 jeans bought by
a customer, then what is the net profit made by Veer after selling all of
the items which he bought from the store?
(a) 2% (b) 7% (c) 6%
(d) 5% (e) 8%

13 Adda247 Publications For any detail, mail us at


Publications@adda247.com
Cracker Book for Bank (IBPS | SBI | RRB PO | Clerk) Mains Exams

38. A shopkeeper has two types Wheat i.e Type A & Type B and cost price
per kg of Type A wheat is 10 times of cost price per kg of type B
wheat. Shopkeeper cheated a costumer by saying that he will give him
200 kg of Type A wheat but shopkeeper gives 200 kg of mixture of
Type A & Type B wheat on 10% above the cost price per kg of Type A
7
wheat. If shopkeeper made a total profit of 71 % in this transaction,
8
then find ratio between quantity of Type B wheat to Type A wheat in
sold mixture?
(a) 2 : 5 (b) 2 : 3 (c) 1 : 3
(d) 2 : 7 (e) 1: 5
1
39. In a store marked price of a shirt is 33 % less than marked price of a
3
jeans. There are two discounts schemes A & B. In Scheme A, if
someone buy one jean and one shirt together, then store gives overall
discount of 35%, while in scheme B if someone buy one shirt and one
jeans separately, store give 20% discount on shirt and 25% on jeans.
If difference between selling price of one jeans and one shirt in
scheme A and B is Rs. 384 then find cost price of one jeans & one shirt,
1
given that store made of 33 % on shirt and 25% on jeans, when it
3
sold one jeans and one shirt in scheme B ?
(a) 720 Rs, 1150 Rs (b) 768 Rs. 1152 Rs.
(c) 786 Rs. 1168 Rs. (d) 796 Rs. 1144 Rs.
(e) 790 Rs. 1108 Rs.

40. The Hero company manufactured cycles, 40% of the total


manufactured cost is on raw material, 20% on labour charges, 20% on
fixed charges and the rest on transportation. The cycle is sold at a
profit of 25%. The price of the raw material increased by 15% and the
labour charges increased by 20% and the cost on the transportation
increased by 50% while the fixed costs remained unchanged. If the
7
manufacturer wants a 13 % profit, then by what percentage should
11
company reduced its expenditure on raw materials (at the increased
price), the selling price remaining the same ?
17 17 17
(a) 19 % (b) 21 % (c) 19 %
23 23 23
17 17
(d) 23 % (e) 27 %
23 23

14 Adda247 Publications For any detail, mail us at


Publications@adda247.com
Cracker Book for Bank (IBPS | SBI | RRB PO | Clerk) Mains Exams

Solutions

1. (a); Nikhil gave Rs. 500 note, & Nishant have to actually pay him Rs.
225. Hence Nishant sold it at Rs. 500 – 225 = 275 to Nikhil.
Now, Let MRP is 100x.
If he will get 10% profit, then SP will becomes 110x.
110x = 275
x = 5.5
∴ MRP is Rs. 250
80
And CP for Nishant is × 25 = Rs. 200.
100

He got Rs. (500 – 125)


= Rs. 375 from Nikhil.
375–200
Hence Profit % = ( ) × 100
200
7
= × 100 = 87.5%
8

2. (e);
CP of Mobile M.P. mobile S.P. of mobile
100% 140% 133%(1st discount) (133 – 126)% = 1400
126%(2nd discount)
7% = 1400
1400
C. P. = × 100
7
= 20000 Rs.
112
S. P. after 20% discount = 20000 ×
100
= 22400
Profit = 2400 Rs.

15 Adda247 Publications For any detail, mail us at


Publications@adda247.com
Cracker Book for Bank (IBPS | SBI | RRB PO | Clerk) Mains Exams

3. (b); Let he manufacture 𝑛 balls and manufacturing cost of each ball


is 𝑦
And 𝑥 is marked price.
Now, ATQ
7 5
𝑥 [1– ] = 𝑦 ×
30 4
23 5
⇒𝑥× =𝑦×
30 4
5 30 75
⇒𝑥 =𝑦× × =𝑦× …(i)
4 23 46
3 1 12
After accident he sells 𝑛 balls on a price of 𝑥 (1– ) = 𝑥 ×
4 13 13
3 3 12
Total SP of 𝑛 balls = 𝑛. 𝑥.
4 4 13
3 75 12
= ( × 𝑛) (𝑦 × )( )
4 46 13
675
= 𝑛𝑦 ×
598
675
𝑛𝑦×598−𝑛𝑦
Hence profit % = × 100
𝑛𝑦
77
= × 100 ≅ 12% profit
598

4. (c); Let the MRP of cycle is 6𝑥.


50
∴ discount is % of 6x = x
3
∴ Rahul bought this cycle for 5x
He earns 2.5x by renting it and resells it at 3x.
ATQ
3x + 2.5x – 5x = 200
0.5x = 200 ⇒ x = 400
∴ MRP = 6x = 6 × 400 = Rs. 2400
16 Adda247 Publications For any detail, mail us at
Publications@adda247.com
Cracker Book for Bank (IBPS | SBI | RRB PO | Clerk) Mains Exams

5. (d); Let the purchasing price of items A is B be Rs x – 50, Rs y – 75


respectively.
When he applied Rs 50 & Rs 75 on them, their actual CP
becomes x & y
ATQ
10 12
x+ y = 84 …(i)
100 100
Also,
20 10 14
x+ y= (x + y)
100 100 100
⇒ 20x + 10y = 14x + 14y
⇒ 6x = 4y
2
x= y
3
Put this value in equation (i)
10 (2y) 12
+ y = 84
100 3 100
20y+36y
= 84
300
84×300
⇒y= = Rs 450
56
2
x = 450 × = Rs 300
3
Initial purchasing price is
= (300 + 450) – (75 + 50)
= Rs 625

6. (c); Let marked price of second article be


10x Rs.
Selling price of first article be 9x Rs.
Selling price of second article
8
= 10x × = 8x Rs.
10

C.P. of fisrt article


9x
= × 5 = 11.25x Rs.
4

17 Adda247 Publications For any detail, mail us at


Publications@adda247.com
Cracker Book for Bank (IBPS | SBI | RRB PO | Clerk) Mains Exams
3
C.P. of second article = 8𝑥 × = 6x Rs.
4

Total C.P. of both articles = 11.25x + 6x


= 17.25x Rs.
Total S.P. of both articles
= 9x + 8x = 17x Rs.
ATQ—
17.25x – 17x = 75
75
x= = 300
0.25

selling price of both articles = 17 × 300


= 5100 Rs.

7. (d); ATQ
𝑥 2𝑥 𝑥 𝑥−6
500 [ 1 + ] + 1500 [1 + ] [1 − ] = 2000 [1 + ]
100 100 100 100
2𝑥 𝑥 2𝑥 2 𝑥−6
500 + 5𝑥 + 1500 [1 + − − ] = 2000 [1 + ]
100 100 100×100 100
3𝑥 2
500 + 5𝑥 + 1500 + 15𝑥 − = 2000 + 20(𝑥 − 6)
10
3𝑥 2
2000 + 20𝑥 − = 2000 + 20𝑥 − 120
10
3𝑥 2
= 120
10

x² = 400
⇒ x =20%

18 Adda247 Publications For any detail, mail us at


Publications@adda247.com
Cracker Book for Bank (IBPS | SBI | RRB PO | Clerk) Mains Exams

Solutions (8-10): Let cost price = 100


Profit percent = 𝑥%
So, selling price ⇒ 100 + 𝑥
M.P. ⇒ 100 + 2𝑥
Now if discount is 12.5%
1 4 4
Then profit percentage increases by 33 % ⇒Profit percent = 𝑥 × = 𝑥
3 3 3
ATQ,
4 7
100 + 𝑥 = (100 + 2𝑥)
3 8
𝑥 = 30
So, if cost price = 100
Selling price = 130
M.P. = 160
Profit = 30
30
Discount % = × 100 = 18.75%
160

8. (c); Profit on one articles = 30


Profit on 20 articles = 30 × 20 ⇒ 600
600 → 300
1 → 0.5
M.P. → 160 → 160 × 0.5 ⇒ Rs.80

9. (e); Let total quantity ⇒ 1000


He gives → 800 → for → 100
Cost price of → 800 → 80
30
Now initial discount = × 100 = 18.75%
160
18.75×4
Reduced discount = = 15%
5
160×85
S.P. = = 136
100
(136–80)
Profit = × 100 = 70%
80

19 Adda247 Publications For any detail, mail us at


Publications@adda247.com
Cracker Book for Bank (IBPS | SBI | RRB PO | Clerk) Mains Exams

10. (b); Let cost price of 20 articles → 20 × 100


= 2000
Actual profit on 20 articles → 20 × 30
= 600
S.P. → 2600
S.P. of each undamaged article
2600
⇒ = 200
20–7
30
Initial discount = × 100 = 18.75%
160

So M.P. of each article should be


200
= × 100 ≈ 246
81.25

Approximately markup% = 146%

11. (d); Let total employee in Bankersadda = 100


Females employee = 30
Male employee = 70
Employee who earned more than Rs. 17000 as monthly salary
= 55
Employee who earned less than Rs. 17000 as monthly salary
= 45
Total female employee earned less than
Rs. 17000 as monthly salary
40
= 30 × = 12
100

Total male employee earned Rs. 17000 or less as monthly salary


= 45-12 = 33
33
Required fraction =
100

20 Adda247 Publications For any detail, mail us at


Publications@adda247.com
Cracker Book for Bank (IBPS | SBI | RRB PO | Clerk) Mains Exams

12. (b); Let original marked price be Rs 100x.


Then, New marked price of article
= Rs 160x.
Selling price of article
7 6 4
= 160𝑥 × × ×
8 7 5

= 96x Rs.
5
C.P. of article = 96x ×
6

= 80x Rs.
2nd selling price
7 4
= 160𝑥 × × = 112x Rs.
8 5

Given
112x – 80x = 800
32x = 800
x = 25
cost price = 80 × 25 = 2000 Rs.

13. (d); Total CP of one DI book = 125 + 15 + 40


= 180 Rs.
C.P. of 1500 books = 1500 × 180
750×180
SP of 750 book =
2

Let required profit percent = x%


750×180 100+x
∴ +( ) × (750 × 180)
2 100
120
= × 180 × 1500
100

21 Adda247 Publications For any detail, mail us at


Publications@adda247.com
Cracker Book for Bank (IBPS | SBI | RRB PO | Clerk) Mains Exams

67500 + 135000 + 1350x = 324000


1350x = 324000 – 202500
121500
x= = 90%
1350
475
14. (a); CP of puzzle book = × 100
125

= 380 Rs.
575
CP of DI book = × 100
115

= 500 Rs.
For 30% profit on both
130
= (380 + 500) × = 1144
100

DI book should be sold


= 1144 – 360 = 784 Rs.

15. (b); Let the MP of a chair and a table be Rs.5x and Rs.8x respectively.
And, the number of chairs and tables bought be 6y and 5y
respectively.
CP of a chair for Abhishek = (100 – 20)% of 5x = Rs.4x
CP of a table for Abhishek = (100 – 25)% of 8x = Rs.6x
Total CP for Abhishek = 4x × 6y + 6x × 5y = 24xy + 30xy = 54xy
SP of a chair for Abhishek = (100 – 25)% of (100 + 50)% of 4x
= 4.5x
SP of a table for Abhishek = (100 – 20)% of (100 + 50)% of 6x
= 7.2x
Number of chairs sold in bunch of four by Abhishek

22 Adda247 Publications For any detail, mail us at


Publications@adda247.com
Cracker Book for Bank (IBPS | SBI | RRB PO | Clerk) Mains Exams
2
= rd of 6y = 4y
3
So, number of table sold for free by Abhishek
1
= th of 4y = y
4
Total SP for Abhishek = 4.5x × 6y + 7.2x × (5y – y) = 27xy +
28.8xy = 55.8xy
55.8xy – 54xy
Profit % = × 100
54xy
1.8xy 1
= × 100 = 3 %
54xy 3

16. (c); According to the question,


MP of a table = 300 + MP of a chair
⟹ 8x = 300 + 5x ⟹ x = 100
Total CP for Abhishek = 108000
⟹ 54xy = 108000
⟹ 54 × 100 × y = 108000 ⟹ y = 20
Number of chairs purchased by Abhishek
= 6y = 120

Solutions (17-18): Let total cost of car X = 100𝑥


Cost of raw material = 25𝑥
(100–25𝑥)×20
Cost of labor = = 15𝑥
100

Cost of engineers = 30𝑥


Cost of machine = 21𝑥
Cost of taxes = 9𝑥
Profit = 28𝑥
So, proposed selling price = 128𝑥
23 Adda247 Publications For any detail, mail us at
Publications@adda247.com
Cracker Book for Bank (IBPS | SBI | RRB PO | Clerk) Mains Exams
25𝑥×13
17. (a); New cost of raw material = = 32.5𝑥
100
30𝑥
Now cost of each engineer = = 7.5𝑥
4
150
Cost on new engineer = 2 × 7.5𝑥 ×
100

= 22.5
So new CP = 100𝑥 + 7.5𝑥 – 7.5𝑥 = 100𝑥
So, % change in S.P. = 0%

15𝑥×120
18. (d); Cost on labor for car Y = = 18𝑥
100

Cost price of car Y = 103𝑥


128𝑥 320
Required ratio = =
103𝑥×120 309

19. (e); Let cost price of article A Rs. 112x and cost price of article B Rs.
135x
ATQ,
112𝑥
Selling price of article A = ×5
4

= 140x Rs.
135𝑥
Selling price of article B = ×5
4

= 168.75x Rs.
Let mark price of article A is 7y
And article B is 9y
100−𝑑
140x = 7y ×( ) ________(I)
100
100–(𝑑+5)
168.75x = 9y ×( ) ________(II)
100

24 Adda247 Publications For any detail, mail us at


Publications@adda247.com
Cracker Book for Bank (IBPS | SBI | RRB PO | Clerk) Mains Exams

From (I) and (II) _________


100–𝑑
140𝑥 7𝑦×( 100 )
= 95–𝑑
168.75𝑥 9𝑦×( 100 )

140×9 100–𝑑
=
168.75×7 95–𝑑
16 100–𝑑
=
15 95–𝑑

16d – 1520 = 15d – 1500


First discount d = 20%
Second discount = (20 + 5) = 25%

20. (d); Let cost price of each mobile = 100x Rs.


Percent profit on first mobile
18750−100𝑥 18750−100𝑥
= [( )] × 100 =
100𝑥 𝑥

Loss percent on second mobile


100𝑥−14250 100𝑋−14250
= [( )]× 100 =
100𝑥 𝑥

ATQ –
18750−100𝑥 100𝑋−14250
=5×
𝑥 𝑥

18750 − 100𝑥 = 500𝑥 − 71250


600x = 90000 ⇒ x = 150 Rs.
Total cost price of both mobile = 2 × 15000 = 30000 Rs.
Total selling price of both mobile = 18750 + 14250 = 33000 Rs.
33000−30000
Profit percentage = ×100 = 10%
30000

25 Adda247 Publications For any detail, mail us at


Publications@adda247.com
Cracker Book for Bank (IBPS | SBI | RRB PO | Clerk) Mains Exams

21. (b); Let population of society A, B and C be 200x, 300x and 500x
respectively
35
Female in society A = × 200x = 70x
100

Male in society A = 200x – 70x = 130x


In society B Let ‘a’ = Number of males
ATQ,
1 4𝑎
a + a = 300x ⇒ = 300𝑥
3 3

⇒ a = 225x = Number of males


Number of females = 300x – 225x = 75x
In society C,
23
Number of females = × 500x = 230x
50
27
Number of males = × 500x = 270x
50

Total number of female = 70x + 75x + 230x


= 375x
Total number of male = 130x + 225x + 270x = 625x
625x –375x
Required% = × 100
625x
250
= × 100 = 40%
625

22. (c); Let marked price of bike = 100x Rs.


Selling price for store A
(100 –30)
= 100x × = 70x
100

Given, 70x = 49000 ⇒ x = 700


marked price of bike = 70000 Rs.
26 Adda247 Publications For any detail, mail us at
Publications@adda247.com
Cracker Book for Bank (IBPS | SBI | RRB PO | Clerk) Mains Exams

ATQ—
Selling price of bike in store B=
(100−𝑑) (100−8)
Marked price of bike× ×
100 10

= 𝑠𝑒𝑙𝑙𝑖𝑛𝑔 𝑝𝑟𝑖𝑐𝑒 𝑜𝑓 𝑏𝑖𝑘𝑒 𝑖𝑛 𝑠𝑡𝑜𝑟𝑒 𝐴+7672


(100 –d) (100 –8)
70000 × ×
100 100

= (49000 + 7672)
644d = 64400 – 56672
7728
d= ⇒ d = 12 %
644

23. (e); Lets MP of article A = 100x Rs.


Selling price of article A
(100−19)
= 100x × = 81x Rs.
100
10
Selling price of article B = 81𝑥 ×
9

= 90x Rs.
90𝑥
Cost price of article B = × 100 = 75x
120
81𝑥
Cost price of article A = × 100
125

= 64.8x Rs.
Given,
81x – 75x = 840
6x = 840 ⇒ x = 140 Rs.
Cost price of article A = 64.8 × 140
= 9072 Rs.

27 Adda247 Publications For any detail, mail us at


Publications@adda247.com
Cracker Book for Bank (IBPS | SBI | RRB PO | Clerk) Mains Exams

Solutions (24-25): Lets cost price of A, B and C type of mobile be Rs. 5x, Rs.
7x and Rs. 9x respectively
130
M.P. of type A mobile = 5𝑥 × = 𝑅𝑠. 6.5𝑥
100
140
M.P. of type B mobile = 7𝑥 × = 𝑅𝑠. 9.8𝑥
100
130
M.P. of type C mobile = 9𝑥 × = 𝑅𝑠. 11.7𝑥
100

S.P. of type A mobile


200 1 11
= 6.5𝑥 × (100– )× = 6.5𝑥 × = 𝑅𝑠. 5.5𝑥
13 100 13

S.P. of type B mobile


150 1
= 11.7𝑥 × (100– )× = 𝑅𝑠. 7.7𝑥
7 100
8
S.P. of type C mobile = 11.7 × = 𝑅𝑠. 10.4𝑥
9

24. (d); Total cost price of Fourteen type A mobile


= 14 × 5x = Rs. 70x
Total selling price of fourteen type A mobile = 14 × 5.5x = 77x
Given total profit = Rs. 17500
7x = 17500 ⇒ x = Rs. 2500
Total profit made by store owner on type B and type C mobile
= (7.7 × 20 – 7× 20) × 2500 + (10.4 × 15 – 9 × 15) × 2500
= 14 × 2500 + 21 × 2500
= 35000 + 52500 = Rs. 87500

28 Adda247 Publications For any detail, mail us at


Publications@adda247.com
Cracker Book for Bank (IBPS | SBI | RRB PO | Clerk) Mains Exams

25. (c); In the transaction —


Given,
In 48 type A mobile, 12 mobile are free.
In 36 type B mobile, 6 mobile are free.
In 39 type C mobile, 9 mobile are free.
Total profit of store owner on selling of 48 type A mobile
= 48 × 5x – 36 × 6.5x
= 240x – 234x = 6x
Total loss of store owner on selling of 36 type mobile
= 36 × 7x – 30 × 9.8x
= 252x – 294x = 42x
Total loss of store owner on selling of 39 type C mobile
= 9x × 39 – 11.7x × 30 = 351x – 351x = 0
Total loss of store owner in this transaction = 42x- 6x = 36x
Total cost price = 48 × 5x + 36 × 7x + 39 × 9x
= 240x + 252x + 351x = 843x
36𝑥 76
Required% = × 100 = 4 %
843𝑥 281

26. (b); Let marked price of article A and B be 400x and 500x
respectively
ATQ—
(100–d) (100–d−18)
400x × = 500x ×
100 100

400 – 4d = 410 – 5d ⇒ d = 10%


90
400x×100
Cost price of article A = × 100
120
29 Adda247 Publications For any detail, mail us at
Publications@adda247.com
Cracker Book for Bank (IBPS | SBI | RRB PO | Clerk) Mains Exams

= 300x Rs.
(100–28)
500x× 100
Cost price of article B = × 100
125

= 288x Rs.
ATQ—
72 90
(500x × – 288x) – (400x × – 300x) = 384
100 100

72x – 60x = 384 ⇒ x = 32


Cost price of article A = 32 × 300 = Rs.9600
Cost price of article B = 32 × 288 = Rs.9216

27. (b); Total CP of one shoes = (500 + 300 + 100)


= 900
CP of 150 shoes = 900 × 150
900×75
CP of 75 shoes =
2

Let’s required percentage =x


900×75 100+𝑥
∴ +( ) × (900 × 75)
2 100
150
= × 900 × 150
100

= 33750 + 67500 + 675x = 202500


= 675x = 202500 – 101250
675x = 101250
101250
𝑥= ⇒ x = 150%
675

30 Adda247 Publications For any detail, mail us at


Publications@adda247.com
Cracker Book for Bank (IBPS | SBI | RRB PO | Clerk) Mains Exams

28. (a); Let marked price of watch at shop P & Q be Rs. 140x and Rs.
100x respectively
Selling price of watch at shop P
90 75
= 140𝑥 × × = 94.5𝑥 𝑅𝑠.
100 100

Selling price of watch at shop Q


(100−𝑑) 90
= 100x × ×
100 100

ATQ –
(100−𝑑) 90 44
94.5x - 100x × × = 94.5𝑥 ×
100 100 105

9450 – 9000 + 90d = 3960


90d = 3510
3510
d= ⇒ d = 39%
90

29. (d); Let cost price of one Mi mobile be Rs. 100x and cost price of one
Samsung mobile be Rs.120x
Marked price of Samsung mobile
125
= 120x × = 150x Rs.
100

Marked price of Mi mobile


140
= 100x × = 140x Rs.
100

Selling price of Samsung mobile


75
= 150x × = 112.5 x Rs.
100
80
Selling price of Mi mobile = 140x ×
100

= 112x Rs.

31 Adda247 Publications For any detail, mail us at


Publications@adda247.com
Cracker Book for Bank (IBPS | SBI | RRB PO | Clerk) Mains Exams

ATQ–
112.5 x – 112x = 160 ⇒ x = 320 Rs.
Marked price of one Samsung & one Mi mobile
= (150 × 320) + ( 140 × 320)
= 48000 + 44800 = 92800 Rs.
30. (c);

Male 60% Female 40%

Government Employees Government employees


20 40
= 60 × = 12% = 40 × = 16%
100 100

Private employees Private employees


5 40
= 48 × = 20% = 24 × = 9.6%
12 100

Unemployed = 48 – 20 =28% Unemployed = 24 – 9.6


= 14.4%
Given —
28% – 14.4% = 3400 ⇒ 13.6% = 3400
Government (male + female) employees
= 12% + 16% = 28%
3400
Government (male + female) employees = × 28 = 7000
13.6

31. (d); Let the savings of A & B be Rs 8 x and Rs 8y respectively. Then


37.5
amount invested by A in XYZ scheme = × 8𝑥 = 3x
100
62.5
Amount invested by B in XYZ = 8y = 5y
100

ATQ,

32 Adda247 Publications For any detail, mail us at


Publications@adda247.com
Cracker Book for Bank (IBPS | SBI | RRB PO | Clerk) Mains Exams

⇒ 6x = 5y
5 6
x = y or y = x
6 5
1
Now B invested 50% of (8y – 5y) = × 3y
2

= 1.5y in scheme PQR.


A wants to invest 2× 1.5y = 3y
6 18 𝑥
=3× x=
5 5

His remaining savings = 8x – 3x = 5x


18x
5
% he could invest in PQR = × 100
5x
1800
= % = 72%
25

32. (e); He bought total of 50 kg sugar.


And ratio of quantity of both is given as 2:3:5. Therefore, he have
10 kg of X, 15 kg of Y and 25 Kg of Z.
And cost price of X, Y & Z is Rs. 30/kg, Rs. 40/kg and Rs. 50/kg
20
Profit earn on 1kg of X is × 30 = Rs. 6
100
10
Loss earn on 1 kg of Y is × 40 = Rs. 4
100

Profit earn on 1 kg of Z is 32/100] × 50


= Rs. 16
But he sold all of X & Y and 40 – (10 + 15) =15 kg of Z.
∴ Total amount of profit earned is
6 × 10 – 4 × 15 + 16 × 15
= 60 – 60 + 240 = Rs. 240

33 Adda247 Publications For any detail, mail us at


Publications@adda247.com
Cracker Book for Bank (IBPS | SBI | RRB PO | Clerk) Mains Exams

33. (d); Let total votes be 6𝑥


People who took part in survey
= 83⅓% × 6𝑥 = 5𝑥
2.5𝑥 claims to vote for A,
0.5𝑥 are uncertain.
People who didn’t took part in survey
= 6𝑥 – 5𝑥 = 𝑥
2 2
And vote for A= ×𝑥 = 𝑥
3 3

Vote for A, from people who are uncertain.


1
= × 0.5𝑥 = 0.1𝑥
5
2
Total votes for A = 2.5x + 0.1x + 𝑥
3
25 1 20
= x+ x+ x
10 10 30
75𝑥+3𝑥+20𝑥 98𝑥
= =
30 30
98𝑥 82𝑥
Votes for B = 6𝑥– =
30 30

ATQ,
98𝑥 82𝑥 16𝑥 16𝑥
– = ⇒ = 640
30 30 30 30

⇒ 𝑥 = 40 × 30 = 1200
Hence total votes are 6 × 1200 = 7200

34. (d); Let total population of village = 100x


Male population = 60x
1
Male above 60 year = 60x × = 4x
15

34 Adda247 Publications For any detail, mail us at


Publications@adda247.com
Cracker Book for Bank (IBPS | SBI | RRB PO | Clerk) Mains Exams
1
Male below 18 years = (60x – 4x) × = 8x
7

Male above 18 but below 50 years


5
= (56x – 8x) × = 30x
8

Male above 50 year but below 60 years


= (56x – 8x – 30x) = 18x
ATQ—
30x – 4x = 7800
26x = 7800 ⇒ x = 300
Female population in village
= 300 × 40 = 12000

35. (d); Let marked price of Jeans be Rs. 9x and shirt be Rs. 7x for
Divyaraj
8
CP of jeans for Divyaraj = 9x × = 8x Rs.
9
6
CP of shirt for Divyaraj = 7x × = 6x Rs.
7

Let total number of jeans purchased by Divyaraj be 5y and shirt


be 8y
Total cost price for Divyaraj
= 8x × 5y + 6x × 8y
= 88xy
SP of jeans on, which Divyaraj sold
3 3
= 8x × × = 9x
2 4

SP of shirt on, which Divyaraj sold

35 Adda247 Publications For any detail, mail us at


Publications@adda247.com
Cracker Book for Bank (IBPS | SBI | RRB PO | Clerk) Mains Exams
3 8
= 6x × × = 8x
2 9
Total S.P. on which Divyaraj sold all items
= 9x × 5y + 8x × 8y
= 45xy + 64xy = 109xy
109xy –88xy
Required% = × 100
88xy
19
= 23 %
22

36. (b); Let cost of each mobile be Rs. ‘a’ in 2016 and number of mobile
sold in 2016 be ‘b’
Total revenue generated by store in 2016
= ab Rs.
In 2017 —
Cost of each mobile = 0.75a Rs.
Total revenue generated by store in 2017
105
= × ab = 94.5 lakh
100

ab = 90 lakh …(i)
Total number of mobile sold by store in 2017
1.05ab
= = 1.4b
0.75a

Given,
Number of mobile sold in 2017 – number of mobile phone sold
in 2016 = 90
1.4b – b = 90
b = 225
Cost price of each mobile in 2016
90,00,000
= = 40000 Rs.
225

36 Adda247 Publications For any detail, mail us at


Publications@adda247.com
Cracker Book for Bank (IBPS | SBI | RRB PO | Clerk) Mains Exams

37. (d); Let the marked price of a Shirt = 500 Rs


And the marked price of a Jeans = 700 Rs
Also, no. of shirt bought by Veer = 9x
And no. of jeans bought by Veer = 8x
4
C.P. of shirt for Veer = × 500 = 400 Rs.
5
3
C.P. of jeans for Veer = × 700 = 525 𝑅𝑠.
4

Total C.P. of shirt and jeans for Veer


= 9𝑥 × 400 + 8𝑥 × 525 = 7800𝑥 Rs.
Total S.P. of shirt and jeans for Veer
= 7𝑥 × 400 × 1.5 × 0.75 + 8𝑥 × 525 × 1.5 × 0.8
= 3150𝑥 + 5040𝑥
= 8190𝑥 Rs.
8190𝑥−7800𝑥
Profit % = × 100
7800𝑥
390
= × 100 = 5%
7800

38. (b); Let cost price per kg of Type B wheat = x Rs.


And, Cost price per kg of type B wheat
= 10x Rs.
Selling price of mixture of Type A & Type B wheat = 11x Rs.
Cost price per kg of mixture of Type A & Type B wheat = 11x
32
× = 6.4𝑥 𝑅𝑠.
55

ATQ –
Let N kg of type B wheat in mixture of 200 kg of Type A & Type B
of wheat
37 Adda247 Publications For any detail, mail us at
Publications@adda247.com
Cracker Book for Bank (IBPS | SBI | RRB PO | Clerk) Mains Exams

6.4𝑥 × 200 = N × 𝑥 + 10x (200 − N)


1280 = N + 2000 – 10N
9N = 920
N = 80 kg
80
Required ratio =
120
2
=
3

39. (b); Let marked price of jeans is 300𝑥 Rs. while marked price of shirt
is 200𝑥.
Selling price of a jeans & a shirt in scheme A
65
= (300𝑥 + 200𝑥) ×
100
= 325𝑥
Selling price of one jeans & one shirt in scheme B
75 80
= 300𝑥 × + 200𝑥 ×
100 100

= 225𝑥 + 160𝑥
= 385𝑥 Rs.
Given, 385𝑥 – 325𝑥 = 384 Rs.
60𝑥 = 384 Rs.
𝑥 = 6.4 Rs.
3
Cost price of a shirt = 160𝑥 ×
4
3
= (160 × 6.4 × )
4

= 768 rs.
4
Cost price of jeans = 225𝑥 ×
5
4
=225 × 6.4 ×
5

= 1152 Rs.

38 Adda247 Publications For any detail, mail us at


Publications@adda247.com
Cracker Book for Bank (IBPS | SBI | RRB PO | Clerk) Mains Exams

40. (b); Let, total manufactured cost of a cycle


= 100x Rs.
Then S.P a cycle = 125x Rs.
Expenditure on raw materials
= 100x × 0.40 = 40𝑥 𝑅𝑠.
Expenditure on labour = 100𝑥 × .20
= 20𝑥 𝑅𝑠.
Fixed charge = 100𝑥 × .20 = 20𝑥 𝑅𝑠.
Expenditure on transportation
= 100x − (40𝑥 − 20𝑥 − 20𝑥 ) = 20𝑥 𝑅𝑠.
New expenditure on raw materials
= 1.15 × 40x = 46x Rs.
New expenditure on labour
= 1.2 × 20x = 24x Rs.
New expenditure on transportation
= 1.5 × 20x = 30x Rs.
7
To get a profit of 13 % at a selling price of Rs. 125x, the C.P. of
11
1100
an item must be equal to 125𝑥 ×
1250

= 110𝑥 Rs.
We need to reduce C.P. (or expenditure) by 10, and this 10 has
to be reduced from expenditure on raw materials.
10𝑥
Required % = × 100
46𝑥
17
= 21 %
23

39 Adda247 Publications For any detail, mail us at


Publications@adda247.com
Cracker Book for Bank (IBPS | SBI | RRB PO | Clerk) Mains Exams

1 Adda247 Publications For any detail, mail us at


Publications@adda247.com
Cracker Book for Bank (IBPS | SBI | RRB PO | Clerk) Mains Exams

Chapter
Simple Interest and Compound Interest
6
BEST APPROACH TO SOLVE THE QUESTIONS

The interest can be defined as amount paid by the borrower to the lender in
addition to the amount which he had borrowed. In competitive exams,
interest is classified into Simple interest (SI) and Compound Interest (CI).

Easiest example, when we borrow money from bank, it doesn’t say pay Rs.
100 as interest every year no matter whether you took a loan of Rs 1000 or
Rs 100000. They offer us a rate generally in % at which we have to pay
interest. This rate is known as Rate of interest (ROI).

For eg. If I borrowed Rs. 500 for 1 year at a rate of 8%/annum. Find the
interest that I have to pay.
Always remember, this 8% we have to calculate on the amount we
borrowed. 8% of Rs. 500 means Rs. 40, that I have to pay as an interest.

Note: Different forms of ROI may be given or we have to change, as 3% for 3


months or 24% for 18 months. For easier calculation we can normalize
these in 12 month/year rate. In above rates, former means 12%/annum
and later means 16%/annum.

Now, if I asked what would be amount of interest that I have to pay, If I


borrow Rs. 500 for 3 years at same rate?

Here arise difference b/w SI and CI.

2 Adda247 Publications For any detail, mail us at


Publications@adda247.com
Cracker Book for Bank (IBPS | SBI | RRB PO | Clerk) Mains Exams

SIMPLE INTEREST:

Simple interest simply means that we have to pay same amount of interest
every time. If I have to pay interest on Rs. 500 at ROI of 8%/annum for 3
years, it means I will pay
40 × 3 = RS 120.

Formula to calculate SI on an amount is


PRT
SI =
100
Here, P is the amount borrowed and generally it is referred as Principal.
R is ROI in %. (100 in equation is used to because of %)
And T is time in years.

NOTE: If it is asked in a question, that what is interest obtained at the ROI


of 10%(or any value) for 5 years on amount of Rs. Z.

In order to save time first calculate mentally 10 × 5 = 50%. Total interest


will be 50 % of Principal amount.

COMPOUND INTEREST:

The one liner difference b/w SI and CI is that interest paid for every year is
not same. Consider the above example.

Interest accrued in 1st year = Rs. 40


In CI principal amount changes, after 1year principal amount will be Rs. 540
not Rs. 500.
8
After second year interest obtained on Rs. 540 is × 540 = Rs. 43.2
100
And interest after 3 years will be calculated on Rs 540+Rs 43.2= Rs 583.2 i.e
8
× 583.2= Rs. 46.65
100
And total interest will be 40 + 43.2 + 46.65= Rs 129.85
The formula for amount obtained after the given time period in CI
𝐑 𝐧
𝐀 = 𝐏 (𝟏 + )
𝟏𝟎𝟎
Here A is amount obtained after interest, P is principal, n is number of years
R is ROI.
CI= A – P.
3 Adda247 Publications For any detail, mail us at
Publications@adda247.com
Cracker Book for Bank (IBPS | SBI | RRB PO | Clerk) Mains Exams

In competitive exams, this approach is rarely used because it will be too


calculative if time is 3 years and ROI is 17%.
17 3 117 3
A = P (1 + ) = P( ) Imagine if you started calculating cube of 117 .
100 100

Best approach to use in CI is resultant ROI, it makes CI as easy as SI. Eg.


If ROI on Rs.10000 in a scheme is Y%, calculate total interest after two
years. Then resultant interest is

Y2
Y + Y + (Y × Y)/100 = 2Y +
100
17×17
Resultant interest % on 17%= 2×17 + = 34+ 2.89= 36.89%.
100
It is more time saving than traditional method. Little use of mind will help
you to solve all kind of problems.
Note that if we can calculate resultant interest % for two years, we can also
calculate it for three years. Actually general formula for calculating
resultant % for two years, if ROI on first year is X% and second year is Y%,
then resultant interest % will be
X×Y
X+Y+
100

For calculating 3year resultant interest, first calculate resultant interest of


first two years and then again use resultant formulae for adding resultant of
first two year and ROI of 3rd year.

REMARK: Check 3 years total interest % for ROI 0f 10% whether it comes
33.1%.

DIRECT FORMULAE TO CALCULATE 3 YEARS RESULTANT INTEREST %


3r. 3 r² r³ __
Here, r is ROI

Note that 4 underscores (_ _ _ _) represent 4 digits after decimal. Further,


make it clear that _ _ (last two digits are for last two digits of value of r³]
And starting two (_ _) are for value of 3r².
Consider eg.
3r. 3 r² r³_
When r = 2
r³ = 08, r² = 4, 3r² = 12
4 Adda247 Publications For any detail, mail us at
Publications@adda247.com
Cracker Book for Bank (IBPS | SBI | RRB PO | Clerk) Mains Exams

Resultant interest is for 3 years


6.1208
Now consider r = 8
r³ = 512, r² = 64, 3r² = 192
use last two digits of r³ for last two digits.
i.e. 3r. _ _ 12, and carry on 5 to 3r².
3r² = 192 + 5 = 197.
Here also, use only its last 2 digit and carry on 1.
3r. 9 7 1 2
3r = 24 + 1 → carry on from 197.
Resultant interest % = 25.9712%
Check for r = 10%.
While solving a question, student should always keep in mind
resultant interest %.
Example: Amount of Rs 4000 becomes Rs 6600 in SI in 5 years. Calculate
ROI.
PRT
Solution: Though general formula of SI i.e. SI = is an easy approach,
100
But try mentally to solve RS 2600 is what % Rs 4000. You can mentally
calculate that
2600 13
= which means 65%.
4000 20
And 65% in 5 years in SI means 13%/year.

Example: Difference b/w interest obtained in SI and CI on an amount at the


ROI of 10% is Rs. 310. Find the principal amount.

Solution:
Calculate resultant % for SI and CI for 3 years at ROI of 10%.
Its 30% for SI and 33.1% for CI. But we know that we calculate interest on
principal. It means that 3.1% of principal is equal to Rs 310. Hence principal
can be calculated.

POINTS TO REMEMBER ALWAYS:


Difference b/w interest obtained on CI and SI for 2 years is PR2/100.
Resultant interest % in CI for 3 years at 10% ROI is 33.1% and
15.7625% at ROI of 5%.
5 Adda247 Publications For any detail, mail us at
Publications@adda247.com
Cracker Book for Bank (IBPS | SBI | RRB PO | Clerk) Mains Exams

Practice Exercise Based on new Pattern

1. Ankur invested X Rs. at the rate of 15% per annum on compound


interest for two years and gets total interest of 5805 Rs. if Ankur
invest (X + 7000) Rs. for another two year at additional rate of 5%,
then what will be compound interest on that investment ?
(a) 10000 Rs. (b) 11000 Rs. (c) 12000 Rs.
(d) 15000 Rs. (e) 18000 Rs.

2. Shikha invested 32000 Rs. at simple interest for 2 years at the rate of
R% and gets an interest of Rs.8000. If he invested total amount
(Principle + Interest) in a scheme, which offered compound interest at
the rate of (R%+2.5%) then find total compound interest obtained by
Shikha after 2 years ?
(a) 12600Rs. (b) 12800Rs. (c) 14400 Rs.
(d) 12000 Rs. (e) 12900Rs.

3. Ankit borrowed Rs. 12000 from Veer on C.I. at 10% per annum for
three years and added some extra amount and lent to Arun on C.I. at
20% per annum for two year. If Ankit got Rs. 3948 more interest than
interest got by Veer.Find how much extra amount was added by Ankit
?
(a) 6000 Rs. (b) 4000 Rs. (c) 5000 Rs.
(d) 8000 Rs. (e) 9000 Rs.
4. Abhi invested some amount on scheme ‘P’ which offer CI at the rate of
15% p.a.. After 2 years he got Rs. 1032 as interest. Abhi invest the
amount he got from scheme ‘P’ in scheme ‘Q’ which offer 15% p.a. at SI
for 4 years. Find the total interest he earned from scheme P and
scheme Q together?
(a) 3571.2 (b) 3715.2 (c) 4232
(d) 3148 (e) 3379.2

6 Adda247 Publications For any detail, mail us at


Publications@adda247.com
Cracker Book for Bank (IBPS | SBI | RRB PO | Clerk) Mains Exams

5. A man invests Rs. 1,200 at 10% p.a. At the end of the year he
withdraws 30% of total amount and pays Rs. 24 as transaction fee. At
the end of 2nd year he withdraws 30% of the amount and pays Rs. 93
as transaction fee. What is the balance at the end of the third year?
(a) Rs. 660 (b) Rs. 825 (c) Rs. 500
(d) Rs. 770 (e) None of these

6. A sum of Rs. 1000 after 3 years at compound interest of 10% p.a.


becomes a certain amount that is equal to the amount that is the
result of 3 year depreciation from Rs. 1728. Find the approximate
difference between the rate of C.I. and rate of depreciation.
(a) 3.33% (b) 0.66% (c) 4%
(d) 2% (e) 6%

7. A person invested a certain amount at simple interest at the rate of 6


per cent per annum earning Rs. 900 as an interest at the end of three
years. Had the interest been compounded every year, how much more
interest would he have earned on the same amount with the same
interest rate after three years?
(a) Rs. 38.13 (b) Rs. 25.33 (c)Rs. 55.08
(d) Rs. 35.30 (e) None of these

1
8. Mayank borrowed Rs. 48000 from a bank at 12 % per annum
2

compound interest at the end of 1st, 2nd and 3rd year, he paid 14000,
13000 and 16000 respectively. If he wanted to clear his loan at the
end of 4th year, what would he pay at the end of the fourth year to
clear his loan ?
(a) 21000 (b) 22500 (c) 16800
(d) 26000 (e) None of these

7 Adda247 Publications For any detail, mail us at


Publications@adda247.com
Cracker Book for Bank (IBPS | SBI | RRB PO | Clerk) Mains Exams

9. Rakesh lent Rs. P to Rahul on an agreement that for first two year,
interest will be calculated on SI at 15% per annum and for next two
years interest will be calculated on C.I at 5% additional rate. If Rahul
paid total amount of Rs. 17971.2 at the end of four year, Find the
amount borrowed by Rahul?
(a) 9000 Rs. (b) 9800 Rs. (c) 9900 Rs.
(d) 9600 Rs. (e) 9200 Rs.

10. Veer invested Rs 22500 for 2 year at the Rate of x% in scheme A at


compound interest annually and gets a total amount of Rs 32400 If he
added Rs 2600 in amount and invested total amount in scheme B at
S.I. for 3 year at same rate . Then find the total simple interest veer gets
from scheme B?
(a) Rs 22500 (b) Rs 22000 (c) Rs 17500
(d) Rs 20000 (e) Rs 21000

11. Veer Invested X Rs. in SBI at the rate of 18% for 2 year and obtained a
total simple interest of 6750 Rs. If he invested 2250 Rs. more for same
2 2
period time at the rate of 14 % for first year and at 16 % for 2nd
7 3
year on compound annually. Then find the total compound interest
obtained by Veer after 2 year ?
(a) 6000 Rs. (b) 5000 Rs. (c) 3000 Rs.
(d) 7000 Rs. (e) 5500 Rs.

12. Satish invested 16000 Rs. in simple interest for 2 years on certain rate
and gets an interest of 4800 Rs, if he invested total amount (Principle
+ Interest) in a scheme, which offered compound interest on 5% more
interest rate as earlier rate. Then find total interest gets by Satish after
2 years ?
(a) 9252 Rs. (b) 9225 Rs. (c) 9512 Rs.
(d) 925 Rs. (e) 9152 Rs.

8 Adda247 Publications For any detail, mail us at


Publications@adda247.com
Cracker Book for Bank (IBPS | SBI | RRB PO | Clerk) Mains Exams

13. Bhavya and Veer invested their principle in two different schemes,
Bhavya invested X Rs. on compound interest for two year at rate of
20% annually and Veer invested 4000 Rs. more than Bhavya on
simple interest for three year at 15% annually, if both gets total
interest of Rs. 9632, then Find the amount invested by Veer ?
(a) 12900 (b) 12400 (c) 8800
(d) 12800 (e) 12600

14. A man invested Rs. x, y and z in three difference schemes which offers
10% p.a., 20% p.a. and 30% p.a. rate respectively and after one year
he earns Rs. 2000 interest. If x, y and z are in arithmetic progression
and sum of x, y and z is Rs. 9,000. Find the difference between ‘x’ and
‘z’.
(a) 1500 (b) 2000 (c) 2500
(d) 2250 (e) 1750

15. P invested Rs. X in a scheme for 2 year which offered simple at the
rate of 15% per annum and Q invested Rs. (X + 2500) in another
scheme for same period of time, which offered compound interest at
the rate of 20% per annum. If from both scheme P and Q got total
interest of Rs. 32550, then find the value of X ?
(a) 41500 (b) 42500 (c) 40500
(d) 40000 (e) 38250

16. Rajat have some money in his hand. He invested 20% of the money in
scheme ‘A’ for 4 year at 6% p.a., 30% of the amount in scheme ‘B’ for 6
year at 12% p.a., remaining in scheme ‘C’ 2 year at 15% p.a. If total
amount Rajat received from scheme ‘A’, ‘B’ and ‘C’ is 11,355, then find
the difference of sum invested in scheme ‘B’ and scheme ‘A’.
(a) 1800 (b) 1500 (c) 1200
(d) 750 (e) 270
9 Adda247 Publications For any detail, mail us at
Publications@adda247.com
Cracker Book for Bank (IBPS | SBI | RRB PO | Clerk) Mains Exams
1
17. Satish borrowed Rs. 24000 from a SBI at the rate of 12 %
2
compounded annually. If at the end of 1st, 2nd and 3rd year, he paid Rs.
7000, Rs. 6500 and Rs. 8000 respectively. If Satish wanted to clear his
loan at the end of 4th year, what would he pay at the end of the fourth
year to clear his loan ?
(a) 11500 Rs. (b) 11250 Rs. (c) 10500 Rs.
(d) 9500 Rs. (e) 14500 Rs.

18. Divyaraj invested an amount into two parts in the ratio of 4 : 3 on


compound interest for two years at the rates of 20% & 15%
respectively. If he exchange rate of interests, then he will get Rs. 705
less interest than earlier interest, then find how much Simple interest
he will get, if he invest total amount at the rate of 17.5% for two
years?
(a) 14500 Rs. (b) 14700 Rs. (c) 14900 Rs.
(d) 14800 Rs. (e) 14900 Rs.

19. Rahul took a loan of Rs.80000/- at rate of of 5% p.a. at SI & lent it at


5% pa at CI. After 3 years, he cleared all his debts and invested the
profit earned in a scheme which offers C.I. After two years he got
Rs.128.1/- as interest from the scheme. Find the rate of interest
offered by scheme.
(a) 8% (b) 7% (c) 6%
(d) 10% (e) 12%

20. A man invested his total savings in two parts. He invested Rs. 30000/-
in a scheme which offers CI at ROI of 10% per annum and remaining
in SI at ROI of 5%per half year. After two years the amount obtained
by him in former scheme is half of later, then calculate amount
invested in later scheme.
(a) Rs. 242000 (b) Rs. 60500 (c) Rs. 12100
(d) Rs. 80525 (e) None of these
10 Adda247 Publications For any detail, mail us at
Publications@adda247.com
Cracker Book for Bank (IBPS | SBI | RRB PO | Clerk) Mains Exams

21. Satish saved 55% of his monthly income, which he further invested in
two different schemes A & B in the ratio of 7 : 5 on CI compounded
annually at the rate of 10% p.a. and 20% p.a. respectively. If after two
years, Satish gets total interest of Rs. 8074 from both the schemes,
then find the total annual saving of Satish?
(a)316,600 Rs. (b)316,400 Rs. (c)316,000 Rs.
(d)316,200 Rs. (e)316,800 Rs.

22. Aman invest Rs. (x – 2000) in scheme ‘P’ which offers 20% CI pa while
Rs. (x + 2000) in scheme ‘Q’ which offers 8% SI pa. After 2 year
difference between interest earn from scheme P and Q is 1600, then
amount invested by Aman in scheme ‘Q’ is what percent more/less
than amount invested by him in scheme ‘P’ ?
1
(a) 75% (b) 133 % (c) 150%
3
1
(d) 50% (e) 33 %
3

23. Ritu invested her total saving in three different FD schemes A, B and C
in the ratio of 5 : 4 : 6 on CI for two years at the rate of 10%, 15% and
20% respectively.If interest is calculated annually and interest from
scheme B is Rs. 744 more than interest from scheme A then, find
difference between interest received from scheme C and scheme B by
Ritu?
(a) Rs. 4185 (b) Rs. 4175 (c) Rs. 3840
(d) Rs. 4580 (e) Rs. 3250

24. Raghv invested Rs. 8400 and Rs. 9600 at the rate of R% and (R + 5)%
respectively on simple interest for two years and gets total interest of
Rs. 6360. If Raghv invested Rs.(8400 + P) and Rs.(9600 + P) at the rate
of R% and (R + 5)% respectively on compound interest for two year,
then he would get total interest of Rs. 8153. Find value of P?
(a) 1800 Rs. (b) 1600 Rs. (c) 2200 Rs.
(d) 2400 Rs. (e) 2600 Rs.
11 Adda247 Publications For any detail, mail us at
Publications@adda247.com
Cracker Book for Bank (IBPS | SBI | RRB PO | Clerk) Mains Exams

Directions (25-26): A person invested Rs. 20000 in a bank which is


offering 10% per annum simple interest. After two years he withdrew the
money from the bank and deposited the total amount in another bank
which gives an interest rate of r% p.a. compounded annually. After 2 years
he received an amount of Rs. 2460 more than what he had invested in that
bank.

25. What is the value of r?


(a) 10% (b) 15% (c) 5%
(d) 12% (e) None of these

26. If the person had invested Rs. 50,000 instead of 20000 in the bank
that offered simple interest, what would have been his net profit after
following the same procedure as given above?
(a) Rs. 16,800 (b) Rs. 16,150 (c) Rs. 16,350
(d) Rs. 16,000 (e) None of these

27. A bank offers 10% p.a. at CI in scheme ‘A’, 20% p.a. at CI in scheme ‘B’
and 40% p.a. at CI in scheme ‘C’. Veer have same amount in his hand.
He invested 32% amount in scheme ‘A’, 20% in scheme ‘B’ and
remaining in scheme ‘C’. After two year he received 6600 as interest.
Find the difference between amount invested by Veer on scheme ‘C’ to
that of in scheme ‘B’.
(a) 5,000 (b) 4,000 (c) 2,000
(d) 3,000 (e) 2,500

28. Bhavya borrowed Rs. 37500 on compound interest with agreement


that, he will not pay amount after 2 year with interest then the rate of
interest increase by 5% as earlier interest rate. If Bhavya paid Rs.
12093.75 as total compound interest after two year. Then find how
much amount Bhavya have to pay after two year for completing his
entire borrowing?
(a) Rs. 58000 (b) Rs. 52000 (c) Rs. 42000
(d) Rs. 45000 (e) Rs. 54000
12 Adda247 Publications For any detail, mail us at
Publications@adda247.com
Cracker Book for Bank (IBPS | SBI | RRB PO | Clerk) Mains Exams

29. A man invested ‘X’ amount in scheme A and ‘Y’ amount in scheme B.
Scheme A offers 8% p.a. at S.I. scheme B offer 20%. p.a. at C.I. Man
invested both schemes for 2 years. The sum of interest he earns after
2 years will be equal to the simple interest he can earn if he invested
‘Y’ amount in scheme A for 7 years. Find the total amount he has
initially if difference between interest earned by him after 2 years is
2304.
(a) 12,300 (b) 12,900 (c) 12,600
(d) 13,500 (e) 13,800

30. Shikha and Sameer have equal amount. Shikha invested on C.I. for two
2
years at the rate of 10% p.a. and Sameer invested 66 % of amount at
3
the rate of R% p.a. on simple interest and remaining amount at the
rate of 6.5% p.a. on simple interest. If interest received by both at the
end of two years are equal, then find the value of ‘R’?
(a) 10% (b) 12.5% (c) 15%
(d) 12% (e) 8%

31. Veer invested Rs. x in SBI for three years on S.I. at the rate of 15% per
annum and Sameer invested Rs. (x + 4000) in BOB for two years on
C.I. at the rate of 8% per annum. If Divyaraj invested equal to sum of
what Veer and Sameer invested in UBI for two years on C.I. at the rate
of 20% per annum. Find the sum invested by Divyaraj, if they all got
total interest of Rs. 20382.4?
(a) Rs. 20000 (b) Rs. 18000 (c) Rs. 28000
(d) Rs. 30000 (e) Rs. 26000

32. Three employee Ankit, Veer and Sameer got Rs. P, Rs. (P+2400) and
Rs. (P+4400) Diwali bonus. Ankit and Veer invested their half of
bonus on CI at the rate of 10% and 20% respectively for two years.
Sameer invested 60% of his bonus on simple interest at the rate of
15% p.a. for three years. If Sameer got Rs. 132 more as interest got by
Ankit and Veer together, then find the bonus got by Sameer?
(a) 12000 Rs. (b) 10000 Rs. (c) 14400 Rs.
(d) 14000 Rs. (e) 18000 Rs.
13 Adda247 Publications For any detail, mail us at
Publications@adda247.com
Cracker Book for Bank (IBPS | SBI | RRB PO | Clerk) Mains Exams

33. Manoj lend Rs. P for three years on S.I. at the rate of 15% per annum
and Rajesh lend Rs. (P + 8000) for two years on C.I. at the rate of 8%
per annum. Suresh borrowed sum equal to of what Manoj and Rajesh
lend, for two years on C.I. at the rate of 20% per annum. If Suresh paid
interest Rs. 5352 more than, what Manoj and Rajesh got total interest
on their sums together. Find total sum borrowed by Suresh?
(a) Rs. 34000 (b) Rs. 44000 (c) Rs. 32000
(d) Rs. 46000 (e) Rs. 30000

34. A person invested his savings of Rs. 32000 in two parts at equal rate
of interest in CI and SI. After 3 years, he withdrew amount invested in
CI, and put the cash in his almirah. After another 2 years, the amount
invested in SI got doubled and he also withdrew them. If the final ratio
of both the amount is 36:65, then find amount invested by him in SI
(in Rs.)
(a) 20000/- (b) 18050/- (c) 19500/-
(d) 21500/- (e) can’t be determined

35. A man invested an amount into three schemes P, Q and R in the ratio
of 5 : 6 : 9 at the rates of 20%, 15% and 10% respectively. Schemes P
and R offered compound interest annually, while scheme Q offered
simple interest annually. Total interest received by man from scheme
P and Q together is Rs. 1899 more than total interest received from
scheme R after two years. If man would invest total amount on C.I at
the rate of 15% p.a., then find the total interest received by the man?
(a) 5800 Rs. (b) 5805 Rs. (c) 5580 Rs.
(d) 5590 Rs. (e) 5900 Rs
14 Adda247 Publications For any detail, mail us at
Publications@adda247.com
Cracker Book for Bank (IBPS | SBI | RRB PO | Clerk) Mains Exams

Solutions

15 2
1. (b); ATQ, 5805 = X [(1 + ) – 1]
100
129X = 5805 × 400
X = 18000 Rs.
Now
Total Interest=(18000+7000)
(15+5) 2
×[(1 + ) – 1]
100
11
= 25000 × ( )
25
= 11000 Rs.

2. (e); Let shikha invested at the rate of R%


ATQ—
8000×100
R= R = 12.5%
32000×2
New Rate = 12.5 + 2.5 = 15%
Total amount = 32000 + 8000 = 40000 Rs.
Equivalent CI of two years at the rate of 15%
15×15
= 15 + 15 +
100
= 32.25%
32.25
Required compound interest = 40000 × = 12900 Rs.
100

10
3. (a); Interest got by Veer=12000[(1 + )3 − 1]
100
= 3972
20×20
Two year CI on 20% = 20 + 20 +
100
= 44%
Atq,
44
(12000 + 𝑥 ) − 3972 = 3948
100
5280+.44x – 3972 = 3948
.44x = (3948 + 3972) – 5280
.44x = 2640
x = 6000 Rs
15 Adda247 Publications For any detail, mail us at
Publications@adda247.com
Cracker Book for Bank (IBPS | SBI | RRB PO | Clerk) Mains Exams

4. (a); Let, sum Abhi have initially = 100x


ATQ,
115 115
100x × × – 100x = 1032
100 100
132.25x – 100x = 1032
1032
⇒x= = 32
32.25
Amount initially Abhi have = 3200
4232×15×4
Interest earned from scheme Q =
100
= 2539.2
Required amount = 2539.2 + 1032
= 3571.2

5. (a); The man invests Rs. 1,200 at 10% p.a.


At the end of 1st year the amount = Rs. 1,320
30
Withdrawal × 1320 + 24 = Rs. 420
100
Amount at the end of second year
= 900 × 1.1 = Rs. 990
30
Withdrawal = × 990 + 93 = Rs. 390
100
∴ Amount at the end of 3 years
= 600 × 1.1 = Rs. 660

6. (d); The amount @ 10% C.I. could become Rs. 1331. Also,
Rs. 1728 depreciated at R% has to become Rs. 1331.
Thus,
100−R 3
1728 × [ ] =1331(approximately).
100

The closest value of R = 8%


Thus, the difference is 2%.

16 Adda247 Publications For any detail, mail us at


Publications@adda247.com
Cracker Book for Bank (IBPS | SBI | RRB PO | Clerk) Mains Exams
900 ×100
7. (c); P =
6×3
P = 5000

Required value = 955.08 – 900 = 55.08

8. (b); 1st year CI


1
48000 × = 6000
8
Amount = 48000 + 6000 = 54000
2nd year principle = 54000 – 14000 = 40000
2nd year CI =
1
40000 × = 5000
8
Amount = 40000 + 5000 = 45000
3rd year principle
= 45000 – 13000 = 32000
3rd year CI
1
= 32000 × = 4000
8
Amount = 32000 + 4000 = 36000
4th year principle = 36000 – 16000 = 20000
At the end of 4th year amount
1
= 20000 × + 20,000
8
= 2500 + 20000
= 22500

9. (d); For first two years total S.I = 15 × 2


= 30%
For next two years total C.I on (15+5)%
20×20
= 20+20+
100
= 44%
Amount after two year

17 Adda247 Publications For any detail, mail us at


Publications@adda247.com
Cracker Book for Bank (IBPS | SBI | RRB PO | Clerk) Mains Exams
𝑃×30
=P+
100
13𝑃
= Rs.
10
Amount after 4 years
13𝑃 144
× = 17971.2
10 100
17971.2
P=
1.872
P = Rs. 9600

10. (e); ATQ,


𝑅 2
32400 = 22500 (1 + )
100
324 100 + 𝑅 2
=( )
225 100
18 100 + 𝑅
=
15 100
15R = 300
R = 20%
20×3
Simple interest = (32400 + 2600) ×
100
= 21000 Rs

X×18×2
11. (d); Principle = = 6750
100
6750×100
⇒X=
18×2
X = 18750 Rs.
According to question —
1
1st year CI = (18750 + 2250) ×
7
= 3000
1
2 year CI = (21000 + 3000) ×
6
1
= 24000 ×
6
= 4000
Total CI after 2 year = 3000 + 4000
= 7000 Rs.

18 Adda247 Publications For any detail, mail us at


Publications@adda247.com
Cracker Book for Bank (IBPS | SBI | RRB PO | Clerk) Mains Exams
4800×100
12. (e); Rate =
16000×2
R = 15%
New rate = 15 + 5 = 20%
2 year CI on 20%
20×20
= 20 + 20 +
100
= 44%
Interest gets Satish
44
= (16000 + 4800) ×
100
= 9152 Rs.

13. (d); ATQ,


(X+4000)45 44X
+ = 9632
100 100
0.45X + 1800 + 0.44x = 9632
0.89X = 9632 – 1800
7832
X=
0.89
X = 8800
Veer principle = 8800 + 4000
= 12800 Rs.

14. (b); x + y + z = 9,000


& 2y = x + z
⇒ y = 3,000
10 20 30
×x+ × 3,000 + × z = 2000
100 100 100
0.1x + 0.3z = 2000 – 600 = 1400
and x + z = 6,000
0.1x + 0.3z = 1400 …(i)
x + z = 6,000 …(ii)
on solving (i) & (ii), we get
x = 2,000
z = 4,000
Difference between x & z = 2,000

19 Adda247 Publications For any detail, mail us at


Publications@adda247.com
Cracker Book for Bank (IBPS | SBI | RRB PO | Clerk) Mains Exams

15. (b); ATQ


15×2x 20 2
+ (x + 2500) [(1 + ) – 1] = 32550
100 100
0.3x + 0.44x + 1100 = 32550
0.74x = 31450
31450
x=
0.74
x = 42500

16. (d); Let the amount Rajat have = 100


ATQ,
20×4×6 30×6×12 50×2×15
100 + + + = 11,355
100 100 100
151.4 = 11355
⇒ 100 = 7500
7500[30 –20]
Required difference =
100
= 750

17. (b); 1st year CI


1
24000 × = 3000 Rs.
8
Amount = 24000 + 3000 = 27000 Rs.
2nd year principle = 27000 – 7000 = 20000 Rs.
2nd year CI =
1
20000 × = 2500 Rs.
8
Amount = 20000 + 2500 = 22500 Rs.
3rd year principle
= 22500 – 6500 = 16000 Rs.
3rd year CI
1
= 16000 × = 2000
8
Amount = 16000 + 2000 = 18000 Rs
4th year principle = 18000 – 8000 = 10000 Rs.
At the end of 4th year amount, which Satish have to pay
1
= 10000 × + 10,000
8
= 1250 + 10000 = 11250 Rs.

20 Adda247 Publications For any detail, mail us at


Publications@adda247.com
Cracker Book for Bank (IBPS | SBI | RRB PO | Clerk) Mains Exams

18. (b); Let Divyaraj invested total amount = 7x Rs.


Equivalent CI of two year at the rate of 20%
20×20
= 20 + 20 + = 44%
100
Equivalent CI of two year at the rate of 15%
15×15
= 15 + 15 + = 32.25%
100
ATQ –
44 32.25 32.25 44
4x × + 3𝑥 × − 4𝑥 × − 3𝑥 × = 705
100 100 100 100
(1.76x + .9675x) – (1.29𝑥 + 1.32𝑥) = 705
2.7275x – 2.61𝑥 = 705
0.1175x = 705
x =6000 Rs.
Total amount invested by Divyaraj = 6000 × 7 = 42000 𝑅𝑠.
17.5×2
Required simple interest = 42000 × =14700 Rs.
100

19. (d); Profit earned in 3 years = Difference in interest obtained


D=P r2 (300+r)
(100)3
80000×25×305
=
1000000
= Rs610
Let scheme offers R% rate of interest
𝑅 2
610 (1 + ) − 610 = 128.1
100
R = 10%

20. (b); Let he invested Rs. Y in second scheme


5% per half year means 10% per annum in SI
Therefore, amount obtained by him after two years
Y(2)(10)
=Y+
100
6𝑌
=
5
If he invested Rs. 30000/- in a scheme, which offers ROI of 10%
per annum
Amount obtained by him after two years is
10 2
= 30000 (1 + )
100

21 Adda247 Publications For any detail, mail us at


Publications@adda247.com
Cracker Book for Bank (IBPS | SBI | RRB PO | Clerk) Mains Exams
11 11
= Rs. 30000 × ×
10 10
= Rs. 36300
Now According to Q
1 (6𝑌)
⇒ 36300 =
2 5
36300×5×2
⇒Y=
6
= Rs. 60500/-

21. (e); Let total monthly income of Satish = 100x Rs.


Amount invested in scheme A
55 7
= 100𝑥 × ×
100 12
385𝑥
= Rs.
12
Amount invested in scheme B
55 5
= 100𝑥 × ×
100 12
275𝑥
= Rs.
12
Equivalent CI of two years on 10%
10×10
= 10 + 10 +
100
= 21%
Equivalent CI of two years on 20%
20×20
= 20 + 20 +
100
= 44%
ATQ—
385𝑥 21 275 44
× + × = 8074
12 100 12 100
1617𝑥+2420𝑥
= 8074
240
4037x = 8074 × 240
𝑥 = 480 Rs.
Annually saving of Satish
55
= 48000 × × 12
100
= 316,800 Rs.

22 Adda247 Publications For any detail, mail us at


Publications@adda247.com
Cracker Book for Bank (IBPS | SBI | RRB PO | Clerk) Mains Exams

22. (d); Compound interest rate for 2 years


20+20
= 20 + 20 + = 44%
100
ATQ,
44 (x+2000)×8×2
(x– 2000) × – = 1600
100 100
44𝑥 16𝑥
⇒ – – 880 – 320 = 1600
100 100
28𝑥
⇒ = 1600 + 880 + 320 = 2800
100
⇒ x = 10,000
12000−8000 4
Required % = × 100 = × 100 = 50%
8000 8

23. (a); Let total saving of Ritu be x Rs.


5𝑥
Invested in Scheme A =
15
𝑥
= Rs.
3
4𝑥
Amount Invested in scheme B = Rs.
15
2𝑥
Invested in scheme C = 𝑅𝑠.
5
10×10
Two year C.I on 10% =10+10+
100
= 21%
15×15
Two year CI on 15% =15+15 +
100
= 32.25 %
20×20
Two year CI on 20% =20+20+
100
= 44%
ATQ,
4𝑥 32.25 𝑥 21
× − × = 744
15 100 3 100
129𝑥 21𝑥
− = 744
1500 300
24x = 744 × 1500
744×1500
x=
24
x = 46500 Rs.
Required difference
46500×6 44 46500×4 32.25
= × − ×
15 100 15 100
= 8184 – 3999
= 4185 Rs.
23 Adda247 Publications For any detail, mail us at
Publications@adda247.com
Cracker Book for Bank (IBPS | SBI | RRB PO | Clerk) Mains Exams

24. (b); ATQ—


8400×𝑅×2 9600×(𝑅+5)×2
+ = 6360
1000 100
168R + 192R + 960 = 6360
360R + 960 = 6360
360R = 5400
R = 15%
Rs. (8400 + P) invested at 15%
And Rs. (9600 + P) invested at 20%
Equivalent CI of two years at the rate of 15%
15×15
= 15 + 15 +
100
= 32.25%
Equivalent CI of two year at the rate of 20%
20×20
= 20 + 20 + = 44%
100
ATQ—
32.25 44
(8400 + 𝑃 ) × + (9600 + 𝑃 ) × = 8153
100 100
270900 + 32.25P + 422400 + 44P= 815300
76.25P = 122000
P = 1600 Rs.

25. (c); Amount withdrawn from bank which offered simple interest =
20000×10×2
20000 + = 24000
100
Compound interest accrued from another bank = 2460
𝑅 2
2460 = 24000 [(1 + ) − 1]
100
⇒r=5%

26. (b); Amount withdrawn after 2 years from bank offering S.I. =
5000×10×2
50000 +
100
= 60000
Amount withdrawn after another 2 years from another bank
5 2
offering C.I. = 60000 (1 + ) = 66150
100
New profit = 66150 − 50000 = 16,150

24 Adda247 Publications For any detail, mail us at


Publications@adda247.com
Cracker Book for Bank (IBPS | SBI | RRB PO | Clerk) Mains Exams

27. (d); Let, Total amount Veer have = ‘x’


ATQ,
11 11 12 12 14 14
0.32x [ × – 1] + 0.20x [ × – 1] + .48x [ × – 1]
10 10 10 10 10 10
= 6600
21 44 96
0.32x [ ] + 0.20x [ ] + 0.48x [ ] = 6600
100 100 100
6.72𝑥 8.8𝑥 46.08𝑥
+ + = 6600
100 100 100
6600×100
⇒x=
61.6
6600×100
Required difference = × [. 48 – .20] = 3,000
61.6

28. (e); ATQ,


r 2
12093.75 = 37500 [(1 + ) – 1]
100
12093.75 r
√ +1 = 1+
37500 100
49593.75 r
√ =1+
37500 100
r
1.15 – 1 =
100
r = 15%
New rate = 15 + 5 = 20%
Amount paid by Bhavya after next two more years
144
= 37500 × = 54000 Rs.
100

29. (c); ATQ


𝑥×8×2 20 2 𝑦×8×7
+ 𝑦 [(1 + ) − 1] =
100 100 100
16x + 44y = 56y
16x = 12y
𝑦 4 3𝑦
= ⇒𝑥=
𝑥 3 4
44y – 0.16x = 2304
3
44y – 0.16 × 𝑦= 2304
4
0.44y – 0.12y = 2304
y = 7200
x = 5400
Required amount = 12,600
25 Adda247 Publications For any detail, mail us at
Publications@adda247.com
Cracker Book for Bank (IBPS | SBI | RRB PO | Clerk) Mains Exams

30. (b); Let Shikha and Sameer have Rs. 100𝑥


Equivalent CI for two years at the rate of 10%
10×10
= 10 + 10 + = 21%
100
ATQ—
21 2 R×2 1 6.5×2
100x × = 100x × × + 100x × ×
100 3 100 3 100
4x×R 13x
21x = +
3 3
63x = 4x × R + 13x
4x × R = 50x
50x
R=
4x
R = 12.5%

31. (c); 2 year C.I. on 8% per annum


8×8
=8+8+ = 16.64%
100
2 year C.I. on 20% per annum
20×20
= 20 + 20 + = 44%
100
ATQ—
x×3×15 (x+4000)16.64 (2x+4000)44
+ + = 20382.4
100 100 100
0.45x + 0.1664x + 665.6 + 0.88x + 1760 = 20382.4
1.4964x = 20382.4 – 2425.6
17956.8
x= = 12000 Rs.
1.4964
Sum of Divyaraj invested = (2 × 12000 + 4000) = 28000 Rs.

32. (d); Bonus of Ankit = P Rs.


Bonus of Veer = (P + 2400) Rs.
Bonus of Sameer = (P + 4400) Rs.
Equivalent CI at 10% for two years
10×10
= 10 + 10 +
100
= 21%
Equivalent CI at 20% for two years
20×20
= 20 + 20 +
100
= 44%

26 Adda247 Publications For any detail, mail us at


Publications@adda247.com
Cracker Book for Bank (IBPS | SBI | RRB PO | Clerk) Mains Exams

ATQ—
60 15×3 P 21 (P+2400) 44
(P + 4400) × × – × − × = 132
100 100 2 100 2 100
27P+118800 65P+105600
– = 132
100 200
54P + 237600 – 65P - 105600 = 26400
11P = 105600
P = 9600
Sameer bonus = (9600 + 4400)
= 14000Rs.

33. (c); Three years SI on 15% = 15×3=45%


8×8
Equivalent two years CI on 8 %= 8 +8+ = 16.64%
100
20×20
Equivalent two years CI on 20% = 20+20+ = 44%
100
ATQ –
44(2𝑃+8000) 45𝑃 16.64(𝑃+8000)
−( + ) = 5352
100 100 100
.88P + 3520 - .45P – .1664P – 1331.2
= 5352
.2636P = 3163.2
3163,2
P= = 12000 𝑅𝑠.
.2636
Suresh borrowed = 12000×2+8000 = 32000 Rs

34. (c); ATQ,


The amount invested by him in SI, got doubled after 5 years.
Therefore ROI was 20% per annum.
Let amount invested by him in SI was x, then amount invested by
him in CI is (32000 – x)
Now,
120 3
(32000–x)( ) 36
100
=
2x 65
(32000–x)216 36
⇒ =
2x×125 65
(32000–x)3 1
⇒ =
x×25 13
⇒ 32000 × 39 = 64x
x = 500 × 39 = 19500/-

27 Adda247 Publications For any detail, mail us at


Publications@adda247.com
Cracker Book for Bank (IBPS | SBI | RRB PO | Clerk) Mains Exams

35. (b); Let man invested in scheme P, Q and R be Rs. 5x, Rs. 6x and Rs.
9x respectively
20×20
Equivalent CI of two years on 20% = 20 + 20 + = 44%
100
10×10
Equivalent CI of two years on 10% = 10 + 10 + = 21%
100
ATQ –
44 15 ×2 21
5𝑥 × + 6x × – 9x× = 1899
100 100 100
2.2x + 1.8x – 1.89x = 1899
2.11x = 1899
1899
x=
2.11
x = 900 Rs.
Total amount = 900 × (5 + 6 + 9)
= 18000 Rs.
If man invested total amount on C.I at the rate of 15% p.a.
15×15
Equivalent CI of two years on 15% = 15 + 15 + = 32.25
100
32.25
Required interest = 18000× = 5805 Rs.
100

28 Adda247 Publications For any detail, mail us at


Publications@adda247.com
Cracker Book for Bank (IBPS | SBI | RRB PO | Clerk) Mains Exams

1 Adda247 Publications For any detail, mail us at


Publications@adda247.com
Cracker Book for Bank (IBPS | SBI | RRB PO | Clerk) Mains Exams

Chapter
Time & Work and Pipe Cistern
7
BEST APPROACH TO SOLVE THE QUESTIONS

• Questions asked from this topic generally to test mental ability of a


student calculate the time in which a man/team complete the given
work while efficiency is given.
• While doing the questions from this topic, a most common approach
that student must use is that
‘if a man completes 1 unit of work in N days, then amount/part of
work done by him in 1 day is 1/𝑁,’
Assume that work of man is to eat 100 chocolates and he eats all of
them in 10 days, then amount of work done (chocolates eaten) in 1
100
day is = 10 𝑐ℎ𝑜𝑐𝑜𝑙𝑎𝑡𝑒𝑠 𝑒𝑎𝑡𝑒𝑛.
10
Or
If Rashmi can make 10 cakes in 5 hours, then amount of work done
10
(Cake made) by her in 1 hour is = 2 cakes.
5
Another easy way to solve questions from this topic is % approach i.e
‘if a man can complete a work in 20 days, then amount of work done
100
by him in 1 day is % = 5%.
20
Consider an example: Rahul can complete a work in 10 days and Arun
can complete the same work in 20 days. Find the time in which the
work will be completed if both of them work together.
There are three approaches to solve this:

(i) TRADITIONAL APPROACH


If Rahul can complete the work in 10 days, then amount of work
done by him in 1 day
1
= 𝑢𝑛𝑖𝑡𝑠.
10
Similarly amount of work done by Arun in 1 day
1
= 𝑢𝑛𝑖𝑡𝑠.
20
Hence when both of them work together, total work done is
2 Adda247 Publications For any detail, mail us at
Publications@adda247.com
Cracker Book for Bank (IBPS | SBI | RRB PO | Clerk) Mains Exams
1 1 3
+ = 𝑢𝑛𝑖𝑡𝑠.
10 20 20
3
Hence if 𝑢𝑛𝑖𝑡𝑠 𝑜𝑓 𝑤𝑜𝑟𝑘 𝑖𝑠 𝑑𝑜𝑛𝑒 𝑖𝑛 1 𝑑𝑎𝑦
20
Then 1 unit of work will be completed in 20/3 days.

(ii) LCM APPROACH


In this method, assume the LCM of days as a number of
chocolates. LCM of 10 and 20 is 20. Now assume that there were
20 chocolates, and If Rahul take 10 days to eat them all, it can be
concluded that he ate 2 chocolates per day. Similarly, Arun can eat
1 chocolate in a day.
Therefore,
They both will eat 3 chocolates in 1 day.
20
Time taken to complete whole work(to eat all of them)= 𝑑𝑎𝑦𝑠.
3
NOTE: Our answer will be same, we consider any number of
chocolates, but for easy calculation purpose we use a number that
is easily divisible by number of days given.

(iii) % APPROACH
This method is kind of same as (i). Consider the work unit as
100% work.
Now note that if Rahul take 10 days to complete 100% of work,
then work done by him in 1 day is 10%. Similarly work done by
Arun in 1 day is 5%. Therefore, both of them working together
will complete 15% of work in 1 day.
100 20
100% of work will be completed in = 𝑑𝑎𝑦𝑠.
15 3

In all these approaches, 1 common thing that occurs is to calculate


amount/part/percentage of work done in 1 day.
LCM approach is best used in the questions in which efficiency of pipe or
volume of tank is given. Consider eg.

Example 1: Pipe A and Pipe B when both opened together can fill a tank in
4 minutes. Pipe A alone can fill it in 16 minutes. Find the time in which pipe
B alone will fill the tank.

3 Adda247 Publications For any detail, mail us at


Publications@adda247.com
Cracker Book for Bank (IBPS | SBI | RRB PO | Clerk) Mains Exams

In traditional way:
1
Sol. Tank filled by Pipe A alone in 1 minutes = 𝑢𝑛𝑖𝑡𝑠.
16
Let pipe B alone can fill it in x minutes.
1
Then tank filled by pipe B alone 1 minute = 𝑢𝑛𝑖𝑡.
𝑥
1 1 1
Therefore, + =
𝑥 16 4
1 3 16
= , 𝑥=
𝑥 16 3

LCM APPROACH
Let total units of tank are 16 liters. It means A can fill 1 liter in a minute.
Assume that B fills x unit in a minute. Tank was filled in 4 minutes’ means
4(𝑥 + 1) = 16
𝑥+1 = 4
x = 3 units per minute
16
time taken is minutes.
3
All this could be done was mentally through LCM method. Give it a try!
MAN-DAYS-HOUR-WORK THEOREM
According to this theorem if more than 1 man of equal efficiency are
working together on a same work, then amount/part of work done by them
is directly proportional to the time given to work by number of man. In
more simple words work done depends on efficiency and total time given.
𝐦 𝐝 𝐡 𝐦 𝐝 𝐡
⇒ 𝟏 𝟏 𝟏= 𝟐 𝟐 𝟐
𝐰𝟏 𝐰𝟐
Here m₁, m₂ are number of men
d₁, d₂ are number of days
h₁, h₂ are working hours in given days
w₁, w₂ are part/amount of work done.
For eg. 10 men in 5 days can do same amount of work as 5 men in 10 days
or if we explore it more we can see 10 men in 6 days working 4 hours a day
can do same amount of work as 30 men in 4 days working 2 hours a day.
Please verify above two examples, by assuming a man can eat 1 chocolate in
1 hour and try to solve given problem.

Example 2: 12 Men working 4 hours a day can complete 48% of work in 8


days. Calculate how many men are required to complete 2 units of a work, if
they have to complete it in 10 days, 8 hours a day.
4 Adda247 Publications For any detail, mail us at
Publications@adda247.com
Cracker Book for Bank (IBPS | SBI | RRB PO | Clerk) Mains Exams

Sol. In this type of problems, by default we have to assume that all men
have equal efficiency and they do the work with uniform speed.
Applying Man-day-hour formulae
m1 d1 h1 m2 d2 h2
=
w1 w2
Here,
m₁ = 12 m₂ = ?
d₁ = 8 d₂ = 10
h₁ = 4 h₂ = 8
w₁ = 48 w₂ = 200

Note that, here w₂ = 200 is used because, 2 units of work means 200%
of work.
12×8×4×200
m2 =
48×8×10
m2 = 20

Another type of concept that is introduced here is when people with


unequal efficiencies are working, for eg. 4 women can do the same amount
of work in a given time as 3 men. For these type of questions, we use ratio
approach, we simply write
4W= 3M
𝑊 3
=> = .
𝑀 4

And then we consider efficiency of a men as 4 units and that of women as 3


units throughout the problem.

Few points to remember:


If the ratio of efficiencies of two persons is x:y then ratio of respective time
taken by them is y:x.
If X is N times efficient than Y, time taken by X to complete a work is
1
𝑡𝑖𝑚𝑒𝑠 𝑜𝑓 time taken by Y.
𝑁
Total work done by a man of efficiency X in N days is 𝑁𝑋 units.
5 Adda247 Publications For any detail, mail us at
Publications@adda247.com
Cracker Book for Bank (IBPS | SBI | RRB PO | Clerk) Mains Exams

Practice Exercise Based on New Pattern

1. Ratio between efficiency of Arun, Yash and Rana is 6 : 4 : 5. All three


starts to work together with same efficiency. But Rana, destroys his
60% of work in every evening, due to which they have to work 20
more days then estimated time. Find the estimated days by them to
complete the work.
(a) 80 (b) 76 (c) 84
(d) 72 (e) 90

2. Pipe P can fill a tank in 24 minutes, pipe Q can fill the same tank in 36
minutes and an outlet pipe can empty the fully filled tank in 48
minutes. Find in how many minutes tank will be filled, if for first
minute only pipe P & Q opened together and in second minute all
three pipes opened in tank alternatively?
4 4 4
(a) 15 𝑚𝑖𝑛𝑢𝑡𝑒𝑠 (b)13 𝑚𝑖𝑛𝑢𝑡𝑒𝑠 (c)18 𝑚𝑖𝑛𝑢𝑡𝑒𝑠
5 5 5
4 4
(d) 16 𝑚𝑖𝑛𝑢𝑡𝑒𝑠 (e)12 𝑚𝑖𝑛𝑢𝑡𝑒𝑠
5 5

Direction (3-4): One day earning of P, Q and R together is Rs.1026 to do a


work. ‘P’ earns more than R which is same as ‘R’ earns more than ‘Q’.
Efficiency of R and Q is 19 : 18.

3. Find the amount earned by R and P together in 5 days to do the same


work?
(a) Rs. 3620 (b) Rs. 3430 (c) Rs. 3510
(d) Rs. 3310 (e) Rs. 3710

4. S, who’s efficiency is average of efficiency of P and Q, can complete a


work in 36 days. If P, Q and R work together, and complete that work
then find the total wage of Q
(a) 3888 Rs. (b) 4104 Rs. (c) 4320 Rs.
(d) 3666 Rs. (e) 4520 Rs.

6 Adda247 Publications For any detail, mail us at


Publications@adda247.com
Cracker Book for Bank (IBPS | SBI | RRB PO | Clerk) Mains Exams

5. (X+4) men can complete a work in 2X days while (X+12) women can
complete same work in (X+8) days. If ratio of efficiency of men to
women is 5 : 4 then find in how many days 12 men and 15 women
together can complete the same work?
(a) 32 days (b) 16 days (c) 48 days
(d) 64 days (e) 80 days

6. A cistern can be filled completely by pipe A and B together in 12


hours. If pipe A works with twice speed while pipe B work with 50%
more speed than cistern can be filled completely in 7 hours. Find the
capacity of cistern if flow of water through pipe A is 2.5ℓ/ minute.
(a) 2800 liter (b) 3150 liter (c) 3300 liter
(d) 3650 liter (e) 4200 liter

7. Ratio of efficiency of A and B in completing a work is 3 : 4. Both


started to work together but A left after 2 days. Another person C
joins B and they together complete the remaining work in 6 days. If A
and B together can complete the work in 8 days then C alone can
complete the work.
27 56 41
(a) days (b) days (c) days
4 3 3
28 49
(d) days (e) days
3 3

8. Pipe A can fill a tank in 45 hr, pipe B is 50% more efficient than A and
pipe C can fill the same tank in 7.5 hr less than B. A and B opened
together for X hr and closed after that and pipe C fill remaining tank in
(X + 9) hr, if the ratio between tank filled by (A + B) together to tank
filled by pipe C is 1 : 2. Find the value of X ?
(a) 3 hr (b) 4 hr (c) 6 hr
(d) 8 hr (e) 7 hr

9. A and B can do a piece of work in 72 days and 64 days respectively. C


2
can do the same work in 2 more days as A & B take together to
17
complete. If first day A & B work together and second day B & C work
together alternatively, then in how many days work will be
completed?
13 13 13
(a) 22 𝑑𝑎𝑦𝑠 (b) 27 𝑑𝑎𝑦𝑠 (c) 32 𝑑𝑎𝑦𝑠
25 25 25
13 13
(d) 25 𝑑𝑎𝑦𝑠 (e) 29 𝑑𝑎𝑦𝑠
25 25
7 Adda247 Publications For any detail, mail us at
Publications@adda247.com
Cracker Book for Bank (IBPS | SBI | RRB PO | Clerk) Mains Exams

10. Four persons started to do a work together. ‘A’ works only in starting
two days after that B, C and D works alternately starting from B. Ratio
of time taken by A, B, C and D if they work alone is 4 : 3 : 2 : 5. If the
work is completed in 12 days then in how many days A and C can
complete the work if they work together ?
(a) 6 days (b) 12 days (c) 10 days
(d) 8 days (e) 4 days

11. A person C can complete 21% of work in 10 days while working with
1
233⅓% of his efficiency. B is 11 % more efficient than C. A, while
9
working with his half efficiency can complete the work in half time as
compared to time taken by B. Find the time taken by A & B together to
complete the 50% of whole work.
(a) 15 days (b) 10 days (c) 20 days
(d) 25 days (e) 22 days

12. Two pipes A and B can fill a cistern in 15 hours and 10 hours
respectively. A tap can empty the full cistern in 30 hours. All the three
taps were open for 2 hours, when it was remembered that the
emptying tap had been left open. It was then closed. How many hours
more would it take for the cistern to be filled ?
(a) 4hr 30 min. (b) 4hr 12 min. (c) 4 hr24 min.
(d) 4hr 35 min. (e) 4hr 54 min.

Directions (13-15): ‘B’ and ‘C’ together started to do a work ‘X’. After 6
days ‘B’ is replaced by ‘A’ who can do work ‘X’ alone in same time in which
‘B’ and ‘C’ together can do. After 4 days more ‘A’ left the work and
1
remaining work is completed by ‘C’ alone in 2 days. ‘B’ did 33 % of work ‘X’
3
in total. ‘C’ and ‘D’ can complete the same work ‘X’ in 20 days while ‘D’ is
60% less efficient than ‘B’.
Efficiency of A, B, C and D remain same for work ‘X’, ‘Y’ and ‘Z’

13. Find the ratio between work done by ‘A’ in 4 days to work done by ‘B’
in 9 days?
(a) 3 ∶ 4 (b) 1 ∶ 2 (c) 1 ∶ 3
(d) 2 ∶ 3 (e) 3 ∶ 1
8 Adda247 Publications For any detail, mail us at
Publications@adda247.com
Cracker Book for Bank (IBPS | SBI | RRB PO | Clerk) Mains Exams

14. Find in how many days ‘A’ can complete another work ‘Y’ if ‘C’ and ‘D’
together can complete work ‘Y’ in 26 days? (2 Marks)
(a) 14.4 days (b) 15.6 days (c) 18.2 days
(d) 16.6 days (e) 19.5 days

15. ‘E’ who is 60% more efficient than ‘B’ can complete another work ‘Z’
1
in 12 days. Find in how many days ‘A’ and ‘C’ together can complete
2
work ‘Z’? (2 Marks)
(a) 20 days (b) 15 days (c) 12 days
(d) 10 days (e) 8 days
1
16. Veer and Sameer alone can do 25% & 33 % of a task in 16 days
3
individually. If Sameer & Satish together can do same task in 16 days,
then find in how many days Veer, Sameer and Satish will complete the
whole task, if they do it on alternate days, starting with Satish and
followed by Sameer & Veer respectively?
(a)38 days (b)42 days (c)36 days
(d)32 days (e)30 days

17. Working alone, A can complete a task in ‘a’ days and B in ‘b’ days. They
take turns in doing the task with each working 2 days at a time. If A
starts they finish the task in exactly 10 days. If B starts, they take half
a day more. How long does it take to complete the task if they both
work together?
1 1 5
(a) 5 days (b) 5 days (c) 5 days
3 7 9
5
(d) 5 days (e) None of these
11

18. Monika can do 40% of a work in 24 days and Anshika can do 12.5% of
same work in 4.5 days. Shikha can complete the same work in equal
time as Monika and Anshika can do together. If all three starts work
alternatively, starting with Shikha and followed by Monika and
Anshika respectively, then find how many days will be required to
complete the work?
1 1 1
(a) 30 𝑑𝑎𝑦𝑠 (b) 24 𝑑𝑎𝑦𝑠 (c) 33 𝑑𝑎𝑦𝑠
3 3 2
1 1
(d) 28 𝑑𝑎𝑦𝑠 (e) 20 𝑑𝑎𝑦𝑠
3 3

9 Adda247 Publications For any detail, mail us at


Publications@adda247.com
Cracker Book for Bank (IBPS | SBI | RRB PO | Clerk) Mains Exams

19. One day efficiency of A is 20% more than B and that of C is 20% less
than B. D can do a piece of work in twice of time as A, B & C complete
same work in together. If first eight days D work alone after that he
left the work and A, B & C complete the remaining work in 12 days.
Find in what time work will be completed, if B and D work in rotation
on each day, starting with D?
1 1 1
(a) 41 𝑑𝑎𝑦𝑠 (b) 38 𝑑𝑎𝑦𝑠 (c) 36 𝑑𝑎𝑦𝑠
3 3 3
1 1
(d) 44 𝑑𝑎𝑦𝑠 (e) 46 𝑑𝑎𝑦𝑠
3 3

20. 5 inlet pipes (same capacity) can fill a tank in same time in which 3
outlet pipes (same capacity) can empty it. If 2 inlet and one outlet pipe
is opened for first minutes and 5 inlet and 2 outlet pipe open for 2 nd
minute and process continues till tank is completely filled in 30
minutes. Find the time in which 2 outlet pipe can empty the
completely filled tank.
(a) 9 minute (b) 12 minutes (c) 6 minutes
(d) 10 minutes (e) 15 minutes

Direction (21-22): P, Q, R and S four pipes can fill a cistern ‘N’ in 7 minutes
if all are opened together. P is 25% less efficient then Q which is 100%
more efferent than R. ‘S’ filled 25% of the cistern. P and Q together can fill
the cistern in ‘x’ minutes while P and S can fill half cistern in ‘y’ minutes.

21. Pipe A and B together can fill another cistern ‘M’ in (x+12) minutes
while pipe B and C together can fill cistern ‘M’ in (y+5) minutes. If pipe
6𝑥
A, B and C together can fill cistern ‘M’ in ( ) minutes, then find
𝑦
efficiency of ‘B’ is what % more/less than efficiency of ‘A’ ?
(a) 50% (b) 75% (c) 100%
(d) 150% (e) 200%

22. ‘D’ men can complete a work in (x+9) day while (y+7) men can
complete same work in ‘E’ days. If y men can complete same work in
(E+12) days then find in how many day (E–D) men can complete the
same work?
(a) 21 days (b) 42 days (c) 35 days
(d) 14 days (e) 84 days
10 Adda247 Publications For any detail, mail us at
Publications@adda247.com
Cracker Book for Bank (IBPS | SBI | RRB PO | Clerk) Mains Exams

Directions (23-24): These questions are based on the information given


below.
Each of A, B, C and D need a unique time to do a certain work. A can do the
work in 𝑥 days and B can do the work in 2𝑥 days. A started the work and do
2
it for 22 days then he is replaced by B and B completed remaining work in
9
same time as C and D together can complete the whole work.
The ratio of the efficiency of C and D is 4 : 5. If C and D work for alternative
1
days starting from C then they can do the total work in 44 days.
2

23. What is the value of 𝑥?


2 1 2
(a) 66 (b) 33 (c) 16
3 3 3
2
(d) 14 (e) none of these
7

24. A and B together can compete 225% of the work in how many days?
2
(a) 66 days (b) 60 days (c) 50 days
3
(d) 25 days (e) none of these

1
25. P can do 50% of a work in 32 days, Q can do 37 % of same work in
2
1
27 days, while R can do 62 %of same work in 30 days. Q and R
2

together started and worked for x days. After x days Q left the work
and P joined R and both completed remaining work in (x +4) days. If
ratio between work done by (Q + R) together to work done by (P + R)
is 5 : 7 then in 2x days R will complete what fraction of work?
1 1 2
(a) (b) (c)
3 4 3
1 1
(d) (e)
2 5

11 Adda247 Publications For any detail, mail us at


Publications@adda247.com
Cracker Book for Bank (IBPS | SBI | RRB PO | Clerk) Mains Exams

26. Veer and Sameer can do a work in 64 days and 72 days respectively.
Satish and Ankit together take 48 days to complete the same work.
Ankit is 40% more efficient than Satish. If Veer and Sameer work for
18 days together, after that both left the work, then find in how many
days remaining work will be completed by Satish and Ankit if they
work on alternated days and Ankit starts the work?
6 6 6
(a) 40 𝑑𝑎𝑦𝑠 (b) 42 days (c) 48 𝑑𝑎𝑦𝑠
7 7 7
6 4
(d) 44 𝑑𝑎𝑦𝑠 (e) 49 𝑑𝑎𝑦𝑠
7 5

27. Veer takes 50% more time to complete a task than that of Satish takes
2
to complete the same task, while Mahendra takes 2 days less than to
5
complete same task as Veer and Satish takes together. If Satish
1
complete 𝑟𝑑 of task in 8 day, then find in how many days work will
3
be completed, if Veer and Mahendra work in rotation, starting with
Mahendra?
(a) 16 days (b) 18 Days (c) 24 days
(d) 12 days (e) 22 days

28. There are 3 pipes A, B and C. A can fill a bucket in 8 minutes. C can fill
18 3
buckets in 18 minutes & pipe B can fill buckets in a minute. These
5 20
pipes are opened in a tank alternatively, 1 minute each starting from
A, then C, then B. If tank is filled after 2 hours, then find the volume of
tank if the capacity of bucket is 5 liters.
(a) 88 liters (b) 95 liters (c) 90 liters
(d) 105 liters (e) 92 liters

29. Efficiency of Ram is 25% more than Shyam who completes a task in
2
60 days. Ghanshyam takes 6 days less than the days taken by Ram
3
and Shyam together to complete the work. If Ram and Shyam work for
16 days, after that both left the task then find in how many days
Ghanshyam will complete remaining work?
(a) 6 days (b) 8 days (c) 4 days
(d) 5 days (e) 10 days

12 Adda247 Publications For any detail, mail us at


Publications@adda247.com
Cracker Book for Bank (IBPS | SBI | RRB PO | Clerk) Mains Exams
1
30. A child can do th of work in same time as his father can do the whole
5
work and efficiency of child’s mother is equal to half the sum of father
and child efficiency. If they complete 70% of work in 24 days, when
they work alternatively, starting with father and child together on
first day followed by father and mother together on second day. Find
in how many days mother can complete 85% of work alone?
(a) 48 days (b) 68 days (c) 60 days
(d) 56 days (e) 54 days

4
31. Ankit and Apoorv together can finish a work in 8 𝑑𝑎𝑦𝑠 while Ankit
7
can do it individually in 15 days. Both work on another task for (x + 4)
90
days and (x + 20) days respectively. If remaining % of the second
7
task is completed by Bhavya in 18 more days with the efficiency of 2
units work/day. Then, find what portion of the second task will be
completed in x days, if all three work together?
29 26 31
(a) (b) (c)
35 35 35
23 27
(d) (e)
35 35

32. One day efficiency of Raj is equal to one day efficiency of Divyaraj and
Veer together. Divyaraj is 25% more efficient than Veer and starts
with 20% more of his efficiency with Veer and works for x days, after
that Divyaraj left the work and remaining work will be completed by
Veer and Sameer together in (x + 8) days. If ratio of work done by
Divyaraj and Veer together to work done by Veer and Sameer is 5 : 7
and efficiency of Sameer is 25% less than Veer. Then find Raj and
Sameer complete work together in how many days?
(a) 14 days (b) 12 days (c) 16 days
(d) 18 days (e) 20 days

13 Adda247 Publications For any detail, mail us at


Publications@adda247.com
Cracker Book for Bank (IBPS | SBI | RRB PO | Clerk) Mains Exams

33. Two pipes P and Q can fill tank A in 28 minutes and 56 minutes
respectively and empty pipe M can empty same tank in 42 minutes. If
all three pipes opened in tank B for (x – 24) minutes together they
filled 90 liter of the tank which is 25% of the quantity tank B. Find in
‘x’ minutes what portion of tank B filled, if all pipe P and Q and M
opened alternatively starting with P, followed by Q and M
respectively?
7 7 5
(a) (b) (c)
12 36 36
3 7
(d) (e)
35 41

34. There are two pipes A & B, pipe A is for filling the swimming pool and
pipe B is to empty the swimming pool. Capacity of swimming pool is
5040 m3 and volume of pipe B is 8 m3/minute more than that of pipe
1
A. If pipe A takes 11 more minutes to fill same swimming pool, than
4
time taken by B to empty the same swimming pool. If pipe B can
empty second swimming pool in 112.5 minutes, then find the capacity
of second swimming pool?
(a) 7200 m3 (b) 6400 m3 (c) 5600 m3
(d) 7800 m3 (e) 8400 m3

35. Two pipes P and Q can fill tank A in 28 minutes and 56 minutes
respectively and empty pipe M can empty the tank in 42 minutes.
Tank A have the capacity of 168 liters. If all three pipes opened in tank
B for (x – 24) minutes together they filled 90 liter of the tank which is
25% of the quantity of tank B. Find in x minutes what portion of tank
B filled, if all pipe P and Q and M opened alternatively in each minute
starting with P, followed by Q and M respectively?
5 7 9
(a) (b) (c)
36 36 38
7 7
(d) (e)
38 39

14 Adda247 Publications For any detail, mail us at


Publications@adda247.com
Cracker Book for Bank (IBPS | SBI | RRB PO | Clerk) Mains Exams

Directions (36-38): Working efficiency of A is 20% more than that of B. B


can complete a work ‘X’ in 36 days.
B and C together started to complete the work ‘X’ and after 10 days they
both left the work and then remaining work is done by A alone in 15 days.
A and C together started to complete another work ‘Y’ and after working for
12 days they both left the work. Remaining work is done by B alone in 16
days. D first completed work ‘X’ and then completed work ‘Y’ in total 38
days.
It is given that efficiency of all, in completing work ‘X’ and work ‘Y’ is same.
1
36. A, B and C working together completed rd of work ‘X’, and then A and
3
C are replaced by D. Now remaining of work ‘X’ is completed by B and
D together. For how many days B worked?
(a) 12 days (b) 10 days (c) 15 days
(d) 4 days (e) 8 days

37. A, C and D working simultaneously completed work ‘X’ in ‘n’ days and
A, B, C and D working simultaneously completed work ‘Y’ in ‘m’ days.
Find the value of (𝑚 + 𝑛).
(a) 15 days (b) 10 days (c) 12 days
(d) 8 days (e) 17 days

38. A person E starts the work ‘X’ and leave after 12 days, then B and C
complete the remaining work in 8 days. What is the ratio of number of
days taken by A and E together to complete the work ‘X’ to the
number of days taken by D, B and C together to complete the both
work ‘X’ and ‘Y’ .
(a) 3 : 5 (b) 5 : 3 (c) 8 : 7
(d) 1 : 2 (e) 2 : 3

Directions (39-40): Ankur & Shubham can do a piece of work 64 days


together and Shubham &Kartik can do the same work in 72 days together.
First 22 days Ankur & Shubham work together, after that Ankur left the
work and Kartik joined Shumbham. Kartik and Shubham work for next six
days, after that Shubham left the work and remaining work complete by
Kartik & Anup in 30 more days. Efficiency of Anup is 75% more than
efficiency of Kartik.
15 Adda247 Publications For any detail, mail us at
Publications@adda247.com
Cracker Book for Bank (IBPS | SBI | RRB PO | Clerk) Mains Exams

39. Kartik and Anup work for x days, Ankur and Shubham work for next
2 1
(x+2 ) days, If remaining 12 % 𝑜𝑓 𝑡𝑜𝑡𝑎𝑙 work complete by Prabhat
3 2
in 6 more day. Prabhat will do, what portion of total work in (x+12)
days?
2 1 3
(a) (b) (c)
3 4 4
2 2
(d) (e)
3 5

40. Ankur and Shubham start work alternatively and work for y days.
𝑦
After that Kartik and Anup replace them and work for next days
2
75
alternatively. If % of total work still remain, which complete by
8
Abhimanyu in 9 more days. Find if Abhimanyu work for (y+16) days,
then in how many days remaining work will be completed by Anup
and kartik? (y is divisible by 4)
48 34
(a) days (b) days (c) 20 days
11 11

(d) 28 days (e) 32 days

16 Adda247 Publications For any detail, mail us at


Publications@adda247.com
Cracker Book for Bank (IBPS | SBI | RRB PO | Clerk) Mains Exams

Solutions

1. (a); Ratio of efficiency of Arun, Yash and Rana is 6 : 4 : 5


Total work done by them in 1 day
= (6 + 4 + 5) units = (15) units.
Let they estimate 𝑥 days to complete the work.
Then total work = 15𝑥
40
But Rana’s 1-day work is only 40%, i.e. × 5 = 2 unit
100
The work done by them in actual
= (6 + 4 + 2) (x + 20) = 12(x + 20)
∴ 15𝑥 = 12𝑥 + 240
3𝑥 = 240
𝑥 = 80
Hence, estimated days are 80.

2. (d);

First minutes (P + Q) = (6 + 4) = 10 unit


Second minutes (P + Q – R)
= (6 + 4 – 3) = 7 unit
17 unit in 2 minutes
So,
17 × 8 = 136 unit in 16 minutes
(144 –136)
Remaining by (P + Q) =
10
4
= minutes
5
4
Total time = 16 +
5
4
= 16 minutes
5

17 Adda247 Publications For any detail, mail us at


Publications@adda247.com
Cracker Book for Bank (IBPS | SBI | RRB PO | Clerk) Mains Exams

Solution (3-4): Let R earns = y


And P earns = y + x
So, Q earns = y – x
ATQ,
y – x + y + y + x = 1026
y = 342
Each person earns according to their efficiencies so
342
Q earns → × 18 = 324
19
342
An P earns = × 20 = 360
19
Ratio of efficiency of P, R and Q
= 20 : 19 : 18
Money per unit work = Rs.18

3. (c); Amount earned by R and P together in 5 days


Work done = (20 + 19) × 5 = 195
Money earned = 195 × 18 = Rs.3510
20+18
4. (a); Efficiency of S = = 19
2
Total work → 19 × 36
Days taken by P, Q and R to do that work
19×36
⇒ = 12 day
(20+19+18)
Q’s earning ⇒ 12 × 324 = Rs.3888

5. (b); (X+4) men can complete work in 2X days


⇒ 1 man can complete same work in 2X(X+4) days
(X+12) women can complete work in (X+8) days
⇒ 1 woman can complete same work in (X+8)(X+12) days
Now ratio of efficiency of men to women is 5 : 4
⇒ Ratio of days taken by men to women is
4:5
4 2X(X+4)
So, =
5 (X+8)(X+12)
2
⇒ 2(𝑋 + 20𝑋 + 96) = 5𝑋(𝑋 + 4)
⇒ 2𝑋 2 + 40𝑋 + 192 = 5𝑋 2 + 20𝑋
⇒ 3𝑋 2 − 20𝑋 − 192 = 0
⇒ 3𝑋 2 − 36𝑋 + 16𝑋 − 192 = 0
18 Adda247 Publications For any detail, mail us at
Publications@adda247.com
Cracker Book for Bank (IBPS | SBI | RRB PO | Clerk) Mains Exams

⇒ 3𝑋(𝑋 − 12) + 16(𝑋 − 12) = 0


⇒ (3𝑋 + 16)(𝑋 − 12) = 0
16
⇒ 𝑋 = 12, −
3
16×24
12 men can complete work in
12
= 32 𝑑𝑎𝑦𝑠
24×20
15 women can complete work in
15
= 32 𝑑𝑎𝑦𝑠
32×32
Required time = = 16 𝑑𝑎𝑦𝑠
32+32

6. (e); Let, pipe A and pipe B alone can fill the tank in x and y hours
respectively.
ATQ,
1 1 1
+ = … (i)
x y 12
2 1.5 1
and, + = … (ii)
x y 7
On solving (i) and (ii)
x = 28 hour ⇒ y = 21 hour
Capacity of cistern = 28 × 60 × 2.5
= 4200 liter

7. (b); Let A and B can do 3𝑥 and 4𝑥 unit of work in one day.


So,
Total work = (3𝑥 + 4𝑥 ) × 8 = 56𝑥
(A + B) two day work = 7𝑥 × 2 = 14𝑥
Remaining work = 42𝑥
In 6 days B will complete
= 6 × 4𝑥 = 24𝑥 units
So, remaining 18𝑥 units are completed by C in 6 day So,
56𝑥 unit will be completed in
56𝑥 56
= 18𝑥 = days
3
6

19 Adda247 Publications For any detail, mail us at


Publications@adda247.com
Cracker Book for Bank (IBPS | SBI | RRB PO | Clerk) Mains Exams

8. (c); A = 45 hr
A : B = 100 : 150 = 2 : 3
Total capacity of tank = 45 × 2 = 90 liter
90
C= – 7.5 = 22.5 hr
3
90 ℓ
C efficiency = =4
22.5 hr
5X 1
According to question ⇒ =
4(X+9) 2
⇒ 10X – 4X = 36
X = 6 hr

9. (b);

576 36 612
C takes = (8+9) + = = 36 days
17 7
576
Efficiency of C = = 16 units/days
367
When First day (A and B) and second day (B and C) work
alternatively
Two day work = (A and B) one day work and (B and C) one day
work
= (8 + 9) + (9 + 16) = 17 + 25
= 42 units
26
In 26 day = × 42 = 546 units
2
(A and B) on 27th day = 17 units
After 27 days remaining work
= (576 – 546 – 17) = 13 units
13
13 units work done by (B and C) on 28th day =
25
13 13
Total time = (27 + ) = 27 days
25 25

20 Adda247 Publications For any detail, mail us at


Publications@adda247.com
Cracker Book for Bank (IBPS | SBI | RRB PO | Clerk) Mains Exams

10. (d); A, B, C and D worked for 2 days together after that A leave and B,
C and D worked alternatively for 10 days starting from B
∴ B worked for 4 days, C for 3 days, and D for 3 days.
Total days A worked = 2
Total days B worked = 4 + 2 = 6
Total days C worked = 3 + 2 = 5
Total days D worked = 3 + 2 = 5
Let, their alone time to complete the work is 4x, 3x, 2x and 5x
respectively.
2 6 5 5
∴ + + + =1
4x 3x 2x 5x
30+120+150+60
⇒ =1
60𝑥
360
⇒𝑥= =6
60
‘A’ can complete the work in 4 × 6 = 24 days
‘C’ can complete the work in 2 × 6 = 12 days
12×24 12×24
Required time = = = 8 days
12+24 36

11. (b); Let the efficiency of C [is C units/days] total work is 100a Units
ATQ,
7
c × 10 = 21a
3
1 7
[233 % = ]
3 3
9𝑎
C= 𝑈𝑛𝑖𝑡𝑠 /𝑑𝑎𝑦
10
1
Also B is 11 % more efficient means
9
10
B= of C
9
10 9𝑎
B = × Units/day
9 10
= a Units / day
A which while working with half efficiency total half time.
∴ A : B = 4 : 1 [efficiency ratio]
Efficiency of A = 4a
Work done by then together is 4a + a
= 5a/day
Time take to complete 50 a Units
50𝑎
= = 10 days
5𝑎

21 Adda247 Publications For any detail, mail us at


Publications@adda247.com
Cracker Book for Bank (IBPS | SBI | RRB PO | Clerk) Mains Exams

12. (c);

In 2 hours, part of the tank filled


= 2(4) = 8 units
(30−8)
Now required time =
5
22
= = 4hour 24mins.
5

Solution (13-15): Let, efficiency of A, B, C and D be ‘a’, ‘b’, ‘c’ and ‘d’
respectively
Total work = 6b + 6c + 4(a +c) + 2c
And also, a = b + c
⇒ Total work = 10b + 16c
ATQ,
1
B did of work in 6 days
3
⇒ B can complete whole work in 18 days
And,
(10b + 16c) = 18b ⇒ 16c = 8b
𝑏 2
⇒ =
𝑐 1
⇒ ‘C’ can complete whole work ‘X’ in 36 days
18×36
A can complete whole work ‘X’ in = 12 day.
18+36
18
D can complete whole work ‘X’ in × 5 = 45 days.
2

Ratio of efficiency of A, B, C and D


1 1 1 1
= ∶ ∶ ∶
12 18 36 45

= 15 : 10 : 5 : 4

22 Adda247 Publications For any detail, mail us at


Publications@adda247.com
Cracker Book for Bank (IBPS | SBI | RRB PO | Clerk) Mains Exams
4×15 60 2
13. (d); Required ratio = = =
9×10 90 3

14. (b); Let efficiency of A, B, C and D be 15x, 10x, 5x and 4x respectively


Total work ‘Y’ = (5x + 4x) × 26
= 9x × 26 =234x
234𝑥
‘A’ can complete work ‘Y’ in = 15.6 day
15

15. (d); Efficiency of ‘E’ = 16x


25
Total work ‘Z’ = 16𝑥 × = 200x
2
‘A’ and ‘C’ together can complete work ‘Z’ in
200𝑥 200𝑥
= (15+5)𝑥 = = 10 days
20𝑥

16. (a); Veer can complete the whole task alone


= 16 × 4 = 64 𝑑𝑎𝑦𝑠
Sameer can complete the same task alone
= 16 × 3 = 48 𝑑𝑎𝑦𝑠
Total work = 192 𝑢𝑛𝑖𝑡𝑠(LCM of 64 and 48)
Efficiency of Satish
192 192
= − = 8 𝑢𝑛𝑖𝑡𝑠/𝑑𝑎𝑦
16 48
If all three work alternatively
First day by Satish = 8 units
Second day by Sameer = 4 units
Third day by Veer = 3 units
Total work in three days = 8 + 4 + 3
= 15 units
36
In total 36 days = × 15 = 180 𝑢𝑛𝑖𝑡𝑠
3
Satish on 37 days = 8 units
Remaining work after 37 days
= 192 – 180 – 8 = 4 units
On 38 days remaining work by Sameer
4
= = 1𝑑𝑎𝑦𝑠
4
Total time = 38 days

23 Adda247 Publications For any detail, mail us at


Publications@adda247.com
Cracker Book for Bank (IBPS | SBI | RRB PO | Clerk) Mains Exams

17. (b); If each works 2 days at a time alternately starting with A, the
work is completed in exactly 10 days.
∴ A works for 6 days and B worked for 4 days.
6 4
+ =1 ………….(i)
a b
If B starts, the work is completed in 10.5 days.
∴ B works for 6 days and A worked for 4.5 days.
6 4.5
+ =1 ………….(ii)
b a
By solving (i) and (ii)
a = 9 days
And, b = 12 days
Time taken by A and B working together to complete the work
1 1
=1 1 =1 1
+ +
a b 9 12
36 1
= = 5 days
7 7

18. (c); Monika can do whole work alone


24
= × 100 = 60 days
40
Anshika can do whole work alone
4.5
= × 100 = 36 days
12.5
Let Total work = 180 units (LCM of days taken by Anshika and
Monika)
180
Efficiency of Monika = = 3 units/day
60
180
Efficiency of Anshika = = 5 units/day
36
Shikha take to complete whole work alone
180
= (3+5) = 22.5 days
Efficiency of Shikha = 8 units/day
when Shikha, Monika & Anshika work alternatively—
Work done on First day by Shikha = 8 units
Work done on First day by Monika = 3 units
Work done on First day by Anshika
= 5 units
In three days = (8 + 3 + 5) = 16 units

24 Adda247 Publications For any detail, mail us at


Publications@adda247.com
Cracker Book for Bank (IBPS | SBI | RRB PO | Clerk) Mains Exams
33
In total 33 days = × 16 = 176 units
3
Remaining work complete by Shikha
(180 –176) 1
= = days
8 2
1
Total time = 33 +
2
1
= 33 days
2

19. (b); Let efficiency of B = 100


So, efficiency of A = 120
80
And efficiency of C = 100 × = 80
100
100+120+80
Efficiency of D = = 150
2
Ratio of efficiency of A, B, C and D
= 6 : 5 : 4 : 7.5
Let one day work of A , B , C & D be 6x units, 5x units, 4x units &
7.5x units respectively
ATQ –
Total work
= 7.5x × 8 + (5𝑥 + 6𝑥 + 4𝑥) × 12
= 240x units
When B & D work alternatively
First day by D = 7.5x units
Second day be B = 5x units
Two day work of = 7.5x + 5x = 12.5x units
In 38 days total work
38
= × 12.5𝑥 = 237.5𝑥 units
2
Remaining work = 240x – 237.5x = 2.5x
2.5𝑥
Remaining work by D on 39 days =
7.5𝑥
1
= 𝑑𝑎𝑦𝑠
3
1
Total time = 38 𝑑𝑎𝑦𝑠
3

25 Adda247 Publications For any detail, mail us at


Publications@adda247.com
Cracker Book for Bank (IBPS | SBI | RRB PO | Clerk) Mains Exams

20. (a); Let efficiency of one inlet pipe = 3x


Efficiency of 5 inlet pipes = 5 × 3x = 15x
15x
Efficiency of 1 outlet pipe = = –5x
3
ATQ,
15 (2 × 3x + 1 × (–5x)) + 15 (5 × 3x + 2 × (–5x)) = Tank capacity
Tank capacity = 90x
90x
Required time = = 9 minutes
10x

Solution (21-22): Let efficiency of R is 2 units/ minute


⇒ Efficiency of Q is 4 units/ minute
So Efficiency of P is 3 units/ minute
P + Q + R together completed 75% of work
⇒ 75% of work = 9 units
⇒ 100% of work = 12 units
Efficiency of S is 3 units/minute
Total work = 7 × (12) = 84 unit
Number of days taken by P, Q, R and S individually: -

28×21
P and Q together can fill the cistern =
28+21
28×21
= ⇒ x = 12 min.
49
28×28
P & S together can fill half cistern =
28+28
y = 7 min.
26 Adda247 Publications For any detail, mail us at
Publications@adda247.com
Cracker Book for Bank (IBPS | SBI | RRB PO | Clerk) Mains Exams

21. (c); A and B together can fill cistern ‘M’ in (x+12) minutes
= 12+12=24 minutes
B and C together can fill cistern ‘M’ in (y+5) minutes
= 7 + 5 = 12minutes
6𝑥
A, B and C together can fill cistern ‘M’ in ( ) minutes
𝑦
6×12 72
= = minutes
7 7

A’s efficiency = 49 – 42 = 7 units/minute


C’s efficiency = 49 – 21 = 28 units/ minute
B’s efficiency = 49 – 7 – 28
= 14 units/minute
14−7
Required % = × 100
7
7
= × 100 = 100%
7

22. (b); ATQ,


D × (12+9) = (7+7) × E
3
⇒D× =𝐸
2
And (7+7) × E = 7 × (E+12)
⇒ 2E = E + 12
⇒ E = 12
⇒D=8
Let (E – D) men can complete same work in ‘a’ days
ATQ,
(12−8) × a = 8 × (12+9)
a = 2 × 21 = 42 days

27 Adda247 Publications For any detail, mail us at


Publications@adda247.com
Cracker Book for Bank (IBPS | SBI | RRB PO | Clerk) Mains Exams

23. (b); Let the C do in one day = 4𝑦 work


Let the D do in one day = 5𝑦 work
2 day work of C + D = 9𝑦
In 44 day they will complete 9𝑦 × 22
= 198𝑦
1
In another days 2𝑦 work will be done
2
200y
C will take = days = 50 day
4y
D will take = 40 days.
C and D will complete work together in
50×40 200
= =
90 9
According to condition
200 200
+ =1
9𝑥 9 × 2𝑥
400+200
=1
18𝑥
1
⇒ 𝑥 = 33
3

1
24. (c); A – 33 𝑑𝑎𝑦𝑠
3
2
B − 66 days
3
200
(A+𝐵)− 𝑑𝑎𝑦𝑠
9
200 9
Required Time − × = 50 𝑑𝑎𝑦𝑠
9 4

25. (d); P can do whole work alone = 64 days


Q can do whole work alone
27
= × 8 = 72 days
3
R can do whole work alone
30
= × 8 = 48 days
5

28 Adda247 Publications For any detail, mail us at


Publications@adda247.com
Cracker Book for Bank (IBPS | SBI | RRB PO | Clerk) Mains Exams

ATQ—
(8+12)𝑥 5
(9+12)(𝑥+4)
=
7
140𝑥 = 105𝑥 + 420
420
𝑥=
35
𝑥 = 12 days
In 2𝑥 days work done by R = 2 × 12 × 12
= 288 units
288 1
Required portion = =
576 2

26. (d);

Let efficiency of Satish is 5x unit/day and that of Ankit is 7x


unit/day
ATQ—
5x + 7x = 12 unit
x = 1 unit/day
Efficiency of Satish = 5 unit/day
Efficiency of Ankit = 7 unit/day
Work done by Veer and Sameer in 18 days
= (9 + 8) × 18 = 306 unit
Remaining work = 576 – 306Ru = 270 unit
When Ankit and Satish work alternatively
Then two days’ work = (7 + 5) = 12 units
44
In 44 days total work done = × 12
2
= 264 unit
(270 –264) 6
On 45th days remaining work done by Ankit = = days
7 7
6
Total time = 44 days
7

29 Adda247 Publications For any detail, mail us at


Publications@adda247.com
Cracker Book for Bank (IBPS | SBI | RRB PO | Clerk) Mains Exams

27. (b); Let Veer take 3𝑥 day and Satish take 2𝑥 days to complete the
task
So, efficiency of Veer and Satish be 2𝑥 units/day & 3𝑥 units/day
respectively.
Whole work completed by Satish in 24 days
24
So, while task done by Veer alone = × 3𝑥 = 36 days
2𝑥
Total task = 72𝑥 units
72𝑥
Veer and Satish do it together =
5𝑥
2
= 14 days
5
Mahendra do same task alone
72 12
= – = 12 days
5 5
72𝑥
Efficiency of Mahendra = = 6𝑥 unit/day
12
When Veer and Mahendra do it alternatively —
First day task = 6𝑥 unit
Second day task = 2𝑥 units
In two day = 8𝑥 unit
72𝑥
Task completed = × 2 = 18 days.
8𝑥

28. (b); When these pipes are opened for two hours, means each of them
open for 40 min. individually.
Pipe A can fill 1 bucket in 8 minutes
& 5 buckets in 40 minutes …(i)
18×40
Similarly pipe C can fill
5×18
= 8 bucket in 40 min. …(ii)
3
And Pipe B can fill × 40 = 6 buckets in 40 minutes …(iii)
20
Hence volume of tank = 5 + 6 + 8
= 19 buckets
= 19 × 5
= 95 liters

30 Adda247 Publications For any detail, mail us at


Publications@adda247.com
Cracker Book for Bank (IBPS | SBI | RRB PO | Clerk) Mains Exams

29. (b); Efficiency of Ram :Shyam = 125 : 100


=5:4
60
Ram takes = × 4= 48 days
5
Let total work = 60×4= 240 unit
Ram and Shyam takes together
240 80
= = days
(5+4) 3
80 20
Ghanshyam takes = − = 20 days
3 3
240
Efficiency of Ghanshyam = = 12 unit/day
20
Rams and Shyam 16 days work
16 × (5 + 4)= 144 unit
240−144
Remaining work by Ghanshyam =
12
96
= = 8 days
12

30. (b); Let efficiency of child and father be x unit/day and 5x unit/day
x+5x
Efficiency of mother =
2
= 3x unit⁄day
ATQ,
They start work alternatively—
First day efficiency =(5x + x) = 6x
Second day efficiency= (5x + 3x) = 8x
6x×12+8x×12
Total work = × 100
70
168x
= × 100
70
= 240x
Mother complete 85% of work in
85
240x×100
=
3x
204x
=
3x
= 68 days

31 Adda247 Publications For any detail, mail us at


Publications@adda247.com
Cracker Book for Bank (IBPS | SBI | RRB PO | Clerk) Mains Exams
36×100×7
31. (e); Total work = = 280 units
90
ATQ -
4 (x + 4) + 3 (x + 20) + 18 × 2 = 280
4x + 16 + 3x + 60 = 280 – 36
7x = 168
x = 24 days
They all together in 24 days
= 24 (4 + 3 + 2)
= 216 units
216
Required portion =
280
27
=
35

32. (c); Ratio of efficiency of Divyaraj : Veer


= 125 : 100
=5:4
3
Efficiency of Sameer = 4 ×
4
= 3 units/day
ATQ,
120
(5× +4)x 5
100
=
(4+3)(x+8) 7
10x 5
=
7x+56 7
70x – 35x = 280
x = 8 days
Total work = 10 × 8 + 7 × 16
= 80 + 112
= 192
Raj efficiency = 9 units/day
Sameer and Raj efficiency = (3 + 9)
= 12 units/day
192
(Sameer + Raj) =
12
= 16 days
32 Adda247 Publications For any detail, mail us at
Publications@adda247.com
Cracker Book for Bank (IBPS | SBI | RRB PO | Clerk) Mains Exams

33. (b); ATQ


For tank A —

ATQ
For tank B —
6 (x – 24) + 3 (x – 24) – 4 (x – 24) = 90
6x – 144 + 3x – 72 – 4x + 96 = 90
5x = (90 + 144 + 72 – 96)
210
x= = 42 minutes
5
Total quantity of tank B = 90 × 4 = 360 liter
Alternatively (P + Q – M) for 42 minutes, means each work pipe
for 14 minutes—
All three in 14 minutes
(P + Q – M) = 14 × 6 + 14 × 3 – 14 × 4
= 70 liter
70 7
Filled portion = =
360 36

34. (a); Let capacity of pipe A = y m3


So, capacity of pipe B = y + 8 m3
Required time to filled the swimming pool
5040
= minutes
𝑦
5040
Required time to empty the swimming pool = minutes
(𝑦+8)
ATQ –
5040 5040 45
– (𝑦+8) =
𝑦 4
112 112 1
− (𝑦+8) =
𝑦 4
3584 = y + 8y
2

y2 + 8y −3584 = 0
y2 + 64y – 56𝑦 − 3584 = 0
y (y + 64) – 56(𝑦 − 64) = 0
(y + 64) (𝑦 − 56) = 0
Y = 56 m3
Capacity of second swimming pool
= (56 + 8) × 112.5 = 7200 m3
33 Adda247 Publications For any detail, mail us at
Publications@adda247.com
Cracker Book for Bank (IBPS | SBI | RRB PO | Clerk) Mains Exams

35. (b); ATQ


For tank A —

ATQ
For tank B —
6 (x – 24) + 3 (x – 24) – 4 (x – 24) = 90
6x – 144 + 3x – 72 – 4x + 96 = 90
5x = (90 + 144 + 72 – 96)
210
x= = 42 minutes
5
Total quantity of tank B = 90 × 4 = 360 liter
Alternatively (P + Q – M) for 42 minutes, means each pipe for 14
minutes—
All three in 14 minutes
(P + Q – M) = 14 × 6 + 14 × 3 – 14 × 4
= 70 liter
70 7
Filled portion = =
360 36

Solutions (36-38):

Lets efficiency of Kartik is x unit/day and Anup is 1.75x unit/day


ATQ—
22 days (Ankur + Shubham) + 6 days (Shubham + Kartik) + 30 day
(Kartik + Anup) = 576
22 × 9 + 6 × 8 + 30 (x + 1.75x) = 576
30 (2.75x) = 576 – 246
330
2.75x =
30
11
x= ⇒ x = 4 unit/day
2.75
efficiency of Anup = 4 × 1.75 = 7 unit/day

34 Adda247 Publications For any detail, mail us at


Publications@adda247.com
Cracker Book for Bank (IBPS | SBI | RRB PO | Clerk) Mains Exams

36. (a); Working efficiency of A = 120% of working efficiency of B


B can complete ‘X’ work = 36 days
A can complete ‘X’ work = 30 days
Let ‘X’ work = 180 unit
A working efficiency = 6 unit/day
B working efficiency = 5 unit/day
15 × 6 + 10 × 5 + 𝐶 × 10 = 180
so, C working efficiency = 4 unit/day
For work ‘Y
Total of work ‘Y’=(6 + 4) × 12 + 16 × 5
= 200 𝑢𝑛𝑖𝑡𝑠
Total units of both work ‘X’ and work ‘Y’
= 180+200= 380 units
380
so,D working efficiency =
38
= 10 unit/day
Now:
A, B and C work together in ‘x’ work
1
= × 180 unit = 60 unit
3
A + B + C = (6 + 4 + 5) unit per day
60
= = 4 days
15
Remaining work = 120 unit
(B + D) ⇒ (5 + 10) unit per day
120
= = 8 days.
15
B work for = 4 + 8 = 12 days.

37. (e); ‘X’ work = 180 unit


A + C + D ⇒ 6 + 4 + 10 = 20 unit/day
180
Days = = 9 days
20
Work = 200 unit
(A + B + C + D) ⇒ (6 + 5 + 4 + 10)
= 25 unit/day
200
= = 8 days
25
Total time = (9 + 8) = 17 days

35 Adda247 Publications For any detail, mail us at


Publications@adda247.com
Cracker Book for Bank (IBPS | SBI | RRB PO | Clerk) Mains Exams

38. (a); Let efficiency of E is Z unit/day


he work for 12 days
work complete = 12Z unit
B and C work for 8 days = (5 + 4) × 8 unit
= 72 unit
Remaining work = 180 - 72 = 108 unit
108
Efficiency of E = = 9 unit⁄day
12
Now,
A and E completed work ‘X’
180
= = 12 days
15
D, B and C completed both work ‘X’ and ‘Y’
200+180
= = 20 days
19
Ratio = 12 : 20 ⇒ 3 : 5

2
39. (c); (Kartik + Anup) × x days + (Ankur + Shubham) × (𝑥 + 2 ) day
3
87.5
= 576 ×
100
3x+8 7
(4 + 7) × x + 9 × ( ) = 576 ×
3 8
11x + 9x + 24 = 504
20x = 480
x = 24 days
1
576×
8
efficiency of Prabhat = = 12 unit/day
6

Prabhat will do in (x + 12) days


= (24 + 12) × 12 = 432 unit
432 3
Required portion = =
576 4

𝑦
40. (a); If Ankur and Shubham work alternatively, then both work for
2
days individually
𝑦
Same, if Kartik and Anup work for day alternatively both work
2
𝑦
for day individually
4

36 Adda247 Publications For any detail, mail us at


Publications@adda247.com
Cracker Book for Bank (IBPS | SBI | RRB PO | Clerk) Mains Exams

ATQ—
y y y y
Ankur × + Shubham × + Kartik × + Anup × =
2 2 4 4
75 1
576 × (100 – ) ×
8 100
or
y y
(Ankur + Shubham) × + (Kartik + Anup) × = 522
2 4
𝑦
9 × 𝑦 + (4 + 7) × = 522 ∗ 2
2
y = 72 days
Efficiency of Abhimanyu
75 1
576× 8 ×100
= = 6 unit/day
9
Abhimanyu work did in (y + 16) days
= (72 + 16) × 6 = 528 unit
576 –528 48
Remaining work done by Anup and Kartik in = = days
11 11

37 Adda247 Publications For any detail, mail us at


Publications@adda247.com
Cracker Book for Bank (IBPS | SBI | RRB PO | Clerk) Mains Exams

1 Adda247 Publications For any detail, mail us at


Publications@adda247.com
Cracker Book for Bank (IBPS | SBI | RRB PO | Clerk) Mains Exams

Chapter
Speed Time and Distance
8
BEST APPROACH TO SOLVE THE QUESTIONS

Speed is defined as the distance covered in per unit time. Mathematically,


𝐷𝑖𝑠𝑡𝑎𝑛𝑐𝑒
Speed= .
𝑇𝑖𝑚𝑒

The whole concept of TSD (Time, speed and distance) lies within this
formula. Three most logical points in TSD are:
1. When speed is constant,
Distance is directly proportional to time i.e. D ∝ 𝑇
2. When distance is constant,
1
Speed is inversely proportional to time i.e. S ∝
𝑇
3. When time is constant,
Speed is directly proportional to distance i.e. S ∝ 𝑇

Another important term that we must know while solving questions in this
chapter is RELATIVE SPEED.
Relative speed is defined as the net speed of two entities when they are in
motion.

If the two entities are moving in opposite direction to each other,


net/relative/resultant speed is sum of the speeds. And when they move in
the same direction, their net/relative speed is the difference of their speeds.

Example: ‘A and B stay next to each other and study in the same school. A
and B start walking towards their school at speeds of 2 km/hr and 3 km/hr
respectively. The faster of the two reaches the school first, turns around and
starts walking back. If A and B meet 200 m away from the home. What is the
distance between their residences and their school?’

2 Adda247 Publications For any detail, mail us at


Publications@adda247.com
Cracker Book for Bank (IBPS | SBI | RRB PO | Clerk) Mains Exams

Solution: Let the distance where they met be ‘x’ metres from the school.
Distance travelled by A = 200m
Distance travelled by B= 200+x+x=200+2x
As the ratio of their speed is 2:3, and time is same, hence the ratio of
distance travelled by the them is also 2:3.
200 2
=
200+2𝑥 3
Solving, x= 50
Distance between their residences to school= 200+ x= 250m

Example: A goods train leaves a station at a certain time. After 6 hours, an


express train leaves the same station and travels in the same direction at a
speed of 90 km/hr. If the express train catches up the goods train in 4
hours, what is the speed of the goods train?

Solution: Distance covered by express train after leaving the station =


90× 4 = 360𝑘𝑚
Goods train covers this distance in 6+4=10 hours.
360
Hence speed of goods train= = 36 𝑘𝑚/ℎ𝑟
10

Key Points to Remember for New Pattern Questions


➢ Average speed is calculated by dividing the total distance
travelled by total time taken.
➢ Always be careful that units of speed, time and distance for
two or more entities should be same while solving a
problem.
➢ To change speed of km/hr into m/second multiply the
5 18
former by and by 𝑖𝑛 𝑣𝑖𝑐𝑒 𝑣𝑒𝑟𝑠𝑎 𝑐𝑎𝑠𝑒.
18 5

3 Adda247 Publications For any detail, mail us at


Publications@adda247.com
Cracker Book for Bank (IBPS | SBI | RRB PO | Clerk) Mains Exams

Practice Exercise Based on new Pattern

1. In how much time a train can cover a distance of 600 km if speed of


train is 25% more than the speed of car and speed of car is 20 % more
than the speed of a truck which cover 640 km in 20 hours in which he
takes 40 minute break after every 100 km.
(a) 9 hours (b) 8 hours 20 mins
(c) 10 hours (d) 8 hours
(e) None of these

2. A train travelling at 144 km/hr crosses another train, having 30 meter


less length and travelling in opposite direction at 126 km/hr in 6
seconds. If longer train cross a railway platform in 20 second then find
smaller train will cross same platform in how many seconds?
(a) 22 seconds (b) 24 seconds (c) 28 seconds
(d) 32 seconds (e) 30 seconds

3. Train A running at the speed of 108 km/hr crosses a man, who


running in the opposite direction at the speed of 12 km/hr in 7.2 sec.
If speed of train A increased by 25% and it takes 48 seconds to cross
another train B, which running at the speed of 90 km/hr in same
direction. Find the length of train B?
(a) 280 meters (b) 360 meters (c) 180 meters
(d) 160 meters (e) 220 meters

4. Train P leaves Delhi at 7 PM and is travelling at a speed of 80 kmph, 3


hours later another train Q leaves Delhi and is travelling in same
direction as train P in how many time (in minutes) train Q will be 18
km ahead train P, if speed of train Q is 120 kmph?
(a) 383 minutes (b) 387 minutes (c) 380 minutes
(d) 377 minutes (e) 375 minutes

4 Adda247 Publications For any detail, mail us at


Publications@adda247.com
Cracker Book for Bank (IBPS | SBI | RRB PO | Clerk) Mains Exams

5. A bus and a car start from a place A toward B simultaneously at 10:00


am. Bus reach at B in 4 hours and return for C, which is exactly in the
middle of A and B, and meet the car after 1 hour after leaving B. Find
the time at which car reach C after returning from B.
(a) 6:00 pm (b) 8:00 pm (c) 10:00 pm
(d) 8:00 am (e) 12:00 pm

6. Two trains A and B running at the speed of 108km/hr and 162km/hr


crossed another train ‘P’, which is standing stationary of length 240
meters in 14 sec and 16 sec respectively. How much time both train A
and B will take to cross each other, if both running in opposite
direction?
(a) 8.4 𝑠𝑒𝑐 (b) 8.6 𝑠𝑒𝑐 (c) 8.8 𝑠𝑒𝑐
(d) 9 𝑠𝑒𝑐 (e) 8.2 𝑠𝑒𝑐

7. There was a race of 3000 meters between A & B on a circular track of


750 meters. First time they meet during the race is after 5 minutes of
starting of race. Find the time taken by B to complete the race, if he
runs at half the speed of A.
(a) 20 minutes (b) 18 minutes (c) 15 minutes
(d) 10 minutes (e) None of these

8. Two cities Rampur and Dhamnagar are 300 km apart, Bhavya starts
from Rampur at 8 : 24 am and an hour later Abhi starts from same
city. After travelling for one hour Abhi reaches at city which Bhavya
had passed 40 min earlier. The city is in the way of Rampur and
Dhamnagar. If they reaches city Dhamnagar at same time. Find their
speed.
(a) 50 km/hr, 75 km/hr (b) 75 km/hr, 75 km/hr
(c) 100 km/hr, 100 km/hr (d) 75 km/hr, 100 km/hr
(e) None of these

5 Adda247 Publications For any detail, mail us at


Publications@adda247.com
Cracker Book for Bank (IBPS | SBI | RRB PO | Clerk) Mains Exams

9. A 240 meters long train crosses a 210 meters long train running in
opposite direction in 6 sec. Ratio between speed of longer train and
smaller train is 7 : 8. If faster train crosses a platform in 9 sec, then
find time taken by slower train to cross a bridge, which is 60 meters
more long than platform?
142 136 90
(a) 𝑠𝑒𝑐 (b) 𝑠𝑒𝑐 (c) 𝑠𝑒𝑐
7 7 7
148 142
(d) 𝑠𝑒𝑐 (e) 𝑠𝑒𝑐
7 7

10. Train P leave place A by is travelling at a speed of 120 kmph. 4 hours


later another train Q leaves place by travelling in same direction as
train P. Train Q will be 36 km ahead of train P in T hours and speed of
train Q is 160 kmph, then find a bus travel what distance in (T + 3.1)
hr, if speed of bus is 75% of speed of train P?
(a) 1360 km (b) 1440 km (c) 1540 km
(d) 1280 km (e) 1720 km

11. Satish started from point A in a boat to reach point B. After 6.5 hours
he covered only 20% of the distance and reach at point M. Now, Satish
started from point M reached at mid point of A and B and came back
to M in 29.25 hours. In what time Satish can cover the distance
between B and A if he started from B?
(a) 58.5 hours (b) 32.5 hours (c) 65 hours
(d) Can’t be determined (e) 50 hours

12. Train X having length 130 m and train Y having length 145 m moving
in opposite direction. They enter into a tunnel which have length
equal to the sum of length of both trains. Trains meet after 10 second
of entering in the tunnel. What percent of train X part is leave out the
tunnel when it meet train Y if they have there speed in the ratio of 5
:6.
11 11 1
(a) 2 % (b) 3 % (c) 4 %
13 13 3
(d) 5% (e) 8%
6 Adda247 Publications For any detail, mail us at
Publications@adda247.com
Cracker Book for Bank (IBPS | SBI | RRB PO | Clerk) Mains Exams

13. A car and a bus starts from point ‘A’. After ‘T’ hours bus is 48 km
ahead of car, while after ‘8’ hours distance between bus and car is
same as distance covered by bus in one hour. If relative speed of car
and bus if they move towards each other is 240, then how much
1
distance can bus cover in (𝑇 – ) hour.
2
(a) 320 km (b) 280 km (c) 384 km
(d) 336 km (e) 256 km

14. Point A to Point B is a downstream journey of 300 km on a stream


which flows at a speed of 5 km/hr. Two boats P and Q starts from point
A and Point B respectively with speed of 25 km/hr and 15 km/hr in
still water. After reaching the opposite point they return to their
starting points, find after how much time will they meet second time?
(a) 7.5hour (b) 15hour (c) 20hour
(d) 10hour (e) None of these

15. Distance between Delhi and Jaipur is 300 km. Aman starts from Delhi
and Rajiv from Jaipur at same time. After two hours, Aman realized he
was travelling slow and therefore increased his speed by 25% and
meet Rajiv at a point 108 km from Delhi. Find the increased speed of
Aman, if Rajiv derived at a constant speed of 75 km/hr.
(a) 40 km/hr (b) 50 km/hr (c) 60 km/hr
(d) 55 km/hr (e) 65 km/hr

Directions (16-17): Time taken by train A to cover a distance is 1.5 times


the time taken by train B to cover the same distance. Train A and train C
starts simultaneously from a station in same direction. Train B, start after
30 min and overtake train C in 1.5 hour later than it cross train A.

16. If speed of train A is 80 km/hr, then what is the speed of train C. (2


Marks)
(a) 60 km/hr (b) 100 km/hr (c) 125 km/hr
(d) 75 km/hr (e) 80 km/hr
7 Adda247 Publications For any detail, mail us at
Publications@adda247.com
Cracker Book for Bank (IBPS | SBI | RRB PO | Clerk) Mains Exams

17. If train A double its speed, then speed of train A is what % more than
the speed of train C. (1 Mark)
(a) 60% (b) 100% (c) 80%
(d) 50% (e) None of these

Directions (18-19): A consignment should deliver on time to customer for


which Mahendra start his journey with uniform speed, after 2 hours
customer call Mahendra to deliver his order 1 hour before the decided time.
Mahendra increased his speed by 50% to deliver it 1 hour before the
decided time.

18. Find the total time taken by Mahendra to deliver the consignment?
(a) 4 hours (b) 5 hours (c) 6 hours
(d) 7 hours (e) 3 hours

19. If initially, customer alto start moving toward Mahendra at speed of


60 km/hr and take his consignment after 3 hours then find the total
distance between Mahendra and Customer initially.
(a) 256 km (b) 150 km (c) 500 km
(d) 450 km (e) 300 km

Directions (20-21): A man decided to run 15 rounds of a circular track of


400 m in certain time with certain speed. He starts running but after
completing some round around the track he reduced his speed by 40% due
to which he takes 4 min extra as scheduled. But if he reduced his speed by
completing 3 more rounds he would have reached 2 min earlier than the
time he actually reached.

20. Find the original speed of man?


(a) 240 m/min (b) 160 m/min (c) 200 m/min
(d) 400 m/min (e) 350 m/min

21. Find the number of rounds at which he decided to reduce his speed?
(a) 7 (b) 9 (c) 10
(d) 8 (e) 11
8 Adda247 Publications For any detail, mail us at
Publications@adda247.com
Cracker Book for Bank (IBPS | SBI | RRB PO | Clerk) Mains Exams

22. A cyclist left point A for point B and travelled at the constant speed of
25 km/h. When he covered the distance of 25/3 km, he was overtaken
by a car that left point A twelve minutes after the cyclist and travelled
at a constant speed too. When the cyclist travelled another 30 km, he
encountered the car returning from B. Assume that the car did not
stop at point B. Find the distance between A and B.
(a) 39.5833 km (b) 41.0833 km (c) 60.833 km
(d) 43.33 km (e) 50 km

23. Two person left simultaneously two places A and B. One of them left A
for B while the other left B for A. Both travel at uniform speed . The
first person on reaching B returns to A and then again travels back to
B and so on. Similarly the second person on reaching A returns to B
and then travels back to A and so on. What will be the distance
covered by the first person when they meet for the third time given
the ratio of the speed of the first person to that of the second person is
3 : 2 and the distance between A and B is 500 m?
(a) 1000 m (b) 1500 m (c) 2500 m
(d) 1200 m (e) 1800 m

24. A train M running at the speed of 108 km/hr crosses a man running at
12 km/hr in opposite direction of train in 12 second and crosses a
platform in 32 sec. If a train N is Standing on same platform and
length of platform is 140 meter more than length of train N. Find in
what time train N will cross train M running in same direction, if train
N passes a pole in 12 second?
(a) 168 sec (b) 164 sec (c) 154 sec
(d) 186 sec (e) 172 sec

Directions (25-26): Satish and Bhavya starts from point P and Q


respectively at same time to reach at point B. Satish overtakes Bhavya after
10 hours at point A. If Bhavya starts 4 hours earlier than Satish then they
will meet at point B after 16 hours when Satish starts. Ratio between
distance between Q and A to Q and B is 1 : 2. Point P, Q, A and B lies on a
straight road.

9 Adda247 Publications For any detail, mail us at


Publications@adda247.com
Cracker Book for Bank (IBPS | SBI | RRB PO | Clerk) Mains Exams

25. If distance between point A and point B is 180km then find time taken
by Bhavya to reach point ‘C’ which is 432km ahead of point ‘B’?
(a) 36 hours (b) 44 hours (c) 24 hours
(d) 32 hours (e) 40 hours

26. If Satish and Bhavya started to move towards each other from P and Q
at 8:00 a.m, then at what time they will meet?
(a) 9 : 00 a.m (b) 9 : 30 a.m (c) 10 : 00 a.m
(d) 10 : 30 a.m (e) 11 : 00 a.m

27. A bus (X) start its journey from P to Q simultaneously another bus (Y)
start from Q to P they reach their respective destination after 3 hour.
Bus X travel with a speed of 40 km/hr in first hour 50 km/hr in
second hour and 60 km/hr in 3rd hour. Bus Y travel with a speed of 60
km/hr in first hour, 40 km/hr in second and 50 km/hr in last hour.
Find out the distance between Q and the point where the buses cross
each other.
(a)87 (b)88 (c)89
2 7
(d)82 (e) 87
9 9

175 2
28. Speed of a car is 𝑚/𝑠 and speed of bus is 14 % less to speed of car,
9 7

bus and car covered different distance in different interval of time and
the sum of the time taken by them to cover their respective distances
is10 hours but distance covered by bus is 80 km more than distance
covered by car. If a train covered total distance which covered by car
2
and bus together in 6 ℎ𝑜𝑢𝑟𝑠, then find speed of train?
3

(a) 96km/hr (b) 84km/hr (c) 78km/hr


(d) 72km/hr (e)64km/hr

10 Adda247 Publications For any detail, mail us at


Publications@adda247.com
Cracker Book for Bank (IBPS | SBI | RRB PO | Clerk) Mains Exams
20
29. Two trains running in opposite directions cross each other in sec.
3
Longer train cross shorter train in 60 sec when both train running in
same direction. If length of shorter train increased by 50% than
longer train cross shorter train in 72 sec, when both running in same
direction. If difference between length of both trains is 60 meters,
then find the time in which longer train will cross a platform whose
7
length is 77 % more than length of longer train?
9
(a) 20 sec (b) 10 sec (c) 15 sec
(d) 14 sec (e) 18 sec
30. Two trains P and Q started from two points A and B towards each
other to cover 300 km distance. Speed of train Q is twice of speed of
train P. If train P runs at its normal speed and train Q reduces its
speed to 50% then it will take 2.5 hrs. more to meet each other. Find
the time (in sec) in which train P can cover a platform of 150m length
if the length of train P is given as 50m.
(a) 18 seconds (b) 36 seconds (c) 72 seconds
(d) 54 seconds (e) 24 seconds
27
31. A train P, 180 meter long train passed a pole in sec and also passed
4
two trains Q and R in 9 sec and 39 sec respectively, where train Q
running in opposite direction of train P and train R is running in same
direction of train P. If length of train Q and R is 240 meter and 210
meter respectively, then in what time train Q will pass train R, if both
runs in opposite direction ?
7 3
(a) 35 sec (b) 9 sec (c) 12 sec
11 11
(d) 15 sec (e) 55 sec
32. Total distance between Delhi to Lucknow is 480 km. A train starts
running with an average speed of 60 km/hr from Delhi to Lucknow
while another train starts Journey after 120 minutes of first train and
reaches Lucknow 30 minutes before first train. If first train stops for
5 minutes on each station and second train did not stop at any station
,then find the ratio between speed of first train to speed of second
train given that total number of station between Delhi and Lucknow
are nine.?
(a) 25 : 33 (b) 25 : 32 (c) 35 : 25
(d) 25 : 31 (e) 21 : 31
11 Adda247 Publications For any detail, mail us at
Publications@adda247.com
Cracker Book for Bank (IBPS | SBI | RRB PO | Clerk) Mains Exams

33. The ratio between length of two trains is 3 : 2 and speed of both trains
is 72 km/hr & 90 km/hr respectively. Faster train crosses slower
trains in 60 sec running in same direction. If slower train crosses a
goods train, which is running at the speed of 108 km/hr in 7.2 sec
running in opposite direction. Then find the time taken by faster train
to cross goods train running in same direction?
(a) 40 sec (b) 64 sec (c) 60 sec
(d) 56 sec (e) 50 sec

34. Two athletes ‘A’ and ‘B’ practice running on 800 meters’ track. If they
start running at their usual speeds, in same direction, at same time,
from same point, first time they meet is in 100 seconds. During warm-
2
up, ‘A’ runs at 18 % of his usual speed and completes 400 meters in
11
50 second. Find the time taken by athlete ‘B’ to complete 9 rounds of
5
400-meter track, if he runs at of his usual speed. Assume athlete ‘A’
4
is faster than ‘B’.
(a) 100 second (b) 450 second (c) 90 second
(d) 80 second (e) 85 second

35. The speed of car is 25% more than the speed of bus. The time
difference between them to cover a certain distance D, is 1 hour. On
particular day, driver noticed that if they are (D- 40) km apart from
each other then they can meet in 2 hours driving in opposite direction
at their normal speed. Find 150% of the speed of bus ?
(a) 150 km/hr (b) 90 km/hr (c) 120 km/hr
(d) 105 km/hr (e) None of these

36. Time taken by car A to cover distance from point X to point Y is 6½


hrs including ½ hour of stoppage. B also started from point X and
stopped 90 km earlier from point Y after travelling for 6½ hrs. Had
they started from same point and at same time, the difference
between their distance covered in 7.5 minutes will be 2.5 km. Find the
distance between two cars, if they start a race for the distance which
is twice the distance between X and Y , assuming that both cars stops
when one of the car finish the race
(a) 360 km (b) 240 km (c) 180 km
(d) 300 km (e) 280 km
12 Adda247 Publications For any detail, mail us at
Publications@adda247.com
Cracker Book for Bank (IBPS | SBI | RRB PO | Clerk) Mains Exams

37. Two trains A and B with their length difference 17 m cross each other
in 2.6 seconds while running in opposite direction. Train A crosses
train B in 13 seconds while they are running in same direction. If train
A takes 2 seconds to cross a pole, then find the sum of speed of both
the trains, assuming that train B is longer than train A.
(a) 34 ms¯¹ (b) 51 ms¯¹ (c) 68 ms¯¹
(d) 85 ms¯¹ (e) 102 ms¯¹

38. ‘Rajdhani express’, 240 m long train crosses a pole in 6 sec and it also
120
crosses two trains ‘Shatabdi express’ and ‘Duranto express’ in 𝑠𝑒𝑐
17
& 42 sec respectively, where ‘Shatabdi express’ is running in opposite
direction of ‘Rajdhani express’ and ‘Duranto express’ running in same
direction as that of ‘Rajdhani express’. If length of ‘Shatabdi express’ &
Duranto express’ is 360 m and 180 m respectively, then find in what
time ‘Shatabdi express’ will cross ‘Duranto express’ while running in
same direction?
(a) 24 sec (b) 28 sec (c) 32 sec
(d) 36 sec (e) 48 sec

Directions (39–40): There are two trains A and B, both train starts
travelling to each other from stations P and Q respectively. Train A reaches
the station Q in total 4 hours while train B reaches the station P in total 4
hours 48 minutes. Speed of train A is 120 km/hours.

39. Train A and B running on two different railroads parallelly in opposite


direction to two different trains C and D respectively. Train A passed
train C, which running at the speed of 60 km/hr. in 6 sec. and train B
passed train D, which running at the speed of 110 km/hr. in 9 sec. if
ratio between length of train A to train C is 3: 2, and that of between
train B to train D is 4 : 3. Find in what time train D passed train C, if
both running in same direction ?(2 marks)
(a) 22.64 sec (b) 24.84 sec (c) 24.24 sec
(d) 28.84 sec (e) 20.84 sec

13 Adda247 Publications For any detail, mail us at


Publications@adda247.com
Cracker Book for Bank (IBPS | SBI | RRB PO | Clerk) Mains Exams

40. Ratio between speed of train A to train E is 6: 5.Train E starts a


journey from Kolkata to Delhi, which is 600 km at 9.45pm and train A
starts same journey after 30 minutes of train E.Train A stop for x
minutes on each three stoppages and train E stop for (x+10) minutes
each on three stoppages between Kolkata to Delhi.Train A reaches
Delhione hour earlier than train E, which reaches at 4.30
am.Findinwhat time train B a total distance of 720 km cover between
Lucknow to Jaipur. If train B stop for (x+10) minutes on four
stoppages between Lucknow to Jaipur?(2 marks)
(a) 4 hours (b) 8 hours (c) 8.2 hours
(d) 10 hours (e) 12 hours

Solutions

1. (c); Stop of truck = 6 × 40 = 240 mint or 4 h.


640 640
Speed of truck = = = 40 𝑘𝑚/ℎ
20−4 ℎ. 16
40×120
Speed of car = = 48 𝑘𝑚/ℎ
100
48×125
Speed of train = = 60 𝑘𝑚/ℎ
100
600
Required time = = 10 ℎ𝑜𝑢𝑟𝑠
60

2. (a); Let length of both trains be L meters and (L – 30) meters


respectively
ATQ,
5 𝐿+ (L – 30)
(144 + 126)× =
18 6
450 = 2L – 30
L = 240
Smaller train length = 210 meters
Let length of platform be P meters
ATQ,
5 240+𝑃
144 × =
18 20
P = 800 – 240 = 560 meters
Let required time = T
ATQ,
5 210+560
126 × =
18 𝑇
770
T = ⇒ T = 22 sec
35
14 Adda247 Publications For any detail, mail us at
Publications@adda247.com
Cracker Book for Bank (IBPS | SBI | RRB PO | Clerk) Mains Exams

3. (b); Let length of train A be L meters


5 𝐿
(108 + 12) × =
18 7.2
L = 240 meters
125
New speed of train A = 108 ×
100
= 135 𝑘𝑚/ℎ𝑟
Let length of train B be S meters
5 240+𝑆
(135 – 90)× =
18 48
S = 360 meters

4. (b); Relative speed of both train = (120 – 80) = 40 kmph


Distance covered by train P from Delhi when train Q was not
moving = 80 × 3 = 240 km
Train Q should be covered (240 + 18) km from Delhi for ahead
18 km to train P from Delhi
258
Required time = km
40
= 6.45 = 6.45 × 60
= 387 minutes

5. (b);

Car travel → 3/4 part of one side distance in 5 hour.


20
1 part = hour
3
Car can travel 1.5 part in
20
× 1.5 = 10 hour
3
Time to reach C after returning form point B → 10:00 + 10
= 8:00 pm

15 Adda247 Publications For any detail, mail us at


Publications@adda247.com
Cracker Book for Bank (IBPS | SBI | RRB PO | Clerk) Mains Exams

6. (c); Let speed of train A and B be ‘x’ meters and ‘y’ meters
respectively
ATQ,
5 x+240
108 × = m⁄s
18 14
30 × 14 = x + 240
x = 180 meters
And,
5 𝑦+240
162 × =
18 16
45 × 16 = y + 240
y = 480meters
Let required time be T sec
5 480+180
(108 + 162) × =
18 T
75 𝑇 = 660
T = 8.8 sec

7. (a); Let speed of A is 2x & speed of B is x


According to question,
750
= 5 min
a−b
⇒ a – b = 150 meters/min
⇒ 2x – x = 150 meters/min
⇒ x = 150 meters/min
It is speed of B hence
3000 3000
Time taken = = = 20 minutes
x 150

8. (d); Let the city in way of Rampur and Dhamnagar is C.

Time Speed
Bhavya 80 3x
Abhi 60 4x
16 Adda247 Publications For any detail, mail us at
Publications@adda247.com
Cracker Book for Bank (IBPS | SBI | RRB PO | Clerk) Mains Exams

ATQ,
300 300
– =1
3x 4x
25
= 1 ⇒ x = 25
x
∴ Bhavya Speed = 3 × 25 = 75 km/hr
Abhi speed = 4 × 25 = 100 km/hr

9. (c); Let speed of longer train and smaller train be 7𝑥 and 8𝑥


respectively.
ATQ—
(240+210)
(7𝑥 + 8𝑥 ) =
6
90𝑥 = 450
𝑥=5
Speed of longer train = 5 × 7 = 35 m/s
Speed of smaller train = 5 × 8 = 40 m/s
Let length of platform be l meter
𝑙+210
40 =
9
40 × 9 = 𝑙 + 210
l=150
Then, length of bridge = 150 + 60 = 210 meters
Let time taken by slower train to cross bridge be T sec.
210+240
T=
35
35T = 660
90
T= sec.
7

10. (b); Relative speed of both trains = (160 – 120) = 40 kmph


Distance covered by train P from place A when train Q was not
moving = 120 × 4 = 480 km
Train Q should cover (480 + 36) km from place A in T hrs
516
Required time, T = = 12.9 hr
40
3
Bus speed = 120 × = 90𝑘𝑚𝑝ℎ
4
Bus covered in ( T+3.1) hours = ( 12.9 + 3.1) × 90 = 1440 km

17 Adda247 Publications For any detail, mail us at


Publications@adda247.com
Cracker Book for Bank (IBPS | SBI | RRB PO | Clerk) Mains Exams

11. (c); Let total distance from A to B = ‘D’


ATQ, Satish cover 20% distance in 6.5 hours So, he can cover
30% distance (M to mid-point of A and B) in
6.5
× 3 = 9.75 hr.
2
Time taken by Satish to come back from mid-point to M
= 29.25 – 9.75 = 19.5 hr
30% distance covered by Satish in 19.5 hr.
19.5
100% distance covered by Satish in × 10 = 65 hr
3

12. (b); Speed of train X and Y 5x and 6x respectively.


Length of tunnel → 130 + 145 = 275 m.
Speed of trains per second
275
= = 27.5 𝑚/s
10
5x + 6x = 27.5
x = 2.5
speed of train X → 12.5 m/s
Speed of train Y → 15 m/s
Distance cover by train X in tunnel = 12.5 × 10 = 125 m.
Length of train X leaves out = 130 – 125 = 5m
5
Required% = × 100
130
11
=3 %
13

13. (a); Let, speed of car = x km/hr


Speed of bus = y km/hr
ATQ,
Distance covered by bus in one hour
= Distance between bus & car in 8 hour
⇒ y = 8 (y – x) [ ∵ y > x]
y = 8y – 8x
8x = 7y …(i)
and x + y = 240 …(ii)
On solving (i) & (ii)
x = 112, y = 128
After T hour bus is 48 km ahead of car
⇒ (128 – 112) T = 48
48
⇒T= =3
16
Required distance = 2.5 × 128 = 320 km
18 Adda247 Publications For any detail, mail us at
Publications@adda247.com
Cracker Book for Bank (IBPS | SBI | RRB PO | Clerk) Mains Exams

14. (c); Time when P and Q meet first time


300 300
= = = 7.5 hr
25+15 40
Distance travelled by P = 7.5 × (25 + 5) = 225 km
Distance travelled by Q = 7.5 × (15 – 5) = 75 km
75
P reach at point B in i.e. 2.5 hour
30
In 2.5hr Q travelled 2.5 × 10 = 25 km
Now Boat P returns and relative distance b/w P and Q is 100 km
and relative speed is 10 km i.e. difference of speed of P (25 -5) =
20 km/hr (upstream) and speed of Q (15 – 5) = 10 km/hr
(upstream)
100
Time taken by P and Q to meet = = 10 hr
10
Total time when P and Q meet second time
= 7.5 +2.5 + 10 = 20 hr
15. (b); Let initial speed of Aman is x km/hr.
Distance travelled by him in 2 hours is 2x km.
While distance travelled by Rajiv in these 2 hours is 75 × 2 = 150
km
When both of them meet, Aman had travelled a distance of 108
km.
Distance travelled by Aman with 25% increase in his speed =
(108 – 2x) km
125 5x
And his increased speed = x( ) = km/hr
100 4
If Aman had travelled 108 km, Rajiv had travelled 192 km.
192 – 150 = 42 km after 2 hours
Time taken by Rajiv to travel 42 km
42 14
= = hours
75 25
This is equal to time taken by Aman to travel (108 – 2x) km
(108–2x) 14
5 =
x 25
4
14×5x
108– 2x =
25×4
7
108 = x + 2x
10
27
x = 108 ⇒ x = 40 km/hr.
10
Therefore, increased speed of Aman was
5
= × 40 = 50 km/hr
4

19 Adda247 Publications For any detail, mail us at


Publications@adda247.com
Cracker Book for Bank (IBPS | SBI | RRB PO | Clerk) Mains Exams

Solutions (16-17); Time ratio between train A and train B = 1.5 : 1


So speed ratio between train A and train B is =1 : 1.5 or 2 : 3
Let speed of train A = 2x km/hr
Speed of train B = 3x km/hr
Relative speed = 3x – 3x = x km/hr
Distance travel by train A in 30 m
2𝑥
⇒ = x km
2
𝑥
Train B cross train A ⇒ = 1 hour
𝑥
So,
Train B cross train C ⇒ 1 + 1.5 = 2.5 hour
Let speed of train C = y km/h
ATQ
2.5 × 3x = 3y
x:y⇒2:5
Ratio of speed of A, B and C
4:6:5

16. (b); Speed of train A = 80 km/hr


80
Speed of train C = × 5 = 100 km/hr
4

17. (a); Let speed of train A = 4x km/hr


So speed of train C = 5x km/hr
(2×4𝑥–5𝑥)
Required % = × 100 = 60%
5𝑥

18. (b); Let speed of Mahendra = 2𝑥 km/hr


So, Distance covered by him in 2 hours = 2𝑥 × 2 = 4𝑥
Let he decide to deliver his consignment in 𝑡 hour
So total distance ⇒ 2𝑥 × 𝑡 = 2𝑥𝑡
ATQ
⇒ 2𝑥𝑡 = 2𝑥 × 2 + (𝑡– 3)3𝑥
3𝑥 → increase speed
𝑡– 3 = time for which he travelled after speed is increased
On Solving equation →
5 ℎ𝑜𝑢𝑟𝑠 = 𝑡

20 Adda247 Publications For any detail, mail us at


Publications@adda247.com
Cracker Book for Bank (IBPS | SBI | RRB PO | Clerk) Mains Exams

19. (d); Distance covered by Mahendra in 3 hours → 2𝑥 × 3 = 6𝑥


Total distance ⇒ 5 × 2𝑥 = 10𝑥
Remaining distance ⇒ 10𝑥 – 6𝑥 = 4𝑥 = Distance covered by
customer
and 4𝑥 = 60 × 3
𝑥 = 45 km
Total distance = 45 × 10 = 450 km

Solutions (20-21): Total distance = 15 × 400 = 6000 m


Let his original speed = x meter/min
Decided time = t min
Let after ‘n' round he reduced his speed 1st decided condition
6000
⇒t= …(i)
x
3x
is the reduced speed after n rounds
5
Now 2nd condition
400×n 6000–400×n
+ 3x = t + 4 …(ii)
x
5
3rd → supposed condition
400(n+3) 6000–400×(n+3)
+ 3x = t + 2 …(iii)
x
5
Putting value of ‘t’ from (i) into (ii) and (iii)
⇒ 3x + 200n = 3000 …(iv)
⇒ 3x + 400n = 4800 …(v)
Solving (iv) and (v)
x = 400 m/min
n=9

20. (d); Original speed of man = 400 m/min

21. (b); Number of rounds after which he reduced his speed = 9

21 Adda247 Publications For any detail, mail us at


Publications@adda247.com
Cracker Book for Bank (IBPS | SBI | RRB PO | Clerk) Mains Exams
25 1
22. (c); Time taken by Cyclist to reach 25/3 km = = hr = 20 min
25 × 3 3
Car has taken to reach 25/3 km = 20 – 12 = 8 min
25
Speed of Car = 3  60 = 62.5 km/h
8
Now time taken by cyclist to go further 30 km
30 6
= = hr = 72 min
25 5
72
Car will go in 72 min =  62.5 = 75 km
60
Now, according to question,
distance between first meeting and second meeting is 30
So,
distance between first meeting and point B will be
75+30
= = 52.5 km
2
Required answer = 52.5 + 8.33 = 60.833 km

23. (b); When they meet for the third time they together cover 5 × 𝐴𝐵
= 5 × 500 = 2500𝑚
The first person will cover
3
× 2500 = 1500 𝑚
2+3

24. (b); Lets length of train M = L meter


ATQ,
(108+12)×5 𝐿
=
18 12
L = 400 m
Let length of platform = Lp
108×5 𝐿𝑝 +400
=
18 32
𝐿𝑝 = 960– 400
𝐿𝑝 = 560 meter
Length of train N = 560 – 140
= 420 meter
Speed of train N = 𝑆𝑛
420
𝑆𝑛 = = 35 m/s
12

22 Adda247 Publications For any detail, mail us at


Publications@adda247.com
Cracker Book for Bank (IBPS | SBI | RRB PO | Clerk) Mains Exams

Relative speed of train M and N, when both running in same


direction
5
= 35 −108 × = 5 𝑚/𝑠𝑒𝑐
18
Let required time is t
ATQ,
400+420
5=
𝑡
820
t=
5
t = 164 sec

Solution (25-26)

Let speed of Satish = ‘a’


And, speed of Bhavya = ‘b’
Let distance between P and Q = x
And, QA and QB be y and 2y respectively.
After 10 hrs Satish overtakes Bhavya at point A.
𝐷𝑖𝑠𝑡𝑎𝑛𝑐𝑒
⇒ 𝑆𝑝𝑒𝑒𝑑 =
𝑇𝑖𝑚𝑒
y= 10 × b
and,
(a - b) × 10 = x …(i)
If Bhavya starts 4 hour earlier than Satish, then Satish overtakes Bhavya at
point B after 16 hours
Distance travelled by Bhavya in 4 hours = 4b
So,
(x + 4b) = (a – b) × 16 …(ii)
On solving (i) & (ii)
20𝑏 2𝑦
𝑥= ⇒𝑥= …(iii)
3 3
Satish can travel and reach point B in 16 hours
⇒ 16a = 2y + x …(iv)
On solving (iii) & (iv)
y = 6a
3𝑥 𝑥 3𝑥
So, 𝑦 = , 𝑎 = , 𝑏 =
2 4 20

23 Adda247 Publications For any detail, mail us at


Publications@adda247.com
Cracker Book for Bank (IBPS | SBI | RRB PO | Clerk) Mains Exams

25. (b); ATQ,


3𝑥
= 180
2
⇒ x = 120 km/hr
3𝑥 3𝑥
+ +432
2 2
Required time = 3𝑥
20
3×120+432
=
18
792
= = 44 hours
18

𝑥
26. (d); Required time = 𝑥 3𝑥
+
4 20
𝑥
= 5𝑥+3𝑥
20
20
= = 2 ℎ𝑜𝑢𝑟 30 𝑚𝑖𝑛𝑢𝑡𝑒
8
Satish & Bhavya meet at 10 : 30 am

27. (d); Distance between P and Q is 150 km.


Now X bus cover 40 km in 1 hour
Y bus cover 60 km in 1st hour
Remaining distance = 50 km
𝑅𝑒𝑚𝑎𝑖𝑛𝑖𝑛𝑔 𝑡ime to cross each other
50 50 5
= = = ℎ𝑟
40+50 90 9
5
Distance which is covered by ′Y ′ in hr
9
5 200
= 40 × = km
9 9
Distance between Q and the point where buses crosses each
other
200 2
= 60 + = 82 km
9 9
24 Adda247 Publications For any detail, mail us at
Publications@adda247.com
Cracker Book for Bank (IBPS | SBI | RRB PO | Clerk) Mains Exams
175 6 25×6
28. (a); Speed of bus = × =
9 7 9
50 50 18
= m⁄s = × = 60 km⁄hr
3 3 5
Let distance travel by car and bus be D and (D + 80)
respectively.
D (D+80)
+ = 10
70 60
6D+7D+560
= 10
420
13D = 4200 – 560
13D = 3640
D = 280 km
[280+(280+80)]×3
Speed of train =
20
= 96 km⁄hr

29. (a); Let length of shorter train be L meters and length of longer train
be (L + 60) meters
And also speed of shorter and longer train be V m/s and U m/s
respectively.
When both trains travelling in opposite direction
(L+L+60)3
= U+V
20
3L+180
= U + V …(i)
20
When both train running in same directions—
L+L+60
= U– V
60
(2L+60)
= U– V …(ii)
60
When length of shorter train increased and both train running in
same direction —
(1.5L+L+60)
= U –V
72
2.5L+60
= U– V …(iii)
72
From (i) and (ii)
3L+180 2L+60
2U = +
20 60
11L+600
U= …(iv)
120
7L+480
U= …(v)
120
25 Adda247 Publications For any detail, mail us at
Publications@adda247.com
Cracker Book for Bank (IBPS | SBI | RRB PO | Clerk) Mains Exams

After solving (iii), (iv) and (v)


L = 120 m, U = 25 m/s and V = 20 m/s
Length of longer train = 120 + 60 = 180 meter
7
Length of platform = 180 + 180 × = 320 meters
9
Time taken by longer train to cross platform
320+180 500
= = = 20 sec
25 25

30. (b); Let, speed of train P = x


Speed of train Q = 2x
Let, Normal time taken by train to meet each other = ‘y’
ATQ,
(x + 2x) y = (x + x) (y + 2.5)
3y = 2(y + 2.5)
y = 5 hr.
300
Now, x + 2x = = 60
5
3x = 60
x = 20 km/hr
50
= m/sec
9
(150+50)
Required time = ×9
50
200
= × 9 = 36 sec
50

31. (c); Lets speed of train P, Q and R be S₁, S₂ and S₃ respectively


180 80 m
Speed of train P (S1 ) = 27 m⁄s =
3 s
4
Speed of train Q (S2 )
80 240+180
+ 𝑆2 =
3 9
420 80
S2 = –
9 3
S2 = 20 m⁄s
Speed of train R (𝑆3 )
80 210+180
– S3 =
3 39
80
S3 = – 10
3

26 Adda247 Publications For any detail, mail us at


Publications@adda247.com
Cracker Book for Bank (IBPS | SBI | RRB PO | Clerk) Mains Exams
50
S3 = m⁄s
3
Lets required time be T sec
50 240+210
Required time = 20 + =
3 T
110 450 450×3
= ⇒T =
3 T 110
3
T = 12 sec
11

32. (b); Speed of first train = 60 km/h


Total distance = 480 km
Time taken by first train to cover that distance without stoppage
480
= = 8ℎ
60
9 station × 5 minute = Total stoppage time
= 45 minute
Train took total time to reach Lucknow
35
= 8 hr 45 minute = ℎ𝑜𝑢𝑟
4
2nd Train reach 30 min. before it start 2 hour late from Delhi
So,
Time taken by 2nd train = 8 h 45 min – 2h
– 30 min.
= 6 hour 15 minute
1 25
= 6 = hour
4 4
480×4 384
Speed of 2nd Train= = km/h
25 5
60
Required Speed ratio of train = 384 =25:32
5

33. (c); Let length of two trains be 3L meter and 2L meter


ATQ—
5 3L+2L
(90– 72) × =
18 60
300 = 5L
L = 60 meter
Length of faster train = 120 meter
Length of slower train = 180 meter
Let length of goods train be G meter

27 Adda247 Publications For any detail, mail us at


Publications@adda247.com
Cracker Book for Bank (IBPS | SBI | RRB PO | Clerk) Mains Exams
5 180+𝐺
(108 + 72) × =
18 7.2
50 × 7.2 = 180 + G
G = 360 – 180
G = 180 meter
Let faster train cross good train in T sec —
5 120+180
(108 – 90) × =
18 T
5T = 300
T = 60 sec

34. (d); Let the speed of athletes ‘A’ and ‘B’ is 𝑎 and 𝑏 respectively
From 1st condition
800
= 100 ⇒ 𝑎– 𝑏 = 8 ms –1
𝑎–𝑏
From 2nd condition
2
When athlete ‘A’ will run at 18 % of his usual speed
11
200 2
= ×𝑎= 𝑎
1100 11
400
= 2 = 50 ⇒ 𝑎 = 44 ms –1
𝑎
11
∴ 𝑏 = 36 ms –1
5
If 𝑏 runs at of his speed, which means with 45 ms –1
4
4×400
Time taken by him = = 80 seconds.
45

35. (c); Let speed of car is 5𝑥 𝑘𝑚/ℎ𝑟 and speed of bus is 4𝑥 𝑘𝑚/ℎ𝑟.
According to first condition,
𝐷 𝐷 5𝐷–4𝐷
– =1⇒ =1
4𝑥 5𝑥 20𝑥
⇒ 𝐷 = 20𝑥 …(i)
Also if they are travelling in opposite direction,
Then
𝐷–40
= 2 ⇒ D – 40 = 18x …(ii)
4𝑥+5𝑥
From (i) & (ii)
18𝑥 + 40 = 20𝑥 ⇒ 𝑥 = 20
Hence speed of bus = 4 × 20 = 80 km/hr
150
150% of speed of bus = × 80 = 120 km/hr
100

28 Adda247 Publications For any detail, mail us at


Publications@adda247.com
Cracker Book for Bank (IBPS | SBI | RRB PO | Clerk) Mains Exams

36. (b); Let the Distance b/w X to Y be D km and speed of car A and B be
A km /hr & B km/hr respectively
13
6A – B = 90 – (i)
2
& according to second condition
7.5 7.5
A– 𝐵 = 2.5
60 60
A – B = 20 – (ii)
Solving (i) & (ii)
A = 80 km, B = 60 km
Distance b/w X & Y is = 6 × 80 = 480 km
960
Time take by A to cover 960 km is = 12 hours
80
Distance covered by B in 12 hours = 12 × 60 = 720 km
Distance b/w them = (960–720) km = 240 km

37. (d); Let the length of train A be L m and speed is a m/s.


& the length of train B will be L + 17 m and speed is b m/s.
ATQ,
2𝐿+17
= 2.6 …(i)
𝑎+𝑏
2𝐿+17
= 13 ...(ii)
𝑎–𝑏
For train A
𝐿
= 𝑎 ⇒ 𝐿 = 2𝑎
2
Put L=2a in (i) and (ii)
4𝑎+17
= 2.6 …(iii)
𝑎+𝑏
4𝑎+17
= 13 .....(iv)
𝑎–𝑏
On solving (iii) and (iv),
a= 51m/s
and, b=34m/s
Hence sum of speed =51+34= 85 𝑚𝑠 –1

38. (d); Let speed of ‘Rajdhani express’ be S m/s


ATQ –
240
S=
6
S = 40 m/s
Speed of ‘Shatabdi express’ be V m/s
29 Adda247 Publications For any detail, mail us at
Publications@adda247.com
Cracker Book for Bank (IBPS | SBI | RRB PO | Clerk) Mains Exams
17(240+360)
(40 +V) =
120
4800 + 120V = 10200
120V = 5400
V = 45 m/s
Speed of Duranto express’ be u m/s
240+180
(40 − U) =
42
40 − U = 10
U = 30 m/s
Let ‘Shatabdi express’ will cross ‘Duranto express’ running in
same direction in T sec
360+180
(45 − 30) =
𝑇
15T = 540
T = 36 sec

Solutions (39-40): Speed of train A = 120km/h


Distance between P and Q = 120 × 4 = 480
480 ×5
Speed of train B = = 100 km/hr
24

39. (b); Relative speed of train A and C, when both running in opposite
direction
5
= (120 + 60) ×
18
= 50 m/sec
Lets length of train A and C is 3L meter and 2L meter
respectively.
ATQ—
3L+2L
50 =
6
5L = 300
L = 60 meter
Length of train C = 60 × 2 = 120 meter
Relative speed of train B and D, when both running in opposite
5 175
direction = (100 + 110) × = m/sec
18 3
Lets length of train B and D is 4L meter and 3L meter
respectively
ATQ—
175 4L+3L
=
3 9
30 Adda247 Publications For any detail, mail us at
Publications@adda247.com
Cracker Book for Bank (IBPS | SBI | RRB PO | Clerk) Mains Exams

7L = 525
L = 75
Length of train D = 75 × 3 = 225 meter
Relative speed of train C and D when both train running in same
direction
5
= (110 – 60) ×
18
125
= meter/sec
9
(120+225)×9
Required time = = 24.84 sec
125

40. (c); Speed ratio of train A and Train E = 6 : 5


So, ratio of time taken by train A and train E (without stoppage)
=5:6
Now → Train E start = 9 : 45
Halt → 3 × (x+ 10) ⇒ 3x + 30 min
Train A start →9 : 45 + 30 → 10 : 15
Halt time ⇒ 3 × x = 3x min
If halt time add, before starting journey
Train E ⇒9 : 45 + 30 + 3x
⇒10 : 15 + 3x
Train A ⇒10 : 15 + 3x
So, time difference on reaching on Delhi is have same proportion
6 – 5 = 1 hour
Train A take → 5 hour
Train E take→ 6 hour
Train A speed = 120 km/h
Train E = 100 km/h
For x –
(3.30 am – 10 .15 pm) – 3x = 5hours
3x = 5. 15 – 5
X = 5minutes
Speed of train B = 100 km/hr
Train cover total distance of 720 between Lucknow to Jaipur
720
= + 4(𝑥 + 10)
100
= 7.2 hours + 4(5+10)
= 7.2 hour + 60 minutes
= 8.2 hours
31 Adda247 Publications For any detail, mail us at
Publications@adda247.com
Cracker Book for Bank (IBPS | SBI | RRB PO | Clerk) Mains Exams

1 Adda247 Publications For any detail, mail us at


Publications@adda247.com
Cracker Book for Bank (IBPS | SBI | RRB PO | Clerk) Mains Exams

Chapter
Boat and Stream
9
BEST APPROACH TO SOLVE THE QUESTIONS

The questions based on boats and streams are simply based on the concept
of relative speed. Generally, two cases arise when a boat moves. Either the
boat moves along the flow of the stream or the boat moves in the direction
opposite to that of stream.

1. If boat moves in the direction of stream. The net speed is the


addition of the speed of stream and boat.
2. If boat moves in the direction opposite to that of stream, the net
speed is the difference of their speeds (speed of boat being more
than that of stream).

In questions related to boat and streams, usually it happens that one may
need to solve complex calculations but using a bit of intelligent guess work
one may save a good amount of time.

A sample problem shows one way of approach:

Example: ‘A man can row 24 km in upstream and 36 km in downstream in


9 hours. The same man can row 8 km in upstream and 48 km in
downstream in 6 hours. Calculate the speed of the stream.’

Sol. The basic approach to this question involves formation of two


equations, which are If we assume speed of stream as r and speed of boat as
x.
24 36
+ =9
𝑥−𝑟 𝑥+𝑟
8 48
+ =6
𝑥−𝑟 𝑥+𝑟

2 Adda247 Publications For any detail, mail us at


Publications@adda247.com
Cracker Book for Bank (IBPS | SBI | RRB PO | Clerk) Mains Exams

It’s cumbersome to solve these set of equations. So we need to look at


distance travelled in downstream or in upstream. In majority of cases it’s
better to take HCF of distance travelled in downstream in both cases. HCF of
36 and 48 is 12. Now we have to break 12 in two parts such that, adding or
subtracting them, completely divides all the four distance. Try (10,2) (9,3)
(8,4) and check whether these fulfill the condition. Or check from option.

In short we can only check this by hit and trial method.

Here, speed of stream=4km/hr and that of boat in still water is 8 km/hr.

Practice Exercise Based on New Pattern

1. A boat goes 28 km downstream and while returning covered only


75% of distance that covered in downstream. If boat takes 3 hr more
to cover upstream than downstream then find the speed of boat in
5
still water (km/hr) if speed of stream is m/sec ?
9
(a) 8 km/hr (b) 2 km/hr (c) 5 km/hr
(d) 4 km/hr (e) 3 km/hr

2. The ratio of time taken by boat A and boat B to swim a certain


distance downstream in a river is 3 : 4 respectively. The time taken by
boat B to cover some other distance in upstream is 50% more than
the time taken by it to cover the same distance in downstream. What
is the ratio of speed of boat A to that of boat B in still water?
(a) 7 : 5 (b) 7 : 4 (c) 7 : 3
(d) 7 : 9 (e) 7 : 2

3. There are three points P, Q and R in a straight line in a river such that
point Q is equidistant from point P and Point R. Boat travelled from
point P to R downstream in 8 hours and from point Q to P upstream in
12 hours. Find the ratio between speed of boat in still water to speed
of current?
(a) 3:1 (b) 2:1 (c) 1:2
(d) 5:2 (e) 2:3
3 Adda247 Publications For any detail, mail us at
Publications@adda247.com
Cracker Book for Bank (IBPS | SBI | RRB PO | Clerk) Mains Exams

4. A boat cover 60 km upstream and 60 km downstream in 22.5 hr with


its usual speed. If boat double its speed then new upstream speed is
150% more than the usual upstream speed. Find the time taken by
boat to cover 80 km in downstream with usual speed.
(a) 12 hr (b) 20 hr (c) 5 hr
(d) 16 hr (e) 10 hr

5. A boat covers total distance of 360km downstream in three equal


parts with the speed of boat be 4x km/hr, 5𝑥 km/hr and 7𝑥 km/hr
respectively. If speed of stream is x km/hr and boat takes total 29.5
hours, then find boat will cover total given distance upstream in three
equal parts with three givens speeds?
(a) 36 hours (b) 40 hours (c) 45 hours
(d) 48 hours (e) 54 hours

6. The speed of two boats A and B in the still water is in the ratio of 3 : 4
and the speed of current is 4km/hr. A start from point P ,45 minutes
earlier than B in downstream direction. If B catch boat A in three
hours then, find boat B takes how much time to cover 120 km distance
each in downstream and in upstream.
(a) 12 hours (b) 16 hours (c) 18 hours
(d) 14 hours (e) 20 hours

7. A boat has to travel from point P to point Q in upstream and reach Q


by 5.00 PM. If boat starts form point P at 2.00 PM with the speed of 20
km/hr and after travelling one hour the speed of current is increased
by 25% of its initial speed. If distance from point P to Q is 36 km and
speed of current is 8 km/hr, then find by what percent boat has to
increase its speed to reach point Q in time?
(a) 5% (b) 15% (c) 12%
(d) 10% (e) 17.5%

Directions (8-9): A boat ‘A’ start from point X, toward point Y in upstream
simultaneously second boat ‘B’ start from point Y to X having speed more
than first boat ‘A’. Difference between speed of both boats in still water is
equal to the speed of stream and they cross each other first time after 2
hours. Given that both boats start their respective return journey by
reaching their destination points.
4 Adda247 Publications For any detail, mail us at
Publications@adda247.com
Cracker Book for Bank (IBPS | SBI | RRB PO | Clerk) Mains Exams

8. If distance between X and Y is 56 km and speed of stream is 4 km/hr


then find the distance from ‘Y’ when boats cross each other 2nd time.
(a) 2.8 km (b) 2.4 km (c) 5.6 km
(d) 3.2 km (e) 4.8 km

9. Find the time taken by boats to cross each other on 2 nd time from the
first time if speed of second boat ‘B’ is 1.5 times of the speed of first
boat ‘A’ in still water?
(a) 5 hours (b) 3 hours (c) 2 hours
(d) 6 hours (e) 4 hours

10. Point A to Point B is a downstream journey of 300 km on a stream


which flows at a speed of 5 km/hr. Two boats P and Q starts from
point A and Point B respectively with speed of 25 km/hr and 15
km/hr in still water. After reaching the opposite point they return to
their starting points, find after how much time will they meet second
time?
(a) 7.5hour (b) 15hour (c) 20hour
(d) 10hour (e) None of these

11. Speed of current is 10 km/hr and speed of a motor boat is 80% more
than speed of current. Motor boat travels 280 km downstream with its
usual speed, after that it’s increased speed by ‘s’ kmph and travelled
for another 280 km then it returns and covers 560 km in upstream. If
boat complete whole journey downstream to upstream in 45 hr, then
find the value of ‘s’?
(a) 10 km/hr (b) 8 km/hr (c) 6 km/hr
(d) 12 km/hr (e) 4 km/hr

12. Speed of current is 5 km/hr and speed of boat in still water is 80%
more than speed of current.A boat travels 50% of total distance of 280
km, in downstream. After that the speed increase by x kmph because
of a man who added an engine. If boat complete whole journey
downstream and upstream in 45 hr, then find the value of x?
(a) 6 km/hr (b) 8 km/hr (c) 9 km/hr
(d) 4 km/hr (e) 5 km/hr
5 Adda247 Publications For any detail, mail us at
Publications@adda247.com
Cracker Book for Bank (IBPS | SBI | RRB PO | Clerk) Mains Exams

13. A lake in which speed of water current is zero has a stretch of 144 km.
Two boats, whose sum of speed in still water is 28 km/hr, starts from
two ends of lake. A fish starts swimming simultaneously with one boat
toward another boat and reach the second boat in 4 hours and then
returns toward first boat and touch it in 48 minutes. Find the speed of
fish.
(a) 12 km/hr (b) 16 km/hr (c) 24 km/hr
(d) 40 km/hr (e) 8 km/hr
3th
14. A boat goes certain distance downstream and then return of the
4
3
distance upstream.It takes of the time in upstream than in
2
1
downstream. If boat increases its speed by 33 % and cover a distance
3
of 60 km in downstream and then return upstream in 16 hours, find
increased speed of boat?
(a) 6 km/hr (b) 14 km/hr (c) 16 km/hr
(d) 12 km/hr (e) 8 km/hr
15. Vikas can swim in still water with twice the speed as that of speed of
water. Time difference to cover a certain distance in upstream and in
downstream is 4 hours at his usual speed. But if he had doubled his
usual speed, then time difference would reduce by 80% of its usual
time difference to cover same distance. Calculate total time taken by
Vikas to cover 6 km upstream and 2 km downstream, at his usual
speed.
3 5
(a) 1 hour (b) hour (c) hour
4 4
4
(d) hour (e) Can’t be determined
3

16. Speeds of three motor boats A, B and C are equal and all cover 8 km of
distance upstream in 48 minutes. Ratio between speed of motor boats
and speed of stream is 6 : 1. On first day A starts in downstream from
point P to Q, which shifts 9 km away from point P each day. On second
day B starts from point P in downstream and reach at point Q in 4.5
hours, then find time take by C on third day to reach at point Q,
(consider speed of stream same on all three days)?
1 1 1
(a) 4 ℎ𝑜𝑢𝑟𝑠 (b) 5 ℎ𝑜𝑢𝑟𝑠 (c) 3 ℎ𝑜𝑢𝑟𝑠
7 7 7
1 1
(d) 2 ℎ𝑜𝑢𝑟𝑠 (e) 9 ℎ𝑜𝑢𝑟𝑠
7 7
6 Adda247 Publications For any detail, mail us at
Publications@adda247.com
Cracker Book for Bank (IBPS | SBI | RRB PO | Clerk) Mains Exams

17. Usual speed of a boat is S km/hr. Boat rowing in a lake (neglect the
current speed) and covered distance of 360 km in three equal parts
with three different speeds in total 7.5 hours. If boat covered first part
with usual speed and second part with 20% increased speed and third
part with further increase of 25% of latest speed. Find the speed of
boat at which it covered third part of journey?
(a) 56km/hr (b) 36km/hr (c) 40km/hr
(d) 60km/hr (e) 48 km/hr

18. Ratio between speed of two boats A & B in still water is 4 : 3 and
speed of current is 4 km/hr .Boat B starts from point P in
downstream, two hours earlier than boat A and it is found that boat A
is16 km ahead of boat B after 12 hours of its start. If boat A start from
point Q in upstream and boat B start from point P in downstream at
same time and distance between P and Q is 108 km, then find in what
time boat A & B meet each other?
6 6 6
(a) 4 ℎ𝑜𝑢𝑟𝑠 (b) 5 ℎ𝑜𝑢𝑟𝑠 (c) 9 ℎ𝑜𝑢𝑟𝑠
7 7 7
6 6
(d) 3 ℎ𝑜𝑢𝑟𝑠 (e) 11 ℎ𝑜𝑢𝑟𝑠
7 7

19. When two ships A & B are travelling in opposite direction crosses
each other in 2 second. Speed of ship B is 50% more than the speed of
ship A in still water. On a particular day, ship A was ahead of ship B by
9 meters and both of them were travelling in downstream. If Ship B
overtook Ship A in 11 seconds and length of ship B is 54 meters then
find the speed of ship A.
(a) 56m/s (b) 36m/s (c) 40m/s
(d) 60m/s (e) 18m/s

20. A boat goes a certain distance downstream and then returns and
covers 40% of distance covered in downstream. Ratio of time taken in
covering downstream and upstream distances is 3 : 2. If speed of boat
in still water is reduced by 50% then it covers 60 km downstream in
10 hours. Find the speed of boat in still water.
(a) 9 km/hr (b) 8 km/hr (c) 6 km/hr
(d) 10 km/hr (e) 12 km/hr

7 Adda247 Publications For any detail, mail us at


Publications@adda247.com
Cracker Book for Bank (IBPS | SBI | RRB PO | Clerk) Mains Exams

Solutions

5 18
1. (c); Speed of current = ×
9 5
= 2 km⁄hr
Let’s still water speed = x km/hr
ATQ,
3
28× 28
4

(x–2) (x+2)
=3
21x + 42 – 28x + 56 = 3x² – 12
–7x + 98 = 3x² – 12
3x² + 7x – 110 = 0
x = 5 km/hr

2. (a); Let total distance be D km.


And, speed of boat A in still water be
x km/hr
Speed of boat B in still water be y km/hr
Speed of stream be r km/hr
ATQ -
𝐷 𝐷 3
∶ =
𝑥+𝑟 𝑦+𝑟 4
𝑦+𝑟 3
Or, = ……..(i)
𝑥+𝑟 4
Also, for a distance D1,
𝐷1 3 𝐷1
= ( )
𝑦−𝑟 2 𝑦+𝑟
or, 2𝑦 + 2𝑟 = 3𝑦 − 3𝑟
or, 𝑦 = 5𝑟
Putting this in (i),
6𝑟 3
=
𝑥+𝑟 4
or, 3𝑥 + 3𝑟 = 24𝑟
or, 𝑥 = 7𝑟
Ratio of speed of boat A and B in still water= 7r : 5r
=7:5

8 Adda247 Publications For any detail, mail us at


Publications@adda247.com
Cracker Book for Bank (IBPS | SBI | RRB PO | Clerk) Mains Exams

3. (b); Let still water speed x km/hr and


Speed of current y km/hr —
Downstream speed = (x + y) km/hr
Upstream speed = (x – y) km/hr
Let distance between P to R is 2ℓ and Q to P is ℓ km
ATQ—
2ℓ
(x+y)
=8 … (i)
and

(x–y)
= 12 … (ii)
From (i) and (ii)
x 2
=
y 1

4. (e); Let usual speed of boat in still water = x


River speed = y
ATQ,
(𝑥−𝑦)250
= (2𝑥 − 𝑦)
100
5𝑥 − 5𝑦 = 4𝑥 − 2𝑦
x = 3y
Now,
60 60
+ = 22.5
x –y x+y
60 60
+ = 22.5
2y 4y
y = 2 km/hr
x = 6 km/hr
80
Required time =
6+2
= 10 hr

5. (c); ATQ—
120 120 120
(4𝑥+𝑥)
+ (5𝑥+𝑥)
+ (7𝑥+𝑥)
= 29.5
120 120 120
+ + = 29.5
5𝑥 6𝑥 8𝑥
5760+4800+3600
=𝑥
240×29.5
𝑥=2
120 120 120
Required time = + +
3𝑥 4𝑥 6𝑥
1 1 1 3
= 120 ( + + ) = 120 × = 45 ℎ𝑜𝑢𝑟𝑠
6 8 12 8
9 Adda247 Publications For any detail, mail us at
Publications@adda247.com
Cracker Book for Bank (IBPS | SBI | RRB PO | Clerk) Mains Exams

6. (b); Let speed of boat A and B in still water be 3𝑥 km/hr and 4𝑥


km/hr respectively
Downstream speed for boat A
= (3𝑥 + 4) km/hr
Downstream speed for boat B
= (4𝑥 + 4) km/hr
Relative speed of boat A and B
= (4𝑥 + 4) – (3𝑥 + 4) = 𝑥 km/hr
ATQ—
45 (3𝑥+4)
× =3
60 𝑥
9𝑥 + 12 = 12𝑥
𝑥 = 4 km/hr
Speed of boat B = 4 × 4 = 16 km/hr
120 120
Required times = (16+4) + (16–4)
= 6 + 10 = 16 hours

7. (d); Boat travels total distance in the hours


Upstream speed of boat = 20 – 8 = 12 km/hr
In one hour, boat will cover a distance of
= 12 × 1 = 12 𝑘𝑚
Remaining distance = 36 – 12 = 24 km
To reach Q at 5.00 PM boat has to cover remaining distance at
speed
24
= = 12 𝑘𝑚/ℎ𝑟
2
125
New speed of stream = 8 × = 10 𝑘𝑚/ℎ𝑟
100
New upstream speed of boat =20 – 10
= 10 km/hr
For 12 km/hr speed boat has to increase its speed by 2 km/hr
2
Required percentage = × 100 = 10%
20

10 Adda247 Publications For any detail, mail us at


Publications@adda247.com
Cracker Book for Bank (IBPS | SBI | RRB PO | Clerk) Mains Exams

Solutions (8-9): Let speed of first boat ‘A’


= a km/hr
Speed of stream = b km/hr
So,
Speed of 2nd boat ‘B’ = 𝑎 + 𝑏 km/hr

Distance XY ⇒ 2 × (a – b + a + b + b) = 4a + 2b

8. (d); Given XY = 56 = 4𝑎 + 2𝑏
2𝑎 + 𝑏 = 28
Now speed of stream = 𝑏 = 4 km/hr
2𝑎 + 4 = 28
𝑎 = 12
Speed of first boat ‘A’ in still water = 12
Speed of 2nd boat ‘B’ in still water = 16
First time crossing

Time taken by 2nd boat ‘B’ to reach at X after first crossing


16 4
= = hour
20 5
4
Distance travelled by first boat ‘A’ in hour
5
4 32
= × 8 = km
5 5
Now,
Distance between first boat ‘A’ and 2nd boat ‘B’
32
16 + ⇒ 22.4 km
5
On return journey of second boat ‘B’, speed of boat = 12 km/hr
Time taken by first boat to reach Y
56–22.4
= = 4.2 hours
8
Distance remaining between both boat after reaching of first
boat ‘A’ at Y
⇒ 56 – 4.2 × (12 – 8) = 5.6 km
Time taken by both boat to cover this distance
5.6
= = 0.2 hours
12+16
Distance from Y = 0.2 × 16 = 3.2 km
11 Adda247 Publications For any detail, mail us at
Publications@adda247.com
Cracker Book for Bank (IBPS | SBI | RRB PO | Clerk) Mains Exams

9. (b); If speed of first boat ‘A’ = 𝑎 km/hr


3
Speed of second boat ‘B’ ⇒ 𝑎 km/hr
2
3 𝑎
So speed of stream ⇒ 𝑎– 𝑎 = km/hr
2 2
Distance travelled by first boat till first crossing
𝑎
⇒ (𝑎– ) × 2
2
⇒ 𝑎 km
𝑎
Speed of first boat in upstream = km/hr
2
Speed of 2 boat in downstream
nd

= 2𝑎 km/hr

𝑎 1
2nd boat reached at X ⇒ = ℎ
2𝑎 2
1 1 𝑎 𝑎
In hour first boat cover ⇒ × = km
2 2 2 4
𝑎
Distance between both boats ⇒ 𝑎 +
4
5𝑎
⇒ km
4
Relative speed of boat ‘A’ and ‘B’ when they cross each other
3𝑎 𝑎 𝑎 𝑎
= ( – – )=
2 2 2 2
5𝑎
Time taken to cross
4
5𝑎×2
⇒ = 2.5 hour
4×𝑎
So total time for 2nd crossing
⇒ 2.5 + 0.5 = 3 hours

10. (c); Time when P and Q meet first time


300 300
= = = 7.5 hr
25+15 40
Distance travelled by P = 7.5 × (25 + 5)
= 225 km
Distance travelled by Q = 7.5 × (15 – 5)
= 75 km
75
P reach at point B in i.e. 2.5 hour
30
In 2.5hr Q travelled 2.5 × 10 = 25 km

12 Adda247 Publications For any detail, mail us at


Publications@adda247.com
Cracker Book for Bank (IBPS | SBI | RRB PO | Clerk) Mains Exams

Now Boat P returns and relative distance b/w P and Q is 100 km


and relative speed is 10 km i.e. difference of speed of P (25 -5) =
20 km/hr (upstream) and speed of Q
(15 – 5) = 10 km/hr (upstream)
Time taken by P and Q to meet
100
= = 10 hr
10
Total time when P and Q meet second time
= 7.5 +2.5 + 10 = 20 hr

11. (d); speed of boat in still water


80
= (10 + 10 × ) km⁄hr
100
= 18 km⁄hr
ATQ—
280 280 560
(18+10)
+ (18+10)+s + (18–10)+s = 45
280 560
+ = 35
28+s 8+s
8 16
+ =1
28+s 8+s
64 + 8s + 448 + 16s = 224 + 28s + 8s + s2
s² + 12x – 288 = 0
s = 12 km/hr

80
12. (a); Still water speed = (5 + 5 × ) km⁄hr
100
= 9 km⁄hr
ATQ—
140 140 280
(9+5)
+ (9+5)+x + (9–5)+x = 45
140 280
+ = 35
14+x 4+x
4 8
+ =1
14+x 4+x
12x + 128 = 56 + 14x + 4x + x²
x² + 6x – 72 = 0
x = 6 km/hr

13 Adda247 Publications For any detail, mail us at


Publications@adda247.com
Cracker Book for Bank (IBPS | SBI | RRB PO | Clerk) Mains Exams

80
12. (a); Still water speed = (5 + 5 × ) km⁄hr
100
= 9 km⁄hr
ATQ—
140 140 280
(9+5)
+ (9+5)+x + (9–5)+x = 45
140 280
+ = 35
14+x 4+x
4 8
+ =1
14+x 4+x
12x + 128 = 56 + 14x + 4x + x²
x² + 6x – 72 = 0
x = 6 km/hr

13. (c);

Let speed of boat 2 and speed of fish is x and y respectively.


Boat 2 and fish meet in 4 hour
So,
(x + y) × 4 = 144 …(i)
Now
ATQ,
Speed of boat 1 = 28 – x
In 4 hour boat 1 cover = 4 × (28 – x)
48
× (𝑦 + (28 – 𝑥 )) = 144 – [4𝑥 + 4(28 − 𝑥 )]
60
4
(𝑦 − 𝑥 ) + 22.4 = 144 − 112
5
𝑦 − 𝑥 = 12 … … . (𝑖𝑖)
Sovling (i) & (ii)
x = 12 km/hr
y = 24 km/hr

14 Adda247 Publications For any detail, mail us at


Publications@adda247.com
Cracker Book for Bank (IBPS | SBI | RRB PO | Clerk) Mains Exams

14. (e); Let distance travelled by boat in downstream be D and speed of


boat in still water be x km/hr and speed of current be y km/hr
ATQ,
3
×𝐷 3 𝐷
4
= ×
(𝑥−𝑦) 2 (𝑥+𝑦)
1 1
=
2(𝑥−𝑦) 𝑥+𝑦
x = 3y
1
New sped of boat = 3y + 3y ×
3
= 4 y km/hr
60 60
+ = 16
(4𝑦+𝑦) (4𝑦−𝑦)
12 20
+ = 16
𝑦 𝑦
32
y = = 2 𝑘𝑚/ℎ𝑟
16
Increased speed of boat = 4×2 = 8 km/hr

15. (e); Let the speed of water is x km/hr, then speed of vikas in still
water is 2x km/hr
From 1st condition,
𝐷 𝐷
– =4
2𝑥–𝑥 2𝑥+𝑥
𝐷 𝐷
⇒ – =4
𝑥 3𝑥
⇒ D = 6𝑥 …(i)
From 2nd condition, when speed of Vikas is 4𝑥 km/hr
𝐷 𝐷 20
– = ×4
4𝑥–𝑥 4𝑥+𝑥 100
𝐷 𝐷 4
⇒ – =
3𝑥 5𝑥 5
⇒ D = 6𝑥 …(ii)
Hence both equations are same so answer can’t be determined.

16. (b); Speed of motor boats in upstream


60
= 8 × = 10 km/hr
48
ATQ,
Let speed of motor boats be 6x km/hr and speed of stream be x
km/hr
6x – x = 10
15 Adda247 Publications For any detail, mail us at
Publications@adda247.com
Cracker Book for Bank (IBPS | SBI | RRB PO | Clerk) Mains Exams

x = 2 km/hr
Downstream speed of all boats
= (6 × 2 + 2)
= 14 km/hr
Let distance between point P to Q on first day = y km
Second day distance = (y + 9)
𝑦+9
14 =
4.5
y= 63 – 9
y = 63 – 9
y= 54 km
Distance travelled on third day = 54 + 9 × 2
= 72 km
Total time taken by boat C on third day to reach point Q
72 1
= = 5 hours
14 7

17. (d); Three equal parts of journey


360
= = 120 km
3
ATQ—
120 120 120
+ S + 6S 6S 1 = 7.5
S S+5 + ×
5 5 4
120 600 120×10
+ + = 7.5
S 6S 12S+3S
3600+3000+2400 15
=
30S 2
9000 15
=
30S 2
9000×2
S=
30×15
S = 40 km/hr
Speed of which boat which it covered third part of Journey
20 20 1
= (40 + 40 × ) + (40 + 40 × ) ×
100 100 4
= 60 km/hr

18. (d); Let speed of boat A & B in still water be 4x km/hr and 3x km/hr
respectively
Downstream speed of boat A
= (4x + 4) km/hr
16 Adda247 Publications For any detail, mail us at
Publications@adda247.com
Cracker Book for Bank (IBPS | SBI | RRB PO | Clerk) Mains Exams

Downstream speed of boat B


= (3x + 4) km/hr
AT Q –
12 (4x + 4) – 16 = 2 (3x + 4) + 12 (3x + 4)
48x + 48 − 16 = 6x + 8 + 36x + 48
6x = 24
x = 4 km/hr
Speed of boat A in still water = 16 km/hr
Speed of boat B in still water= 12 km/hr
Relative speed = (16 – 4) + (12 + 4)
= 28 km/hr
108
Required time =
28
6
= 3 ℎ𝑜𝑢𝑟𝑠
7

19. (e); Let speed of ship A in still water is 2x m/s , and its length is ℓ m
And speed of ship B is 3x m/s and its length is 54 and speed of
water is Y m/s.
ATQ, when both of them are travelling in opposite direction.
Downstream speed of ship A = (2x + Y) m/s [assume ship A is
travelling in downstream and ship B in upstream]
Upstream speed of ship B = (3x – Y) m/s
Their relative speed = 2x + Y + 3x – Y
= 5x m/s
ATQ,
54+ℓ
= 2 …(i)
5x
Similarly when both are travelling in downstream their relative
speed is
= 3x + Y – 2x –Y= x m/s
ATQ,
54+9+ℓ
= 11
𝑥
63 + ℓ = 11x …(ii)
From (i)
ℓ = 10x – 54
put this value in eqn. (ii)
= 63 + 10x – 54 = 11x
x = 9 m/s
Hence speed of ship A in still water = 2 × 9 = 18 m/s
17 Adda247 Publications For any detail, mail us at
Publications@adda247.com
Cracker Book for Bank (IBPS | SBI | RRB PO | Clerk) Mains Exams

20. (b); Let speed of boat in still water and speed of current be
respectively x km/hr and y km/hr
And Let time taken to cover given distance in downstream and
upstream be = 3m hours and 2m hours
40% of distance in upstream is covered in 2 m hour
100% of distance upstream is covered in 5m hours
According to question
x−y 3
=
x+y 5
5x - 5y = 3x + 3y
x 4
⇒ =
y 1
And
60
x = 10
(2+y)
x
+y=6
2
4𝑦
+𝑦=6
2
6y = 12
y = 2 km/hr
so, x = 8 km/hr

18 Adda247 Publications For any detail, mail us at


Publications@adda247.com
Cracker Book for Bank (IBPS | SBI | RRB PO | Clerk) Mains Exams

1 Adda247 Publications For any detail, mail us at


Publications@adda247.com
Cracker Book for Bank (IBPS | SBI | RRB PO | Clerk) Mains Exams

Chapter
Mensuration
10
BEST APPROACH TO SOLVE THE QUESTIONS

Questions related to mensuration usually don’t involve any short-cut


methods for solving a problem. The reason being how can one use short
tricks when all one has to do is to deal with formulae. To solve questions
faster and in a lucid manner, one must avoid doing unnecessary and
multiple calculations. Rather one must calculate only where it is needed. In
case, ratio or percentage is asked in the question, then it becomes more
easy to do away with hefty calculations and arrive at the answer by using
basic elimination methods.

‘The length, breadth and height of a room are in the ratio 3:2:1. If the
breadth and height are halved while the length is doubled, then the
percentage change in total area of the four walls of the room.’

Area of four walls of the room= 2(lh+bh), where l,b and h are length,
breadth and height of the room respectively.

As we have been asked the percentage change, let’s take length, breadth and
height of the room to be 6,4 and 2 respectively.

Then changed values of length, breadth and height would be 12, 2 and 1.
Initial area of four walls= 2 (6 × 2 + 4 × 2) = 40
Final area of four walls= 2 (12× 1 + 2 × 1)= 28
28 is 70% of 40, which means there has been a reduction of 30%.

‘A solid cylinder has height 10 meters and radius R meters. Some part
of it is melted and casted into two cones. One of the cones has radius
3cm and height 4cm. The other cone has height 8 cm and radius 6cm. If
the ratio of initial curved surface area of cylinder to the sum of curved
surface areas of two cones is 4:1, then find the radius of the cylinder.’

2 Adda247 Publications For any detail, mail us at


Publications@adda247.com
Cracker Book for Bank (IBPS | SBI | RRB PO | Clerk) Mains Exams

Slant height of cone having radius 3cm is 5cm (PLEASE SOLVE IT


PROPERLY)
Slant height of cone having radius 6 cm is 10 cm
Curved surface area of a cone is πrl and that of cylinder is 2πrh
2𝜋×𝑅×10 4
ATQ, =
𝜋(3×5+6×10) 1
➢ R=15cm

Practice Exercise Based on new Pattern

1. The ratio of the radius and height of a cone is


2
5 : 12. Its volume is 314 cm. Then its slant height is ?
7
(a) 9 cm (b) 10 cm (c) 11 cm
(d) 12 cm (e) 13 cm

2. If the perimeter of a rectangle and a square are equal and the ratio of
2 adjacent sides of a rectangle is 2:1. Then find the ratio of area of
rectangle and area of square.
(a) 1:1 (b) 1:2 (c) 2:3
(d) 8:9 (e) 9:4

3. The parameter of a square is equal to the parameter of a rectangle


having dimensions 24 cm × 16 cm. Find the circumference of a circle
whose diameter is equal to the side of square ?
360 410 470
(a) cm (b) cm (c) cm
7 7 7
440 460
(d) cm (e) cm
7 7

4. Total surface area of a cylinder mounted with a hemispherical bowl


on one end is 2552 cm². If height of cylinder is 8 cm then find the
volume of the solid body?
1 1 1
(a) 10443 (b) 10677 (c) 10547
3 3 3
1 1
(d) 10977 (e) 10787
3 3

3 Adda247 Publications For any detail, mail us at


Publications@adda247.com
Cracker Book for Bank (IBPS | SBI | RRB PO | Clerk) Mains Exams

5. Sum of Area of circle and a rectangle is equal to 622 sq cm. The


diameter of the circle is 14 cm, then what is the sum of circumference
of the circle and the perimeter of the rectangle if the length of
rectangle is 26 cm?
(a) 142cm (b)132cm (c)152cm
(d) 140cm (e)134cm

6. A copper sphere of diameter 42 cm is drawn into a wire of diameter


70 mm. Then, the length of the wire is?
(a) 243 cm (b) 343 cm (c) 443 cm
(d) 972 cm (e) None of these

7. A cylindrical roller rolls over a square field. It takes 500 rounds to


cover the complete field. If the radius of the cylindrical roller is 0.63
meter and length is 7.7 m then find out the side of square
(approximately).
(a) 135 m (b) 125 m (c) 100 m
(d) 150 m (e) 110 m

8. A cuboid is cut along its base such that, now it become a cube. Cube
again cut along its base, now ratio of height to length becomes 1 : 𝑎.
Find the value of ‘𝑎’.
(a) 𝑎 > 1 (b) 𝑎 = 1 (c) 𝑎 ≥ 1
(d) 𝑎 < 1 (e) 𝑎 ≤ 1

9. Volume of a cone and cylinder, having same base, are in ratio of 4 : 3.


If height and radius of cone is 4 : 3 then find the ratio of total surface
area of cone and that of cylinder.
(a) 1 : 1 (b) 3 : 1 (c) 5 : 3
(d) 2 : 1 (e) 4 : 1

10. If the volume and curved surface area of a cylinder 616 m3 and 352m2
respectively, what is the total surface area of the cylinder (in m2 )?
(a) 429 (b) 419 (c) 435
(d) 421 (e) 417
4 Adda247 Publications For any detail, mail us at
Publications@adda247.com
Cracker Book for Bank (IBPS | SBI | RRB PO | Clerk) Mains Exams

11. Area of a rectangle is 252 cm². If length of the rectangle is equal to


side of a square, whose perimeter 72 cm then find the volume of cone,
whose radius is equals to breadth of rectangle and height equals to
length of rectangle?
(a) 4752 cm2 (b) 3586 cm2 (c) 4026 cm2
(d) 4246 cm2 (e) 3696 cm2

12. Sum of length, breadth and height of cuboid is 12 cm and length of its
diagonal is 5√2. Then find the total surface area of cuboid.
(a) 94 cm² (b) 84 cm² (c) 72 cm²
(d) 64 cm² (e) 90 cm²

13. Circumference of a circular garden is 66 cm and area of circular


garden is 25% more than perimeter of a square hall. Find the area of
square hall ?
(a) 4802.49 sq cm (b) 4704.48 sq. cm
(c) 4820.49 sq.cm (d) 4822.49 sq.cm
(e) none of these

14. Internal radius of pipe is 3 cm and the external radius of pipe is 5 cm


and the length of pipe is 15 cm then find total surface area of pipe ?
(a) 278 π cm² (b) 275 π cm² (c) 268π cm²
(d) 272 π cm² (e) none of these

15. A solid cube is melted to form 3 spheres, with radius, 1 cm, 2 cm and 3
cm and additional irregular shape was formed. Calculate the volume
of irregular shape (upto two decimal place) if diagonal of cube is 6√3
cm.
(a) 65.14 𝑐𝑚3 (b) 70.35 𝑐𝑚3 (c) 54.75 𝑐𝑚3
(d) 50.00 𝑐𝑚3 (e) 78.90 𝑐𝑚3

16. 20% increase in both radius and height of the cylinder increases the
total surface area of cylinder by 677.6 cm², If the ratio of radius to
height is 1 : 4, then find the radius of cylinder
(a) 21 cm (b) 10.5 cm (c) 3.5 cm
(d) 14 cm (e) 7 cm
5 Adda247 Publications For any detail, mail us at
Publications@adda247.com
Cracker Book for Bank (IBPS | SBI | RRB PO | Clerk) Mains Exams

17. A Cone cylinder and hemisphere have equal radius and height. Find
the ratio of total surface area of cylinder, cone and hemisphere.
(a) 4 ∶ √2: √6 (b) 4 ∶ √2 ∶ 3
(c) √2 + 1 ∶ 4 ∶ 3 (d) 3 ∶ 4 ∶ √2 + 1
(e) 4 ∶ √2 + 1 ∶ 3

18. Sum of circumference of a circle and perimeter of a rectangle is 220


1
cm while area of circle is 1386 sq. cm. If length of rectangle is 33 %
3
more than radius of the given circle then find the area of rectangle?
(a) 408 𝑐𝑚2 (b) 418 𝑐𝑚2 (c) 428 𝑐𝑚2
(d) 448 𝑐𝑚2 (e) 438 𝑐𝑚2

19. Four identical coins are placed in a square. For each coin, area of each
coin is equal to circumference of each coin. Then, find the area of the
square that is not covered by the coins

(a) 16(π − 1) (b) 16(8 − π) (c) 16(4 − π)


π
(d) 16 (4 − ) (e) None of these
2

20. The surface area of a sphere is 423.5 cm2 less than total surface area
of a hemisphere. If ratio between radius of hemisphere and sphere is
3 : 2, then find the radius of hemisphere?
(a) 5.5 cm (b) 5 cm (c) 4 cm
(d) 7 cm (e) 10.5 cm

21. A cylindrical jar, whose base has a radius of 15 cm is filled with water
up to a height of 20 cm. A solid iron spherical ball of radius 10 cm is
dropped in the jar to submerge completely in water. Find the increase
in the level of water (in cm) is
17 5 8
(a) 5 (b) 5 (c) 5
27 7 9
25 7
(d) 5 (e) 5
27 27

6 Adda247 Publications For any detail, mail us at


Publications@adda247.com
Cracker Book for Bank (IBPS | SBI | RRB PO | Clerk) Mains Exams

22. The dimensions of a field are 20 m and 9 m. A pit 10 m long, 4.5 m


wide and 3 m deep is dug in one corner of the field and the earth
(mud) removed has been evenly spread over the remaining area of
the field. What will be the rise the height of field as a result of this
operation ?
(a) 1.5 m (b) 2 m (c) 3 m
(d) 4m (e) 1 m

23. Difference between perimeter of two rectangles is 8 cm. If breadth of


both rectangle is 16 cm and length are in the ratio of 7 : 6 then find the
total surface area of cylinder whose radius is half of the length of
larger rectangle and height equal to length of smaller rectangle?
(a) 3344 cm2 (b) 3668 cm2 (c) 3444 cm2
(d) 3468 cm2 (e) 3462 cm2

Directions (24-25); There are three toys which are in cylindrical, conical
and spherical shape. Height of all the toys is same. Ratio between the height
of cylinder to radius of cylinder is 4 : 3.

24. Find the ratio between total surface area of cylinder to that of sphere.
(a) 17 : 15 (b) 21 : 16 (c) 28 : 23
(d) 21 : 8 (e) 15 : 4

25. Volume of cone is what percent of the volume of sphere if radius of


cone is equal to that of cylinder
(a) 112.5% (b) 100% (c) 125%
8
(d) 75% (e) 88 %
9

26. A well whose shape is of equilateral triangle having side 8 cm has to


be dug 44√3 cm deep. The earth taken out of the well is used to fill a
cylindrical tank having diameter 8 cm. Find the height up to which
earth is filled in the tank.
(a) 10.5 cm (b) 21 cm (c) 42 cm
(d) 84 cm (e) 32 cm

7 Adda247 Publications For any detail, mail us at


Publications@adda247.com
Cracker Book for Bank (IBPS | SBI | RRB PO | Clerk) Mains Exams

27. A circle is inscribed in a square. If the difference between area of the


square and circle is 262.5 cm², then find the area of the rectangle
whose perimeter is same as that of circle while length of rectangle is
20% more than the breadth of rectangle (in cm²)
(a) 1500 (b) 1400 (c) 700
(d) 750 (e) 3000

28. Inside a square plot a circular garden is developed which exactly fits
in the square plot and the diameter of the garden is equal to the side
of the square plot which is 28 metre. What is the area of the space left
out in the square plot after developing the garden?
(a) 98 m2 (b) 146 m2 (c) 84 m2
(d) 168 m2 (e) 68 m2

29. The area of a rectangle gets reduced by 9m2 if its length is reduced by
5 m and breadth is increased by 3m. If we increase the length by 3 m
and breadth by 2 m, the area is increased by 67 m2. The length of the
rectangle is :
(a) 9 m (b) 15.6 m (c) 17 m
(d) 18.5 m (e) 19 m

30. A toymaker makes a toy in which a cone is mounted on the base of a


hemisphere. If the total surface area of the toy is 858 cm² then find the
volume of the toy, given that the diameter of the toy is 14 cm?
2 2
(a) 1950 cm3 (b) 1250 cm3 (c) 1400 cm3
3 3
3
(d) 1500 cm (e) 1200 cm3

31. If sum of circumference of a circle and perimeter of rectangle is 204


cm and perimeter of rectangle is 50% of magnitude of area of square
and side of square is 12 cm .If length of rectangle is 8 cm more than
that of breadth, then find sum of area of circle and rectangle ?
(a) 1694sq cm (b) 1450sq cm (c) 1550sq cm
(d) 1750sq cm (e) 1460sq cm
8 Adda247 Publications For any detail, mail us at
Publications@adda247.com
Cracker Book for Bank (IBPS | SBI | RRB PO | Clerk) Mains Exams

32. A sphere is melted into ‘n’ number of small spheres. Total surface area
of these small spheres is what percent more or less than total surface
area of large sphere, if the ratio of radius of small sphere to radius of
large sphere is 1 : 3.
(a) 50% (b) 100% (c) 150%
(d) 200% (e) 75%

33. Ratio of height of cylinder to that of diameter is 2: 3. This cylinder is


formed by melting a sphere having same radius as the radius of circle.
Ratio of magnitude of area of circle to magnitude of its circumference
is 21: 2. Find height of cylinder.
(a) 14 units (b) 21 units (c) 42 units
(d) 28 units (e) 35 units

34. If the curved surface area of first cone is thrice that of second cone
and slant height of the second cone is thrice that of the first, find the
ratio of the area of their base.
(a) 81 : 1 (b)9 : 1 (c)3 : 1
(d)27 : 1 (e) None of these

35. There are two cylindrical vessels with same capacity and same
dimensions. If radius of one vessel increased by 7 cm and height of
second vessel increases by 15 cm, then capacity of both vessel
increased by equality ‘Q’ cm3. If actual height of both vessels is 12 cm,
then find value of Q?
(a) 9840 cm3 (b) 9240 cm3 (c) 9460 cm3
(d) 9160 cm3 (e) 9780 cm3

36. There are four light poles in four corners of a park, four at midpoint of
each side and one pole in the middle of that square park. Area of park
is 19600 cm2. If side of park is 50 times of radius of each light pole,
then find the total base area covered by these poles if pole’s base is
circular in shape?
(a) 197.12 cm2 (b) 172.48 cm2 (c) 246.4 cm2
(d) 220.76 cm2 (e) 221.76 cm2

9 Adda247 Publications For any detail, mail us at


Publications@adda247.com
Cracker Book for Bank (IBPS | SBI | RRB PO | Clerk) Mains Exams

37. Area of a square is 306.25 cm2 and breadth of rectangle is 3.5 cm less
than side of square. Area of rectangle is 252 cm2 and height and radius
of a cylindrical vessel equal to length & breadth of rectangle
respectively. If vessel filled, contains mixture of milk and water in
which milk is 75%, then find the volume of water contained by
cylindrical vessel?
(a) 2572 cm3 (b) 2472 cm3 (c) 2772 cm3
(d) 2872 cm2 (e) 2878 cm2

Directions (38-39); A hemispherical bowl is filled with hot water to the


brim. The contents of the bowl are transferred into a cylindrical vessel
whose radius is 50% more than its height.

38. If diameter of the bowl is the same as that of the cylindrical vessel,
then the volume of the hot water in the cylindrical vessel is
(a) 60% of the cylindrical vessel
(b) 80% of the cylindrical vessel
(c) 100% of the cylindrical vessel
(d) 50% of the cylindrical vessel
(e) none of these

39. This hemispherical bowl is joined at one end of the cylindrical vessel
and the solid obtained by combining these figure is filled completely
with water. If a drain pipe is connected to it then pipe will empty 539
cm³ of water is one minute. If radius of the solid is 21 cm, then in what
time whole of the water will be emptied from the solid. Given that
diameter of cylinder and hemisphere are same.
(a) 72 min (b) 60 min (c) 90 min
(d) 75 min (e) None of these

40. A cylinder whose height is equal to its radius is full of milk. Its milk is
poured into a hemi spherical bowl of same radius as of cylinder and
remaining milk is poured into another hemi-spherical bowl of same
volume as of previous one. What percentage of the volume of second
hemispherical bowl remained empty?
(a) 40% (b) 66⅔% (c) 33⅓%
(d) 50% (e) 100%
10 Adda247 Publications For any detail, mail us at
Publications@adda247.com
Cracker Book for Bank (IBPS | SBI | RRB PO | Clerk) Mains Exams

41. In a house, there is a semicircular corridor, a circular hall and a


circular garden. Sum of perimeter of corridor and hall is 102 m.If
radius of hall is 50% more than radius of corridor then find area of
circular garden, given radius of garden is 300% more than sum of
radius of corridor and radius of hall together.
(a) 6300𝜋 sq.m (b) 5400𝜋 sq.m (c) 5700𝜋 sq.m
(d) 4200𝜋 sq.m (e) 4900𝜋 sq.m

42. Height of a cylindrical vessel is equal to side of a square, whose area is


256 cm². Radius of cylindrical vessel is equal to average of radius of a
sphere and hemisphere. If ratio between the radius of hemisphere and
that of the sphere is 2 : 1 and difference between their total surface
area is 4928 cm², then find volume of cylindrical vessel ?
(a) 21,288 cm³ (b) 20,176 cm³ (c) 22,176 cm³
(d) 26,176 cm³ (e) 24,288 cm³

43. Radius of a cylinder is half of the side of square, whose perimeter is 6


cm more than perimeter of rectangle. If ratio between length to
breadth of rectangle is 7 : 2 and between length of rectangle to side of
square is 3 : 2. Find volume of cylinder, given height of cylinder is
equal to breadth of rectangle?
(a) 24,938 cm3 (b) 24,948 cm3 (c) 24,848 cm3
(d) 24,996 cm3 (e) 24,918 cm3

44. A solid sphere of some radius is melted and re-casted into a number of
small cylinders and cones. The height of the cone is 33⅓% less than
the height of the cylinder which is 6 cm and the ratio of radius of cone
to the radius of cylinder is √3 : 1 and the ratio of number of cylinder
to the number of cone is 2 : 3. Find the total surface area of the sphere
if the number of cylinders are 12 and the radius of cylinder is 4 cm.
1 2 4
(a) 1810 cm² (b) 1810 cm² (c) 1810 cm²
7 7 7
11 9
(d) 905 cm² (e) 905 cm²
14 14

11 Adda247 Publications For any detail, mail us at


Publications@adda247.com
Cracker Book for Bank (IBPS | SBI | RRB PO | Clerk) Mains Exams

45. Radius of a cylindrical vessel is 1.5 cm less than breadth of rectangle,


while height of cylindrical vessel is equal to length of rectangle, whose
area is 168 cm2. If ratio between length & breadth of rectangle is 7 : 6,
then find the think ness of 1 disc, if these discs are kept one above
another in cylindrical vessel and 10 discs get completely fit in
cylindrical vessel.(Let radius of discs equal to radius of cylindrical
vessel)?
(a) 1.2 cm (b) 1.6 cm (c) 1.8 cm
(d) 1.4 cm (e) 1.0 cm

46. The ratio between radius of two hemispheres solid tin pieces is 2 : 3 and
difference between volume of both is 836/21 cm3. These two
𝜋
hemispheres are melted into a cylindrical vessel and used 74 cm3
3
extra tin material for polishing the vessel. If ratio between height &
radius of cylindrical vessel is 3 : 4, then find the total surface area of
cylindrical vessel?
(a) 154 cm2 (b) 132 cm2 (c) 176 cm2
(d) 208 cm2 (e) 198 cm2

Solutions

1. (e); Let, radius = 5X and height = 12X cm.


2200
Volume =
7
1 22 2200
= × × 12X × 5X × 5X =
3 7 7
X=1
r = 5 and h = 12
ℓ = √52 + 122
= √25 + 144
= √169
= 13 cm

2. (d); let side of a rectangle=a, 2a, side of a square=b


2
2(a+2a)=4b , a= b
3
8
Area of rectangle=a×2a=2a2 or ⌈ ⌉b2
9
Area of square=b 2

Ratio = 8:9
12 Adda247 Publications For any detail, mail us at
Publications@adda247.com
Cracker Book for Bank (IBPS | SBI | RRB PO | Clerk) Mains Exams

3. (d); Parameter of square = parameter of rectangle


= 2 (24 + 16)
4a = 80 cm
Side of square = 20 cm
20
Radius of circle = = 10 cm
2
Circumference of circle
22 440
2πr = × 20 = cm
7 7

4. (b); T.S.A = 2πr² + πr² + 2πrh = 2552


3πr² + 2πr × 8 = 2552
2552
3r² + 16r = ×7
22
3r² + 16r = 812
3r² + 16r – 812 = 0
3r² + 58r – 42r – 812 =0
r(3r² + 58) – 14(3r + 58) = 0
58
r = 14, −
3
2
Required volume = πr 3 + πr 2 h
3
2 22 22
= × × (14 )3 + × (14)2 × 8
3 7 7
1
= 5749 + 4928
3
1
= 10677
3

14
5. (b); Radius of circle = = 7
2
Area of circle
22
= × 7 × 7 = 154 sq. cm
7
Rectangle area = 622 – 154 = 468 sq. cm
Breadth of rectangle
468
= = 18 cm
26
Required sum
= 2πr + 2(ℓ + b)
22
= 2 × × 7 + 2(26 + 18)
7
= 44 + (44) × 2 = 132 cm.

13 Adda247 Publications For any detail, mail us at


Publications@adda247.com
Cracker Book for Bank (IBPS | SBI | RRB PO | Clerk) Mains Exams

6. (e); Let length of the wire = h


70
Radius = = 35 mm = 3.5 cm
2
4
Volume of the wire = πr 2 h = π𝑟 3
3
4
and, volume of sphere = π × (21)3
3
2 4
π𝑟 h = π × 21 × 21 × 21
3
h = 1008 cm

7. (b); Radius of cylinder = 0.63 m


Length = 7.7 m
Let side of square is ‘a’ meter
ATQ,
Curved surface area of cylinder × 500 = Area of field
22
2 × × 0.63 × 7.7 × 500 = a2
7
a = 123.47 m
approximately (a) ≈ 125 𝑚

8. (a); Ratio of length, breadth and height of cuboid →


𝑎∶𝑎∶1
As, a cuboid is cut into a cube this means after cutting ratio
become 1 : 1 : 1 or a : a : a
In starting height is more than length and breadth. We conclude
this because of the condition that base is same, and cuboid is cut
along its base, so height is reduced.
After cutting this cube again along its base a : a : a becomes a : a :
1⇒𝑎 >1

9. (a); Same base → Same radius


Let radius = r
ATQ—
1 2
3
πr H 4
=
πr2 h 3
H and h are the height of cone and cylinder respectively.
H 4
=
h 1
Ratio of height and radius of cone
H:r=4:3
14 Adda247 Publications For any detail, mail us at
Publications@adda247.com
Cracker Book for Bank (IBPS | SBI | RRB PO | Clerk) Mains Exams

h=x
h = 4x
r = 3x
slant height of cone = 5x
πrℓ+πr2 1
Ratio of T. S. A = = ⇒1∶1
2πrh+2πr2 1

𝜋𝑟 2 ℎ 616
10. (a); =
2𝜋𝑟ℎ 352
𝑟 = 3.5 m
𝜋𝑟 2 ℎ = 616
616
ℎ= = 16 m
11×3.5
Total S.A. = 2𝜋𝑟ℎ + 2𝜋𝑟 2
= 2𝜋𝑟(ℎ + 𝑟)
22
= 2 × × 3.5(3.5 + 16)
7
= 429 m2

11. (e); Let, length and breadth of rectangle be ‘l’ and ‘b’
respectively
Given (ℓ × b) = 252 cm²
and, 4𝑙 = 72 cm
𝑙 = 18 cm
length of rectangle = 18 cm
252
breadth of rectangle, 𝑏 = = 14 𝑐𝑚
18
Radius of cone = 14 cm
Height of cone = 18 cm
1 1 22
Volume of cone = 𝜋𝑟 2 ℎ = × × 14 × 14 × 18 = 3696 cm²
3 3 7

12. (a); Length + breadth + height = 12 cm


and
√ℓ2 + 𝑏 2 + ℎ2 = 5√2
ℓ² + b² + h² = 50 cm
𝑙 + 𝑏 + ℎ = 12
Square both sides
(𝑙 + 𝑏 + ℎ)2 = 122
𝑙 2 + 𝑏 2 + ℎ2 + 2 (ℓb + bh + hℓ) = 122
12² = 50 + 2 (ℓb + bh + hℓ)
2 (ℓb + bh + hℓ) = 94 cm²
15 Adda247 Publications For any detail, mail us at
Publications@adda247.com
Cracker Book for Bank (IBPS | SBI | RRB PO | Clerk) Mains Exams

13. (a); Radius of circular garden


66×7 21
= = cm
2×22 2
Area of circular garden (πr²)
22 21 21
= × × = 346.5 sq cm
7 2 2
4
Perimeter of square = 346.5 ×
5
=277.2 cm
4a = 277.2 cm
a = 69.3 cm
Area of square hall
= 69.3 × 69.3
= 4802.49 sq cm

14. (d); r = 3 cm, R = 5 cm, h = 15 cm


Total surface area of pipe
= 2πrh + 2πRh + 2(πR² – πr²)
= (2π × 3 × 15) + (2π × 5 × 15) + 2(5²π – 3²π)
= 90π + 150π + 32π
= 272π cm²

15. (a); If the diagonal of cube is 6√3 cm


Then side of cube is 6cm.
Now let volume of irregular shape is x cubic centimeter.
4
216 = π((1)3 + (2)3 + (3)3 ) + x
3
4 22
216 = × × [36] + x
3 7
1056
𝑥 = 216 −
7
456
𝑥= = 65.14 cubic centimeter
7

16. (e); Let the radius and height of cylinder be are r & h respectively
Now 20% income in both mean
20
𝑛𝑒𝑤 𝑟𝑎𝑑𝑖𝑢𝑠 = (1 + ) 𝑟 = 1.2 𝑟
100
Also 1.2h.
2π × 1.2 r(1.2r + 1.2h)
44 × 2πr (h + r) = 67760

16 Adda247 Publications For any detail, mail us at


Publications@adda247.com
Cracker Book for Bank (IBPS | SBI | RRB PO | Clerk) Mains Exams

r(h+r)= 245
Let radius and height be x and 4x
𝑥 × 5𝑥 = 245
x = 7 cm

17. (e); Let ratio of cone, cylinder and hemisphere = r


Height = r (Because height of hemisphere is equal to its radius)
Required ratio →
2πr(r + h) : πr (ℓ +r) : 3πr²
⇒ 4r : √2r+r : 3r
4 : √2 +1 : 3

18. (d); 2πr + 2 (ℓ + b) = 220 cm


πr² = 1386 sq. cm
1386×7
r2 =
22
r = 21 cm
Length of rectangle
4
= 21 × = 28 cm
3
22
2× × 21 + 2(28 + b) = 220
7
132 + 56 + 2b = 220
32
b= = 16 cm
2
Area of rectangle = (28 × 16) = 448 𝑐𝑚2

19. (c); Let r be the radius of each circle.


Then by given condition,
πR2 = 2πR ⇒ R = 2
∴ The length of the side of the square = 8
Now the area covered by 4 coins = 4 × π (2)2 = 16 π
And area of the square = 64
∴ The area which is not covered by the coins
= 64 – 16π = 16 (4–π)

17 Adda247 Publications For any detail, mail us at


Publications@adda247.com
Cracker Book for Bank (IBPS | SBI | RRB PO | Clerk) Mains Exams

20. (e); Total surface area of sphere = 4πr²


Total surface area of hemisphere = 3πr²
Let radius of hemisphere and sphere be 3𝑥 cm
And 2𝑥 cm respectively.
ATQ—
3πr³ – 4πr² = 423.5 cm²
22 22
3 × × (3𝑥 )2 – 4 × × (2𝑥 )2 = 423.5
7 7
𝑥 = 3.5 cm
21
Radius of hemisphere = cm =10.5
2

21. (d); Let level of water will be increased by h.


4
π × (15)2 × h = π(10)3
3
4 10×10×10 25
⇒h= × =5 cm
3 15×15 27

22. (e); Volume of earth removed = 10 × 4.5 × 3


= 135m3
Remaining area = [20 × 9 − 10 × 4.5] = 180 − 45 = 135
Let, rise in height be h m
Therefore,
ℎ × 135 = 135
ℎ = 1m

23. (a); Lets length of two rectangles be 7x cm 6x cm respectively


ATQ
2 (7x + 16) – 2 (6x + 16) = 8 cm
2x = 8 cm
x = 4 cm
4×7
Radius of cylinder = = 14 cm
2
Height of cylinder = 4 × 6 = 24 cm
Total surface area of cylinder = 2πr (r + h)
22
= 2 × × 14(14 + 24)
7
= 88 × 38 = 3344 cm²

18 Adda247 Publications For any detail, mail us at


Publications@adda247.com
Cracker Book for Bank (IBPS | SBI | RRB PO | Clerk) Mains Exams

Solutions : (24-25); Let height of cylinder = 4𝑥


So radius of cylinder = 3𝑥
Height of cone = 4𝑥
4𝑥
Radius of sphere = = 2𝑥
2

24. (d); T.S.A of cylinder = 2𝜋(𝑟𝑎𝑑𝑖𝑢𝑠)(𝑟𝑎𝑑𝑖𝑢𝑠 + ℎ𝑒𝑖𝑔ℎ𝑡)


T.S.A of sphere = 4𝜋(𝑟𝑎𝑑𝑖𝑢𝑠)2
2𝜋×3𝑥×(4𝑥+3𝑥) 21
Required ratio = 2
= = 21 : 8
4𝜋(2𝑥) 8

1
25. (a); Volume of cone = × 𝜋(3𝑥 )2 × 4𝑥
3
4
Volume of sphere = 𝜋(2𝑥)3
3
1
×𝜋×36×𝑥 3
3
Required % = 4 × 100 = 112.5%
×𝜋×8×𝑥 3
3
26. (c); Volume of Equilateral triangle well = volume of cylindrical tank
filled.
Let h = height of cylinder
ATQ,
√3 22
× 8 × 8 × 44√3 = ×4×4×h
4 7
⇒ h = 42 cm

27. (d); Let, the side of square = 2a


⇒ Diameter of circle = 2a
ATQ,
22
4a2 – × a × a = 262.5
7
28a2 –22a2
⇒ = 262.5
7
⇒ 6a² = 262.5 × 7
⇒ a² = 306.25
⇒ a = 17.5 cm
22
Perimeter of circle = 2 × ×a
7
22
=2× × 17.5 = 110
7

19 Adda247 Publications For any detail, mail us at


Publications@adda247.com
Cracker Book for Bank (IBPS | SBI | RRB PO | Clerk) Mains Exams

Let, Breadth of rectangle = x


⇒ length of rectangle = 1.2x
ATQ,
2(1.2x + x) = 110
55
⇒ 𝑥= = 25
2.2
Required area = 1.2 × 25 × 25 = 750 cm²

28. (d); Area of the space left out = Area of square – Area of circle

22
= (28)2 − × 14 × 14
7
11×14
= 28 (28 − )
7
= 28(28 − 22)
= 28 × 6
= 168 m2

29. (c); (ℓ − 5)(𝑏 + 3) = ℓ𝑏 − 9 ……..(i)


(ℓ + 3)(𝑏 + 2) = ℓ𝑏 + 67 ……..(ii)
Solving (i) and (ii),
ℓ𝑏 + 3ℓ − 5𝑏 − 15 = ℓ𝑏 − 9
or, 3ℓ − 5𝑏 = 6 ………….(iii)
ℓ𝑏 + 2ℓ + 3𝑏 + 6 = ℓ𝑏 + 67
or, 2ℓ + 3𝑏 = 61 ………(iv)
Solving (iii) and (iv),
ℓ = 17m

20 Adda247 Publications For any detail, mail us at


Publications@adda247.com
Cracker Book for Bank (IBPS | SBI | RRB PO | Clerk) Mains Exams

30. (a); Total surface area of the toy = C.S.A of cone + C.S.A of
Hemisphere
Let, slant height of cone be l cm.
πrℓ + 2πr² = 858 cm2
πr(ℓ + 2r) = 858 cm2
ℓ =25 cm
height of cone = 24cm
volume of the toy
1 2
= πr 2 h + πr 3
3 3
1 2(
= πr h + 2r)
3
2
= 1950 cm3
3

31. (a); 2πr + 2(ℓ + b) = 204 cm


2 (ℓ + b) = 144 × ½
2 (ℓ + b) = 72
ℓ + b = 36
(b + 8) + b = 36
b = 14
ℓ = 22
Area of rectangle = 14 × 22 = 308 sq. cm
Circle radius
2πr = 204 – 72
πr = 66
r = 21 cm
22
Area of circle = × 21 × 21
7
= 1386 sq. cm
Required sum = (308 + 1386) sq. cm
= 1694 sq. cm

32. (d); Let the radius of large sphere is 3𝑥 and small sphere is 𝑥.
Then if large sphere is melted into n small spheres.
4 4
𝜋(3𝑥 )3 = 𝑛 𝜋(𝑥 )3
3 3
27𝑥 3
𝑛 = 3 = 27
𝑥
Surface area of large sphere = 4𝜋(3𝑥 )2 = 36𝜋𝑥 2
21 Adda247 Publications For any detail, mail us at
Publications@adda247.com
Cracker Book for Bank (IBPS | SBI | RRB PO | Clerk) Mains Exams

Surface area of 27 small spheres = 27 × 4 𝜋 (𝑥 )2


= 108𝜋𝑥 2
108𝜋𝑥 2 –36𝜋𝑥 2
Required %= × 100 = 200%
36𝜋𝑥 2

33. (d); Let the radius of circle is R


Atq,
πR2 21
= ⇒ R = 21
2πR 2
Volume of sphere of same radius
4
= πR3
3
Let, radius of cylinder is r and height is h
4
πR3 = πr 2 h
3
4
r 2 h = × 21 × 21 × 21
3
= 4 × 7 × 21 × 21
r 2 h = 28 × 21 × 21 …(i)
In question
h : D = 2 : 3 [D is diameter of cylinder]
h : r = 2 : 3/2
= 4: 3
4x: 3x
Put this in equation (i)
36x3 = 28 × 21 × 21
x³ = 7 × 7 × 7
x=7
height = 28 units

34. (a); Let radius of 1st cone be r1 and that of second cone be r2.
𝜋𝑟1 ℓ = 3(𝜋𝑟2 . 3ℓ)
or, 𝜋𝑟1 ℓ = 9𝜋𝑟2 ℓ
𝑟1 9
⇒ =
𝑟2 1
Ratio of areas = 81 : 1
22 Adda247 Publications For any detail, mail us at
Publications@adda247.com
Cracker Book for Bank (IBPS | SBI | RRB PO | Clerk) Mains Exams

35. (b); Let initial radius of both vessels r cm and volume is V cm3
V = 𝜋𝑟 2 12
For first vessel, 𝑉1 = 𝜋(𝑟 + 7)2 12
Second Vessel, 𝑉2 = 𝜋(𝑟 + 7)2 12
Second Vessel, 𝑉2 = 𝜋𝑟 2 (12 + 15)
Given,
𝑉1 = (𝑉 + 𝑄 ) 𝑐𝑚3 _______(I)
𝑉2 = (𝑉 + 𝑄 ) 𝑐𝑚3 ________(II)
From (i) and (ii) We get
𝑉1 = 𝑉2
𝜋(𝑟 + 7)2 12 = 𝜋𝑟 2 (27)
r = 14 cm
So increased capacity (Q)
22 22
= × (14 + 7)2 × 12 − × 14 × 14 × 12
7 7
= 16632 – 7392
= 9240 cm3

36. (e); Given, area of square park = 19600 cm²


a² = 19600
𝑎 = 140 cm
140
Radius of each light pole = = 2.8 cm
50
Area covered by One pole in park
22
= × 2.8 × 2.8 = 24.64 cm²
7
Total area covered by these nine poles
= 9 × (24.64)
= 221.76 cm²

37. (c); Given, Area of square = 306.25 cm2


Side of square = 17.5 cm
Breadth of square = 17.5 − 3.5 = 14 cm
252
Length of rectangle = = 18 𝑐𝑚
14
Radius & height of cylindrical vessel be 14 cm & 18 cm
respectively
22 1
Volume of cylindrical vessel = × 14 × 14 × 18 × = 2772 cm3
7 4

23 Adda247 Publications For any detail, mail us at


Publications@adda247.com
Cracker Book for Bank (IBPS | SBI | RRB PO | Clerk) Mains Exams

38. (c); let height of vessel = h


3
Let radius of bowl = h = radius of vessel
2
2
Volume of hot water = 𝜋ℎ3
3
Required percentage
2 3 3
𝜋( ℎ)
3 2
= 3 2
× 100
𝜋(2ℎ) ×ℎ

= 100%

39. (a); Let radius be 3r then height will be 2r


Value of figure formed
2
= π × (3r)3 + π(3r)2h
3
2
= π × 27r 3 + π9r 2 × 2r
3
= 36 πr³ = 3r = 21
r=7
22
= 36 × ×7×7×7
7
= 36 × 22 × 49 cm³
Required time
36×22×49
=
539
= 72 minute

40. (d); Let radius of cylinder and hemisphere is ‘r’


Also height of cylinder = ‘r’
Total milk = πr² × r = πr³
Milk poured in 1 hemi-sphere
2
= πr 3
3
1
Remaining milk = πr 3
3
When this milk poured in 2nd sphere then it remained empty
2 1 1
πr 3 – πr 3 = πr 3
3 3 3
Required% = 50%
24 Adda247 Publications For any detail, mail us at
Publications@adda247.com
Cracker Book for Bank (IBPS | SBI | RRB PO | Clerk) Mains Exams

41. (e); Perimeter of corridor and hall = 102 m


Let radius of hall is R m and corridor is r m
ATQ—
2πR + πr + 2r = 102 m
22 3r 22r
2× × + + 2r = 102 m
7 2 7
204r
= 102
14
r=7m
21
So, R = m
2
Radius of circular garden
21 400
=( + 7) ×
2 100
35
= × 4 = 70 m
2
Area of Garden = πr²
= 70 × 70 𝜋
= 4900 𝜋 sq. m

42. (c); Given, height of cylindrical vessel = side of square


a2 = 256 cm²
a = 16 cm
Let radius of hemisphere is 2r cm and sphere is r cm.
ATQ—
3π(2r)² – 4πr² = 4928 cm²
12r² – 4r² = 1568
r² = 196
r = 14
Radius of cylindrical vessel
28+14
= = 21 cm
2
Volume of cylindrical vessel
22
= × 21 × 21 × 16
7
= 22176 cm³

25 Adda247 Publications For any detail, mail us at


Publications@adda247.com
Cracker Book for Bank (IBPS | SBI | RRB PO | Clerk) Mains Exams

43. (b); Ratio between length, breadth and side of square = 21 : 6 : 14


Let length and breadth of rectangle be 21x cm and 6x cm
respectively.
And, side of square be 14x cm
ATQ –
4(14x) – 2 (21x + 6x) = 6
56x – 54x = 6
x = 3 cm
14×3
Radius of cylinder = = 21 cm
2
Height of cylinder = 6 × 3 = 18 𝑐𝑚
Volume of cylinder = πr2h
22
= × 21 × 21 × 18
7
= 24948 cm3
44. (b); Let the radius of sphere be x cm.
4 1 2
⇒ 𝜋(𝑥 )3 = 18 ( 𝜋 × (4√3) × 4) + 12(𝜋 × (4)2 × 6)
3 3
4
)3
⇒ (𝑥 = 1152 + 1152 cm³
3
⇒ x = 12 cm
Then, total surface area of sphere = 4πx²
22
= 4 × × (12)2
7
2
= 1810 cm²
7

45. (d); Let length & breadth of rectangle be 7x & 6x respectively


Given area of rectangle = 168 cm2
7x × 6𝑥= 168 cm2
42x2 = 168
x2 = 4
x = 2 cm
Radius of cylindrical vessel = 6 × 2 − 1.5 = 10.5 𝑐𝑚
Height of cylindrical vessel = 7 × 2 = 14 𝑐𝑚
Volume of cylindrical vessel = 𝜋 × 10.5 × 10.5 × 14
= 1543.5π cm3
Let think ness of each discs be x cm
So, 10 × 𝜋 × x × 10.5 × 10.5 = 1543.5π cm3
1543.5
x=
10.5×10.5×10
x = 1.4cm
26 Adda247 Publications For any detail, mail us at
Publications@adda247.com
Cracker Book for Bank (IBPS | SBI | RRB PO | Clerk) Mains Exams

46. (c); Let the radius of two hemisphere be 2x & 3x


ATQ,
2 2 836
π(3x)3 – π(2x)3 = cm3 [∵ volume of hemisphere = ⅔ π r³)
3 3 21
2 836
⇒ π[19x3 ] =
3 21

⇒x=1
∴ radius are 2 cm & 3 cm.
Now,
2 2 74
[ π(2)3 + π(3)3 ] + [ πcm3 ] = πR2 H
3 3 3
2 2
⇒ π[8 + 27] + π 37 = πR2 H
3 3
2
⇒ 𝜋[35 + 37] = 𝜋𝑅2 𝐻
3

⇒ R²H = 48
⇒ R : H = 4 : 3 (given)
Let R = 4a, H = 3a
⇒ 16a². 3a = 48
𝑎=1
∴ R = 4, H = 3
T.S.A. of cylinder = 2πR(R + H)
22
=2× × 4(7)
7

= 176 cm²

27 Adda247 Publications For any detail, mail us at


Publications@adda247.com
Cracker Book for Bank (IBPS | SBI | RRB PO | Clerk) Mains Exams

1 Adda247 Publications For any detail, mail us at


Publications@adda247.com
Cracker Book for Bank (IBPS | SBI | RRB PO | Clerk) Mains Exams

Chapter
Permutation, Combination and
11 Probability

BEST APPROACH TO SOLVE THE QUESTIONS

The best way to define probability is number of favorable outcomes divided


by total number of possible outcomes. Eg. When we conduct an experiment
of throwing a dice and we have to calculate probability of getting 2.

Now number of favorable outcome is 1 i.e. ‘2’


And total number of possible outcomes is 6 i.e it may appear ‘1, 2, 3, 4, 5 or
1
6’ on throwing a dice. Hence probability of getting a ‘2’ is =
6

If we were asked what is the probability of getting an even number.


Number of possible outcome = 3 (2, 4 or 6)
3 1
Probability of getting an even number on throwing a dice = =
6 2

But the probability in bank exam is not this much easy, we encounter with
way more complicated questions.
Example 1: A C++ program shuffles all the letters of a word. When the
word INDIA was entered, a new word was given as an output.
What is the probability that new word formed is having all the
vowels together?
Solution: We need to calculate two things
(i) Total number of possible outcomes
(ii) Total number of favorable outcomes
Imagine how many different words can be formed by re-
arranging the words of INDIA.
And then how many different possible words can be formed
when ‘IIA’ of ‘INDIA’ are together.
To solve these kind of questions we have PERMUTATION AND
COMBINATION.

2 Adda247 Publications For any detail, mail us at


Publications@adda247.com
Cracker Book for Bank (IBPS | SBI | RRB PO | Clerk) Mains Exams

Permutation and Combination:


Basic definition: Permutation means different possible arrangement of
things taken all or some of them together.
While Combination means different way of selection of things taken all or
some of them together.
Eg. Of arrangement/permutation:
There were two chairs numbered 1 and 2. In how many ways A, B and C
could sit on them?
SOLUTION: AB or BA, AC or CA and BC or CB. (First person denotes to sit on
chair named A). Hence there are 6 possible ways.
Eg. Of selection/ Combination
There were two chairs and three persons. In how many different ways they
can sit on chairs?
Solution: Either AB, BC or AC will sit on two chairs. Hence there are 3
possible ways.

The point is arrangement when sequence matters is known as permutation


or selecting some or all the persons is called combination.
Before further discussion, let us revise few terms of permutation and
combination.
! is called factorial.
N! where N is a natural number is N × (N-1) × (N-2)….. ×1
0! = 1
N N!
Pr = (N–r)!, it is used in permutation cases
N N!
Cr = , it is used in combination cases.
C!(N–r)!
E.g. In previous cases.
3 3! 3×2×1
P2 = (3–2)! = = 6 arrangements.
1
3 3!
C2 = = 3 cases of selection.
2!(3–2)!

NOTE:
Though permutation and combination in itself are too big, here we will
discuss few of the types of questions that are asked in exam. Look at the
previous example of INDIA.

3 Adda247 Publications For any detail, mail us at


Publications@adda247.com
Cracker Book for Bank (IBPS | SBI | RRB PO | Clerk) Mains Exams

Example 2: What is the probability that after rearranging the word ‘INDIA’
a new word is formed in which all vowels are together?
SOLUTION: When n things in which p are of one type, q are of one type ,
n!
then total no. of arrangements of those n things is =
p!q!
Taking two Is and one A as a single entity, we have in all 3
entities (N,D and the group of is and A).
Total no. of ways of arranging these three entities is 3!.
3!
Two Is and one A can be rearranged among themselves in
2!
ways.
Hence, total no. of ways in which all vowels are together is
3!
equal to 3!× =18.
2!
Total no. of ways in which all letters of word INDIA can be
5!
arranged = =60.
2!
18 3
Hence required probability= =
60 10

Example: What is the probability that a word formed after rearranging


“ARRANGEMENT’’ such that all vowels are together?
SOLUTION: Following the same method as that of previous question,
Total no. of ways of rearranging the letters of word
11! 11!
ARRANGEMENT= =
2!2!2!2! 16
No. of ways of arranging letters such that all vowels are
8! 4! 8!4!
together= × =
2!2! 2!2! 16
8!4!
16 4
Required probability= 11! =
165
16

KEY POINTS: Number of permutation of n things taken all at a time is n!


N
Pr = NCr × r!

4 Adda247 Publications For any detail, mail us at


Publications@adda247.com
Cracker Book for Bank (IBPS | SBI | RRB PO | Clerk) Mains Exams

Practice Exercise Based on New Pattern

1. There are five mangos and six oranges in a bucket. What will be
probability of Picking up four fruits which contains at least two orange?
53 43 59
(a) (b) (c)
66 66 66
49 3
(d) (e)
66 5

1 2 3
2. Three shooter having probability of shot a target , and respectively.
3 5 7
What is the probability that exactly one of them did not hit the target?
12 29 19
(a) (b) (c)
35 105 35
41 1
(d) (e)
105 5

3. Rahul, Sachin and Sanjeev speaks 2 times, 5 times and 3 times truth out
of 5 times, 6 times and 8 times respectively. Find the probability that
they contradict each other when asked to speak on a fact?
3 5 7
(a) (b) (c)
16 16 16
13 9
(d) (e)
16 16

4. Probability of choosing one boy from a group of 26 girls and ‘x’ boys is
3
. If one more boy joined the group then find the probability of choosing
5
one boy and one girl from that group?
8 6 4
(a) (b) (c)
33 11 11
14 16
(d) (e)
33 33

5. In how many ways word “EATEN” can be arranged with or without


meaning in which at least two vowels will come together?
(a) 48 (b) 54 (c) 72
(d) 108 (e) 90

6. Sum of digits of a 5-digit number is 43. Find the probability that such a
number is divisible by 11?
(a) 1/3 (b) 1/5 (c) 2/5
(d) 2/15 (e) 2/3
5 Adda247 Publications For any detail, mail us at
Publications@adda247.com
Cracker Book for Bank (IBPS | SBI | RRB PO | Clerk) Mains Exams

7. Three-unit squares are chosen at random on a chessboard. What is the


probability that two of them are of same color while remaining one is of
another color?
(a) 16/21 (b) 8/21 (c) 5/21
(d) 4/7 (e) 32/63
8. Two dice are thrown, find the probability of getting an even number
after multiplying numbers obtained from the dice.
(a) ¼ (b) ¾ (c) ⅔
(d) ½ (e) ⅙

9. In how many ways 9 hindi books are placed between 10 English books
such that no two Hindi book comes together?
(a) 60 (b) 55 (c) 45
(d) 96 (e) 75

10. Fourteen applications apply for a job out of which there are 9 men and
5 women. If only three applications selected for the job then find the
probability that at least one of the selected application is of a woman?
60 10 80
(a) (b) (c)
91 13 91
40 50
(d) (e)
91 91

11. In a bucket there are three type of fruits i.e, mango, banana and papaya.
1
Probability of selecting one mango out of total fruits in bucket is and
3
3
the probability of selecting one papaya out of total fruits in bucket is .
8
If number of banana in the bucket is seven, then find the number of
papaya in the bucket.
(a) 4 (b) 5 (c) 11
(d) 9 (e) 7

12. A bucket contains seventeen rectangular boxes, from number 1 – 17. If


three boxes drawn at random from bucket without repeated, then what
will be probability of that all three rectangular boxes represent even
number boxes?
9 6 7
(a) (b) (c)
85 85 85
11 21
(d) (e)
85 85
6 Adda247 Publications For any detail, mail us at
Publications@adda247.com
Cracker Book for Bank (IBPS | SBI | RRB PO | Clerk) Mains Exams

13. Two boxes contain 4 and 16 balls respectively. Two balls in the first box
and four in the second box are black. If a box is chosen randomly and
two balls are drawn at random from it, what is the probability that at
least one ball is black ?
11 43 77
(a) (b) (c)
20 120 120
9 7
(d) (e)
20 23

14. Probability of getting one Red ball from a bag full of balls is 2/13 and
Black balls in the bag is 5. If white ball is 30% less than Brown ball and
40% more then the black balls then find the number of Red balls.
(a) 5 (b) 4 (c) 6
(d) 10 (e) 13

15. Probability of choosing one boy from a group of 26 girls and ‘x’ boys is
3
. If one more boy joined the group then find the probability of choosing
5
one boy and one girl from that group?
8 6 4
(a) (b) (c)
33 11 11
14 16
(d) (e)
33 33

16. In a meeting of HCL company, in the month of June, there are 6 CEO’s ,
5
Y MD’s and 7 HR’s. Probability of calling one MD on stage is . When
18
the meeting is conducted in next month (Y + 1) CEO’s, (Y + 3) MD’s and
(Y + 2) HR’s attended the meeting. If two representatives are called
together on stage, then what will be probability of both being CEO’s?
1 1 1
(a) (b) (c)
16 14 12
1 1
(d) (e)
18 20

17. There are three groups of children in a school i.e. A, B and C. In group A,
there are (x + 2) girls and x boy. In group B, there are 2 boys and 2 girls,
whereas in group C, there are 1 girl and 2 boys respectively. One child
is selected at random from each group. Find the value of x, if the
3
probability of selecting 1 girl and 2 boys is .
8
(a) 2 (b) 3 (c) 1
(d) 4 (e) 5
7 Adda247 Publications For any detail, mail us at
Publications@adda247.com
Cracker Book for Bank (IBPS | SBI | RRB PO | Clerk) Mains Exams

18. In a room, 10 people sit in which 4 left hander and 6 right hander. Find
the probability that left hander shake hand with only left handers.
1 7 17
(a) (b) (c)
15 45 45
5 2
(d) (e)
9 15

19. In a cold drink cartoon 3 pepsi, 5 limca and 7 sprites bottels. If one
bottle is drawn at random from the cartoon what is the probability that
it is either sprite or pepsi ?
5 3 2
(a) (b) (c)
7 4 3
8
(d) (e) None of these
9
20. Find the difference between number of words formed from ALAIS
when all vowels are together, and words formed from ENGLISH when
no vowels are together?
(a) 3528 (b) 3618 (c) 3572
(d) 3592 (e) 3582

21. In a box some Munch chocolate, some Dairy milk chocolate and Kit Kat
chocolate. The probability of selecting one Munch chocolate out of box
3
is and the probability of selecting one Kit Kat chocolate out of box is
10
2
. The number of Dairy milk chocolate in box is 6. Find probability of
5
selecting three chocolate from the box such that at most two chocolate
are same?
64 64 42
(a) (b) (c)
85 95 85
87 67
(d) (e)
95 85

22. Find the probability of forming the word from ANANAB such that N
never comes together.
2 4 3
(a) (b) (c)
3 5 5
2 4
(d) (e)
5 5

8 Adda247 Publications For any detail, mail us at


Publications@adda247.com
Cracker Book for Bank (IBPS | SBI | RRB PO | Clerk) Mains Exams

23. A bag contains 5 red, 4 green and 3 black balls. If three balls are drawn
out of it at random, find the probability of drawing exactly 2 red balls?
5 7 6
(a) (b) (c)
22 24 25
2 7
(d) (e)
11 22

Directions (24-25): A man can send a message by using six flags. He had 4
black flags, 3 blue flags and 1 green flag.

24. In how many ways a message can be transmitted?


(a) 108 (b) 185 (c) 20
(d) 136 (e) 110

25. In how many ways a message can be transmitted by using all three
colored flags?
(a) 150 (b) 90 (c) 100
(d) 70 (e) 125

Directions (26-27): A box contains two types of ball i.e., red and Blue.
1
Probability of choosing one red and one blue ball is while probability of
2
3
choosing two red balls is .
20

26. Find the probability of selecting two balls without replacement from
the box such that at least one of them is red?
9 11 13
(a) (b) (c)
20 20 20
17 3
(d) (e)
20 4
27. If 5 Red balls and 6 blue balls are added in the box, then find the
probability of choosing two blue balls from the box at present?
1 1 1
(a) (b) (c)
2 3 4
1 1
(d) (e)
5 6

9 Adda247 Publications For any detail, mail us at


Publications@adda247.com
Cracker Book for Bank (IBPS | SBI | RRB PO | Clerk) Mains Exams

Directions (28-30): There are three sections A, B and C in a class. Every


section has some boy and some girl students in it. Probability of a girl being
2
selected when one student is selected randomly from section A is , that
5
4 5
from section B is and that from section C is .
9 9
28. If the ratio of total number of students in sections A, B and C is 10 : 12 :
9, then what is the probability of a girl being selected when one student
is selected randomly from the students from all the three sections
together? (2 Marks)
14 11 13
(a) (b) (c)
31 23 31
43
(d) (e) Cannot be determined
93
29. If the number of girls in sections A is same as the number of boys in
section C, then what is the ratio of number of boys in section A to the
number of boys in section C? (2 Marks)
(a) 2 : 3 (b) 3 : 4 (c) 3 : 2
(d) 4 : 3 (e) 3 : 5
30. If 20 girls leaves section B, then the probability of a boy being selected
5
when one student is selected randomly from this section will be . What
8
is the number of boys in section B? (2 Marks)
(a) 120 (b) 100 (c) 75
(d) 60 (e) Cannot be determined

Directions (31-32): There are some horses, hens and elephants in a park.
Ratio between number of heads in park to number of legs in park are 7 : 20.
31. Find the probability of choosing a hen over all the animals.
3 2 3
(a) (b) (c)
7 7 8
4 2
(d) (e)
7 9
9
32. If probability of choosing 2 horses over all the animals are , then find
119
the number of elephant present in park. (Number of horses is double
than that of the number of elephants in park).
(a) 5 (b) 10 (c) 20
(d) 15 (e) 25
10 Adda247 Publications For any detail, mail us at
Publications@adda247.com
Cracker Book for Bank (IBPS | SBI | RRB PO | Clerk) Mains Exams

Directions (33-34): There are two bags, one of which contains 3 black and
4 white balls, while the other contains 4 black and 3 white balls. A dice is
cast, if the face 1 or 3 turns up, a ball is taken from the first bag and if any
other face turns up, a ball is chosen from the second bag.

33. The probability of choosing a black ball is: (2marks)


11 11 11
(a) (b) (c)
24 21 27
11 3
(d) (e)
29 8

34. The probability of choosing a white ball is: (2marks)


11 11 10
(a) (b) (c)
21 29 21
14 11
(d) (e)
23 23

Directions (35-37) : India wins every 6 matches out of 10 matches against


England. Virat Kohli (Indian Player) make a century in every 3 matches out
of 5 matches and in which he makes century, India wins 80% of the
matches. Alastair cook (England’s player) make 1 century in every 4
matches and England wins 60% of that matches.

35. Find the probability of India’s winning against England when Kohli and
Cook both make centuries.
20 17 36
(a) (b) (c)
623 625 625
24 18
(d) (e)
625 625

36. Find the probability that India wins against England when Kohli failed
to make a century.
8 3 6
(a) (b) (c)
25 5 25
4 12
(d) (e)
25 25

37. Find the probability of winning England against India when Kohli make
a century but cook does not.
9 16 2
(a) (b) (c)
250 250 5
14 21
(d) (e)
250 250
11 Adda247 Publications For any detail, mail us at
Publications@adda247.com
Cracker Book for Bank (IBPS | SBI | RRB PO | Clerk) Mains Exams

Direction (38-39): In a stationary shop there are four types of colored


sheets of red, blue, green and white colors. The probability of selecting one
1
red sheet out of the total sheets is ,the probability of selecting one blue
3
2
sheet out of the total sheets is and the probability of selecting one white
7
1
sheet out of the total sheets is . The number of green sheets in the bag is
4
22.
38. If all the sheets are numbered as 1,2,3,….and so on and one sheet is
picked up at random, then find the probability of picking up a sheet
which is numbered as a multiple of 13 or 17. (2 marks)
1 1 5
(a) (b) (c)
8 7 8
9
(d) (e) none of these
10

39. What is the total number of sheets in the bag? (1 marks)


(a) 117 (b) 168 (c) 154
(d) 120 (e) none of these

Direction (40-41) : A coach invited 9 players of the team on dinner. All 10


persons seated in a circular table.
40. If in how many ways captain and vice-captain seat opposite to each
other?
(a) 80640 (b) 5040 (c) 3624
(d) 40320 (e) 10080
41. In how many ways team can seat, if caption and vice-captain seat either
side of coach.
(a) 80640 (b) 5040 (c) 3624
(d) 40320 (e) 10080
Directions (42-44): A joker have 10 balls, 10 ribbon and 10 plates of red,
yellow and blue color in ratio 2 : 3 : 5, 5 : 2 : 3 and 3 : 5 : 2 respectively. He
does his performance by using one item of each category.

42. What is the probability that he performs with only red color?
(a) 0.3 (b) 0.02 (c) 0.25
(d) 0.03 (e) 0.15
12 Adda247 Publications For any detail, mail us at
Publications@adda247.com
Cracker Book for Bank (IBPS | SBI | RRB PO | Clerk) Mains Exams

43. What is the probability that he chooses yellow color for ribbons?
(a) 0.0333 (b) 0.025 (c) 0.2
(d) 0.25 (e) 0.03

44. If he chooses 3 balls, 5 ribbons and 1 plate then find the probability that
all items are of same color.
(a) 1 (b) 0 (c) 0.05
(d) 0.5 (e) 0.002

Direction (45-47): - Kishan have two trays of eggs i.e. tray A and tray B.
Tray A have same number of fresh eggs as tray B have rotten eggs. Rotten
eggs in tray A is 40% less than that of in tray B. Fresh eggs in tray B is 14
more than that in tray A. Total number of eggs Kishan have is 140.
45. Fresh eggs in tray B is what percent more than that in tray A?
(a) 140% (b) 80% (c) 60%
(d) 40% (e) 20%

46. Find the probability of choosing two eggs from tray ‘A’ such that at least
one egg should be fresh?
19 17 27
(a) (b) (c)
22 22 44
25 21
(d) (e)
44 22

47. Find the probability of choosing one rotten egg?


23 7 19
(a) (b) (c)
48 16 48
25 9
(d) (e)
48 16

Direction (48- 50)- There are three bucket P, Q & R contains three fruits
Mangos, Bananas & Oranges.
In bucket P, contains x Mangos and y Bananas. Number of bananas are 3
more than the number of Mangos. If Ram selected one fruits for eating, then
3 1
the probability of getting orange is . The value of x is 37 % less than y.
16 2
In bucket Q, number of Mangos is 40% more than that of bucket P. If Satish
selecting two fruits at random for eating then the probability of getting both
18
Bananas is . Total number of fruits in bucket Q is 29.
203
13 Adda247 Publications For any detail, mail us at
Publications@adda247.com
Cracker Book for Bank (IBPS | SBI | RRB PO | Clerk) Mains Exams

In bucket R, the ratio of number of mangos to number of orange is 3 : 4.


Total number of bananas & orange is 33 . If Veer selecting one fruit for
3
eating the probability of getting mangos is .
14

48. What is the total number of bananas present in all the three buckets?
(a) 30 (b) 38 (c) 36
(d) 32 (e) 28

49. If all mangoes of bucket P is placed in the bucket Q then find the
probability of choosing a mango from bucket Q?
5 9 3
(a) (b) (c)
17 17 17
7 6
(d) (e)
17 17

50. If 25% oranges of bucket R are spoiled and thrown away then find the
probability of choosing a good orange from bucket R?
3 5 7
(a) (b) (c)
13 13 13
6 2
(d) (e)
13 13

Solutions

1. (a); Favorable case = (20, 2M) or (30, 1M) or 40


∴ Probability
6C × 5C 6C × 5C 6C
2 2 3 1
= 11C + 11C + 11 4
4 4 C4
15×10 20×5 15 265 53
= + + = =
330 330 330 330 66

2. (b); Required probability


1 2 4 1 3 3 2 3 2 29
= × × + × × + × × =
3 5 7 3 7 5 5 7 3 105

2
3. (d); Probability that Rahul speaks truth =
5
5
Probability that Sachin speaks truth =
6
3
Probability that Sanjeev speaks truth =
8

14 Adda247 Publications For any detail, mail us at


Publications@adda247.com
Cracker Book for Bank (IBPS | SBI | RRB PO | Clerk) Mains Exams

Rahul, Sachin and Sanjeev do not contradict with each other if they
all are correct or they all are incorrect means if they all speak truth
or they all speak lie.
2 5 3 1
Probability that they all speak truth = × × =
5 6 8 8
3 1 5 1
Probability that they all speak lie= × × =
5 6 8 16
1 1 16 –2 –1 13
Required probability = 1 – – = =
8 16 16 16

4. (e); ATQ,
x 3
=
26+x 5
⇒ 5x = 26 × 3 + 3x
⇒ 2x = 26 × 3
⇒ x = 39
Total no. of students now in group
= 26 + 39 + 1 = 66
26 40 16
Required probability = 2 × × =
66 65 33

5!
5. (b); Total number of ways = = 60 ways
2!
Selection of vowels can occur in three pair i.e. ( AE, EE, EA)
Number of ways in which two vowels will come together
3×4!
= = 3 × 4 × 3 = 36
2!
Above, we had not multiplied 2 in numerator because further
arrangement of vowels selected i.e.
(AE, EE, EA) will give the same arrangement
Number of ways in which three vowels will come together
3!×3!
= = 3 × 2 × 3 = 18
2!
Required no. of ways = 36 + 18 = 54
Alternate,
No. of ways in which no vowel will came together
3!
=2× =6
2!
Required no. of ways = 60 – 6 = 54
15 Adda247 Publications For any detail, mail us at
Publications@adda247.com
Cracker Book for Bank (IBPS | SBI | RRB PO | Clerk) Mains Exams

6. (b); Let, numbers in 5-digit number is a, b, c, d, e.


In order to get sum of 43, the combinations exist are
9, 9, 9, 9, 7 = 5 numbers
9, 9, 9, 8, 8 = 10 numbers
Total 15 numbers can be formed having sum 43.
For number which are divisible by 11 main condition is
(a + c + e) – (b + d) = 11 …(i)
and (a + b + c + d + e) = 43 …(ii)
On solving (i) and (ii)
a + c + e = 27, b + d = 16
a, c and e can be 9, 9, 9
while b, d can 7, 9 or 8, 8 or 9, 7 respectively
So, there are three numbers which are divisibly by 11 having sum
43 i.e., 97999, 99979, 98989.
3 1
Required probability = =
15 5

7. (a); Total number of ways = ⁶⁴C₃ = 41664


Favorable ways = 2 × ³²C₂ × ³²C₁
= 2 × 496 × 32 = 31744
31744 16
Required probability = =
41664 21
(³²C₂ × ³²C₁ is taken because if two are black, then one will be of
white or if two are white, then one will be of black)

8. (b); Favourable case →


(Dice 1 → even and dice 2 → odd)
(Dice 1 → even and dice 2 → even)
(Dice 1 → odd and dice 2 → even)
3 3 3 3 3 3 3
= × + × + × =
6 6 6 6 6 6 4

9. (b); Let us denote English book by E and Hindi books by H


The possible arrangements is
HEHEHEHEHEHEHEHEHEHEH
There are → English book, H can occupy 10 + 1 = places
So no. of ways → ¹¹C₉
11×10
= = 55 ways
2
16 Adda247 Publications For any detail, mail us at
Publications@adda247.com
Cracker Book for Bank (IBPS | SBI | RRB PO | Clerk) Mains Exams

10. (b); Total number of ways to select three persons = 14C3


Required case = 2 men 1 women or 1 man 2 women or 3 women
= ( 9C2 × 5C1 ) + ( 9C1 × 5C2 ) + ( 5C3 )
= (36 × 5) + (9 × 10) + (10)
= 180 + 90 + 10
= 280 14C3 = 14C3 = 364
280 10
Required probability = =
364 13

11. (d); Let number of papaya and mango in bucket be ‘P’ and ‘M’
respectively
Number of banana in bucket = 7
1
Probability of selecting one mango =
3
3
and selecting one papaya =
8
So,
M 1
(7+M+P)
=
3
3M = 7 + M + P
2M – P = 7 …(i)
Similarly—
P 3
(7+M+P)
=
8
8P = 21 + 3M + 3P
5P – 3M = 21 …(ii)
Solving (i) and (ii) we get
Number of Mango = 8, Banana = 7, Papaya = 9
Total number of papaya in bucket = 9

12. (c); Total even number boxes between 1 to 17


= 8 (2, 4, 6, 8, 10 …..16)
Total boxes = 17
8
Probability of first box represent even number box =
17
7
Probability of second box represent even number box =
16
6
Probability of third box represent even number box =
15
8 7 2 7
Required probability = × × =
17 16 5 85

17 Adda247 Publications For any detail, mail us at


Publications@adda247.com
Cracker Book for Bank (IBPS | SBI | RRB PO | Clerk) Mains Exams

13. (c); At least one black can be chosen in three ways:


1. first one is black, second is non-black
2. first one is non-black, second is black
3. both are black.
1
Probability of selecting a box is
2
Now, probability of choosing at least one black ball from first box
1 2 2 2 2 2 1 5
= ×( × + × + × )=
2 4 3 4 3 4 3 12
Probability of choosing at least one black ball from 2nd box
1 4 12 12 4 4 3 27
= ×( × + × + × )=
2 16 15 16 15 16 15 120
5 27 50+27 77
Required probability = + = =
12 120 120 120

14. (b); Let number of red balls is 2x


Total balls = 13x
Now ATQ
Black balls = 5
5×140
White balls = =7
100
7
Brown balls = × 100 = 10
70
Now→ 2x + 5 + 7 + 10 = 13x
x=2
Red balls = 4

15. (e); ATQ,


x 3
=
26+x 5
⇒ 5x = 26 × 3 + 3x
⇒ 2x = 26 × 3
⇒ x = 39
Total no. of students now in group
= 26 + 39 + 1 = 66
26 40 16
Required probability = 2 × × =
66 65 33
16. (b); In June —
Total CEO’s = 6
MD’s = y ⇒ HR’s = 7
Now,
18 Adda247 Publications For any detail, mail us at
Publications@adda247.com
Cracker Book for Bank (IBPS | SBI | RRB PO | Clerk) Mains Exams

ATQ,
y 5
= ⇒ y=5
13+y 18
In July —
CEO’s ⇒ 5 + 1 = 6
MD’s ⇒ 5 + 3 = 8
HR’s ⇒ 5 + 2 = 7
6C 6×5 1
2
Required probability = 21C = =
2 21×20 14

17. (c); Possible cases


Girl Boy Boy or Boy
Girls Boy or Boy Boy Girl
3 x+2 2 2 x 2 2 x 2 1
=( × × )+( × × )+( × × )
8 2x+2 4 3 2x+2 4 3 2x+2 4 3
(2x+2)×4×3 2(x+2) 2x x
⇒ =( + + )
2×8 3 3 3
(x+1)×9
⇒ = 5x + 4
2
⇒ 9x + 9 = 10x + 8
⇒ x=1

18. (e); Total no. of ways in which hand shakes


10(10 –1)
= = 45
2
No. of ways in which left hander shakes hand with left handers
4(4 –1)
= =6
2
6 2
Required probability = =
45 15

19. (c); Total number of bottle in cartoon


= 3 + 5 + 7 = 15
∴ Probability of either sprite bottle or pipe bottle
7C + 3C 7+3 2
1 1
= 15C = =
1 15 3

19 Adda247 Publications For any detail, mail us at


Publications@adda247.com
Cracker Book for Bank (IBPS | SBI | RRB PO | Clerk) Mains Exams

20. (e); AAILS when all vowels are together


3!
= 3! × = 18 words
2!
Total words from ENGLISH = 7! = 5040
Words from ENGLISH when no vowels are together
= 5040 – [when all vowels are together]
= 5040 – [6! × 2] = 3600
∴ Required difference = 3600 – 18 = 3582

21. (d); Let total chocolate in box be 10c. Munch chocolate be a and kit-kat
chocolate be b .
a 3
Probability of selecting Munch chocolate = =
10c 10
a = 3c ... (i)
Probability of selecting Kit Kat chocolate
b 2
= =
10c 5
b = 4c … (ii)
ATQ,
a + b + 6 = 10c
⇒ 3c + 4c + 6 = 10c ⇒ c = 2
So, total chocolate = 20
6C + 8C + 6C
3 3 3
Required probability = 1 − 20 C
3
(20+56+20) 1044 87
=1− = =
1140 1140 95

22. (a); Total number of words possible from ANANAB


6!
= = 60
3!2!
No. of word in which N is always together
5!
= = 4 × 5 = 20
3!
No. of in which N is never together = 60 – 20 = 40
40 2
Required probability = =
60 3

20 Adda247 Publications For any detail, mail us at


Publications@adda247.com
Cracker Book for Bank (IBPS | SBI | RRB PO | Clerk) Mains Exams

23. (e); Exactly 2 red balls can be selected in two ways.


(i) 2 red ball & 1 green ball
(ii) 2 red ball & 1 black ball
Total no. of possible outcome = ¹²C₃
12×10×11
= = 220 ways
3×2
Total favourable outcome
= ⁵C₂ × ⁴C₁ + ⁵C₂ × ³C₁
5×4 5×4
= ×4+ × 3 = 40 + 30 = 70 ways.
2 2
70 7
Probability = =
220 22

24. (b); Case I: When all 4 black flag selected


Two ways could be
1 green flag + 1 blue flag or 2 blue flags
6! 6!
No. of ways to arrange = +
4! 4!2!
Case II:
When all 3 blue flag selected
3 ways could be
2 black flags + 1 green flag or 3 black flags
6! 6!
No. of ways = +
3!2! 3!3!
Case III: When one green selected
6!
⇒ 3 black + 2 blue [other cases already included]
3!2!
Adding all
⇒ 30 + 15 + 60 + 20 + 60 = 185

25. (a); All colored used ⇒


1 green flag + 2 black flags + 3 blue flags
1 green flags + 3 black flags + 2 blue flags
1 green flag + 4 black flags + 1 blue flag
6! 6! 6!
⇒ + +
2!3! 3!2! 4!
⇒ 60+ 60 + 30 = 150

21 Adda247 Publications For any detail, mail us at


Publications@adda247.com
Cracker Book for Bank (IBPS | SBI | RRB PO | Clerk) Mains Exams

Solution (26-27): Let, Red balls = x


Blue balls =y
ATQ,
x y 1
2× × = ... (i)
(x+y) (x+y−1) 2
And,
x×(x−1) 3
= ... (ii)
(x+y)(x+y−1) 20
Dividing (ii) by (i)
x−1 3
⇒ =
y 5
⇒ 5x – 5 = 3y …(iii)
And, from (i)
4xy = (x + y) (x + y – 1) ... (iv)
From (iii) and (iv)
5x−5 5x−5+3x 5x−5
4x ( )=[ ] [x + − 1]
3 3 3
20x 8x−5 8x−8
⇒ (x − 1) = [ ][ ]
3 3 3
⇒ 60x² – 60x = 64x² - 64x – 40x + 40
⇒ -4x² + 44x – 40 = 0
⇒ x² – 11x + 10 = 0
⇒ x²– 10x – x + 10 = 0
⇒ x(x – 10) – 1 (x-10) = 0
⇒ x = 1, 10
x = 1 x = 10
⇒ y = 0 ⇒ y = 15But y can’t be zero.
⇒ Number of red balls = 10
Number of blue balls = 15

26. (c); Required probability


10 C + 10 C 15 C 45+150 195 13
2 1 1
y= 25 C = = =
2 300 300 20

21 C 21×20 1
2
27. (b); Required probability = 25+11 C = =
2 36×35 3
22 Adda247 Publications For any detail, mail us at
Publications@adda247.com
Cracker Book for Bank (IBPS | SBI | RRB PO | Clerk) Mains Exams

28. (d); Probability of a girl being selected from a section


Total girls in the section
=
Total students in the section
Let the number of girls, number of boys and total number of
students respectively:
For section A: 2x, 3x and 5x.
For section B: 4y, 5y and 9y.
For section C: 5z, 4z and 9z.
According to the question,
Ratio of total number of students in the three sections:
⟹ 5x : 9y : 9z = 10 : 12 : 9
⟹x:y:z=6:4:3
Let the values of x, y and z be 6k, 4k and 3k respectively.
Total number of girls in all the three sections = 2x + 4y + 5z = 12k +
16k + 15k = 43k
Total number of students in all the three sections
= 5x + 9y + 9z = 30k + 36k + 27k = 93k
Probability of a girl being selected from the students from all the
three sections together
Total girls in all sections 43k 43
= = =
Total students in all sections 93k 93

29. (c); According to the question,


Number of girls in sections A = Number of boys in section C
⟹ 2x = 4z ⟹ x = 2z
Number of boys in section A : Number of boys in section C = 3x : 4z
= 6z : 4z = 3 : 2

30. (b); Probability of a boy being selected from this section B after 20 girls
5
left the section =
8
Number of boys in section B 5
⟹ =
Total number of students in section B−20 8
5y 5
⟹ =
9y − 20 8
⟹ 40y = 45y – 100 ⟹ y = 20
Number of boys in section B = 5y = 100
23 Adda247 Publications For any detail, mail us at
Publications@adda247.com
Cracker Book for Bank (IBPS | SBI | RRB PO | Clerk) Mains Exams

Solution (31-32): Let number of hens, horses and elephant in park is — a,


b and c respectively.
Let number of heads and number of legs be 7x and 20x respectively
So,
a + b + c = 7x
b + c = 7x– a …(i)
2a + 4(b + c) = 20x …(ii)
Solving (i) and (ii)
2a + 4 × (7x– a) = 20x
2a + 28x – 4a = 20x
a = 4x

4x 4
31. (d); Probability of choosing hen over all the animals = =
7x 7
32. (a); No. of hens = 4x
No. of horses and elephants together = 7x– 4x = 3x
If number of horses is double than that of the number of elephants then
Number of horses = 2x
Number of elephant = x
ATQ
2xC 9
2
7xC =
2 119
2x(2x–1) 9
=
7x(7x–1) 119
x=5

2 3 1
33. (b); Probability of choosing a black ball from bag 1 = × =
6 7 7
4 4 8
Probability of choosing a black ball from bag 2 = × =
6 7 21
1 8 11
Total probability = + =
7 21 21

2 4 8
34. (c); Probability of choosing a white ball from bag 1 = × =
6 7 42
4 3 12
Probability of choosing a white ball from bag 2 = × =
6 7 42
10
Total probability =
21
24 Adda247 Publications For any detail, mail us at
Publications@adda247.com
Cracker Book for Bank (IBPS | SBI | RRB PO | Clerk) Mains Exams
6
35. (e); Probability that India wins against England =
10
Condition applied
3
Probability that Kohli makes century =
5
4
Probability that India win when Kohli makes century =
5
1
Probability of Cook’s century =
4
Probability of other country wining when Cook make century
3 2
= 1– =
5 5
6 3 4 1 2 18
Combining all = × × × × =
10 5 5 4 5 625

2
36. (c); Probability when Kohli failed to make a century =
5
So, Probability that India wins against England when Kohli failed to
6 2 6
make a century = × =
10 5 25

3
37. (a); Probability that Kohli will make a century =
5
1
Probability that India lose when Kohli make century =
5
3
Probability that Cook doesn’t make century =
4
4
Probability that of Winning of England against India =
10
4 3 3 1 9
Overall probability = × × × =
10 4 5 5 250

38. (a); Let, total no. of sheets be 84x,


1
Then no. of red sheets = × 84x = 28x
3
2
no. of blue sheets = × 84x = 24x
7
1
no. of white sheets = × 84x = 21x
4
Now,
28x + 24x + 21x + 22 = 84x
or, 11x = 22
or, x = 2
Total no. of sheets = 84x = 168
There are 12 multiples of 13 and 9 multiples of 17 from 1 to 168.

25 Adda247 Publications For any detail, mail us at


Publications@adda247.com
Cracker Book for Bank (IBPS | SBI | RRB PO | Clerk) Mains Exams

Also, there is no multiple of 13 and 17 from 1 to 168. (because,


13× 17 = 221)
21 1
Hence, required probability= =
168 8

39. (b); Total no. of sheets = 168

40. (e);

When two places fixed so we can arrange other 8 in → (8!) ways


and captain and vice-captain
Total ways ⇒ 8! ⇒ 40,320

41. (e);

3 places fixed ⇒
So total place ⇒ 10 – 3 = 7
Number of way to arrange ⇒ 7!
Captain and vice-captain change their place in two ways
So, total ways ⇒ 7 ! × 2 = 10,080

Solutions (42-44)

Balls = 10 Ribbon = 10 Plates = 10


Red = 2 Red = 5 Red = 3
Yellow = 3 Yellow = 2 Yellow = 5
Blue = 5 Blue = 3 Blue = 2
26 Adda247 Publications For any detail, mail us at
Publications@adda247.com
Cracker Book for Bank (IBPS | SBI | RRB PO | Clerk) Mains Exams

42. (d); Performance with only red color


One ball of red color, one ribbon of red color and one plate of red
color
2 5 3 3
Required Probability = × × = = 0.03
10 10 10 100

43. (c); Condition is yellow color for ribbon while plate and ball can be of
any color so, probability of choosing ball and plate is 1 while
probability of choosing one yellow color ribbon is
2
= = 0.2
10

44. (b); Now


Joker choose → 3 balls, 5 Ribbon and 1 plate
Condition → all are of some color
⇒ 5 ribbons of same color is only of red color
But, 3 red ball cannot be possible so
Required Probability = 0
Solution (45-47) : - Let fresh eggs in tray A = 100x = Rotten eggs in
tray B
Rotten eggs in tray A = 60x
Fresh eggs in tray B = 100x + 14
Total eggs = 100x + 100x + 60x + 100x + 14 = 140
360x = 126 ⇒ x = 0.35
Tray A Tray B
Fresh Rotten Fresh Rotten
35 21 49 35

49−35 14
45. (d); Required % = × 100 = × 100 = 40%
35 35

35 34 35×21 17 21 19
46. (a); Required probability = × +2× = + =
56 55 56×55 44 44 22

1 21 1 35 3 5 19
47. (c); Required probability = × + × = + =
2 56 2 84 16 24 48

27 Adda247 Publications For any detail, mail us at


Publications@adda247.com
Cracker Book for Bank (IBPS | SBI | RRB PO | Clerk) Mains Exams

Direction (48– 50): In bucket P –


Given, y = x + 3 … (i)
3y
x= … (ii)
8
By solving (i) & (ii) we get
Mangos (x) = 5
Bananas (y) = 8
Let the number of Orange be z.
z 3
Then, =
13+z 16
16z = 39 + 3z
z=3
In bucket Q –
140
Mangos = 5 × =7
100
Let the number of Bananas be ‘a’ and number of Orange be ‘b’
7 + a + b = 29 ⇒ a + b = 22
aC 18 a×(a−1) 18
2
29C = ⇒ =
2 203 2×406 203
a × (a − 1) = 72
Bananas (a) = 9 ⇒ Orange (b) = 12
In bucket R –
Let Mangos & Orange be 3x & 4x respectively
3xC 3 3x 3
1
= ⇒ =
33+ 3xC1 14 33+3x 14
14x = 33 + 3x ⇒ x = 3
Mangos = 9 ⇒ Orange = 12
Bananas = 33 – (9 + 12 ) = 21

48. (b); Total bananas in all three buckets = 8 + 9 + 21 = 38

5+7 6
49. (e); Required probability = =
29+5 17

3
50. (a); In bucket R Good orange = 12 × = 9
4
Total fruits = 9 + 21 + 9= 39
9 3
Probability = =
39 13

28 Adda247 Publications For any detail, mail us at


Publications@adda247.com
Cracker Book for Bank (IBPS | SBI | RRB PO | Clerk) Mains Exams

Adda247 Publications For any detail, mail us at


1
Publications@adda247.com
Cracker Book for Bank (IBPS | SBI | RRB PO | Clerk) Mains Exams

Chapter

12 Inequality

BEST APPROACH TO SOLVE THE QUESTIONS

In examination, we come across questions in which we need to find two


quantities and compare them to reach at the correct answer. Usually such
questions are asked in two ways. In one type of questions, a common data is
provided based on which we are supposed to find both quantities. In other
type of questions each quantity comes up with some data pertaining to it
only.

In these questions, it’s important not to mingle or use data of one


quantity in finding other quantity in cases where separate data is given for
both the quantities. Where common data is provided for both quantities, we
have to use it for calculating both the quantities. After calculating, we
compare both quantities and choose the correct answer from the options
available.

Example: A and B starting a business together. B invested 60% more than


that of A. Ratio of time period of investment of A and B is 8 : 9.
Quantity I: A’s share of profit out of total profit of Rs. 25,200.
Quantity II; B’s share of profit out of total profit of Rs. 14,000.
(a) Quantity I > Quantity II
(b) Quantity I < Quantity II
(c) Quantity I ≥ Quantity II (d) Quantity I ≤ Quantity II
(e) Quantity I = Quantity II or No relation

Solution: (e); Ratio of profit share of A and B


x×8 x×1.6×9
5 : 9
5
Quantity I: A’s profit = 25,200 × = 9000
14
9
Quantity II: B’s profit = 14,000 × = 9000
14
Quantity I = Quantity II

Adda247 Publications For any detail, mail us at


2
Publications@adda247.com
Cracker Book for Bank (IBPS | SBI | RRB PO | Clerk) Mains Exams

Over the years, questions that test a candidate’s basic algebraic


knowledge have been and are being asked. In these questions, two algebraic
equations are provided. The equation may be linear, quadratic or cubic. At
times, we may come across equations containing higher powers of variable,
but they aren’t too difficult to solve.

These equations generally contain one variable. From the given couple of
equations we need to find the values of two variables. Usually one of the
equations gives us the value of one of the variables and the other gives the
value of the other variable. After finding the values of two variables we
compare them and select the correct option accordingly.

Example: I. 21x² – 22x + 5 = 0


II. 63y² – 54y + 11 = 0

Solution (a); I. 21x² – 22x + 5 = 0


⇒ 21x² – 15x – 7x + 5 = 0
⇒ 3x (7x – 5) – 1 (7x – 5) = 0
⇒ (3x – 1) (7x – 5) = 0
1 5
⇒ 𝑥 = or
3 7
II. 63y² – 54y + 11 = 0
⇒ 63y² – 21y – 33y + 11 = 0
⇒ 21y (3y – 1) – 11 (3y – 1) = 0
⇒ (21y – 11) (3y – 1) = 0
11 1
⇒𝑦 = or
21 3
∴ no relation can be established.

Adda247 Publications For any detail, mail us at


3
Publications@adda247.com
Cracker Book for Bank (IBPS | SBI | RRB PO | Clerk) Mains Exams

Practice Exercise Based on new Pattern

PART A
1. B is thrice as efficient as C. B and C can compete a work together in
45/2 days. A takes 50% more days than the days taken by A and B to
complete the same work together.
Quantity 1: No. of days taken by fastest among them to complete the
work alone.
Quantity 2: Time taken by A and C to complete the work together.
(a) Quantity I > Quan tity II (b) Quantity I < Quantity II
(c) Quantity I ≥ Quantity II (d) Quantity I ≤ Quantity II
(e) Quantity I = Quantity II or No relation

2. Quantity I→ The profit earned by selling an item (in Rs) if the


2
difference b/w the SP and the CP is 117 % of 600.
3
Quantity II→ The cost price of an article (in Rs) if the selling price of
the article is 1000 Rs and he got 25% profit after selling the item.
(a) Quantity I > Quantity II (b) Quantity I < Quantity II
(c) Quantity I  Quantity II (d) Quantity I = Quantity II
(e) No relation

3. The largest possible right circular cylinder is cut out from a wooden
cube of edge 7 cm.
Quantity I: volume of the cube left over after cutting out the cylinder
Quantity II: Surface area of cube remained after cutting out the
cylinder.
Note: compare the magnitudes of both quantities.
(a) Quantity I > Quantity II (b) Quantity I < Quantity II
(c) Quantity I ≥ Quantity II (d) Quantity I ≤ Quantity II
(e) Quantity I = Quantity II or No relation
Adda247 Publications For any detail, mail us at
4
Publications@adda247.com
Cracker Book for Bank (IBPS | SBI | RRB PO | Clerk) Mains Exams

4. Quantity I- ‘Area of rectangle’ Sum of circumference of circle and


perimeter of rectangle is 220 cm. and area of circle is 1386 sq. cm. If
1
length of rectangle is 33 % more than radius of circle.
3
Quantity II- ‘Perimeter of square’ Circumference of a circle is 132 cm
and area of circle is 710 sq. cm more than area of square
(a) Quantity I > Quantity II (b) Quantity I < Quantity II
(c) Quantity I ≥ Quantity II (d) Quantity I ≤ Quantity II
(e) Quantity I = Quantity II or no relation

5. Quantity I – Veer takes two times more than the time taken by Sameer
to complete a work. Sameer and veer started to work alternatively
starting with Sameer on first day and Veer on second day. If both
completed work in 36 days, then find in how many days Sameer and
Veer will complete the work together, if both work with their double
efficiency?

Quantity II – Tap P can fill a swimming pool in 8 days and tap Q can fill
the same swimming pool in 24 days, while tap R can empty the
swimming pool in 16 days. If all three taps are opened together in
swimming pool, then in how much time swimming pool will filled
completely?
(a) Quantity I > Quantity II (b) Quantity I < Quantity II
(c) Quantity I ≥ Quantity II (d) Quantity I ≤ Quantity II
(e) Quantity I = Quantity II or no relation

6. Quantity I: P alone can do a piece of work in 72 days and Q alone can


do same work in 48 days, R is 50% more efficient than Q then find in
how many days total work will be completed if all three work on
alternate days starting with P and ending with R ?
Quantity II: A alone can do a task in 48 days B is 25% less efficient than
A and C can do the same task in 40 days lesser than B does. Then find in
how many days all three will complete the task together?
(a) Quantity I > Quantity II (b) Quantity I < Quantity II
(c) Quantity I ≥ Quantity II (d) Quantity I ≤ Quantity II
(e) Quantity I = Quantity II or no relation
Adda247 Publications For any detail, mail us at
5
Publications@adda247.com
Cracker Book for Bank (IBPS | SBI | RRB PO | Clerk) Mains Exams

7. Quantity I: A bucket carried one dozen of mangoes. One third of


mangoes become bad, if three mangoes taken out from the bucket at
random. Then what is probability that one mango out of three mangoes
picked up is good?
Quantity II: Find the probability of selecting a face card from a well
shuffled pack of card given that 2 additional Joker card also counts as a
face card?
(a) Quantity I > Quantity II (b) Quantity I < Quantity II
(c) Quantity I ≥ Quantity II (d) Quantity I ≤ Quantity II
(e) Quantity I = Quantity II or no relation

8. Quantity I – A train running at the speed of 72 km/hr passed a man in


14 sec and a platform in 32 sec. Find the length of platform ?
Quantity II – A train moves at the speed of 108 km/hr, passes a
platform and a bridge in 15 sec and 18 sec respectively. If the length of
platform is 50% of length of bridge, then find the length of train.
(a) Quantity I > Quantity II (b) Quantity I < Quantity II
(c) Quantity I ≥ Quantity II (d) Quantity I ≤ Quantity II
(e) Quantity I = Quantity II or no relation

9. Quantity I — Difference between the speeds of P and Q if 2 places A


and B are 60 km apart. P and Q start from A at same time & meet 1st
time at a place 12 km from B & they reach A after immediate return
from B. The speed of slower person is 48 km/hr.
Quantity II — Average speed of train if a distance of 600 km is to be
covered in 2 parts. In 1st phase 120 km is travelled by train and rest by
car and it took total of 8 hrs, but if 200 km is covered by train and rest
by car it takes 20 min more.
(a) Quantity I > Quantity II (b) Quantity I < Quantity II
(c) Quantity I ≥ Quantity II (d) Quantity I ≤ Quantity II
(e) Quantity I = Quantity II or No relation
Adda247 Publications For any detail, mail us at
6
Publications@adda247.com
Cracker Book for Bank (IBPS | SBI | RRB PO | Clerk) Mains Exams

10. Quantity I — Value of 𝑥, if ABCD is a rectangle and AB= 10 unit, AD= 6


unit

Quantity II — value of 𝑦, if volume of the cone is 16𝜋 unit3


Radius = 4 unit

(a) Quantity I > Quantity II (b) Quantity I < Quantity II


(c) Quantity I ≥ Quantity II (d) Quantity I ≤ Quantity II
(e) Quantity I = Quantity II or No relation

11. Quantity I – What is value of (A + B) if 25% of 48% of 55% of A is 1188


and 21% of 35% of B is equal to 5 more than square of 32.

Quantity II – Ratio of male population to female population of a town


in the year 2016 was 8 : 9. If in the year 2017 male population
increased by 25 % and female population increased by 40% as
compared to previous year so total population of town in the year 2017
was 45200. What was the total population of town in 2016.
(a) Quantity I > Quantity II (b) Quantity I < Quantity II
(c) Quantity I ≥ Quantity II (d) Quantity I ≤ Quantity II
(e) Quantity I = Quantity II or no relation
Adda247 Publications For any detail, mail us at
7
Publications@adda247.com
Cracker Book for Bank (IBPS | SBI | RRB PO | Clerk) Mains Exams

12. Quantity I – A man have two bucket which contains some fruits, first
bucket contains 5 mango and 7 banana and second bucket contains 4
mango and 5 banana. Also man have a pack of six card with numbering
1 - 6, if a card is taken out and it is even number, then he choose a fruit
from first bucket but if card is odd number, then man choose a fruit
from second bucket. Find the probability of choosing a mango.

Quantity II – In egg tray carried two dozen of eggs. One third of eggs
are rotten, if three eggs taken out from the tray at random. What is
probability that two eggs out of three eggs picked up is good?
(a) Quantity I > Quantity II
(b) Quantity I < Quantity II
(c) Quantity I ≥ Quantity II
(d) Quantity I ≤ Quantity II
(e) Quantity I = Quantity II or no relation

13. ‘A’ can complete a work alone in 12 days. A and B together can complete
2
the work in 6 days. C takes 5 more days than B to complete the work
3
alone. C is 50% more efficient than D.
Quantity I= ‘x’: A, B and C worked for alternate days starting from A
and finish the work in ‘x’ days

Quantity II= ‘y’: B and C worked together for 6 days and left and then D
completes the remaining work. ‘y’ is total time taken to complete the
work.
(a) Quantity I > Quantity II (b) Quantity I < Quantity II
(c) Quantity I ≥ Quantity II (d) Quantity I ≤ Quantity II
(e) Quantity I = Quantity II or No relation

14. Quantity I= ‘x’: A shopkeeper mark up an article 40% above its cost
price. At the time of sale, he gave 20% discount. By this, shopkeeper
earn Rs 168. ‘x’ is the difference between mark up price and cost price.
Quantity II= ‘y’ : Satish invested Rs.12000 in scheme ‘A’ at 12% S.I P.A.
for 4 years and Rs.25,000 in scheme B at 11% S.I P.A for 2 year. ‘y’ is the
difference between the interest earn from both schemes.
Adda247 Publications For any detail, mail us at
8
Publications@adda247.com
Cracker Book for Bank (IBPS | SBI | RRB PO | Clerk) Mains Exams

(a) Quantity I > Quantity II (b) Quantity I < Quantity II


(c) Quantity I ≥ Quantity II (d) Quantity I ≤ Quantity II
(e) Quantity I = Quantity II or No relation

15. ABCD is a rectangle and O is the midpoint of the line EF and diagonal
AC. AO=20cm and AB=24cm

Quantity I: Area of shaded region


Quantity II: 98 cm²
(a) Quantity I > Quantity II (b) Quantity I < Quantity II
(c) Quantity I ≥ Quantity II (d) Quantity I ≤ Quantity II
(e) Quantity I = Quantity II or No relation

16. Quantity I= ‘x’: Veer distributed Rs.80,000 among his wife, two
daughter and three sons. His wife got 300% more than the amount
every son got. If 30% of the total amount given to the sons then ‘x’ is the
amount every daughter got
Quantity II= ‘y’: Yogesh start a business with Rs 10800. After 4 months
veer join him with Rs 37800. After 2 months Amit join them with Rs
28800. After 1 year they got total Rs 54880 as profit. ‘y’ is the share of
Yogesh in profit.
(a) Quantity I > Quantity II (b) Quantity I < Quantity II
(c) Quantity I ≥ Quantity II (d) Quantity I ≤ Quantity II
(e) Quantity I = Quantity II or No relation

Adda247 Publications For any detail, mail us at


9
Publications@adda247.com
Cracker Book for Bank (IBPS | SBI | RRB PO | Clerk) Mains Exams

17. Quantity I: Profit share of ‘A’ out of total annual profit of Rs. 56,500. A,
B and C enter into a partnership. ‘A’ invests Rs. 4000 for the whole year,
‘B’ puts in Rs. 6000 at the first and increasing to Rs. 8000 at the end of 4
months, whilst C puts in at first Rs. 8000 but withdraw Rs. 2000 at the
end of 9 months.

Quantity II: Amount which when lend on C.I. at 20% interest being
compounded annually for 3 years, gives total interest equal to Rs.9100
(a) Quantity I > Quantity II
(b) Quantity I ≥ Quantity II
(c) Quantity I < Quantity II
(d) Quantity I = Quantity II or No relation
(e) Quantity I ≤ Quantity II

18. ‘A’, ‘B’ and ‘C’ together can complete a work in 48 days if they work
1
alternatively. ‘A’ is 25% more efficient than ‘B’ who is 33 % less
3
efficient than ‘C’.
Quantity I: Difference between days taken by ‘A’ alone and ‘C’ alone to
complete the work.
Quantity II: Days in which ‘A’, ‘B’ and ‘C’ together can complete half of
the work.
(a) Quantity I > Quantity II (b) Quantity I < Quantity II
(c) Quantity I ≥ Quantity II (d) Quantity I ≤ Quantity II
(e) Quantity I = Quantity II or No relation

19. X started from a point A towards point B. After 2 hours. Y started from
B towards A. By the time X travelled one-fifth of the total distance, Y had
also travelled the same. Y’s speed is thrice of that of X’s speed.
Quantity I: Difference in time (in hours) taken by X and Y to reach their
respective destinations.
Quantity II: 12 hours
(a) Quantity I = Quantity II or No relation
(b) Quantity I ≤ Quantity II
(c) Quantity I ≥ Quantity II
(d) Quantity I < Quantity II
(e) Quantity I > Quantity II
Adda247 Publications For any detail, mail us at
10
Publications@adda247.com
Cracker Book for Bank (IBPS | SBI | RRB PO | Clerk) Mains Exams
𝑋
20. 5X men can complete a work in days while 2Y men can complete same
2
4𝑌
work in days.
5
Quantity I: Value of ‘Y+20’.
Quantity II: Value of ‘1.25X'
(a) Quantity I ≥ Quantity II
(b) Quantity I = Quantity II or No relation
(c) Quantity I > Quantity II
(d) Quantity I < Quantity II
(e) Quantity I ≤ Quantity II

21. Total surface area of a cylinder is 200% more than that of its curved
surface area. Volume of cylinder is 2156 𝑐𝑚3
Quantity I: Volume of cone whose base radius and height is same as
that of radius and height of cylinder respectively.

Quantity II: Volume of hemisphere whose radius is same as that of


radius of cylinder.
(a) Quantity I > Quantity II
(b) Quantity I = Quantity II or No relation
(c) Quantity I ≥ Quantity II
(d) Quantity I < Quantity II
(e) Quantity I ≤ Quantity II

22. When two ships A & B are travelling in opposite direction crosses each
other in 2 second. Ship B is 50% faster than ship A in still water. On a
particular day, ship A was ahead of ship B by 9 meters and both of them
were travelling in downstream. if Ship B overtook Ship A in 11 seconds.
‘X’ is the speed of ship A and if length of ship B is 54 meters.
Quantity I: 16 m/s
Quantity II: Value of ‘X’ in m/s
(a) Quantity I = Quantity II (b) Quantity I ≥ Quantity II
(c) Quantity I ≤ Quantity II (d) Quantity I > Quantity II
(e) Quantity I < Quantity II
Adda247 Publications For any detail, mail us at
11
Publications@adda247.com
Cracker Book for Bank (IBPS | SBI | RRB PO | Clerk) Mains Exams

23. Quantity I= 𝒙

(o is the center of the circle)


Quantity II = 𝒚

(a) Quantity I ≥ Quantity II (b) Quantity I > Quantity II


(c) Quantity I ≤ Quantity II (d) Quantity I = Quantity II
(e) Quantity I < Quantity II

24. ‘B’ kg of Rs. 36 per/kg rice that mixed with 8 kg of Rs. 42 per/kg rice, in
order to earn profit of 10% while selling the mixture at Rs. 44 per/kg.
Quantity I: Value of ‘B+20’.
Quantity II: Value of ‘6B'

25. Quantity I –Vessel A contains (Q + 36) liter mixture of milk & water in
the ratio of 7 : 2, while vessel B contains (2Q + 42) liter mixture of milk
& water in the ratio of 2 : 3. If 40% & 46% of mixture from vessel A and
B taken out respectively, then remaining mixture in vessel B is 150% of
remaining mixture in vessel A. Find the total initial quantity of milk in
mixture of vessel A & vessel B together?

Adda247 Publications For any detail, mail us at


12
Publications@adda247.com
Cracker Book for Bank (IBPS | SBI | RRB PO | Clerk) Mains Exams

Quantity II – Two vessels contains mixture of mango juice and orange


juice in the ratio of 5 : 3 and 5 : 4 respectively. If 40 liter mixture from
first vessel taken out and mixed in second vessel, so new ratio of mango
juice and orange juice in second vessel becomes 25 : 19. Find initial
quantity of mixture in second vessel?
(a) Quantity I > Quantity II (b) Quantity I < Quantity II
(c) Quantity I ≥ Quantity II (d) Quantity I ≤ Quantity II
(e) Quantity I = Quantity II or no relation

26. Quantity I – Center of semicircle B is O1 and radius is 28 cm. find area


of shaded figure. BO1 is perpendicular to AC

Quantity II – Length of a rectangle is two times of side of square, while


ratio between side of square to breadth of rectangle is 8 : 7. If perimeter
of rectangle is 28 cm more than perimeter of square, then area of
rectangle.
(a) Quantity I > Quantity II (b) Quantity I < Quantity II
(c) Quantity I ≥ Quantity II (d) Quantity I ≤ Quantity II
(e) Quantity I = Quantity II or no relation

27. 5y² + 21y + 18 = 0 and


16(𝑥+2) ÷ 4(𝑥+3) = 64(𝑥+3) × 4(𝑥+4)
Quantity I: Value of y.
Quantity II: Value of x.
(a) Quantity I > Quantity II (b) Quantity I < Quantity II
(c) Quantity I ≥ Quantity II (d) Quantity I ≤ Quantity II
(e) Quantity I = Quantity II or no relation

Adda247 Publications For any detail, mail us at


13
Publications@adda247.com
Cracker Book for Bank (IBPS | SBI | RRB PO | Clerk) Mains Exams

28. Quantity I: Find the cost of B per kg if a product C is made of two


ingredients A and B in the proportion of 2 : 5. The price of A is three
times that of B. The overall cost of C is Rs. 5.20 per kg including labour
charges of 80 paisa per kg.

Quantity II: Find the cost of manufacturing each article if a


manufacturer estimates that on inspection 12% of the articles he
produces will be rejected. He accepts an order to supply 22,000 articles
at Rs. 7.50 each. He estimates the profit on his outlay including the
manufacturing of rejected articles, to be 20%.
(a) Quantity I > Quantity II (b) Quantity I < Quantity II
(c) Quantity I ≥ Quantity II (d) Quantity I ≤ Quantity II
(e) Quantity I = Quantity II or no relation

29. Quantity I: Value of x

Quantity II: Value of y


(20)3 +32 ×103 +25×50
=𝑦+1
54
(a) Quantity I > Quantity II (b) Quantity I < Quantity II
(c) Quantity I ≥ Quantity II (d) Quantity I ≤ Quantity II
(e) Quantity I = Quantity II or No relation

30. Quantity I: What is probability of getting two red balls when they are
drawn from a bag which contains 3 Red balls, 5 blue balls and 4 white
balls.
Quantity 2: Value of x.
1 2 1 1 3 11
5 +5 −5 = +5 −
2 7 14 𝑥 14 2
(a) Quantity I > Quantity II (b) Quantity I < Quantity II
(c) Quantity I ≥ Quantity II (d) Quantity I ≤ Quantity II
(e) Quantity I = Quantity II or No relation
Adda247 Publications For any detail, mail us at
14
Publications@adda247.com
Cracker Book for Bank (IBPS | SBI | RRB PO | Clerk) Mains Exams

31. Total surface area of cube and sphere are equal.


Quantity I: Volume of cube ÷ √𝜋
Quantity II: volume of sphere
(a) Quantity I > Quantity II (b) Quantity I < Quantity II
(c) Quantity I ≥ Quantity II (d) Quantity I ≤ Quantity II
(e) Quantity I = Quantity II or No relation

32. Figure I and II shows two semicircles of radius 14 cm with O and P as


their centres respectively.

Quantity I: Area of shaded region in figure I


Quantity II: Area of triangle in figure II
(a) Quantity I > Quantity II (b) Quantity I < Quantity II
(c) Quantity I ≥ Quantity II (d) Quantity I ≤ Quantity II
(e) Quantity I = Quantity II or No relation

33. Quantity I – A can complete a task in 24 days and B can do same task in
1
18 days. another two persons C & D complete 58 % of the same task in
3
7 days and efficiency of D is 40% more than that of C. Find in how many
days A, B and D will be complete the task together?

Quantity II – Ankit is 60% less efficient than Satish and complete a


piece of work in 22.5 days. Ankit and Saatish start work together and
after 4.5 days both left the work, if veer complete the remaining of
work in 4.5 days then find in how many days the whole work will be
completed, if all three work together ?
(a) Quantity I > Quantity II (b) Quantity I < Quantity II
(c) Quantity I ≥ Quantity II (d) Quantity I ≤ Quantity II
(e) Quantity I = Quantity II or no relation
Adda247 Publications For any detail, mail us at
15
Publications@adda247.com
Cracker Book for Bank (IBPS | SBI | RRB PO | Clerk) Mains Exams

34. Quantity I – A shopkeeper has two articles A & B. Marked price of


article B is 20% more than marked price of article A, shopkeeper sold
article A at 25% discount and article B at 20% discount. He made 20%
2
loss on article A and 6 % 𝑝𝑟𝑜𝑓𝑖𝑡 on article B. If total loss of shopkeeper
3
was Rs. 765, then find marked price of article B?

Quantity II – A shopkeeper gives a discount of 24% on marked price


of Shirt and cost price of Jeans is 25% more than selling price of Shirt. If
shopkeeper sold Jeans at 10 % profit and selling price of Jeans was Rs.
1140 more than selling price of Shirt , then find the cost price of article
Jeans ?
(a) Quantity I > Quantity II (b) Quantity I < Quantity II
(c) Quantity I ≥ Quantity II (d) Quantity I ≤ Quantity II
(e) Quantity I = Quantity II or no relation

35. Quantity I – A bag contains four green balls, three red balls and five
blue balls. If three balls taken out at random what is probability of at
least one ball is green and at least one ball is blue color.

Quantity II – There are five red toys and six green toys in a cartoon.
What will be the probability of selection of four toys which contains at
least two green toys?
(a) Quantity I > Quantity II (b) Quantity I < Quantity II
(c) Quantity I ≥ Quantity II (d) Quantity I ≤ Quantity II
(e) Quantity I = Quantity II or no relation

36. Quantity I – Six years ago ratio between age of Rohit and Prakash was
1 1
7 : 8, while six years hence ratio between 𝑡ℎ of Rohit age and 𝑟𝑑 of
6 3
Prakash age will be 9 : 20. The age of Rohit two years hence will be.

Quantity II – Ratio between age of A, B and C is 16 : 9 : 7. Three years


hence average of all three age will be 35 years. The age of A two years
hence will be?
(a) Quantity I > Quantity II (b) Quantity I < Quantity II
(c) Quantity I ≥ Quantity II (d) Quantity I ≤ Quantity II
(e) Quantity I = Quantity II or no relation
Adda247 Publications For any detail, mail us at
16
Publications@adda247.com
Cracker Book for Bank (IBPS | SBI | RRB PO | Clerk) Mains Exams

37. A diagram is given below. AC is the diameter of circle. AD || BC and


∠𝐴𝐶𝐷 = 90° ∠𝐴𝐶𝐵 ≤ 45°

Quantity I: -. ∠𝐴𝐷𝐶
Quantity II: - 45°
(a) Quantity I > Quantity II (b) Quantity I < Quantity II
(c) Quantity I ≥ Quantity II (d) Quantity I ≤ Quantity II
(e) Quantity I = Quantity II or No relation

38. Quantity I: ‘x’: There is an interview conducted by HCL. Out of total


applicants six are female. Probability of selecting two candidates such
7
that at least one is male is . ‘x’ is total number of males.
8

Quantity II: ‘y’ : A, B and C together start a business. Ratio between


investment of A and B is 15 : y while ratio between investment of B & C
is 9 : 11. A left business after 8 months of starting of business and B left
before 7 months of completion of year. Out of total profit of Rs. 2080, A
got Rs. 900.
(a) Quantity I > Quantity II (b) Quantity I < Quantity II
(c) Quantity I ≥ Quantity II (d) Quantity I ≤ Quantity II
(e) Quantity I = Quantity II or No relation

39. Quantity I: ‘x’: Two mutually perpendicular chords PQ and EF meet at a


point S inside a circle. PS = 12 cm, SQ = 8 cm and FS = 6 cm. ‘x’ is the
area of circle.
Quantity II: ‘y’: In a circle, two parallel chords are drawn on opposite
sides of diameter. Distance between these chords is 14cm. Length of
one chord is 16cm while length of another chord is 12cm. ‘y’ is the area
of circle

Adda247 Publications For any detail, mail us at


17
Publications@adda247.com
Cracker Book for Bank (IBPS | SBI | RRB PO | Clerk) Mains Exams

(a) Quantity I > Quantity II (b) Quantity I < Quantity II


(c) Quantity I ≥ Quantity II (d) Quantity I ≤ Quantity II
(e) Quantity I = Quantity II or No relation

40. Quantity I: ‘x’ :A dishonest shopkeeper sales his good by weighting


15% less and cheat his wholesaler by giving 15% less amount. ‘x’ is his
approximate total profit % if he sells it at cost price.
Quantity II: ‘y’ : A dishonest milkman, add 20 litre water in pure milk,
after selling half of mixture. He again added 20 liter of water in
remaining mixture now milk and water ratio in mixture becomes 4 : 3.
‘y’ is his overall profit % on selling total mixture.
(a) Quantity I > Quantity II
(b) Quantity I < Quantity II
(c) Quantity I = Quantity II or No relation
(d) Quantity I ≥ Quantity II
(e) Quantity I ≤ Quantity II

41. Quantity I: ‘x’ : A and B can do a work in 24 days together, in which


40% of work is done by B. C who is 25% more efficient than A, do the
same work alone in ‘x’ days.

Quantity II: ‘y’ : P, Q and S work for x, x+1 and x+2 days and their
efficiency are in the ratio of x : x+1 : x+2 respectively. If P got 16 Rs for
his work, then ‘y’ is the total money earned by all three in Rs..
(Compare Magnitude of x and y)
(a) Quantity I > Quantity II
(b) Quantity I ≥ Quantity II
(c) Quantity I = Quantity II or No relation
(d) Quantity I < Quantity II
(e) Quantity I ≤ Quantity II

42. Quantity I:’x’ -. Two circles are concentric with center ‘O’. Their radii
are 8cm and 10cm respectively. ‘B’ and ‘C’ are the points of contact of
two tangents drawn from bigger circle to smaller circle from point ‘A’
lying on bigger circle. ‘x’ is area of quadrilateral ABOC formed in 𝑐𝑚2

Adda247 Publications For any detail, mail us at


18
Publications@adda247.com
Cracker Book for Bank (IBPS | SBI | RRB PO | Clerk) Mains Exams

Quantity II: - ‘y’ - The lengths of two parallel sides of a trapezium are 6
cm and 8 cm. If the height of the trapezium be 6 cm, then its area is ‘y’
𝑐𝑚2
(a) Quantity I > Quantity II (b) Quantity I < Quantity II
(c) Quantity I ≥ Quantity II (d) Quantity I ≤ Quantity II
(e) Quantity I = Quantity II or No relation

43. Quantity I: ‘x’ -. Kishan invested Rs. 20,000 in a bank offering 22% p.a.
at Simple interest. After 2 years he withdraws his money and invested
in another bank which is offering ‘x%’ p.a. at compound interest. After 3
years, interest earned by him is Rs.1350 less than amount invested by
him in this bank.
5
Quantity II: - ‘y’ – Bhavya sell a diary at Marked price and earns 85 %
7
profit while if he gives 'y%’ discount on Marked price then he will earn
‘y%’ profit.
(a) Quantity I > Quantity II (b) Quantity I < Quantity II
(c) Quantity I ≥ Quantity II (d) Quantity I ≤ Quantity II
(e) Quantity I = Quantity II or No relation

44. Quantity I —‘x’ : B alone can do the work in ‘x’ days. A can complete a
work in 5 more days than B while A does the same work in 9 more days
than C. If A and B can complete the whole work in same time as time
taken by C alone to do the whole work.

Quantity II —‘y’ : ‘y’ is the days taken by 8 men and 14 women to reap
7
𝑝𝑎𝑟𝑡 of 360-hectare land by working 7 hrs per day if 6 men and 10
12
5
women can reap part of the land in 15 days by working 6 hrs per day.
12
It is also given that work of 2 men is equal to that of 3 women.
(a) Quantity I > Quantity II (b) Quantity I < Quantity II
(c) Quantity I ≥ Quantity II (d) Quantity I ≤ Quantity II
(e) Quantity I = Quantity II or No relation
Adda247 Publications For any detail, mail us at
19
Publications@adda247.com
Cracker Book for Bank (IBPS | SBI | RRB PO | Clerk) Mains Exams

45. Quantity I — ‘x’ : ‘x’ is the difference between the speeds of P and Q.
Distance between A and B are 60 km. P and Q start from A at same time
& meet 1st time at a place 12 km from B. They return to A immediately
after reaching B. The speed of slower person is 48 km/hr.

Quantity II —‘y’ : ‘y’ is the average speed of train if a distance of 600


km is to be covered in 2 parts. In 1st phase 120 km is travelled by train
and rest by car and it took total of 8 hrs, but if 200 km is covered by
train and rest by car it takes 20 min more.
(a) Quantity I > Quantity II (b) Quantity I < Quantity II
(c) Quantity I ≥ Quantity II (d) Quantity I ≤ Quantity II
(e) Quantity I = Quantity II or No relation

46. Quantity I: ‘x’ : ‘𝒙′ is the number of days taken by A alone to complete a
work. A and B together can complete the same work in 16 days while A
1
and C can complete the same work in 15 days. Ratio of efficiency of B
3
and C is 3 : 4.
Quantity II: ′𝒚′: P, Q and R takes ‘y’ days to complete a work. Ratio of
efficiency of P, Q and R are 2 : 3 : 4 respectively and P and Q together
takes 18 days to complete that work.
(a) Quantity I > Quantity II (b) Quantity I < Quantity II
(c) Quantity I ≥ Quantity II (d) Quantity I ≤ Quantity II
(e) Quantity I = Quantity II or No relation

47. Quantity I: ‘x’ : A article is sold at 100 Rs. profit whose cost price is
‘Rs.𝒙’. If article is sold at 20% more than its S.P. then S.P becomes 2𝑥.

Quantity II: ′𝒚′ : Article is sold at y% profit whose M.P. is 400% above
its C.P. and discount given on M.P. is 60%.
(a) Quantity I > Quantity II (b) Quantity I < Quantity II
(c) Quantity I ≥ Quantity II (d) Quantity I ≤ Quantity II
(e) Quantity I = Quantity II or No relation
Adda247 Publications For any detail, mail us at
20
Publications@adda247.com
Cracker Book for Bank (IBPS | SBI | RRB PO | Clerk) Mains Exams

48. Quantity I: ‘x’: Sum of ‘𝒙′ consecutive number is 4040. First number is
11.
Quantity II: ′𝒚′: Sum of ′𝒚′ consecutive even numbers is 7120. First
number is 10.
(a) Quantity I > Quantity II (b) Quantity I < Quantity II
(c) Quantity I ≥ Quantity II (d) Quantity I ≤ Quantity II
(e) Quantity I = Quantity II or No relation

49. Quantity I: ‘x’: A man invested 𝑹𝒔. ′𝒙′ in two scheme which offers 40%
S.I. for 2 years and 44% S.I. for 5 years respectively. Difference between
interest earned from both scheme is 18,900.

Quantity II: ′𝒚′: Interest earned in a scheme is ′𝑦′ which offers 12%
rate of interest for 3 years on a sum. 30% of the sum is 12000.
(a) Quantity I > Quantity II (b) Quantity I < Quantity II
(c) Quantity I ≥ Quantity II (d) Quantity I ≤ Quantity II
(e) Quantity I = Quantity II or No relation

50. The largest sphere that can fit in a cube of edge 7 cm.
Quantity I: ‘x’: ‘x’ is the volume of vacant space left over after putting
the sphere.(𝑖𝑛 𝑐𝑚3 )
Quantity II: ′𝒚′: ’y’ is the surface area of the sphere. (in 𝑐𝑚2 )
(a) Quantity I > Quantity II (b) Quantity I < Quantity II
(c) Quantity I ≥ Quantity II (d) Quantity I ≤ Quantity II
(e) Quantity I = Quantity II or No relation

PART B
Directions (Q1-5): In each of these questions, two equations I. and II. are
given. You have to solve both the equations and give answer
1. I. 3𝑥+5 . 92𝑥−4 = 95𝑥−14
II. 2𝑦 2 − 15𝑦 − 28 = 3𝑦 2 − 23𝑦 − 13
(a) x < y
(b) x ≥ y
(c) No relation can be established between x and y.
(d) x ≤ y
(e) x > y

Adda247 Publications For any detail, mail us at


21
Publications@adda247.com
Cracker Book for Bank (IBPS | SBI | RRB PO | Clerk) Mains Exams
25 15
2. I. − +2=0
x² x
40 13
II. + 1=
y² y
(a) x < y
(b) x ≥ y
(c) No relation can be established between x and y.
(d) x ≤ y
(e) x > y

3. I. 2(y+4) . 12(2y+4) = 9(y+2) . 16(y+4)


II. x 2 – 10x + 24 = 0
(a) x ≤ y
(b) x > y
(c) No relation can be established between x and y.
(d) x < y
(e) x ≥ y

48 14
4. I. − +1=0
x2 x
45 1
II. + =2
y² y

(a) No relation can be established between x and y.


(b) x ≤ y
(c) x < y
(d) x > y
(e) x ≥ y

5. I. (𝑥 − 4)2 = 9
II. (2𝑦 + 3)2 = 25
(a) No relation can be established between x and y.
(b) x < y
(c) x ≤ y
(d) x ≥ y
(e) x > y

Adda247 Publications For any detail, mail us at


22
Publications@adda247.com
Cracker Book for Bank (IBPS | SBI | RRB PO | Clerk) Mains Exams

Directions (6-10): In each of these questions, two equations I. and II. are
given. You have to solve both the equations and give answer
(a) if x>y
(b) if x≥y
(c) if x<y
(d) if x ≤y
(e) if x = y or no relation can be established between x and y.

6. I. 5x² + 3x – 36 = 0 II. 2y² – 13y + 20 = 0

7. I. x² – 7x + 12 = 0 II. 2y² – 11y + 15 = 0

8. I. 2x² + 11x + 15 = 0 II. 2y² + 9y + 10 =0

9. I. 3x² + 7x – 40 = 0 II. 5y² – 29y + 42 = 0

10. I. 3x² – 23x + 42 = 0 II. 2y² – 19y + 45 = 0

Directions (11-15): In each of these questions, two equations I. and II. are
given. You have to solve both the equations and give answer
(a) if x>y
(b) if x≥y
(c) if x<y
(d) if x ≤y
(e) if x = y or no relation can be established between x and y.

11. I. 𝑥 2 − 5𝑥 + 4 = 0 II. 𝑦 2 + 5𝑦 + 6 = 0

12. I. 2𝑥 2 − 𝑥 − 15 = 0 II. 3𝑦 2 − 23𝑦 + 42 = 0

13. I. 𝑥 2 − 15𝑥 + 54 = 0 II. 𝑦 2 + 15𝑦 − 54 = 0

14. I. 𝑥 2 + 14𝑥 + 40 = 0 II. 𝑦 2 − 5𝑦 − 24 = 0

15. I. 𝑥 2 − 225 = 0 II. 𝑥 2 + 𝑦 2 = 306

Adda247 Publications For any detail, mail us at


23
Publications@adda247.com
Cracker Book for Bank (IBPS | SBI | RRB PO | Clerk) Mains Exams

Directions (16-20): In each of these questions, two equations I. and II. are
given. You have to solve both the equations and give answer
(a) if x>y
(b) if x≥y
(c) if x<y
(d) if x ≤y
(e) if x = y or no relation can be established between x and y.

16. I. 2x² – 7x + 6 = 0 II. 3y² – 19y + 28 = 0

17. I. x² – 13x + 36 =0 II. 3y² – 19y + 30 = 0

25
18. I. x³– 120 = 96 II. y– =0
𝑦

19. I. x² – 2x – 48 = 0 II. y² – 15y + 54 = 0

20. I. 2x² + 25x + 72 = 0 II. 3y² + 22y + 40 = 0

Adda247 Publications For any detail, mail us at


24
Publications@adda247.com
Cracker Book for Bank (IBPS | SBI | RRB PO | Clerk) Mains Exams

Solutions

1. (a); Suppose B takes ‘x’ days,


Then C will take ‘3x’ days
1 1 1
+ =
3𝑥 𝑥 22.5
Or, x = 30
Now, Let A and B together take ‘2y’ days,
Then A will take ‘3y’ days,
1 1 1
+ =
3𝑦 30 2𝑦
Or, y = 5
Hence, A will take 3y = 3 × 5 = 15 days.
Quantity I = 15
90
Quantity II = = 90/7
6+1
Quantity I > Quantity II

117×3+2
2. (b); Quantity I→ × 600 = 706 Rs
300
25𝑥
Quantity II→ 𝑥 + = 1000
100
100
𝑥= × 1000 = 800 Rs
125
∴ Quantity I < Quantity II

3. (b); Quantity I: Volume of cube left


3 7 2
=7 −𝜋( ) ×7
2
22 49×7
= 343 − ×
7 4
= 343 − 269.5
= 73.5 cm3
Quantity II: Surface area of cube left
2 7 2 7
= 6 × 7 − 2. 𝜋 ( ) + 2𝜋 ( ) 7
2 2
= 294 − 77 + 154
= 371 cm2
Quantity II > Quantity I

Adda247 Publications For any detail, mail us at


25
Publications@adda247.com
Cracker Book for Bank (IBPS | SBI | RRB PO | Clerk) Mains Exams

4. (a); Quantity I – 2πr + 2 (ℓ + b) = 220 cm


πr² = 1386 sq. cm
1386×7
r2 =
22
r = 21 cm
Length of rectangle
4
= 21 × = 28 cm
3
22
2× × 21 + 2(28 + b) = 220
7
132 + 56 + 2b = 220
32
b= = 16 cm
2
Area of rectangle = (28 × 16) = 448 cm

Quantity II–2πr = 132


132×7
r=
2×22
r = 21 cm
πr² – a² = 710
22
× 21 × 21 – a2 = 710
7
a² = 1386 – 710
a² = 676 cm
a = 26 cm
Perimeter of square = 4 × 26 = 104 cm
Quantity I > Quantity II

5. (b); Quantity I— Let Veer take 3𝑥 days and Sameer take 𝑥 days.
Efficiency of Veer and Sameer be 𝑥 unit/day and 3𝑥 units/day
respectively
Total work = 3𝑥 × 18 + 𝑥 × 18 = 72x units
If both do with double efficiency
Then,
72𝑥
= (3𝑥×2+2𝑥) = 9 days

Adda247 Publications For any detail, mail us at


26
Publications@adda247.com
Cracker Book for Bank (IBPS | SBI | RRB PO | Clerk) Mains Exams

Quantity II—

When all three opened together


Total work in one day = (6 + 2 – 3) = 5 units
48 3
Required time = = 9 days
5 5
Quantity I < Quantity II

6. (a); Quantity I -

150
R efficiency = 3 × = 4.5 w⁄d
100
3 days work = (P + Q + R)
= 2 + 3 + 4.5 = 9.5 work
Required days = 9.5 × 15 = 142.5 work
Remaining work = 144 – 142.5 = 1.5
1.5 3
Next day by P = =
2 4
3 3
Total days = 15 × 3 + = 45 days
4 4

Quantity II. - A : B = 100 : 75 = 4 : 3


Total work = 4 × 48 = 192
192
B alone = = 64 days
3
C alone = 64 – 40 = 24 days
192
C efficiency = = 8 w⁄d
24
192 192 4
(A + B + C)together = = = 12 days
(4+3+8) 15 5
So, Quantity I > Quantity II

Adda247 Publications For any detail, mail us at


27
Publications@adda247.com
Cracker Book for Bank (IBPS | SBI | RRB PO | Clerk) Mains Exams

7. (b); Quantity I - No. of selection of 3 mango out of one dozen mangoes


= ¹²C₃ = 220
No. of selection of 2 bad mango out of 4 bad mangoes = ⁴C2 = 6
No. of selection of a good mango from 8 good mangoes = 8
8×6 12
Required probability = =
220 55
Quantity II - Face card = 12
Joker = 2
14 7
Probability = =
54 27
So, Quantity I < Quantity II

8. (e); Quantity I - Let length of train be L meter


ATQ—
5
Speed = 72 × = 20 m/sec
18
L
20 =
14
L = 280 meter
Let length of platform be P meter
P+280
20 =
32
P = 640 – 280
P = 360 meter

𝐐𝐮𝐚𝐧𝐭𝐢𝐭𝐲 𝐈𝐈 – Let’s length of train is X meter and length of bridge


and platform are 2L and L meter respectively
5
Speed of train = 108 × = 30 m⁄s
18
When train passed the bridge
Distance covered = X + 2L
X + 2L = 540 …(i)
When train passes the platform
Distance covered = X + L
X + L = 450 …(ii)
From (i) and (ii)
L = 90 meter
X = 360 meter
So, Quantity I = Quantity II
Adda247 Publications For any detail, mail us at
28
Publications@adda247.com
Cracker Book for Bank (IBPS | SBI | RRB PO | Clerk) Mains Exams

9. (b);

Let P is faster than Q


Then P covers 72 km distance in the same time as Q covers 48 km
distance
Ratio of the speed = 72 : 48 = 3 : 2
48
∴ Speed of faster train i.e., P = × 3 = 72 km/hr
2

Quantity 1→ Difference between P and Q


= 72 – 48 = 24 km/hr.
Let speed of train = T km/hr
Let speed of car = C km/hr
120 480
∴ + = 8 ………..(i)
𝑇 𝐶
200 400 1
+ = 8 ………(ii)
𝑇 𝐶 3
On solving (i) and (ii)
T = 60 km/hr
∴ Quantity I < Quantity II

6
10. (b); Quantity I → = 3
2
1
And, 𝜋𝑟 2 ℎ = 16𝜋
3
ℎ=3
∴ 𝑦 = √42 + 32 = √16 + 9
𝑦=5
∴ Quantity I < Quantity II

25 48 55
11. (a); Quantity I— 𝐴 × × × = 1188
100 100 100
1188×100×100×100
A=
25×48×55
A = 18000
21 35
𝐵× × = (32)2 + 5
100 100
21 25
𝐵× × = 1024 + 5
100 100

Adda247 Publications For any detail, mail us at


29
Publications@adda247.com
Cracker Book for Bank (IBPS | SBI | RRB PO | Clerk) Mains Exams
1029×100×100
𝐵=
21×25
B = 19600
(A + B) = 18000 + 19600 = 37600

Quantity II— Let male and female population of town in 2016 be


8x and 9x respectively
125 140
8𝑥 × + 9𝑥 × = 45200
100 100
63𝑥
10𝑥 + = 45200
5
113𝑥 = 226000
𝑥 = 2000
Total population of town in 2016 = 16000 + 18000
= 34000
Quantity I > Quantity II

12. (a); Quantity-I : Probability of choosing Mango


1 5 1 4 31
= × + × =
2 12 2 9 72
Quantity-II : Total no. of cases to choose 3 egg = ²⁴C₃
Favourable cases = ¹⁶C₂ × ⁸C₁
16C × 8C 15
2 1
Probability = 24C =
3 253
Quantity I > Quantity II

13. (e); A can complete work in= 12 days


3 1 9−5 4
‘B’ 1 day work is = − = =
20 12 60 60
60
B can complete the work in = = 15 days
4
C can complete the work in ‘20’ days
20×150
D can complete the work in = 30 days
100

Quantity I: ‘A + B + C three days’ work


1 1 1 5+4+3 12 1
= + + = = =
12 15 20 60 60 5
‘A + B + C’ can complete the work in total ’15 days
Adda247 Publications For any detail, mail us at
30
Publications@adda247.com
Cracker Book for Bank (IBPS | SBI | RRB PO | Clerk) Mains Exams

Quantity II: Remaining work for D


6 6 60−24−18 3
=1− − = =
15 20 60 10
3 30×3
of work is completed by D in
10 10
= 9days
Total time = 6 + 9 = 15 days
Quantity I = Quantity II

14. (a); Quantity I: let C.P = 100


Mark Up price = 140
80
Selling price = 140 × = 112
100
ATQ,
12 → 168
x = 40 → 560
x = 560

Quantity II: Interest from scheme A


12000×12×4
= = 5760
100
Interest from scheme B
25000×11×2
= = 5500
100
y= 5760 – 5500 = 260
x>y
Quantity I > Quantity II
15. (b); Quantity I:
OF=√202 − 12²=√400 − 144 = √256 = 16
Area of shaded region
Area of ∆AFE – Area of ∆ AFO
1 1
= × 12 × 32 − × 12 × 16
2 2
= 192 – 96 = 96 cm²
Quantity II > Quantity I

Adda247 Publications For any detail, mail us at


31
Publications@adda247.com
Cracker Book for Bank (IBPS | SBI | RRB PO | Clerk) Mains Exams

16. (a); Quantity I: Total amount sons got


30
= × 80000 = 24000
100
24000
Each son got = = 8000
3
400
Wife got = 8000 × = 32000
100
Total amount daughters got
80,000 – 24000 – 32000 = 24000
24000
Each daughter got = = 12000
2
x = 12,000
Quantity II: Ratio of investment or profit
= 10800 × 12 : 37800 × 8 : 28800 × 6
=3:7:4
54880
Share of Yogesh in profit = ×3
14
y = 11760
x>y
Quantity I > Quantity II

17. (c); ATQ,


Quantity I: Ratio of profit share
A : B : C
(4000×12) (6000×4)+(8000×8) (8000×9)+(6000×3)
24 : 44 : 45
24
Share of A = × 56,500 = Rs. 12,000
113
Quantity II:
20 3
9100 + P = P (1 + )
100
⇒ 9100 + P = 1.728P
or, P = Rs. 12,500
𝐐𝐮𝐚𝐧𝐭𝐢𝐭𝐲 𝐈𝐈 > 𝐐𝐮𝐚𝐧𝐭𝐢𝐭𝐲 𝐈
Adda247 Publications For any detail, mail us at
32
Publications@adda247.com
Cracker Book for Bank (IBPS | SBI | RRB PO | Clerk) Mains Exams

18. (e); ‘A’, ’B’ and ‘C’ alternatively can complete a work in 48 days
⇒ ‘A’, ‘B’ and ‘C’ together can complete same work in 16 days
Ratio between efficiency of A, B and C is 5 : 4 : 6
Ratio between days taken by A, B and C alone to complete the
same work is 12 : 15 : 10
Let A, B and C alone can complete work in 12x, 15x and 10x days
respectively.
1 1 1 1
⇒ + + = ⇒𝑥=4
12𝑥 15𝑥 10𝑥 16
Quantity I: Required difference
= (12 − 10)4 = 8 𝑑𝑎𝑦𝑠
Quantity II: A, B and C together can complete same work in 16
days, so they can complete half work in 8 days.
Quantity I = Quantity II

19. (d); Let the speed of X be x kmph. Distance travelled by X in 2 hours =


2x km.
1th
Suppose X takes ‘t’ hours to travel of the distance AB.
5
1
Y would take (t–2) hours to travel th of the distance AB.
5
As Y’s speed is thrice that of X’s speed.
𝑡−2 1
= ⇒ t=3
𝑡 3
1𝑡ℎ
of the distance AB = 3x km.
5
AB =15x km
15𝑥
Time taken by x to cover 15x km = = 15 hours
𝑥
15𝑥
Time taken by Y to cover 15x km =
3𝑥
= 5 ℎ𝑜𝑢𝑟𝑠.
∴ Difference in the time = 10 hours.
Quantity I : Difference in the time = 10 hours.
Quantity II : 12 hours
Quantity I < Quantity II
Adda247 Publications For any detail, mail us at
33
Publications@adda247.com
Cracker Book for Bank (IBPS | SBI | RRB PO | Clerk) Mains Exams
𝑋 4𝑌
20. (c); Total work = 5𝑋 × = 2𝑌 ×
2 5
𝑋2 16 𝑋 4
⇒ = ⇒ =
𝑌2 25 𝑌 5
Let X = 4a and Y = 5a
Quantity I: Y+20 = 5a+20
Quantity II: 1.25X = 5a
Quantity I > Quantity II

21. (b); T.S.A of cylinder = 2𝜋𝑟(𝑟 + ℎ)


C.S.A of cylinder = 2𝜋𝑟 2
ATQ,
3 2𝜋𝑟(𝑟+ℎ)
=
1 2𝜋𝑟 2
⇒ ℎ = 2𝑟
𝑉𝑜𝑙𝑢𝑚𝑒 𝑜𝑓 𝑐𝑦𝑙𝑖𝑛𝑑𝑒𝑟 = 𝜋𝑟 2 ℎ = 2156
⇒ 𝑟 = 7𝑐𝑚, ℎ = 14𝑐𝑚
1 2156
Quantity I: Volume of cone = 𝜋𝑟 2 ℎ =
3 3
Quantity II: Volume of hemisphere
2 2156
= 𝜋𝑟 3 =
3 3
Quantity I = Quantity II

22. (e); Let speed of ship A in still water is 2x m/s , and its length is ℓ m
And speed of ship B is 3x m/s and its length is 54 and speed of
water is Y m/s.
ATQ, when both of them are travelling in opposite direction.
Downstream speed of ship A = (2x + Y) m/s [assume ship A is
travelling in downstream and ship B in upstream]
Upstream speed of ship B = (3x – Y) m/s
Their relative speed = 2x + Y + 3x – Y = 5x m/s
ATQ,
54+ℓ
=2 …(i)
5x
Similarly when both are travelling in downstream their relative
speed is
= 3x + Y – 2x –Y= X m/s
Adda247 Publications For any detail, mail us at
34
Publications@adda247.com
Cracker Book for Bank (IBPS | SBI | RRB PO | Clerk) Mains Exams

ATQ,
54+9+ℓ
= 11
𝑥
63 + ℓ = 11x …(ii)
From (i)
ℓ = 10x – 54
put this value in eqn. (ii)
= 63 + 10x – 54 = 11x
x = 9 m/s
Hence speed of ship A in still water = 2 × 9 = 18 m/s
Quantity I = 16 m/s
Quantity II = 18 m/s
Quantity II > Quantity I

23. (b); 𝑥 = 180 − (90 + 30) = 90 − 30 = 60°


𝑦 = 180 − (60 + 40) (angles subtended by same arc in the same
segment are equal)
= 80°
∴ Quantity I < Quantity II

24. (e); Selling mixture at Rs. 44/kg with 10% profit means, the actual
price of mixture is Rs. 40/kg
Let B kg of Rs. 36/kg are mixed
Then
36𝐵+8×42
= 40
𝐵+8
36B + 336 = 40B + 320
4B = 16
B=4
Quantity I = B + 20 = 24
Quantity I = 𝟔 × 𝑩 = 𝟐𝟒
Quantity I = Quantity II
Adda247 Publications For any detail, mail us at
35
Publications@adda247.com
Cracker Book for Bank (IBPS | SBI | RRB PO | Clerk) Mains Exams

25. (b); Quantity I –


ATQ –
60 150 54
(Q + 36) × × = [(2Q + 42) × ]
100 100 100
3(60Q + 2160) = 2 (108Q + 2268)
180Q + 6480 = 216Q +4536
36Q = 1944
Q = 54 l
Total initial quantity of milk in mixture of vessel A & vessel B
7 2
= (54 + 36) × + (2 × 54 + 42) × = 70 + 60 = 130 l
9 5

Quantity II –
ATQ –
Let mixture of mango juice and orange juice in second vessel be 5y
liter and 4y respectively
5
5𝑦+40×8 25 5𝑦+25 25
3 = ⇒ =
4𝑦×40× 19 4𝑦+15 19
8
19y + 95 = 20y + 75
y = 20 l
Initial quantity of mixture in second vessel = 20 × 9 = 180 𝑙
Quantity I < Quantity II

26. (e); Quantity I–Area of semicircle – area of triangle


22 28×28 1
= × − × 56 × 28 = 1232 – 784 = 448 cm²
7 2 2

Quantity II – Ratio of Length, breadth of a rectangle & side of


square = 16 : 7 : 8
Let Length, breadth of a rectangle & side of square be 16x cm, 7x
cm & 8x cm respectively
ATQ –
2(16x + 7x) – 4 × 8𝑥 = 28 𝑐𝑚
46x – 32𝑥 = 28
x = 2 cm
Area of rectangle = (16× 2 ) × (2 × 7) = 448 cm2
Quantity I = Quantity II
Adda247 Publications For any detail, mail us at
36
Publications@adda247.com
Cracker Book for Bank (IBPS | SBI | RRB PO | Clerk) Mains Exams

27. (c); Quantity I: 5y² + 21y + 18 = 0


5y² + 5y + 15y + 18 = 0
Y (5y + 6) +3 (5y + 6) = 0
(y + 3) (5y + 6) = 0
–6
y = –3 or
3
Quantity II: 16(x+2)÷4(x+3) = 64(x+3)×4(x+1)
24x+8–2x–6 = 26x+18+2x+2
8x + 20 = 2x + 2
∴ x = –3.
∴ Quantity I ≥ Quantity II

28. (b); Quantity I: Let the price of B per kg be Rs. X. Then, the price of A
per kg = Rs. 3X
1kg of C contains 2/7 kg of A and 5/7 kg of B
Price of 1 kg of C = (2/7) × 3X + (5/7)X
= (11/7)X
By the given condition, 11X/7
= 5.20 – 0.80 = Rs. 4.40
⇒ X = 4.40 × (7/11) = Rs. 2.80
Hence the price of B per kg = Rs. 2.80.
Quantity II: If 100 articles are manufactured then 12 will be
rejected
Total selling price of 88 articles = 88 × 75
660×100
Total cost price = = 550
120
cost of manufacturing per article = 5.5 Rs
Quantity II > Quantity I

29. (b); Quantity I: ∠CAD = 90° – ∠ACD = 62


(x) = ∠CBD = 90° – ∠CAD
x = 28°
8000+9000+1250
Quantity II: =𝑦+1
625
y = 28.2
Quantity II > Quantity I
Adda247 Publications For any detail, mail us at
37
Publications@adda247.com
Cracker Book for Bank (IBPS | SBI | RRB PO | Clerk) Mains Exams
3C 1
30. (b); Quantity I: 12 2 =
C2 22
1 1 2 1 3 1
Quantity II: 5 ( + + − − )=
2 2 7 14 14 𝑥
1 1
5= ⇒𝑥 =
𝑥 5
Quantity II > Quantity I

31. (b); Let side of cube and radius of sphere be a and r respectively.
⇒ 6a² = 4πr²
2
a=√ π r
3

Quantity I: Volume of cube


=a×a×a
2 2
= π × √ π r 3 ÷ √𝜋
3 3
3
=(2/3 ) π r 3 2

4
Quantity II : volume of sphere πr 3
3
∴ Quantity II > Quantity I

32. (a); Quantity I: Area of shaded region


Area of circle – Area of triangle
Base of triangle = 2 × 14 = 28
22 14×14 1
= × – × 28 × 14
7 2 2
2
⇒ 112 cm

Quantity II: Radius = 14


1
Area of triangle = × 14 × 14 = 98 cm²
2
Quantity I > Quantity II

Adda247 Publications For any detail, mail us at


38
Publications@adda247.com
Cracker Book for Bank (IBPS | SBI | RRB PO | Clerk) Mains Exams

33. (a); Quantity I – Total work = 72 units (Lcm of days taken by A & B)
72
Efficiency of A = = 3 𝑢𝑛𝑖𝑡𝑠/𝑑𝑎𝑦
24
72
Efficiency of B = = 4 𝑢𝑛𝑖𝑡𝑠/𝑑𝑎𝑦
18
Let efficiency of C = x unit/day
175 1
7x + 7(1.4x) = 72 × = 42 𝑢𝑛𝑖𝑡𝑠
3 100
x = 2.5 units
Efficiency of D = 1.4 × 2.5 = 3.5 𝑢𝑛𝑖𝑡𝑠
72 6
(A + B + C) together = = 6 𝑑𝑎𝑦𝑠
(3+4+3.5) 7
Quantity II – Ratio of efficiency of Satish :
Ankit = 100x : 40x = 5x : 2x
Total work = 22.5 × 2x = 45x units
ATQ,
(Ankit + Satish) × 4.5 = 4.5 (5x + 2x) = 31.5x work
Remaining work = 45x– 31.5 x = 13.5x
13.5
Veer efficiency = = 3 units⁄day
4.5
45x
(Ankit + Satish + Veer) =
(5x+2x+3x)
= 4.5 days
So, Quantity I > Quantity II

34. (a); Quantity I – Let marked price of article


A = 100x
Marked price of article B = 120x
3
S. P. of article A = 100x × = 75x
4
4
S. P. of article B = 120 × = 96x
5
5
C. P. of article A = 75 × = 93.75x
4
15
C. P. of article B = 96x × = 90x
16
ATQ—
(93.75x + 90x) – (75x + 96x) = 765
183.75x – 171x = 765
12.75x = 765
x = 60
M.P. of article B = 60 × 120 = 7200 Rs.
Adda247 Publications For any detail, mail us at
39
Publications@adda247.com
Cracker Book for Bank (IBPS | SBI | RRB PO | Clerk) Mains Exams

Quantity II – MP of shirt = 100x Rs


76
SP of shirt = 100 × = 76𝑥 Rs
100
5
Cost price of jeans = 76𝑥 × = 95𝑥 Rs
4
110
Selling price of jeans = 95𝑥 ×
100
= 104.5x Rs
ATQ –
104.5x - 76x = 1140 Rs
28.5x = 1140 Rs.
X = 40 Rs.
Cost price of jeans = 95×40 = 3800 Rs.
So, Quantity I > Quantity II

35. (b); Quantity I – Required cases = (1 green, 1 red, I blue) or ( 2 green ,


1 blue) or (1 green , 2 blue)
4C × 3C × 5C 4C × 5C 4C × 5C 3 3 2 13
1 1 1 2 1 1 2
= 12C + 12C + 12C = + + =
3 3 3 11 22 11 22

Quantity II – Favorable case = (2G, 2R) or (3G, 1R) or 4G


∴ Probability
6C × 5C 6 C × 5C 6C
2 2 3 1 4
= 11C + 11C + 11C
4 4 4
15×10 20×5 15 265 53
= + + = =
330 330 330 330 66
So, Quantity I < Quantity II

36. (e); Quantity I – Let age of Rohit and Prakash six years ago be 7x years
and 8x years respectively
ATQ –
𝟕𝒙+𝟏𝟐
𝟔 𝟗
𝟖𝒙+𝟏𝟐 =
𝟐𝟎
𝟑
140x + 240 = 144x + 216
x = 6 years
Age of Rohit two years hence will be
= (7× 6 + 8) = 50 𝑦𝑒𝑎𝑟𝑠
Adda247 Publications For any detail, mail us at
40
Publications@adda247.com
Cracker Book for Bank (IBPS | SBI | RRB PO | Clerk) Mains Exams

Quantity II – Let age of A, B and C be 16y, 9y and 7y respectively.


ATQ –
(16y + 9y + 7y) = 35 × 3 − 3 × 3
32y = 96
y = 3 years
Age of A two years hence = 3 × 16 + 2 = 50 years
So, Quantity I = Quantity II

37. (c); ∠𝐴𝐵𝐶 = 90° (Semicircle property)


∠𝐶𝐴𝐵 + ∠𝐴𝐶𝐵 + ∠𝐴𝐵𝐶 = 180°
∠𝐶𝐴𝐵 + ∠𝐴𝐶𝐵 = 90°
As ∠𝐴𝐶𝐵 ≤ 45° So, ∠𝐶𝐴𝐵 ≥ 45°
∠𝐴𝐶𝐷 + ∠𝐶𝐴𝐷 + ∠𝐴𝐷𝐶 = 180°
∠𝐶𝐴𝐷 + ∠𝐴𝐷𝐶 = 90°
But ∠𝐴𝐶𝐵 = ∠𝐶𝐴𝐷 (AD || BC)
∠𝐴𝐶𝐵 + ∠𝐴𝐷𝐶 = 90°
As ∠𝐴𝐶𝐵 ≤ 45° So, ∠𝐴𝐷𝐶 ≥ 45°
Quantity I ≥ Quantity II

38. (a); Quantity I: Let, no. of males = x


ATQ,
x 6 x(x–1) 7
2× × + (x+6)(x+5) =
x+6 x+5 8
12x+x2 –x 7
(x+6)(x+5)
=
8
88x + 8x² = 7 (x² + 5x + 6x + 30)
x² + 11x – 210 = 0
x² + 21x – 10x – 210 = 0
x (x + 21) – 10 (x + 21) = 0
x = 10, –21

Adda247 Publications For any detail, mail us at


41
Publications@adda247.com
Cracker Book for Bank (IBPS | SBI | RRB PO | Clerk) Mains Exams

Quantity II: Ratio between investment of A, B and C is 135 : 9y :


11y
Profit sharing of A, B and C is
135 × 8 : 9𝑦 × 5 : 11𝑦 × 12
= 360 : 15𝑦 : 44𝑦
ATQ,
900 360
=
2080 360+59𝑦
⇒ (360 + 59y) = 104 × 8
⇒ 59y = 472
⇒y=8
Quantity I > Quantity II

39. (a); Quantity I:

As, PQ and EF are two line that intersect at S.


So, PS × SQ = ES × SF
12 × 8 = ES × 6
ES = 16 cm
From center O draw
OB ⊥ PQ and OA ⊥ EF
(12+8)
So, PB = PQ = = 10 cm
2
BS = OA = 2 cm
EF = 16 + 6 = 22 cm
22
EA = = 11 cm
2
In triangle ∆𝑂𝐴𝐸
OA² + EA² = OE²
2² + 11² = OE²
OE² = 125
Adda247 Publications For any detail, mail us at
42
Publications@adda247.com
Cracker Book for Bank (IBPS | SBI | RRB PO | Clerk) Mains Exams

OE = √125 = 5√5 cm
2
Area of circle = πr²= 𝜋(5√5)
22
= 125 × = 392.86 cm ²
7

Quantity II:

Let AB and CD are the chord drawn on opposite sides of diameter.


Length of CD is 16 cm and length of AB is 12 cm.
ATQ
𝐶𝐷 16
FC = = = 8𝑐𝑚
2 2
𝐴𝐵 12
AE = = = 6𝑐𝑚
2 2
EF = 14 cm
But AO = OC = radius of circle
𝑂𝐴2 = 𝐴𝐸 2 + 𝑂𝐸 2 = 𝑂𝐶 2 = 𝐶𝐹 2 + 𝑂𝐹 2
Let OE and OF be ‘a’ and ‘b’ respectively
𝑎2 − 𝑏2 = 82 − 62 = 28
And 𝑎 + 𝑏 = 14
So, 𝑎 = 8, 𝑏 = 6
Radius of circle = √62 + 82 = √36 + 64
= √100 = 10𝑐𝑚
Area of circle = 𝜋(10)2 = 314.28 𝑐𝑚2
Quantity I > Quantity II

Adda247 Publications For any detail, mail us at


43
Publications@adda247.com
Cracker Book for Bank (IBPS | SBI | RRB PO | Clerk) Mains Exams

40. (b); Quantity I:


Let cost price is 1000 of 1000 gm. he pays to whole seller ⇒ 850
Rs. for 1000 gm.
Effective cost price = 0.85 Rs. per gm.
Now he sells 850 gm. instead of 1000 gm.
Effective cost price of 850 gm. = 0.85 × 850 = 722.5
1000 –722.5
Profit(x) = × 100 = 38.40 ≈ 38%
722.5

Quantity II:
Let milkman have → q litre of milk
Initially he add → 20 litre of water
𝑞
Sale half Remaining mixture = + 10
2
Again add 20 litre water
𝑞
2 4
So, =
10+20 3
40
Total profit(y) = × 100 = 50%
80
Quantity II > Quantity I

41. (d); Quantity I: If B do 40% of work thus A do 60% of work


Ratio of efficiency of A and B = 3 : 2
Total work = 24 × (3 + 2) = 120 units
3×125 15
Efficiency of C = = units/day
100 4
120 × 4
Time taken by C = = 32 days
15

Quantity II: P work for → x day


Efficiency → x
So,
x × x ⇒ 16 Rs.
x ⇒ 4 Rs.
Money earned by P, Q and R
⇒ x² + (x + 1)² + (x + 2) ² = 4² + 5² + 6²
⇒ 77 Rs.
Quantity II > Quantity I

Adda247 Publications For any detail, mail us at


44
Publications@adda247.com
Cracker Book for Bank (IBPS | SBI | RRB PO | Clerk) Mains Exams

42. (a); Quantity I

OB = 8cm and OA = 10cm


As, AB and AC are tangents on smaller circle
⇒ ∠𝐴𝐵𝑂 = ∠𝐴𝐶𝑂 = 90°
⇒ 𝐴𝐵 = √102 − 82 = 6
Area of quadrilateral ABOC
= 2 × 𝐴𝑟𝑒𝑎 𝑜𝑓 𝑡𝑟𝑎𝑖𝑛𝑔𝑙𝑒 𝐴𝐵𝑂
Area of triangle ABO
1 1
= × 𝑂𝐵 × 𝐴𝐵 = × 8 × 6 = 24𝑐𝑚2
2 2
Area of quadrilateral ABOC
= 2 × 24 = 48𝑐𝑚2

Quantity II: Area of trapezium


1
= (sum of parallel sides) × perpendicular distance
2
1
= (6 + 8) × 6 = 42𝑐𝑚2
2
Quantity I > Quantity II

43. (b); Quantity I


20,000×22×2
Interest earned by Kishan after 2 years = = 8800
100
Amount invested by KIshan in another bank = 20,000 + 8800 =
28,800
ATQ,

Adda247 Publications For any detail, mail us at


45
Publications@adda247.com
Cracker Book for Bank (IBPS | SBI | RRB PO | Clerk) Mains Exams

𝑥 3
⇒ 28,800 [1 + ] = 28,800 + 28,800
100
−1350
𝑥 3
⇒ 28,800 [1 + ] = 56,250
100
𝑥 3 56,250
⇒ [1 + ] =
100 28,800
𝑥 3 125
⇒ [1 + ] =[ ]
100 64
𝑥 5
⇒ [1 + ] =[ ]
100 4
⇒ 𝑥 = 25%
5
Quantity II: Marked price is 185 % 𝑜𝑓 𝑡ℎ𝑒 𝑐𝑜𝑠𝑡 𝑝𝑟𝑖𝑐𝑒 𝑜𝑓 𝑎𝑟𝑡𝑖𝑐𝑙𝑒
7
Let Cost price = 100x and profit % is y% after y% discount
1300𝑥
⇒ 100𝑥 + 𝑥𝑦 = (100 − 𝑦)
700
1300𝑥
⇒ 100𝑥 + 𝑥𝑦 = (100 − 𝑦)
700
⇒ 700 + 7𝑦 = 1300 − 13𝑦
⇒ 20𝑦 = 600 ⇒ 𝑦 = 30%
Quantity II > Quantity I

44. (b); A B C
Time 𝑥 + 5 𝑥 𝑥 − 4
1 1 1
∴ + =
𝑥+5 𝑥 𝑥−4
𝑥 = 10
∴ Quantity I → 10 days
Given 2M = 3W
(8M+14W)×x×7 (6M+10W)×15×6
∴ 7 = 5
×360 ×360
12 12
171 2
𝑥= ⇒ = 13
13 13

2
Quantity II → 13
13
∴ Quantity I < Quantity II

Adda247 Publications For any detail, mail us at


46
Publications@adda247.com
Cracker Book for Bank (IBPS | SBI | RRB PO | Clerk) Mains Exams

45. (b);

Let P is faster than Q


Then P covers 72 km distance in the same time as Q covers 48 km
distance
Ratio of the speed = 72 : 48 = 3 : 2
48
∴ Speed of P = × 3 = 72 km/hr
2
Quantity I→ Difference between P and Q = 72 – 48 = 24 km/hr.
Let speed of train = T km/hr
Let speed of car = C km/hr
120 480
∴ + =8 ………..(i)
𝑇 𝐶
200 400 1
+ =8 ………(ii)
𝑇 𝐶 3
On solving (i) and (ii)
T = 60 km/hr
∴ Quantity I < Quantity II

46. (a); 𝑥: Let efficiency of A = 𝑎


and efficiency of B and C be 3𝑦 and 4𝑦
Now
1
(𝑎 + 3𝑦) × 16 = 15 (𝑎 + 4𝑦)
3
𝑎 = 20𝑦
(20𝑦+3𝑦)×16
Time taken by A = = 18.4 days
20𝑦
𝑦: Let efficiency of P, Q and R be 2𝑎, 3𝑎, 4𝑎 respectively
Total work = (2𝑎 + 3𝑎) × 18
5𝑎×18
Required time = = 10 days
9𝑎
Quantity I > Quantity II
Adda247 Publications For any detail, mail us at
47
Publications@adda247.com
Cracker Book for Bank (IBPS | SBI | RRB PO | Clerk) Mains Exams

47. (a); 𝑥:
Cost price = Rs. 𝑥
S.P. = 𝑥 + 100
S.P. after increasing it by 20%
120
2𝑥 = (𝑥 + 100)
100
𝑥 = 𝑅𝑠. 150
𝑦:
Let C.P. = Rs.100
So M.P. = Rs.500
500×40
S.P. = = 200
100
200–100
Profit % = × 100 = 100%
100
Quantity I > Quantity II

48. (e); 𝑥 :
𝑎 = 11
𝑑=1
𝑛
𝑆𝑛 = [2𝑎 + (𝑛– 1)𝑑]
2
𝑥
4040 = [2 × 11 + (𝑥– 1)1]
2
𝑥 = 80
𝑦:
𝑎 = 10
𝑑=2
So,
𝑦
7120 = [2 × 10 + (𝑦– 1)2]
2
𝑦 = 80
Quantity I = Quantity II

49. (b); 𝑥 :
Amount Invested = 𝑥
Time in first scheme = 2 years
Rate = 40%
𝑥×2×40 4𝑥
S.I. = =
100 5
Time in 2nd scheme = 5 years
Adda247 Publications For any detail, mail us at
48
Publications@adda247.com
Cracker Book for Bank (IBPS | SBI | RRB PO | Clerk) Mains Exams

Rate = 44%
𝑥×5×44 11𝑥
S.I. = =
100 5
11𝑥 4𝑥
Difference = – = 18,900
5 5
𝑥 = Rs. 13,500
𝑦:
𝑦 = Interest earned
12000
P= × 100 = 40,000
30
40000×3×12
Interest = = 14,400
100
Quantity II > Quantity I

50. (a); 𝑥 :
Edge of cube = 7 cm
7
So, largest sphere radius = cm
2
Volume of vacant space
4 22 7 7 7 490
=7×7×7− × × × × = cm³
3 7 2 2 2 3
𝑦 : Surface area of sphere = 4πr²
22 7 7
= 4 × × × = 154 𝑐𝑚2
7 2 2
Quantity I > Quantity II

PART B
1. (b); I. 3𝑥+5 . 92𝑥−4 = 95𝑥−14
⇒ 3𝑥+5 . 34𝑥−8 = 310𝑥−28
⇒ 3𝑥+5+4𝑥−8 = 310𝑥−28
⇒ 35𝑥−3 = 310𝑥−28
⇒ 5𝑥 − 3 = 10𝑥 − 28
⇒ 5𝑥 = 25
⇒ 𝑥=5
II. 2𝑦 2 − 15𝑦 − 28 = 3𝑦 2 − 23𝑦 − 13
⇒ 𝑦 2 − 8𝑦 + 15 = 0
⇒ 𝑦 2 − 3𝑦 − 5𝑦 + 15 = 0
⇒ 𝑦(𝑦 − 3) − 5(𝑦 − 3) = 0
⇒ (𝑦 − 5)(𝑦 − 3) = 0
⇒ 𝑦 = 5 ,3 ⇒ x ≥ y

Adda247 Publications For any detail, mail us at


49
Publications@adda247.com
Cracker Book for Bank (IBPS | SBI | RRB PO | Clerk) Mains Exams
25 15
2. (d); I. − +2=0
x² x
⇒2x 2 − 15x + 25 = 0
⇒2x 2 − 10x − 5x + 25 = 0
2x (x − 5) − 5(x − 5) = 0
(2x − 5)(x − 5) = 0
5
x = ,5
2
40 13
II. +1=
y² y
2
⇒ y − 13y + 40 = 0
⇒ y 2 − 8y − 5y + 40 = 0
⇒ y(y − 8) − 5(y − 8) = 0
(y − 5)(y − 8) = 0
y = 5, 8
y ≥ x

3. (e); 𝟐𝐲+𝟒 . 32y+4 . 42y+4 = 32y+4 . 22y+16


𝒚 + 4 + 4𝑦 + 8 = 4𝑦 + 16
𝐲=4
𝒙𝟐 – 10 + 24 = 0
𝒙𝟐 – 6𝑥– 4𝑥 + 24 = 0
𝒙(𝑥– 6)– 4(𝑥– 6) = 0
(𝑥– 4)(𝑥– 6) = 0
𝒙 = 4, 6
𝒙≥𝑦

48 14
4. (d); I. − +1=0
x² x
2
⇒ x − 14x + 48 = 0
⇒ x 2 − 8x − 6x + 48 = 0
⇒ x(x − 8) − 6(x − 8) = 0
⇒ (x − 8)(x − 6) = 0
x = 8, 6
45 1
II. + =2
y² y
2
⇒ 2y − y − 45 = 0
⇒ 2y 2 − 10y + 9y − 45 = 0

Adda247 Publications For any detail, mail us at


50
Publications@adda247.com
Cracker Book for Bank (IBPS | SBI | RRB PO | Clerk) Mains Exams

⇒ 2y(y − 5) + 9(y − 5) = 0
⇒ (2y + 9)(y − 5) = 0
9
𝐲 = 5, −
2
𝐱 > y

5. (d); I. (𝑥 − 4)2 = 9
⇒ 𝑥 − 4 = ±3
⇒ 𝑥 = 7, 1
II. (2𝑦 + 3)2 = 25
⇒ 2𝑦 + 3 = ±5
⇒ 𝑦 = 1, −4
⇒x≥y

6. (c); I. 5x² + 3x – 36 = 0
5x² + 15x – 12x – 36 = 0
5x (x + 3) – 12 (x + 3) = 0
(5x – 12) (x +3) = 0
x = 12/5, –3
II. 2y² – 13y + 20 = 0
2 y ² – 8 y – 5 y + 20 = 0
2 y (y – 4) – 5(y – 4) = 0
(2 y – 5) (y – 4) = 0
y = 5/2, 4
y>x

7. (b); I. x² – 7x + 12 = 0
x² – 4x – 3x + 12 = 0
x(x – 4) –3 (x – 4) = 0
(x – 3) (x – 4) = 0
x = 3, 4
II. 2y² – 11y + 15 = 0
2y² – 6y – 5y + 15 = 0
2y(x – 3) – 3 (y – 3) = 0
(2y – 5) (y – 3) = 0
y = 5/2, 3
x≥y
Adda247 Publications For any detail, mail us at
51
Publications@adda247.com
Cracker Book for Bank (IBPS | SBI | RRB PO | Clerk) Mains Exams

8. (d); I. 2x² + 11x + 15 = 0


2x² + 6x + 5x + 15 = 0
2x (x + 3) +5 (x + 3) = 0
(2x + 5) (x + 3) = 0
x = –5/2, –3
II. 2y² + 9y + 10 = 0
2y² + 4y + 5y + 10 = 0
2y (y + 2) +5 (y + 2) =0
(2y + 5) (y + 2) = 0
Y = –5/2, –2
y≥x

9. (c); I. 3x² + 7x – 40 = 0
3x² + 15x – 8x – 40 = 0
3x (x + 5) – 8x – 40 = 0
(3x – 8) (x + 5) = 0
x = 8/3, –5
II. 5y² – 29y + 42 = 0
5y – 14y – 15 y + 42 =0
y(5y – 14) – 3 (5y – 14) = 0
(y – 3) (5y – 14) = 0
y = 3, 14/5
y>x
10. (e); I. 3x² – 23x + 42 = 0
3x² – 9x – 14x + 42 = 0
3x( x – 3) – 14(x – 3) =0
(3x – 14) (x – 3) = 0
x =3, 14/3
II. 3x² – 19y + 45 = 0
2y² – 10y – 9y + 45 = 0
2y (y – 5) – 9 (y – 5) =0
(2y – 9) (y – 5) = 0
y=9/2, 5
No relation can be established between 𝑥 𝑎𝑛𝑑 𝑦

Adda247 Publications For any detail, mail us at


52
Publications@adda247.com
Cracker Book for Bank (IBPS | SBI | RRB PO | Clerk) Mains Exams

11. (a); I. 𝑥 2 − 5𝑥 + 4 = 0
𝑥 2 − 4𝑥 − 𝑥 + 4 = 0
𝑥(𝑥 − 4) − 1(𝑥 − 4) = 0
(𝑥 − 1)(𝑥 − 4) = 0
𝑥 = 1, 4

II. 𝑦 2 + 5𝑦 + 6 = 0
𝑦 2 + 3𝑦 + 2𝑦 + 6 = 0
𝑦(𝑦 + 3) + 2(𝑦 + 3) = 0
(𝑦 + 2)(𝑦 + 3) = 0
𝑦 = −2, −3
𝑥>𝑦

12. (d); I. 2𝑥 2 − 𝑥 − 15 = 0
2𝑥 2 − 6𝑥 + 5𝑥 − 15 = 0
2𝑥 (𝑥 − 3) + 5(𝑥 − 3) = 0
(2𝑥 + 5)(𝑥 − 3) = 0
−5
𝑥 = ,3
2

II. 3𝑦 2 − 23𝑦 + 42 = 0
3𝑦 2 − 14𝑦 − 9𝑦 + 42 = 0
𝑦(3𝑦 − 14) − 3(3𝑦 − 14) = 0
(𝑦 − 3)(3𝑦 − 14) = 0
14
𝑦 = 3,
3
𝑦≥𝑥

13. (a); I. 𝑥 2 − 15 + 54 = 0
𝑥 2 − 9𝑥 − 6𝑥 + 54 = 0
𝑥(𝑥 − 9) − 6(𝑥 − 9) = 0
(𝑥 − 6)(𝑥 − 9) = 0
𝑥 = 6, 9
II. 𝑦 2 + 15𝑦 − 54 = 0
𝑦 2 + 18𝑦 − 3𝑦 − 54 = 0
𝑦(𝑦 + 18) − 3(𝑦 + 18) = 0
(𝑦 + 18)(𝑦 − 3) = 0
𝑦 = −18, 3
𝑥>𝑦
Adda247 Publications For any detail, mail us at
53
Publications@adda247.com
Cracker Book for Bank (IBPS | SBI | RRB PO | Clerk) Mains Exams

14. (c); I. 𝑥 2 + 14𝑥 + 40 = 0


𝑥 2 + 10𝑥 + 4𝑥 + 40 = 0
𝑥(𝑥 + 10) + 4(𝑥 + 10) = 0
(𝑥 + 4)(𝑥 + 10) = 0
𝑥 = −4, −10
II. 𝑦 2 − 5𝑦 − 24 = 0
𝑦 2 − 8𝑦 + 3𝑦 − 24 = 0
𝑦(𝑦 − 8) + 3(𝑦 − 8) = 0
(𝑦 + 3)(𝑦 − 8) = 0
𝑦 = −3, 8
𝑦>𝑥

15. (e); I. 𝑥 2 − 225 = 0


𝑥 2 = 225
𝑥 = ±15

II. 𝑥 2 + 𝑦 2 = 306
225 + 𝑦 2 = 306
𝑦 2 = 306 − 225
𝑦 2 = 81
𝑦 = ±9
No relation can be established between x and y.

16. (c); I. 2x² – 7x + 6 = 0


2x² – 4x – 3x + 6 = 0
2x (x –2) –3 (x – 2) = 0
(2x – 3) (x – 2) =0
3
𝑥 = 𝑜𝑟 2
2
II. 3y² – 19y + 28 = 0
3y² – 12y – 7y + 28 = 0
3y (y – 4) –7 (y – 4) = 0
(3y – 7) (y – 4) = 0
7
y= ,4
3
y>x

Adda247 Publications For any detail, mail us at


54
Publications@adda247.com
Cracker Book for Bank (IBPS | SBI | RRB PO | Clerk) Mains Exams

17. (a); I. x² – 13x + 36 =0


x² – 9x – 4x + 36 =0
x (x – 9) – 4 (x – 9) =0
(x – 4) (x – 9) = 0
x = 4, 9

II. 3y² – 19y + 30 = 0


3y² – 10y – 9y + 30 = 0
y(3y – 10) – 3 (3y – 10)=0
(y – 3) (3y – 10) = 0
10
y = 3,
3
x>y
18. (a); I. x³ = 120 + 96 = 216
x=6
II. y² – 25 = 0
y² = 25
y = ±5
x>y

19. (e); I. x² – 2x – 48 = 0
x² – 8x + 6x – 48 = 0
x(x – 8) +6 (x – 8) =0
(x + 6) (x – 8) = 0
x = 8, –6

II. y² – 15y + 54 = 0
y² – 9y – 6y + 54 = 0
y(y – 9) –6 (y – 9) = 0
(y – 6) (y – 9) = 0
y = 6, 9
No relation can be established between x and y

20. (c); I. 2x² + 25x + 72 = 0


2x² + 16x + 9x + 72 = 0
2x (x + 8) +9 (x + 8) = 0
Adda247 Publications For any detail, mail us at
55
Publications@adda247.com
Cracker Book for Bank (IBPS | SBI | RRB PO | Clerk) Mains Exams

(2x + 9) (x + 8) = 0
9
x=− ,–8
2
II. 3y² + 22y + 40 = 0
3y² + 10 y + 12y + 40 = 0
y (3y + 10) + 4 (3y + 10) = 0
(y + 4) (3y + 10) = 0
10
y = – 4, –
3
y>x

Adda247 Publications For any detail, mail us at


56
Publications@adda247.com
Cracker Book for Bank (IBPS | SBI | RRB PO | Clerk) Mains Exams

Chapter

13
What is data interpretation: When data is organized into tables and charts
it is done with the purpose of making it meaningful. The objective of data
interpretation is to assess whether a student can understand bars and
charts and Answer some questions based on them. This act of organizing
and interpreting data to get meaningful information under a given set of
conditions is Data interpretataion.

About data interpretation: This is the calculation intensive portion, it


consists of a myrid of graph. charts and tables and analyze data. The key
to crack this area is to quickly Identify the key pieces of information that
you will require to work on.
Basic key that will help you to solve this topic:
• Calculation
• Square • cube
• table • BODMAS
• Percentage
• Profit and loss
• Ratio and proportion
• Average
Types of Data Interpretation:
• Data table
• Line graph
• Pie charts
• Bar graph
• Mixed graph
• Line with pie chart • Table with Bar
• Table with Line, etc. • Radar graph
• Triangular graph
• Case study (Puzzle)
• Venn Diagram • Table format
2 Adda 247 Publications For any detail, mail us at
Publications@adda247.com
Cracker Book for Bank (IBPS | SBI | RRB PO | Clerk) Mains Exams

Approach for data interpretation:


• First you look carefully at the table or graph and the direction. Note
the years to which, the data refers to and the units. Sometimes the
figures may be given in thousands. While the Answer may be millions
Resulting in mistakes.
• The level of approximation that can be done is assessed from the
choices. If the answer is wide, time should not be wasted in working
out exact figures. If the choice ‘none of the above exists, a close
approximation may be required’.
• Read the question carefully, it will give an indication as to which
row and column should be seen. A carefull reading of the question
will reveal exactly what is to be done and the units in which the
answer is required.
• There may be one or two very large question requiring calculations.
Attempt these at the last.
• Revise bar charts, table and line graphs before attempting D.I.
question remember that the D.I. section is a scoring one and also
time saving.
Data Table: The numbers in the bracket give the maximum marks in each
subject.
The following table gives the percentage of marks obtained by seven students in six
different subjects in an examination. Study it and answer the question given below
it.
Students Subject (Maximum Marks)
Maths Chemistry Physics Geography History Computer
(150) (130) (120) (100) (60) Science (40)
Golu 90 50 90 60 70 80

Mithi 100 80 80 40 80 70

Suraj 90 60 70 70 90 70

Gapplu 80 65 80 80 60 60

Mahi 80 65 85 95 50 90

Khushi 70 75 65 85 40 60

Sheetal 65 35 50 77 80 80

3 Adda 247 Publications For any detail, mail us at


Publications@adda247.com
Cracker Book for Bank (IBPS | SBI | RRB PO | Clerk) Mains Exams

Line Graph:
Answer the question based on the given line graph
Following line graph shows the ratio of export to import of company A
and company B over the year

2
Company A
1.75

1.5 Company B
1.25

.75

.5

.25

0
Years 2005 2006 2007 2008 2009 2010

1. In how many of the given years were the exports more than the imports
for company A?
(a) 2 (b) 3 (c) 4 (d) 5

2. If the imports of company A in 2007 were increased by 40%. What


would be the ratio of exports to the increased imports?
(a) 1.50 (b) 1.25 (c) 1.75 (d) 1.35

3. If the exports of company B in 2008 was Rs 237 Crore, what was the
amount of imports in that years ?
(a) 189.6 (b) 188.8 (c) 184.6 (d) 182.8
4 Adda 247 Publications For any detail, mail us at
Publications@adda247.com
Cracker Book for Bank (IBPS | SBI | RRB PO | Clerk) Mains Exams

4. In 2005, the export of company A was double that of company B. If the


imports of company A during the year was 180 crore. What was the
approximate amount of imports of company B during that year?
(a) 190 crore (b) 210 crore (c) 225 crore (d) 200 crore

5. In which year were the exports of company A minimum proportionate


to its imports.
(a) 2008 and 2009 (b) 2009 (c) 2008 (d) 2005

Solution 1: required ratio, more than 1 for the years; 2005, 2006 and 2007,
ie; for 3 years

Solution 2: In 2007, for company A the ratio of exports to import = 175 :


100
Let exports of company A = 175 x Þ and imports of company
A = 100x
New imports of company = 140% of 100x = 140x
175x
There for requied ratio = = 1.25
140x

Solution 3: Let import of company B in 2008 = x


237 237
Then, = 1.25 Þ x = Þ 189.6 crore
x 1.25

Solution 4: In 2005 for company A Þ Export = 1.75 × 180 crore


1.75 ´ 180
in 2005 for company B export = 2

Ex
= 157.5 crore, = .75
But
Im
157.5
Import of company B = = 210 crore
.75

Solution 5: 2008 and 2009, A option.


Pie Charts or Circle Graphs:
Distribution of candidate who were enrolled in MBA and the candidate
(out of those enrolled) who passed the exam in different institutes

5 Adda 247 Publications For any detail, mail us at


Publications@adda247.com
Cracker Book for Bank (IBPS | SBI | RRB PO | Clerk) Mains Exams

X X
P P
16% 12%
V 22% V 18%
12% 15%
T Q T Q
8% 15% 9% 17%
S R S R
17% 10% 16% 13%

Total number of candidates Total number of candidates


enrolled = 8550 who passed the exam = 5700

1. What percentage of candidates passed the exam from institute T out of


the total no. of candidate enrollled from the same institute?

(a) 50% (b) 52.5% (c) 75% (d) 80%


2. Which institute has the highest % of candidates passed to the candidates
enrolled?
(a) Q (b) R (c) V (d) T

3. The no. of candidates passed from institute S and P together exceeds


the no. of candidates enrolled from institutes T and R together by?
(a) 228 (b) 279 (c) 399 (d) 407

4. What is % of candidates passed to the candidate enrolled for institutes


Q and R together ?
(a) 68% (b) 80% (c) 74% (d) 65%

5. What is the ratio of candidates passed to the candidates enrolled from


institute P ?
(a) 9 : 11 (b) 14 : 17 (c) 6 : 11 (d) 9 : 17
6 Adda 247 Publications For any detail, mail us at
Publications@adda247.com
Cracker Book for Bank (IBPS | SBI | RRB PO | Clerk) Mains Exams

9%of 5700
Solution 1: Required % = ´ 100 = 75%
8%of8550
Solution 2: The % of candidates passed to candidates enrolled can be
determined for each institute as under.
17%of 5700
For Q Þ 15%of 8550 ´ 100 = 75.56% ;

13%of 5700
For R Þ ´ 100 = 86.67%
10%of 8550

9%of 5700
For T Þ ´ 100 = 75% ;
8%of 8550

15%of 5700
For V Þ ´ 100 = 83.33%
12%of 8550
So the highest of these is 86.67% corresponding to institutes
R.
Solution 3: Required Difference = [(16% + 18%) of 5700] – [(8% + 10%) of
8550] = 1938 – 1539 = 399
30%of 5700
Solution 4: 25%of 8550 ´ 100 = 80%

18%of 5700 6
Solution 5: = = 6 : 11
22%of 8550 11

7 Adda 247 Publications For any detail, mail us at


Publications@adda247.com
Cracker Book for Bank (IBPS | SBI | RRB PO | Clerk) Mains Exams

Bar Graph: Percentage profit earned by two companies X & Y over the
given years.
Income Expenditure
Pr ofit% = ´ 100
Expenditure
Company X
70 65
Company Y 60
60
55 55
50 50 50 50
Profit%

50 45 45
40
40
35
30

20

2006 2007 2008 2009 2010 2011


1. The income of two company X and Y in 2010 were in the rato of 3 : 4.
respectively. What is the respective ratio of their expenditures in 2010?
(a) 7 : 22 (b) 14 : 19 (c) 15 : 22 (d)27 : 35

2. If the expenditure of company Y in 2007 was Rs 220 crore, what was its
income in 2007?
(a) Rs. 312 crore (b) Rs. 297 crore (c) Rs. 283 crore (d)Rs. 275 crore

3. If the expenditures of company X and Y in 2006 were equal and the


total income of the two companies in 2006 was Rs 342 crore, what was
the total profit of the two company together in 2006?
(a) Rs. 240 crore (b) Rs. 171 crore (c) Rs. 120 crore (d)Rs. 102 crore

4. The expanditure of company X in the year 2008 was Rs 200 crore and
the income of company X in 2008 was the same as its expenditure in
2011. The income of company X in 2011 was?
(a) Rs. 465 crore (b) Rs. 385 crore (c) Rs. 335 crore (d)Rs. 295 crore

5. If the income of two company were equal in 2009, then what was the
ratio of expenditure of company X to that of company Y in 2009?
(a) 6 : 5 (b) 5 : 6 (c) 11 : 6 (d) 16 : 15
8 Adda 247 Publications For any detail, mail us at
Publications@adda247.com
Cracker Book for Bank (IBPS | SBI | RRB PO | Clerk) Mains Exams

165%of E1 3 E 1 15
Solution 1: = Þ Therefore, = = 15 : 22
150%of E 2 4 E 2 22

Solution 2: 220 + 35% of 220 Þ 220 + 77 = 297 crore

Solution 3: Let the expenditure of each companies x and y in 2006 be 100


Then we have 100 + 40 + 100 + 45 = 342 crore
342
Þ Required profit = 285
×85 = 102 crore

Solution 4: Expenditure of Company X in year 2008 is 200 crore, Then


income of company X in year 2008
= 200 + 55% of 200 = 310 crore
Hence, the expenditure of company X in year 2011 is 310 crore,
Therefore, the required income of X in year 2011 = 310 + 50%
of 310 = 465 crore
150 160
Solution 5: X= Y
100 100
x 160 16
be the incomes of two companies in 2009; y = 150 = 15 = 16
: 15

9 Adda 247 Publications For any detail, mail us at


Publications@adda247.com
Cracker Book for Bank (IBPS | SBI | RRB PO | Clerk) Mains Exams

Practice Exercise Based on new Pattern

Direction (1 – 5): Table given below shows total number of students in two
sections of five different schools, percentage distribution of students in
these different sections (X & Y) and also ratio between boys to girls in each
section. Read the table carefully and answer the questions:

Percentage Ratio between Ratio between


Total
Schools of students boys: girls boys : girls in
students
in Section ‘X’ in section ‘X’ Section ‘Y’
LPS 450 48% 2:1 4:5
CMS 360 55% 7:2 1:2
DPS 420 60% 5:4 11 : 10
SKD 300 55% 1:2 2:1
Loyola 250 50% 3:2 2:3

1. Total number of boys in section ‘X’ from CMS & Loyola together are
what percent of total girls in section ‘Y’ from DPS & SKD together?
(a) 183.2 % (b) 180.2 % (c) 188.2 %
(d) 189.2 % (e) 185.2%

2. Find the ratio between total number of girls in section ‘Y’ from CMS &
SKD together to total number of boys in section ‘X’ from DPS & SKD
together?
(a) 65 : 51 (b) 51 : 65 (c) 51 : 67
(d) 51 : 62 (e) 51 : 49

3. Out of total number of boys in section ‘X’ from D.P.S school ratio
between number of boys who got first, second and third division is 1 : 2
: 4. Find total number of boys who got first and third division together
in section ‘X’ from D.P.S school?
(a) 40 (b) 60 (c) 80
(d) 100 (e) 120
Adda247 Publications For any detail, mail us at
10
Publications@adda247.com
Cracker Book for Bank (IBPS | SBI | RRB PO | Clerk) Mains Exams

4. Find the difference between average number of girls in section ‘X’ from
SKD & Loyola together and average number of boys in section ‘Y’ from
LPS & CMS together?
(a) 5 (b) 4 (c) 3
(d) 2 (e) 1

5. Total girls in section ‘Y’ from CMS & LPS together are how much more
than total boys in section ‘X’ from both DPS & SKD together?
(a)33 (b) 43 (c)53
(d) 63 (e) 45

Direction (6 – 10): Table given below shows total number of applicants,


who have applied for CDS exam in the five years. Percentage of applicants
appeared in exam, percentage of applicants qualified in tier I and tier II.
Line graph shows percentage of applicants got selection in CDS exam. Read
the data carefully and answer the questions:

Appeared % of applicants % of students


Total
Years applicants qualified qualified in
Applicants
in tier I in tier I tier II
2013 2400 85% 25% 40%
2014 3200 75% 30% 25%
2015 4800 90% 20% 25%
2016 7500 80% 25% 20%
1
2017 8400 60% 12 % 40%
2

Note – All applicants qualified in tier I appeared for tier II.


50
(Percentage)

40

30

20
2013 2014 2015 2016 2017
Adda247 Publications For any detail, mail us at
11
Publications@adda247.com
Cracker Book for Bank (IBPS | SBI | RRB PO | Clerk) Mains Exams

6. Number of applicants, who have got the final selection in the year
2017, is how many percent less than number of applicants, who have
got the final selection in the year 2014?
2 2 2
(a) 22 % (b) 24 % (c) 28 %
9 9 9
2 2
(d) 18 % (e) 21 %
9 9

7. Find the difference between number of applicants, who have got final
selection in the years 2013 & 2016 together and number of applicants,
who have got final selection in the year 2014 & 2015 together?
(a) 5 (b) 8 (c) 3
(d) 6 (e) 11

8. Find the ratio between applicants qualified in tier II but did not get final
selection in the year 2017 to applicants qualified in tier I but did not
qualify for tier II in the year 2014?
(a) 7 : 20 (b) 7 : 22 (c) 7 : 19
(d) 7 : 18 (e)7 : 16

9. Total applicants appeared for tier I in the year 2017 is what percent
more than total applicants appeared for tier II in the year 2015?
1 1 1
(a) 498 % (b) 438 % (c) 485 %
3 3 3
1 1
(d) 483 % (e) 487 %
3 3

10. Find the total number of applicants, who did not qualify in tier I in the
year 2014, 2016 & 2017 together?
(a) 10390 (b) 10590 (c) 10390
(d) 10290 (e) 10190

Direction (11 – 15): Given below pie graph shows total number of tickets
of ‘Sanju’ movie sold by five multiplex on opening days, while bar graph
shows percentage distribution of three different type of class ticket sold by
each store. Read given data and answer the questions:
Adda247 Publications For any detail, mail us at
12
Publications@adda247.com
Cracker Book for Bank (IBPS | SBI | RRB PO | Clerk) Mains Exams

Total sold tickets - 12500

A
12%
E
B
36%
18%

D C
10% 24%

Jubali Silver Gold


100
90
80
70
60
Percentage

50
40
30
20
10
0
A B C D E
Multiplex

11. Total Jubali class ticket sold by multiplex C & E together is what percent
more than total Gold class ticket sold by multiplex A & C together?
1 1 1
(a) 105 % (b) 102 % (c) 100 %
7 7 7
1 1
(d) 97 % (e) 107 %
7 7

Adda247 Publications For any detail, mail us at


13
Publications@adda247.com
Cracker Book for Bank (IBPS | SBI | RRB PO | Clerk) Mains Exams

12. Find difference between total Silver class ticket sold by multiplex A, C &
E together and total silver gold class ticket sold by multiplex B, C & E
together?
(a) 400 (b) 200 (c) 650
(d) 600 (e) 450

13. Find sum of average number of Jubli class ticket sold by multiplex B & C
and average number of Gold class ticket sold by multiplex C & E?
(a) 2075 (b) 2175 (c) 2275
(d) 2375 (e) 2205

14. Total Silver class tickets sold by multiplex A & D is what percent less
than total Gold class ticket sold by B & E?
4 4 4
(a) 42 % (b) 40 % (c) 38 %
9 9 9
4 4
(d) 44 % (e) 36 %
9 9

15. The ratio of price of one Gold ticket in multiplex D, C & A is 10 : 6 : 7,


and all three multiplex earned Rs. 40000 from the selling of all Gold
Class tickets. Find difference between price of one Gold ticket in
Multiplex A and D?
(a) 10 Rs. (b) 15 Rs. (c) 5 Rs.
(d) 20 Rs. (e) 25 Rs.

Direction (16 – 20): Given below table shows total number of five different
brands of mobiles sold by a store on Sunday. Also given percentage of 2G
mobiles and ratio between 3G and 4G mobile phone sold out of total
number of sold mobiles. Read the table carefully and answer the following
questions:
Number of total Percentage of 2G Ratio of 3G : 4G
Brand
sold mobiles mobiles mobiles
Samsung 750 20% 2:3
Lava 840 25% 3:4
MI 1050 30% 1:4
Huwai 960 15% 1:2
Oppo 640 35% 1:3
Adda247 Publications For any detail, mail us at
14
Publications@adda247.com
Cracker Book for Bank (IBPS | SBI | RRB PO | Clerk) Mains Exams
2
16. Total number of Samsung mobiles sold by store on Saturday is 𝑟𝑑 of
3
the number of same brand of mobile sold on Sunday. If total number of
3G Samsung mobile sold on Saturday is 40 less than that of total
number of 3G Samsung mobile sold on Sunday, then find total number
of 4G Samsung mobile sold on Sunday is what percent more/less than
total number of 4G Samsung mobile sold on Saturday (on Saturday
store sold only 3G and 4G mobile)?
(a) 25% (b) 20% (c) 10%
(d) 15% (e) 5%

17. Total number of MI mobile sold on Monday is equal to average number


of Huwai & Oppo mobile sold on Sunday. If ratio between number of MI
2G, 3G and 4G mobiles sold on Monday is 4 : 5 : 7, then find the ratio
between total number of MI 3G mobile sold on Monday to total number
of Lava 3G mobile sold on Sunday?
(a) 25 : 29 (b) 25 : 27 (c) 25 : 36
(d) 25 : 31 (e) 25 : 33

18. If ratio of total number of Oppo 4G mobiles sold by store on Sunday,


Monday and Tuesday is 6 : 4 : 7, then find total number of Oppo 4G
mobile sold by store on Monday & Tuesday together is what percent
less/more than the total number of Samsung & Lava 4G mobile sold by
store on Sunday together?
5 5 5
(a) 18 % (b) 16 % (c) 24 %
9 9 9
5 5
(d) 20 % (e) 12 %
9 9

19. Find the difference between average numbers of Samsung & Lava 2G
mobile and average numbers of Huwai & Oppo 3G mobile sold by store
on Sunday?
(a) 12 (b) 16 (c) 10
(d) 8 (e) 4

20. If total number of Vivo mobile sold by store on Sunday is 56 more than
total number of 4G Lava & Huwai mobile sold by store on Sunday, then
find total number of Oppo mobile phone sold by store on Sunday is
Adda247 Publications For any detail, mail us at
15
Publications@adda247.com
Cracker Book for Bank (IBPS | SBI | RRB PO | Clerk) Mains Exams

what percent more/less than total number of Vivo mobile sold by store
on Sunday ?
1 1 1
(a) 35 % (b) 38 % (c) 33 %
3 3 3
1 1
(d) 39 % (e) 41 %
3 3

Direction (21- 25): Table given below shows total number of tickets which
were estimated to be sold in five stadiums. Out of total tickets sold,
percentage of A1 tickets sold, and ratio between A2 and A3 is given. Study the
data carefully and answer the following questions.

Stadiums Tickets which can be sold A1 tickets sold (in %) A2 : A3


Nehru 1800 15% 9:8
Patel 2600 25% 4:5
Eden 2200 20% 5:3
Kaloor 2000 20% 7:5
Raipur 2700 30% 3:4

Note: - 200 tickets remain unsold in each stadium


Tickets which can be sold = A1 tickets + A2 tickets + A3 tickets + Unsold
tickets
21. Total A1 and A3 tickets sold in Patel stadium is what percent more than
total number of A1 and A2 tickets sold in Kaloor stadium?
2 2
(a) 16 % (b) 50% (c) 66 %
3 3
1
(d) 25% (e) 33 %
3

22. Total A2 tickets sold in Nehru stadium is how much less than A3 tickets
sold in Raipur stadium?
(a) 220 (b) 240 (c) 260
(d) 280 (e) 300

23. Find the ratio between total A3 tickets sold in Eden and Kaloor stadium
together to total A2 tickets sold in Patel and Eden stadium?
(a) 3 : 2 (b) 2 : 3 (c) 20 :23
(d) 23 : 20 (e) 8 : 9
Adda247 Publications For any detail, mail us at
16
Publications@adda247.com
Cracker Book for Bank (IBPS | SBI | RRB PO | Clerk) Mains Exams

24. Find the average number of total A1 tickets sold by all five stadiums
together?
(a) 514 (b) 490 (c) 470
(d) 450 (e) 430

25. A1 and A2 tickets sold in Nehru stadium is what percent more/less than
A1 and A2 tickets sold in Raipur stadium?
(a) 36% (b) 64% (c) 56.25%
(d) 43.75% (e) 28%

Direction (26-30): Given below bar graph shows percentage distribution


of weekly income of four different Bollywood movies. Table shows total
income of these four movies and percentage of income earning by these
movies from foreign. Read the data carefully and answer the questions.

First week Second week Third week


100
80
60
40
20
0
A B C D
Movies Total income (in crores) % of income from foreign
A 210 26 ⅔%
B 180 25%
C 200 20%
D 240 30%

26. What is the ratio between income of movie B in second week to income
of movie C in third week?
(a) 13 : 5 (b) 13 : 3 (c) 27 : 5
(d) 27 : 10 (e) 27 : 13
Adda247 Publications For any detail, mail us at
17
Publications@adda247.com
Cracker Book for Bank (IBPS | SBI | RRB PO | Clerk) Mains Exams

27. Out of these four movies, which movie beard highest percentage drop
in its income in second week with respect to first week?
(a) A (b) B (c) C
(d) D (e) None of these

28. What is the ratio of income of movie A from foreign to the income of
movie C from foreign?
(a) 8 : 5 (b) 8 : 7 (c) 7 : 5
(d) 28 : 27 (e) 7 : 8

29. If ratio of foreign income of B of given three weeks is 1 : 2 : 2


respectively. Then find the income earn by movie B in second week in
India?
(a) 36 crores (b) 18 crores (c) 26 crores
(d) 30 crores (e) 32 crores

30. Income of movie D in second week and income of movie C in third week
together is approximately what percent more or less then total income
of movies A and B of second week?
(a) 55 (b) 78 (c) 62
(d) 44 (e) 38

Directions (31-35): Study the following graph carefully and answer the
question. The following bar graph shows the budget allocation (in crore) for
education, sport and Health care from year 2018-22.

Adda247 Publications For any detail, mail us at


18
Publications@adda247.com
Cracker Book for Bank (IBPS | SBI | RRB PO | Clerk) Mains Exams

Education Sport Health care


250
220 220
200
200 190 190
170 170
160 160
150
Budget (in crore)

150 125 130


125 130
105
100

50

0
2018 2020 2019
2021 2022
Year
31. What percent is the average budget of sport during 2018 to 2022 than
that of average budget of Education throughout all the years?
(approximately)
(a) 84% (b) 88% (c) 87%
(d) 90% (e) 94%

32. In 2022, budget allocated for sport for male to female are in the ratio of
3 : 2 and it proposed that budget allocated for males to be increased by
25% in 2023 compared to previous year for sport then, what is the
percentage increase in sport budget allocation for 2023 with no change
in sport budget allocated for female in 2023 as compared to previous
year?
(a) 20% (b) 18% (c) 15%
(d) 12% (e) 10%

33. Find the ratio between total budget allocated for Education from year
2018 to 2020 together to total budget allocated for Health care from
year 2020 to 2022 together ?
(a) 17 : 29 (b) 18 : 29 (c) 21 : 29
(d) 27 : 29 (e) 17 : 27
Adda247 Publications For any detail, mail us at
19
Publications@adda247.com
Cracker Book for Bank (IBPS | SBI | RRB PO | Clerk) Mains Exams

34. If in 2019 budget allocated for only three sports i.e. Cricket, Badminton
and Hockey in the ratio of 5 : 4 : 4 respectively and budget allocated for
Education in 2022 for Urban and rural area in the ratio of 5 : 6
respectively. Then find budget allocated for Education for rural area in
2022 how much more than budget allocated for Cricket & Badminton
together in 2019 ?
(a) 30 cr (b) 35 cr (c) 40 cr
(d) 45 cr (e) 25 cr

35. Find the sum of average of budget allocated for Health care throughout
all the year 2018-22 and average of budget allocated for Education
throughout all the year 2018-2022 ?
(a) 336 (b) 338 (c) 340
(d) 342 (e) 348

Direction (36 – 40): Given below table shows the percentage of number of
cars sold by four different motor companies, percentage of number of total
returned cars out of total number of sold cars in two successive years(2016,
2017) . Read the graph carefully and answer the following questions.

Note; (I) Total number of manufactured cars = number of Unsold cars +


number of Sold cars
(II) Total actual number of cars sold = Total number of sold cars
– number of returned cars

Percentage Percentage of Percentage of Percentage of


of number number of number of number of
of cars sold cars sold in cars returned cars returned
in 2016 2017 in 2016 in 2017
TATA 80% 75% 15% 10%
HYUNDAI 65% 70% 20% 15%
MARUTI-
80% 85% 12.5% 17.5%
SUZUKI
HONDA 75% 70% 15% 22.5%

Adda247 Publications For any detail, mail us at


20
Publications@adda247.com
Cracker Book for Bank (IBPS | SBI | RRB PO | Clerk) Mains Exams

36. The ratio between total number of cars manufactured by TATA and
HONDA in the year 2016 is 6 : 7 and the total actual number of cars sold
by HONDA is 4590 units more than that of TATA in the same year. If
total number of cars returned to HYUNDAI in the year 2017 is 1920
units less than total number of cars returned to TATA in 2016 and total
number of cars returned to MARUTI SUZUKI in the year 2017 is 4830
units more than total number of cars returned to HONDA in 2016, then
find the difference between total number of manufactured cars by
HYUNDAI and MARUTI SUZUKI in the year 2017?
(a) 36,000 (b) 42,000 (c) 44,000
(d) 40,000 (e) 32,000

37. Total number of cars manufactured by TATA in the year 2017 is 40%
more than that of total number of cars manufactured by MARUTI
SUZUKI in the year 2016 and total number of cars returned to TATA in
2017 is 120 units more than total number of cars returned to MARUTI
SUZUKI in the year 2016. If total number of cars returned to HONDA in
the year 2017 is 2010 units more than total number of cars returned to
MARUTI SUZUKI in the year 2016, then find total number of cars
manufactured by HONDA in the year 2017?
(a) 24,000 (b) 20,000 (c) 28,000
(d) 18,000 (e) 16,000

38. The ratio between total number of cars manufactured by HYUNDAI,


MARUTI SUZUKI & HONDA in the year 2016 is 2 : 3 : 4 and average
number of cars returned to these three companies in the same year is
4040 units. If total number of cars manufactured by HYUNDAI, MARUTI
SUZUKI & HONDA in the year 2017 is increased by 20%, 25% and
12.5% respectively over that of the previous year, then find the average
number of cars manufactured by HYUNDAI, MARUTI SUZUKI & HONDA
in the year 2017?
(a) 42,400 (b) 42,800 (c) 42,600
(d) 42,000 (e) 41,600
Adda247 Publications For any detail, mail us at
21
Publications@adda247.com
Cracker Book for Bank (IBPS | SBI | RRB PO | Clerk) Mains Exams

39. The ratio between total number of cars manufactured by HYUNDAI &
HONDA in the year 2016 is 8 : 9 and the ratio of number of cars
manufactured by HYUNDAI & HONDA in the year 2016 to 2017 is 2 : 3
and 3 : 5 respectively. If total actual number of cars sold by HYUNDAI &
HONDA together in the year 2016 is 39590 units then find total number
of cars manufactured by HONDA in the year 2017 is what percentage
more/less than total number of cars manufactured by HYUNDAI in the
same year?
(a) 20% (b) 15% (c) 10%
(d) 25% (e) 35%

40. The ratio between total number of cars manufactured by TATA,


HYUNDAI & HONDA in the year 2017 is 7 : 6 : 8 and total actual number
of cars sold by these companies in the year 2017 is 75810 units, then
find total number of cars manufactured by TATA, HYUNDAI & HONDA
together in the year 2017?
(a) 1,24,000 (b) 1,25,000 (c) 1,20,000
(d) 1,26,000 (e) 1,36,000

Direction (41 – 45): Pie-chart shows the distribution of total bikes


manufactured by five different companies. Also Table shows percentage
‘150 CC’ and ‘200 CC’ & total bike sold by these five companies. Study the
data carefully and solve the following questions

Hero
Yamaha 15%
22%

Bajaj
TVS
25%
18%
Honda
20%

Adda247 Publications For any detail, mail us at


22
Publications@adda247.com
Cracker Book for Bank (IBPS | SBI | RRB PO | Clerk) Mains Exams

Total sold
Companies 150CC Sold bikes 200CC Sold bikes
bikes
Hero 35% 65% —
Bajaj 40% 60% —
Honda — — 80%
TVS 75% — —
Yamaha 70% — 75%

Note All companies manufactured only two types of bike ‘150 CC’ & ‘200
CC’. Some values are missing you have to calculate it according to
questions.
Total bikes sold of any company= Total 150 CC bikes sold + Total 200
CC bikes sold

41. Total ‘200 CC’ Honda bikes sold by store is 25% more than total ‘150
CC’ bikes sold by same store. If difference between ‘200 CC’ & ‘150 CC’
bike sold by Honda is 640 then find total ‘200 CC’ bike sold by Yamaha?
(a) 1785 (b) 1782 (c) 1780
(d) 1787 (e) 1791

42. If Hero & Bajaj sold 80% & 90% of total manufactured bikes
respectively and difference between total ‘200 CC’ bikes sold by these
two companies is 4104, then find total ‘150 CC’ bike sold by Bajaj is
what percent more than total ‘150 CC’ bike sold by Hero?
10 10 2
(a) 88 % (b) 92 % (c) 94 %
21 21 7
2 10
(d) 114 % (e) 96 %
7 21

43. If Tvs sold 80% of total manufactured bikes and difference between
total ‘150 CC’ bikes sold by Tvs and Yamaha is 450, then find bike
manufactured by Hero & Bajaj together?
(a) 28000 (b) 26000 (c) 32000
(d) 24000 (e) 36000
Adda247 Publications For any detail, mail us at
23
Publications@adda247.com
Cracker Book for Bank (IBPS | SBI | RRB PO | Clerk) Mains Exams

44. If Hero, Bajaj & Tvs sold 75%, 80% & 90% of total manufactured bikes
respectively and total number of ‘150 CC’ bikes sold by these three
companies is 23124, then find the total bike sold by Honda?
(a) 15568 (b) 16164 (c) 17162
(d) 15360 (e) 17172

45. If total 84000 bikes manufactured by all five companies and ratio
between ‘150 CC’ to ‘200 CC’ bike manufactured by Honda is 3 : 2, then
find total ‘200 CC’ bike sold by Yamaha is what percent less than total
‘150 CC’ bike manufactured by Honda?
(a) 56.75% (b) 54.75% (c) 62.75%
(d) 59.75% (e) 58.75%

Directions (46-50): Given below are two pie-charts which shows the
percentage distribution of employees in Adda247 who travel to their office
in Gurgoan by two different means i.e. by metro and by cab on different
days of week. First pie chart shows data for Metro and second pie chart
shows data for Cab

Saturday Monday
15% 20%

Friday
(50/3)%
Tuesday
Thursday Wed. (70/3)%
10% 15%

Adda247 Publications For any detail, mail us at


24
Publications@adda247.com
Cracker Book for Bank (IBPS | SBI | RRB PO | Clerk) Mains Exams

Saturday
Monday
10%
25%
Friday
20%
Tuesday
(40/3)%
Thursday Wed.
(20/3)% 25%

Note: 1. Ratio of total employees travelling to Gurgaon by metro to by cab is


5 : 4.
2. If difference of persons travelling by metro and by cab on Saturday
is 420.
46. If number of employees travelling on Sunday by cab decreases by ‘Y’
with respect to employees travelling on Friday by cab and number of
employees travelling on Sunday by metro is twice than that of
travelling on Friday by metro . Then find value of ‘Y’?( Given that total
employees on Sunday is 75% of total employees on wed.)
(a) 1280 (b) None of these (c) 1385
(d) 1415 (e) 1255

47. What is the difference of number of employees travelling by metro on


Wednesday and Thursday together and number of employees travelling
by cab on same days together?
(a) 50 (b) 20 (c) None of these
(d) 40 (e) 60

48. If total fare per person travelling by metro & cab are Rs.120, then ratio
of total amount spent on Thursday by all employees travelling by cab to
total amount spent by all employees on same day travelling by metro?
(given that ratio of fare per person travelling by metro to by cab is 5 : 7)
(a) 21 : 23 (b) 56 : 73 (c) None of these
(d) 56 : 75 (e) 53 : 73
Adda247 Publications For any detail, mail us at
25
Publications@adda247.com
Cracker Book for Bank (IBPS | SBI | RRB PO | Clerk) Mains Exams

49. If total fare on Monday by all employees travelling by metro is Rs.


48000. Then find the total fare on same day by all employees travelling
by cab? (if per person fare for each employee is same)
(a) Rs. 48000 (b) Rs. 42000 (c) Rs. 56000
(d)None of these (e) Rs. 26000

50. Total number of employees travelling by metro on Friday and Saturday


together is approximately what percent more or less than number of
employees travelling by cab on same days together?
(a) 48% (b)28% (c) 42%
(d) 38% (e) 32%

Directions (51-55): Table given below shows number of students


appeared in preliminary examination of an exam ‘ASK’ in six different cities
and percentage of students failed in preliminary, mains and in only
reasoning. Those students who cleared the Preliminary examination can
give the Mains examination. In Mains examination of ASK, there are only
two subjects i.e. Reasoning and Quant. Student has to clear both the
subjects to pass in Mains examination of ‘ASK’.

Students Students
Students Students
Failed in Failed in
appeared in Failed in
Mains only
Preliminary Preliminary
examination Reasoning
exam exam (in %)
(in %) (in %)
A 96,000 25% 87.5% 25%
B 80,000 40% 75% 37.5%
2
C 1,20,000 35% 80% 16 %
3
D 1,60,000 15% 75% 25%
E 1,25,000 28% 70% 40%
1
F 72,000 33 % 68.75% 25%
3

Adda247 Publications For any detail, mail us at


26
Publications@adda247.com
Cracker Book for Bank (IBPS | SBI | RRB PO | Clerk) Mains Exams

51. Total number of students who failed only in Quant in City ‘A’ is three
times of total number of students passed the mains exams in city ‘B’
while total number of students who failed in both subjects in City ‘B’ is
3000 less then total number of students who failed is both subjects in
city ‘A’. Find number of students who failed only in Quant in city ‘A’ is
what percent more than number of students who failed in Reasoning in
city ‘B’?
(a) 250% (b) 50% (c) 150%
(d) 200% (e) 100%
52. In city D, total number of students who failed in Reasoning is ‘x’ more
than total number of students who failed in only Quant. If total number
of students who passed in mains exam in city D is ‘x’ then find the ratio
between total number of students who failed in Reasoning to total
number of students who failed in only Quant in city ‘D’.
(a) 1 : 3 (b) 3 : 1 (c) 1 : 1
(d) 1 : 2 (e) 2 : 1

53. Total number of students who failed in Quant in city E is how much
more than total number of students who failed in Quant in city F?
(a) 3000 (b) 6000 (c) 9000
(d) 12000 (e) 15000

54. Ratio between total number of students who failed in Quant in city ‘B’
to total number of students who failed in at most one subject in city ‘C’
is 30 : 91. Find what percent of students failed in both subject in city ‘C’
in mains exam?
(a) 25% (b) 15.5% (c) 17.5%
(d) 19.5% (e) 30%

55. Total number of students who failed in at most one subjects in city F is
24000 less than total number of students who failed in at least one
subject in city A. Find total number of students who failed in both
subjects in city ‘F’ is how much less than total number of students who
failed in only reasoning in city ‘A’.
(a) 3000 (b) 6000 (c) 9000
(d) 12000 (e) 15000
Adda247 Publications For any detail, mail us at
27
Publications@adda247.com
Cracker Book for Bank (IBPS | SBI | RRB PO | Clerk) Mains Exams

Direction (56-60): - A school has four hostels in which there are two
sections one for boys and one for girls. Line chart given below shows
increase/decrease in total number of students in 2013 in these hostels as
compare to 2012(previous year). Table given below shows ratio between
number of boys to number of girls in these hostels in 2013. Study the data
carefully and answer the following questions

300
Hostel Boys : Girls
250
2… Aravali 2:3
200
150 Nilgiri 4:1

100 Udaygiri 5:3

50 Shivalik 4:7
Aravali Nilgiri Udaygiri Shivalik

56. In Aravali, number of girls in 2013 is 20% more than that in 2012. If
change in the number of girls in Aravali is 25% greater than change in
number of boys in Aravali then find the Number of boys in Aravali in
2013.
(a) 900 (b) 600 (c) 500
(d) 750 (e) 800

57. Number of students is increases and decreases in Aravali and Nilgiri in


2013 respectively as compare to 2012. Number of girls who left Aravali
joins Nilgiri (No other change in Number of girls) such that ratio
between increase and decreases in Number of boys in Aravali and
Nilgiri in 2013 as compare to 2012 is 16 : 15, then find the change in
Number of boys in Nilgiri?
(a) 350 (b) 320 (c) 300
(d) 420 (e) 400

Adda247 Publications For any detail, mail us at


28
Publications@adda247.com
Cracker Book for Bank (IBPS | SBI | RRB PO | Clerk) Mains Exams

58. In 2012, Number of boys in Udaygiri and Shivalik are equal and
Number of girls in Shivalik is 540 more than the Number of girls in
Udaygiri. If in 2013, Number of boys in both hostels are equal then find
the Number of total student in Shivalik, If In 2013 strength of both
hostels increases as compare to previous year
(a) 1650 (b) 1200 (c) 960
(d) 1500 (e) 900

59. In 2012 and in 2013 ratio of total strength of Udaygiri to that of Nilgiri
is 4 : 5 and 5 : 8 respectively. Find the strength of Nilgiri in 2012 if
strength of Udaygiri decreases
(a) 1600 (b) 1200 (c) 1250
(d) 1400 (e) 1750

60. In2013, strength of all hostels increases as compare to 2012. If total


strength of hostels in 2012 is 1030, then find the Number of boys in
Aravali in 2013 is how much more than the Number of boys in Udaygiri
in 2013, given that ratio of strength of Aravali, Nilgiri, Udaygiri and
Shivalik is 2 : 3 : 2 : 3 respectively
(a) 80 (b) 100 (c) 120
(d) 90 (e) 10

Adda247 Publications For any detail, mail us at


29
Publications@adda247.com
Cracker Book for Bank (IBPS | SBI | RRB PO | Clerk) Mains Exams

Solutions

1. (a); Total boys in section ‘X” from CMS & Loyola


55 7 50 3
= 360 × × + 250 × ×
100 9 100 5
= 154 + 75 = 229
Total girls in section ‘Y’ from DPS & SKD
(100 –60) 10 (100–55) 1
= 420 × × + 300 × ×
100 21 100 3
= 80 + 45 = 125
229
Required percentage = × 100 = 183.2%
125

2. (b); Total girls in section ‘Y’ from CMS & SKD


(100–55) 2 (100 –55) 1
= 360 × × + 300 × ×
100 3 100 3
= 108 + 45 = 153
Total boys in section ‘X’ from DPS & SKD
60 5 55 1
= 420 × × + 300 × ×
100 9 100 3
= 140 + 55 = 195
153
Required ratio = = 51 : 65
195

3. (d); Total number of boys in section ‘X’ from D.P.S school


60 5
= 420 × × = 140
100 9
5
Required number of students = 140 × = 100
7

4. (e); Average number of girls in section ‘X’ from SKD & Loyola
55 2 50 2
300×100×3+250×100×5 110+50
= = = 80
2 2
Average number of boys in section ‘Y’ from LPS & CMS
(100–48) 4 (100–55) 1
450× 100
×9+360× 100 ×3
=
2
104+54
= = 79
2
Required difference = 80 – 79 = 1

Adda247 Publications For any detail, mail us at


30
Publications@adda247.com
Cracker Book for Bank (IBPS | SBI | RRB PO | Clerk) Mains Exams

5. (b); Total girls in section ‘Y’ from CMS & LPS together
(100–48) 5 (100–55) 2
= 450 × × + 360 × × = 130 + 108 = 238
100 9 100 3
Total boys in section ‘X’ from DPS & SKD
60 5 55 1
= 420 × × + 300 × ×
100 9 100 3
= 140 + 55 = 195
Required difference = 238 – 195 = 43
60 1
6. (a); Applicants got final selection in the year 2017= 8400 × × ×
100 8
40 25
× = 63
100 100
Applicants got final selection in the year 2014
75 30 25 45
= 3200 × × × × = 81
100 100 100 100
81−63
Required percentage = × 100
81
18 2 2
= × 1100 = × 100 = 22 %
81 9 9

7. (d); Total applicants got final selection in the year’s 2013 & 2016
together
85 25 40 25 80 25 20 30
= 2400 × × × × + 7500 × × × ×
100 100 100 100 100 100 100 100
= 51 + 90 = 141
Total applicants got final selection in the year 2014 & 2015
together
75 30 25 45 90 20 25 25
= 3200 × × × × + 4800 × × × ×
100 100 100 100 100 100 100 100
= 81 + 54 = 135
Required difference = 141 – 135 = 6

8. (a); Applicants qualified in tie II but did not get final selection in the
years 2017
60 1 40 (100−25)
= 8400 × × × × = 189
100 8 100 100
Applicants qualified in tier but did not qualify for tier II in the year
2014
75 30 (100−25)
= 3200 × × × = 540
100 100 100
189
Required ratio = = 7 : 20
540

Adda247 Publications For any detail, mail us at


31
Publications@adda247.com
Cracker Book for Bank (IBPS | SBI | RRB PO | Clerk) Mains Exams

9. (d); Total appeared applicants for tier I in the year 2017


60
= 8400 × = 5040
100
Total appeared applicants for tier II in the year 2015
90 20
= 4800 × × = 864
100 100
5040−864
Required percentage = × 100
864
4176 1
= × 100 = 483 %
864 3

10. (b); Total number of applications did not qualify in tier I in the year
2014, 2016 and 2017
75 (100−30) 80 (100−25) 60
= 3200 × × + 7500 × × + 8400 × ×
100 100 100 100 100
(100−12.5)
100
= 1680 + 4500 + 4410 = 10590

11. (e); Total Jubali class ticket sold by multiplex C & E


24 35 36 25
= 12500 × × + 12500 × ×
100 100 100 100
= 1050 + 1125 = 2175
Total gold class ticket sold by multiplex A & C
12 30 24 20
= 12500 × × + 12500 × × = 450 + 600 = 1050
100 100 100 100
2175−1050 1
Required percentage = × 100 = 107 %
1050 7

12. (d); Total silver class ticket sold by multiplex A, C and E


12 50 36 35 24 45
= 12500 × × + 12500 × × + 12500 × ×
100 100 100 100 100 100
= 750 + 1350 + 1575 = 3675
Total gold class ticket sold by multiplex B, C & E
18 30 24 20 36 40
= 12500 × × + 12500 × × + 12500 × ×
100 100 100 100 100 100
= 675 + 600 + 1800 = 3075
Required difference = 3675 – 3075 = 600
Adda247 Publications For any detail, mail us at
32
Publications@adda247.com
Cracker Book for Bank (IBPS | SBI | RRB PO | Clerk) Mains Exams

13. (b); Average number of jubali ticket sold by Multiplex B & C


18 40 24 35
12500× × +12500× ×
100 100 100 100 900+1050
= = 975
2 2
Average number of gold class ticket sold by multiplex C & E
24 20 36 40
12500× × +12500× × 600+1800
100 100 100 100
= = = 1200
2 2
Required sum = 975 + 1200 = 2175

14. (d); Total silver class ticket sold by multiplex A & D


12 50 10 50
= 12500 × × + 12500 × × = 750 + 625 = 1375
100 100 100 100
Total Gold glass ticket sold by B & E
18 30 36 40
= 12500 × × + 12500 × × = 675 + 1800 = 2475
100 100 100 100
2475−1375 1100 4
Required percentage = × 100 = × 100 = 44 %
2475 2475 9

15. (b); Let price of one Gold ticket in multiplex D, C and A be Rs.10x, Rs6x
and Rs7x respectively.
ATQ,
Total earning = 40000
10 10 24 20 12 30
12500× × ×10x+12500× × × 6x + 12500 × × × 7x
100 100 100 100 100 100
1250x + 3600x + 3150x = 40000
40000
x= ⇒ x = 5 Rs
8000
Required difference = 5 × 10 – 5 × 7 = 15 Rs.

16. (b); Total Samsung mobile sold by store on Saturday


2
= 750 × = 500
3
Total Samsung 3G mobile sold on Saturday
80 2
= 750 × × – 40 = 240 – 40 = 200
100 5
Total Samsung 4G mobile sold on Saturday
= 500 – 200 = 300
Total Samsung 4G mobile sold on Sunday
80 3
= 750 × × = 360
100 5
360–300 60
Required percentage = × 100 = × 100 = 20%
300 300

Adda247 Publications For any detail, mail us at


33
Publications@adda247.com
Cracker Book for Bank (IBPS | SBI | RRB PO | Clerk) Mains Exams

17. (b); Total MI mobile sold on Monday


960+640 1600
= = = 800
2 2
MI 3G mobile sold on Monday
5
= 800 × (4+5+7) = 250
250 250
Required ratio = 75 3 = = 25 : 27
840×100×7 270

18. (d); Total Oppo 4G mobile sold on Monday and Tuesday


640×65 3 (4+7)
= × × = 572
100 4 6
Total Samsung & Lava 4G mobile sold by store on Sunday
750×80 3 75 4
= × + 840 × ×
100 5 100 7
= 360 + 360 = 720
720 –572
Required percentage = × 100
720
148 5
= × 100 = 20 %
720 9

19. (d); Average number of Samsung & Lava 2G mobile sold by store on
Sunday
20 25
(750×100+840×100) 150+210
= = = 180
2 2
Average number of Huwai & Oppo 3G mobile sold by store on
Sunday
85 1 65 1
960×100×3+640×100×4 272+104
= = = 188
2 2
Required difference = 188 – 180 = 8

20. (c); Total Vivo mobile sold by store on Sunday


75 4 85 2
= (840 × × + 960 × × ) + 56
100 7 100 3
= (360 + 544) + 56 = 960
960–640
Required percentage = × 100
960
320 1
= × 100 = 33 %
960 3

Adda247 Publications For any detail, mail us at


34
Publications@adda247.com
Cracker Book for Bank (IBPS | SBI | RRB PO | Clerk) Mains Exams

21. (e); Total A1 and A3 tickets sold in Patel stadium


25 75 5
= 2400 × + 2400 × × = 600 + 1,000 = 1600
100 100 9
Total number of A1 and A2 tickets sold in Kaloor stadium
20 80 7
= 1800 × + 1800 × × = 360 + 840 = 1200
100 100 12
1600−1200 400 1
Required % = × 100 = × 100 = 33 %
1200 1200 3
22. (d); A2 tickets sold in Nehru stadium
85 9
= 1600 × × = 720
100 17
A3 tickets sold in Raipur stadium
70 4
= 2500 × × = 1,000
100 7
Required difference = 1000 − 720 = 280

23. (b); A3 tickets sold in Eden and Kaloor stadium together


80 3 80 5
= 2000 × × + 1800 × × = 600 + 600 = 1200
100 8 100 12
A2 tickets sold in Patel and Eden stadium together
75 4 80 5
= 2400 × × + 2000 × × = 800 + 1000 = 1800
100 9 100 8
1200 2
Required ratio = =
1800 3

24. (c); Total number of A1 tickets sold by all five stadiums together
15 25 20 20 30
= 1600 × + 2400 × + 2000 × + 1800 × + 2500 ×
100 100 100 100 100
= 240 + 600 + 400 + 360 + 750 = 2350
2350
Required average = = 470
5

25. (a); A1 and A2 tickets sold in Nehru stadium


15 85 9
= 1600 × + 1600 × × = 960
100 100 17
A1 and A2 tickets sold in Raipur stadium
30 70 3
= 2500 × + 2500 × × = 1500
100 100 7
1500−960
Required % = × 100 = 36%
1500

Adda247 Publications For any detail, mail us at


35
Publications@adda247.com
Cracker Book for Bank (IBPS | SBI | RRB PO | Clerk) Mains Exams

26. (d); Income of movie B in second week


30
= × 180 = 54 crores
100
Income of movie C in third week
10
= × 200 = 20 crores
100
Required ratio = 27 : 10

27. (d); Income of movie A in first week


50
= × 210 = 105 crores
100
Income of movie A in second week
30
= × 210 = 63 crores
100
42
% drop = × 100 = 40%
105
Similarly,
Income of movies B in first week
40
= 180 × = 72 crore
100
Income of movies B in second week
30
= 180 × =54 crore
100
18
% drop = × 100 = 25%
72
Income of movie C in first week
60
= 200 × = 120 crores
100
Income of movie C in second week
30
= 200 × = crores
100
60
% drop = × 100 = 50%
120
70
Income of movie D in week 1 = × 240
100
= 168 crore
10
income of movie D in week 2 = × 240
100
= 24 crore
144 5
% drop = × 100 = 85 %
168 7
Hence D is the answer.
Adda247 Publications For any detail, mail us at
36
Publications@adda247.com
Cracker Book for Bank (IBPS | SBI | RRB PO | Clerk) Mains Exams
80
28. (c); Foreign income of movie A = × 210 = 56 crores
300
20
Foreign income of movie C = × 200 = 40 crores
100
Required ratio = 7 : 5
25
29. (a); Total income of B from foreign = × 180 = 45 crores
100
2
Foreign income in second week = × 45 crores = 18 crores
5
Income earn by movie B in second week in India
30
= × 180 – 18 = 36 crores
100

30. (c); Second week income of movie D


10
= × 240 = 24 crore
100
10
Third week income of movie C = × 200 = 20 crore
100
Second week income of movie A
30
= × 210 = 63 crore
100
Second week income of movie B
30
= × 180 = 54 crore
100
117–44 73
Required % = × 100 =
117 117

31. (e); Average budget for sport


125+130+150+170+160 735
= = = 147
5 5
Average budget for Education
105+125+130+200+220 780
= = = 156
5 5
147
Required% = × 100 = 94.23 ≈ 94%
156

32. (c); Budget allocated for sport for male in 2022


3
= 160 × = 96 𝑐𝑟.
5
Budget allocated for sport for male in 2023
5
= 96 × = 120 cr
4
24
Required increase = × 100 = 15%
160

105+125+130 360
33. (b); Required ratio = = = 18 ∶ 29
170+190+220 580

Adda247 Publications For any detail, mail us at


37
Publications@adda247.com
Cracker Book for Bank (IBPS | SBI | RRB PO | Clerk) Mains Exams

34. (a); Budget allocated for Cricket and Badminton together in 2019
(5+4)
= 130 × (5+4+4) = 90 cr
Budget allocated for Rural Education in 2022
6
= 220 × = 120 cr
11
Required difference = 120 – 90 = 30 cr

35. (d); Average of budget allocated for Health care in the year 2018-22
160+190+170+190+220 930
= = = 186
5 5
Average of budget allocated for Education in year 2018-22
105+125+130+200+220 780
= = = 156
5 5
Required sum = 186 + 156 = 342

36. (e); Let, total number of manufactured cars by TATA & HONDA in the
year 2016 be 6x and 7x units respectively.
ATQ,
75 (100−15) 80 (100−15)
7𝑥 × × − 6𝑥 × ×
100 100 100 100
= 4590
75 85 80 85
7𝑥 × × − 6𝑥 × × = 4590
100 100 100 100
357𝑥 102𝑥
− = 4590
80 25
1785𝑥−1632𝑥
= = 4590
400
4590×400
𝑥= ⇒ x = 12,000 units
153
Total number of cars returned to HYUNDAI in 2017 = (12000 ×
80 15
6) × × − 1920
100 100
= 8640 – 1920 = 6720
Total number of cars returned to MARUTI SUZUKI in 2017
75 15
= (12000 × 7) × × + 4830 = 9450 + 4830 = 14,280 units
100 100
Let total number of cars manufactured by HYUNDAI in 2017 be x
units
70 15
𝑠𝑜, 𝑥 × × = 6720
100 100
6720×100×100
𝑥= ⇒ x = 64,000 units
70×15

Adda247 Publications For any detail, mail us at


38
Publications@adda247.com
Cracker Book for Bank (IBPS | SBI | RRB PO | Clerk) Mains Exams

Let total number of cars manufactured by MARUTI SUZUKI in 2017


be y units
so,
85 17.5
𝑦× × = 14280
100 100
14280×100×100
𝑦= ⇒ y =96,000 units
85×17.5
Required difference = 96000 – 64000 = 32,000 units

37. (c); Let total number of cars manufactured by MARUTI SUZUKI in the
year 2016 be 100x units
So, total number of cars manufactured by TATA in 2017 be 140x
units
ATQ,
75 10 80 12.5
140𝑥 × × − 100𝑥 × × = 120
100 100 100 100
21𝑥
− 10𝑥 = 120 ⇒ x = 240
2
Total number of cars returned to HONDA in 2017 = (240 × 100) ×
80 12.5
× + 2010 = 2400 + 2010 = 4410 units
100 100
Let total number of cars manufactured by HONDA in year 2017 be x
units
ATQ,
70 22.5
𝑥× × = 4410
100 100
4410×100×100
𝑥= ⇒ x = 28,000 units
70×22.5

38. (c); Let total number of cars manufactured by HYUNDAI, MARUTI


SUZUKI & HONDA in 2016 be 2y, 3y and 4y respectively.
ATQ,
65 20 80 12.5 75 15
2𝑦×100×100+3𝑦×100× 100 +4𝑦×100×100
= 4040
3
13𝑦 3𝑦 9𝑦
+ + = 12120
50 10 20
101y = 1212000 ⇒ y = 12,000
Required average
120 125 112.5
(12000×2)× +(12000×3)×100+(12000×4)× 100
100
=
3
28800+45000+54000 127800
= = = 42,600
3 3
Adda247 Publications For any detail, mail us at
39
Publications@adda247.com
Cracker Book for Bank (IBPS | SBI | RRB PO | Clerk) Mains Exams

39. (d); Let total number of cars manufactured by HYUNDAI & HONDA in
the year 2016 be 8x and 9x units respectively.
ATQ,
65 (100−20) 75 (100−15)
8𝑥 × × + 9𝑥 × × = 39590
100 100 100 100
104𝑥 459𝑥
+ = 39590
25 80
3959𝑥
= 39590 ⇒ x = 4,000 units
400
Total number of cars manufactured by HONDA in 2017
36000
= × 5 = 60,000
3
Total number of cars manufactured by HYNDUAI in 2017
32000
= × 3 = 48,000
2
60000−48000 12000
Required percentage = × 100 = × 100 = 25%
48000 48000

40. (d); Let total number of cars manufactured by TATA, HYUNDAI &
HONDA in the year 2017 is 7y , 6y & 8y units respectively
ATQ –
75 100−10 70 100−15 70 100−22.5
7𝑦 × × + 6𝑦 × × + 8𝑦 × × = 75810
100 100 100 100 100 100
189𝑦 357𝑦 217𝑦
+ + = 75810
40 100 50
945𝑦+714𝑦+868𝑦
= 75810
200
2527y = 75810 × 200 ⇒ y = 6,000
Total number of cars manufactured by TATA, HYUNDAI & HONDA
together in the year 2017
= 7 × 6000 + 6 × 6000 + 8 × 6000
= 42000 + 36000 + 48000 = 1,26,000

41. (b); Let total bike manufactured by all five companies = 100x
So, total bike manufactured by Honda
= 20x
Ratio between total ‘200 CC’ & ‘150 CC bikes sold by Honda = 5 : 4
80 5 80 4
20x × × − 20𝑥 × × = 640
100 9 100 9
80𝑥 64𝑥
− = 640 ⇒ x = 360
9 9
Total ‘200 CC’ bike sold by Yamaha
22 75 (100−70)
= 36000 × × × = 1782
100 100 100
Adda247 Publications For any detail, mail us at
40
Publications@adda247.com
Cracker Book for Bank (IBPS | SBI | RRB PO | Clerk) Mains Exams

42. (d); Let total bike manufactured by all five companies = 100x
Total ‘150 CC’ bike sold by Bajaj
25 90 40
= 100x × × × = 9x
100 100 100
Total ‘150 CC’ bike sold by Hero
15 90 35
= 100x× × × = 4.2x
100 100 100
9𝑥−4.2𝑥 2
Required percentage = × 100 = 114 %
4.2𝑥 7

43. (d); Let total bike manufactured by all five companies = 100x
ATQ –
75 70 80 75
22x × × − 18𝑥 × × = 450
100 100 100 100
11.55x − 10.8x = 450
0.75x = 450 ⇒ x = 600
Total bike manufactured by Hero & Bajaj
(15+25)
= 60000 × = 24000
100

44. (d); Let total bike manufactured by all five companies = 100x
75 35 80 40 90 75
15x × × + 25𝑥 × × + 18𝑥 × × = 23124
100 100 100 100 100 100
3.9375x + 8x + 12.15x = 23124
x = 960
Total bike sold by Honda
20 80
= 96000 × × = 15360
100 100

45. (e); Total ‘200 CC’ bike manufactured by Honda


20 3
= 84000 × × = 10080
100 5
Total ‘150 CC’ bike sold by Yamaha
22 75 (100−70)
= 84000 × × × = 4158
100 100 100
10080−4158
Required percentage = × 100 = 58. 75%
10080

Adda247 Publications For any detail, mail us at


41
Publications@adda247.com
Cracker Book for Bank (IBPS | SBI | RRB PO | Clerk) Mains Exams

Solutions (46-50):
Let total employees travelling by metro & by Cab be 5𝑥 & 4𝑥 respectevely.
15 10
5𝑥 × x – 4𝑥 × x = 420
100 100
75𝑥 40𝑥
− = 420 ⇒ X=1200
100 100
∴ Total employees travelling by metro = 1200 x 5 = 6000
& total employees travelling by cab = 1200 x 4 = 4800

50
46. (c); Total employees travelling on Friday by metro = x 6000 = 1000
300
20
Total employees travelling on Sunday by cab = x 4800–y=960 – y
100
ATQ,
960 – y + 2 x 1000
75 15 25
= [ × 6000 + × 4800]
100 100 100
2960 – y = .75 x 2100
Y = 2960 - 1575 = 1385

47. (b); No. of employees travelling by metro on Wednesday & Thursday


together.
15 10
= × 6000 + × 6000
100 100
= 900 + 600 = 1500
No. of employees travelling by Cab on same days together.
25 20
= × 4800 + × 4800
100 300
= 1200 + 320 = 1520
Required difference = 1520 – 1500 = 20

48. (d); Fare per person travelling by metro


5
= × 120 = Rs. 50
12
Fare per person travelling by cab .
7
= × 120 = 𝑅𝑠 70
12
20
300
×4800×70
Require ratio = 10 = 56 : 75
100
×6000×50

Adda247 Publications For any detail, mail us at


42
Publications@adda247.com
Cracker Book for Bank (IBPS | SBI | RRB PO | Clerk) Mains Exams

49. (a); Total employees travelling by metro on Monday


20
= × 6000 = 1200
100
48000
∴ Per person fare = = 𝑅𝑠. 40
1200
∴ Total fare on Monday of all employees travelling by cab
25
= × 4800 × 40 = 𝑅𝑠. 48000
100

50. (e); Total No. of employees travelling by metro on Friday & Saturday
together
50 15
= × 6000 + × 6000
300 100
= 1000 + 900 = 1900
Total no. of employees travelling by cab on same days together
20 10
= × 4800 + × 4800
100 100
= 960 + 480 = 1440
1900−1440
Required Percentage = × 100
1440
approximately 32%

51. (b); In city A,


Number of students appeared in mains exam
75
= 96000 × = 72000
100
Number of students passed in mains exam
12.5
= 72000 × = 9000
100
In city B,
Number of students appeared in mains exam
60
= 80,000 × = 48000
100
Number of students passed in mains exam
25
= 48000 × = 12000
100
Total number of students who failed only in quant in city A = 3 ×
12000 = 36000
Total number of students who failed in both subjects in city ‘A’
25
= 72000 – 9000 – 36000 – 72000 × = 9000
100
Total number of students who failed in both exam in city ‘B’ = 9000
– 3000= 6000
Adda247 Publications For any detail, mail us at
43
Publications@adda247.com
Cracker Book for Bank (IBPS | SBI | RRB PO | Clerk) Mains Exams

Total number of students who failed in Reasoning = 48000 ×


37.5
+ 6000
100
= 18000 + 6000 = 24000
36000−24000
Required % = × 1000 = 50%
24000
Vein diagram for failed students:-

52. (e); In city D,


Number of students appeared in mains exam
85
= 1,60,000 × = 1,36,000
100
Number of students passed in mains exam
25
= 1,36,000 × = 34,000 = 𝑥
100
Total number of students who failed in reasoning = 34,000 + x
Total number of students who failed in Quant
75
= 1,36,000 × − 34,000 = 68,000
100
Let Total number of students who failed in only Quant = a
And
Let Let Total number of students who failed in both subjects = b

Adda247 Publications For any detail, mail us at


44
Publications@adda247.com
Cracker Book for Bank (IBPS | SBI | RRB PO | Clerk) Mains Exams

ATQ,
𝑎 + 𝑏 = 68,000
And 34,000 + 𝑏 = 𝑎 + 34,000
So 𝑎 = 𝑏 = 34,000
Total number of students who failed in Reasoning = 34,000 +
34,000 = 68,000
Total number of students who failed in only Quant = 34,000
68,000 2
Required Ratio = =
34,000 1

53. (b); Total number of students who failed in Quant in city ‘E’ = Total
number of students who failed in Mains exam – Total number of
students who failed in only Reasoning
72 70−40
= 1,25,000 × × = 27,000
100 100
Total number of students who failed in Quant in city ‘F’ = Total
number of students who failed in Mains exam – Total number of
students who failed in only Reasoning
2 68.75−25
= 72,000 × × = 21,000
3 100
Required difference = 27,000 − 21,000 = 6,000

54. (e); Total number of students who failed in Quant in city ‘B’
60 75−37.5
= 80,000 × × = 18,000
100 100
Total number of students who failed in at most one subject in city
18,000
‘C’ = × 91 = 54,600
30
In city ‘C’
Total number of students who failed in at most one subject in city
‘C’ = Total number of students who failed in Reasoning only + Total
Adda247 Publications For any detail, mail us at
45
Publications@adda247.com
Cracker Book for Bank (IBPS | SBI | RRB PO | Clerk) Mains Exams

number of students who failed in only Quant + Total number of


students who passed in both subjects
Total number of students who passed in both subjects
65 20
= 1,20,000 × × = 15,600
100 100
Total number of students who failed in only Reasoning
65 50
= 1,20,000 × × = 13,000
100 300
Total number of students who failed in only Quant
= 54600 − 15,600 − 13,000 = 26,000
Total number of students who failed in Quant
65 80 50
= 120000 × ×[ − ] = 49,400
100 100 300
Total number of students who failed in both exams
= 49,400 − 26,000 = 23,400
23,400
Required % = × 100 = 30%
78000

55. (c); Total number of students who failed in atleast one subject in city ‘A’
75 87.5
= 96,000 × × = 63,000
100 100
Total number of students who failed in at most one subject in city
‘F’ = 63,000 − 24,000 = 39,000
Total number of students who failed in only Quant in city ‘F’
2 31.25+25
= 39,000 − 72,000 × × = 12,000
3 100
Total number of students who failed in both subjects
2 68.75−25
= 72,000 × × − 12,000 = 21,000 − 12,000 = 9,000
3 100
Total number of students who failed in only reasoning in city ‘A’
75 25
= 96,000 × × = 18,000
100 100

Adda247 Publications For any detail, mail us at


46
Publications@adda247.com
Cracker Book for Bank (IBPS | SBI | RRB PO | Clerk) Mains Exams

Required difference = 18,000 − 9,000 = 9,000

56. (b); Let Number of girls and boys in Aravali in 2012 be x and y
respectively.
In 2013
Number of girls = 1.2 x
Change in girls = 0.2x
0.2𝑥
Change in boys is = × 100 = 0.16x
125
Total increment = 0.2x + 0.16x
0.36x = 270
270×100
x= ⇒ x = 750
36
Number of boys in Aravali in 2013
2
= × 1.2 × 750 = 600
3

57. (c); Let Number of Girls in Aravali be x.


Let Number of girls who left Aravali and joins Nilgiri is ‘a’
Total strength of Aravali increases which means change in number
of boys is
= 270 + a
Adda247 Publications For any detail, mail us at
47
Publications@adda247.com
Cracker Book for Bank (IBPS | SBI | RRB PO | Clerk) Mains Exams

Total strength of Nilgiri decreases, which means change in number


of boys in Nilgiri is = 250 + a
270+𝑎 16
= ⇒ a = 50
250+𝑎 15
Number of boys increases in Nilgiri = 250 + 50 = 300

58. (a); Let Number of boys in Udaygiri in Shivalik in 2012 = x


Let Number of Girls in Udaygiri in 2012 = y
So, Number of Girls in Shivalik = y + 540
In 2013,
Let Number of boys in Udaygiri and Shivalik = 20a
⇒ Number of Girls in Shivalik = 35a
And, Number of Girls in Udaygiri = 12a
ATQ
x + y + 150 = 32a … (i)
x + (y + 540) + 300 = 55a … (ii)
Solving (i) and (ii)
a = 30
Shivalik strength = 55a = 55 × 30 = 1650

59. (e); Let in 2012


Total strength of Udaygiri and Nilgiri be 4x and 5x respectively
And total strength of Udaygiri and Nilgiri in 2013 be 5y and 8y
respectively
ATQ,
4x – 5y = 150 … (i)
Two cases formed
Either
8y – 5x = 250 ... (ii)
Or 5x – 8y = 250 ... (iii)
On solving (i) and (iii) it does give-ve integer value
So, on solving (i) and (ii)
x = 350 ⇒ y = 250
Strength of Nilgiri in 2012 = 1750

Adda247 Publications For any detail, mail us at


48
Publications@adda247.com
Cracker Book for Bank (IBPS | SBI | RRB PO | Clerk) Mains Exams

60. (d); Total strength in 2013 = 1030 + 970


= 2000
Number of boys in Aravali in 2013
2000 2
= × 2 × = 160
10 5
Number of boys in Udaygiri in 2013
2000 5
= × 2 × = 250
10 8
Required Difference = 250 – 160 = 90

Adda247 Publications For any detail, mail us at


49
Publications@adda247.com
Cracker Book for Bank (IBPS | SBI | RRB PO | Clerk) Mains Exams

1 Adda247 Publications For any detail, mail us at


Publications@adda247.com
Cracker Book for Bank (IBPS | SBI | RRB PO | Clerk) Mains Exams

Chapter
Arithmetic Data Interpretation
14
BEST APPROACH TO SOLVE THE QUESTIONS

INTRODUCTION: DI stands for DATA INTERPRETATION. Interpretation


means representing a data in a sensitive way after analyzing it well. In all
the banking exam or particularly quant section of bank, DI plays vital role.
The basic DI consists of pie table, bar graph, numerical value table or graph.
Look at this example:

900
800
700
Number of mobiles

600
500
400
300
200
100
0
Mon Tue Wed Thu Fri Sat

Instead of telling ‘number of mobiles sold’ for each day it is represented in


the graph. So that it is more visible and interested for a reader in this form.
Common form of DI are ratios, percentages or average.

ARITHMETIC DI

Arithmetic DI means representing arithmetic equations and conditions


in bar graphs, line graph, pie graph or in any other form of DI. The questions
on arithmetic DI also cover major portion in mains of any banking exam. So
2 Adda247 Publications For any detail, mail us at
Publications@adda247.com
Cracker Book for Bank (IBPS | SBI | RRB PO | Clerk) Mains Exams

it is necessary to understand the actual concept. Easy way to understand


this is through an example.

Total Road construct = 3600 meters


E A
46° 72°

D
90°
B
80°
C
72°
In this pie chart, portion of ‘Road construct’ is given for 5 different
72
companies. For eg. Company A constructs × 3600 = 720 meters road.
360
Now to make question more complicated different individual for each team
may be given. Let us consider through a bar graph.

32
28
24
No. of Labour in 1 team

20
16
12
8
4
0
A B C D E
Name of teams

3 Adda247 Publications For any detail, mail us at


Publications@adda247.com
Cracker Book for Bank (IBPS | SBI | RRB PO | Clerk) Mains Exams

Here we have considered number of individuals in each team. Now they


may ask you questions like “How many meters 1 member of team A
constructs”. It is definitely easy one. We just had to do
720
= 40 meters.
18
or to make it more complicated they may ask you
“How many day will 8 members of team C will take to construct 3600
meters road?”
Do it yourself.

HINT: Calculate road constructed by an individual of team C in 1 day as we


have calculated previously. Then calculate how many days 8 men will take
to construct given road.
A key point to solve any arithmetic DI is that to understand what had been
given in DI. Then consider these as an individual separate questions from
arithmetic, also these questions are way easier. Now let us consider another
example:

25% Series 1
20%
20%
15%
15% 12%
10%
10%
5%
5%

0%
A B C D E

Look carefully, here a % graph is given and nothing have been mentioned.
Examiner may mention it “% of distance covered by these (A, B, C, D & E)
individuals in 1 hour and total distance is 100 km” OR
“ % of work done by individuals in given time” OR
“ Profit/loss % earned on selling these items for shopkeeper”

4 Adda247 Publications For any detail, mail us at


Publications@adda247.com
Cracker Book for Bank (IBPS | SBI | RRB PO | Clerk) Mains Exams

“ Interest rate given by a bank to different individuals” or many other


things.
Point is arithmetic DI is not as hard as student consider it in exams, it is
always easier than actual arithmetic questions that are asked in exam.
Before solving a question, we must exactly know the information he had
given. For eg. Consider that this graph shows profit % for selling these
items. In question, if examiner ask to calculate actual selling price, then he
must mention the CP or some other clue in exam.

Example: Profit earned on selling item C is Rs. 160, what is amount of


profit earned for item D, if both items have same selling price?
Solution: This is a typical DI question. Now look, if we know that 20%
means Rs 160 we can calculate actual CP of item C. (Profit % is
always calculated on CP).
100
CP OF ITEM C = 160 × = 800.
20
It means SP of item C is Rs. 960
Also SP of item D is Rs. 960. And then we can calculate CP and
amount of profit of item D.

5 Adda247 Publications For any detail, mail us at


Publications@adda247.com
Cracker Book for Bank (IBPS | SBI | RRB PO | Clerk) Mains Exams

Practice Exercise Based on new Pattern

Direction (1 - 4): Given below bar graph shows number of hours taken by
six person to complete a task individually. Read the data carefully and
answer the questions:

50

40

30

20

10

0
Neeraj Aniket Saurabh Gopal Veer Sameer

1. Neeraj, Saurabh and Gopal start working together but due to bad health
1 1
the efficiency of Neeraj and Gopal decreases by 12 % and 33 %
2 3
respectively. Then find in how many hours task will be completed by all
three?
1 1 1
(a) 11 hours (b) 10 hours (c) 12 hours
4 4 4
1 1
(d) 9 hours (e) 13 hours
4 4

2. Veer and Sameer started working together on another task, where


Sameer work with 25% less efficiency. Veer and Sameer work for y
hours and remaining work complete by Aniket in (y + 1) hours, if ratio
of work done by Veer and Sameer together and by Aniket alone is 2 : 1,
then in how many hours Neeraj will complete same task alone?
1 1 1
(a) 15 hours (b) 13 hours (c) 17 hours
2 2 2
1 1
(d) 11 hours (e) 9 hours
2 2
6 Adda247 Publications For any detail, mail us at
Publications@adda247.com
Cracker Book for Bank (IBPS | SBI | RRB PO | Clerk) Mains Exams

3. If Divyaraj can do 50 % more work in one hour as Neeraj can do in one


hour, while Manish can do 25% less work in one hour as Aniket can do
in one hour. Saurabh started working alone and after some time he left
and remaining work completed by Divyaraj & Manish together in 11.75
hours more than time for which Saurabh worked. Then find total time
in which work is completed?
1 1 1
(a) 14 hours (b) 12 hours (c) 10 hours
4 4 4
1 1
(d) 16 hours (e) 8 hours
4 4
4. Neeraj, Aniket, Veer and Sameer worked together on a work in first
hour, while Saurabh & Gopal together destroyed the same work in
second hour. If this work continues till the total work completed. Find
how many hours required to complete the whole work?
270 270 270
(a) 22 hours (b) 20 hours (c) 18 hours
401 401 401
270 270
(d) 24 hours (e) 16 hours
401 401

Direction (5 - 9): The given bar-graph shows the number of marbels of


different colors which are contained by two different bags X and Y. Read the
data carefully and answered the following questions.

X Y
7
6
5
4
3
2
1
0
Yellow Red Black Blue

7 Adda247 Publications For any detail, mail us at


Publications@adda247.com
Cracker Book for Bank (IBPS | SBI | RRB PO | Clerk) Mains Exams

5. If two marbels are picked at random from bag Y then what is the
probability that both marbels are blue in color?
21 19 63
(a) (b) (c)
190 190 190
11 10
(d) (e)
95 99

6. If two marbels are picked at random from bag X then what is the
probability that both are yellow in color?
3 4 7
(a) (b) (c)
53 51 59
5 8
(d) (e)
51 59

7. If two marbels are picked at random from bag Y then what is the
probability that one is red and one is black in color?
1 6 7
(a) (b) (c)
19 95 95
8 13
(d) (e)
95 190

8. If two marbels are picked at random from bag X then what is


probability that either both are black or both are blue in color?
1 4 9
(a) (b) (c)
15 105 115
3 1
(d) (e)
37 17

9. If three marbels are picked at random from bag Y then what is the
probability that one is black, one is blue and one is yellow in color?
17 49 53
(a) (b) (c)
95 285 381
13 47
(d) (e)
95 360
Directions (10-14): Given below pie chart (I) shows percentage
distribution of total filled quantity of water in seven tanks. Line graph
shows time (minutes) taken by seven pipe A, B, C, D, E, F and G in filling
tanks P, Q, R, S, T, U and V respectively. Give the answer of the question
according to given data :
8 Adda247 Publications For any detail, mail us at
Publications@adda247.com
Cracker Book for Bank (IBPS | SBI | RRB PO | Clerk) Mains Exams

Total quantity of seven tanks = 1500 ℓ


P
V
15%
12%
U Q
18% 12%

R
T 8%
15% S
20%

65
60
55
Time (in minutes)

50
45
40
35
30
25
20
A B C D E F G
Pipes

10. A man has a tank which has quantity of water equal to the quantity of
water in P and T together. He opened pipe B and D together for filling
his tank. After X minutes man closed both pipes and opened pipe C and
E for filling remaining tank. If tank filled in next (x + 15) minutes then
find how many liter of water filled by B and D together ?
(a) 160 l (b)180 l (c)150l
(d)120l (e) 144 l
9 Adda247 Publications For any detail, mail us at
Publications@adda247.com
Cracker Book for Bank (IBPS | SBI | RRB PO | Clerk) Mains Exams

11. For filling tank S, man opened pipe D which fills the tank with 20%
more efficiently. After X minutes man close the pipe D and opened pipe
C and G together. Pipe C fills the tank with 25% more efficiently. If pipe
C and G fills remaining tank in 15 minutes then find the value of X ?
(a) 12 minutes (b) 10 minutes (c) 15 minutes
(d) 8 minutes (e) 18 minutes

12. Quantity of another tank K is 120% of total quantity of all seven tank.
For filling the bigger tank man opened pipes alternatively in such a way
that three pipe B, D and G together for first minute, pipe A alone for
second minute and pipe F and C together for third minutes. Find the
total time required in filling the tank alternatively?
(a) 151 7 minutes (b)141 7 minutes (c)151 minutes
9 9
(d)150 minutes (e) 170 minutes

13. A man have a tank M, which has equal quantity of water as tank S and U
together . For filling tank M man opened pipe A, C and G together for
17
some minutes man thought tank will be fully filled, but it takes 7
19
minutes more because of a leak. Find in what time leak will empty the
fully filled tank?
(a) 3hr (b) 3 2 hr (c)2 1 hr
3 6
1
(d) 3 1 hr (e) 4 hr
6 3

14. A man have a tank of 570 liter. Man opened three pipes B, D and C
together for filling the tank but pipe C fills the tank at 175% of its initial
Efficiency. There is a leak at a height of 2/3 from the bottom of tank
,which empty the tank with half of the efficiency of what B, D and C
filling the tank. Man noticed the leak after 32 minutes and closed all
pipes and leak. Find how many liter of tank remain unfilled?
(a) 90l (b) 76l (c) 78l
(d) 85l (e) 96l

10 Adda247 Publications For any detail, mail us at


Publications@adda247.com
Cracker Book for Bank (IBPS | SBI | RRB PO | Clerk) Mains Exams

Directions (15 − 𝟏𝟗): Given below the table shows Investment of five
person, time and share of profit. Some data are missing, calculate that
according to the question and answer the questions –

Person Investment (Rs) Time (months) Profit share (Rs.)


P — 12 65700
Q 32000 — —
R — — —
S 24000 — —
T — — 24300

15. P, Q and S invested in a business together, investment of P is 75% of


investment of S and all three invested for same time. If Q and S gets
extra 12% and 15% of total profit respectively and remaining profit is
distributed according to their share, then find the total profit.
(a) 270000 Rs. (b) 370000 Rs. (c) 375000 Rs.
(d) 275000 Rs. (e) 325000 Rs.

16. Person Q and R invested in the ratio of 4 : 3 and ratio between time
period of Investment Q to R is 5 : 3. If both person agree that 65% of the
total profit should be divided equally and remaining profit is to be
divided into ratio of their capital. If Q gets 10010 Rs. more to R. then
find the total profit share of R ?
(a) 27765 Rs. (b) 28190 Rs. (c) 42497 Rs.
(d) 34307 Rs. (e) 32695 Rs.

17. Q started a business with his investment, after some month S came to
joined with him and invest his amount in business. At the end of 3 year,
the ratio of profit of Q to S is 3 : 2. Find after how many month S joined
the business ?
(a) 4 months (b) 6 months (c) 3 months
(d) 5 months (e) 6 months
11 Adda247 Publications For any detail, mail us at
Publications@adda247.com
Cracker Book for Bank (IBPS | SBI | RRB PO | Clerk) Mains Exams

18. If T received Rs. 24300 as profit out of the total profit of Rs. 40500
which T and R earned at the end of one year. If T invested Rs. 81000 for
9 months, whereas R invested his amount for the whole year, what was
the amount invested by R ?
(a) 30500 Rs. (b) 40500 Rs. (c) 35500 Rs.
(d) 32500 Rs. (e) 40050 Rs.

19. If Q and R invested into the ratio of 8 : 9 and R and S Invested into the
ratio of 3 : 2. At the end of the year if they all got a total profit of 37030
Rs. then find the share of profit of Q, R and S individually ?
(a) 14490 Rs, 9660 Rs, 12880 Rs. (b) 9660 Rs, 17710
Rs, 11005 Rs.
(c) 12880 Rs, 14490 Rs, 9660 Rs. (d) 12434 Rs, 13594
Rs, 11006 Rs.
(e) None of these
Direction (20- 23): Data about investments of different persons is given
below. Study the data and solve the questions carefully.
→ Investment of Neeraj is 50% more than investment of Aman while
investment of Neeraj is 25% less than that of Sandeep.
Total Investment = 40,000

Rakesh,
30% Saurabh,
40%

Sandeep, 30%

20. Neeraj and Saurabh started a business together. After 8 months of


business they invested Rs 1000 per month for every month. If annual
profit is Rs 8000 then find Saurabh’s profit out of total profit?
(a) Rs 2950 (b) Rs 4000 (c) Rs 5050
(d) Rs 4450 (e) Rs 4850

12 Adda247 Publications For any detail, mail us at


Publications@adda247.com
Cracker Book for Bank (IBPS | SBI | RRB PO | Clerk) Mains Exams

21. Aman and Rakesh started a business together. Rakesh left the business
‘x’ months before the completion of year while Aman increased his
investment by Rs 2000 after 8 months and then after 2 months more he
increased his investment by Rs 2000 again. If ratio between profit
share of Aman and Rakesh is 7 : 8 then Rakesh worked for how many
months?
(a) 8 months (b) 4 months (c) 6 months
(d) 2 months (e) 10 months

22. Sandeep and Neeraj started a business together. After 8 months, Neeraj
is replaced by another partner, Satish whose investment is Rs 4000
more than investment of Saurabh. If Satish worked for 3 months, then
find the profit share of Sandeep if annual profit is Rs 11,500.
(a) Rs 2500 (b) Rs 3000 (c) Rs 4500
(d) Rs 5000 (e) Rs 6000

23. Aman invested his amount in a scheme which after 20% p.a. at C.I. for 2
years while Saurabh invested his amount in a car whose value will
depreciate at 20% p.a. every year. Find the total value of amount Aman
and Saurabh will have after 2 years.
(a) Rs. 31,680 (b) Rs. 18,880 (c) Rs. 26,880
(d) Rs. 16,320 (e) Rs. 22,880

Direction (24 – 25): Given below table shows quantity of four different
liquid in four different vessels. Read the data carefully and answer the
questions.

Vessels Milk Water Orange Juice Mango juice


(ml) (ml) (ml) (ml)
P 240 — 144 —
Q 160 80 — 210
R — 144 192 —
S 160 — — 120

13 Adda247 Publications For any detail, mail us at


Publications@adda247.com
Cracker Book for Bank (IBPS | SBI | RRB PO | Clerk) Mains Exams

24. In vessel P quantity of water is 50% of quantity of milk in same vessel,


while quantity of mango juice in vessel P is 6.25% less than quantity of
Orange juice in vessel R. In vessel S quantity of water is 50% more than
quantity of milk in same vessel and quantity of Orange juice in vessel S
is 40 ml less than quantity of water in same vessel. If 114 ml mixture
from vessel P and 180 ml from vessel S taken out and mixed in vessel T,
then find percentage of orange juice in vessel T?
25 25 25
(a) 29 % (b) 25 % (c) 27 %
147 147 147
25 25
(d) 23 % (e) 21 %
147 147

1
25. Quantity of milk in vessel R is 33 % less than quantity of water in
3
same vessel and quantity of orange juice in vessel Q is 25% less than
quantity of milk in same vessel. For making 1086 ml mixture solution
mixture of vessel Q & R mixed together, if ratio of quantity of total
mango juice in vessel R to total quantity of four liquid in vessel R is 7 :
43, then find the ratio between total mango juice in resulting mixture to
total given mixture?
49 49 49
(a) (b) (c)
161 151 181
49 49
(d) (e)
121 111

Direction (26-30): - Table given below shows details of 3 types of item of a


garment shop MRP, discount% (on MRP) and certain offer.
Shopkeeper have shoes, Jeans & T-Shirts in 2 types of each.
S₁, S₂; J₁, J₂; and T₁, T₂ represents the types of two types of shoes, jeans and
T-shirts respectively.

Items Shoes Jeans T-shirts


Types S₁ S₂ J₁ J₂ T₁ T₂
MRP (Rs) 1200 1647 1200 — 400 600
1
Discount 0% 11 % 0% 25% 10% 0%
9
Shop offer      

Note: Shop offer: Buy 2 get 2 free


 → Offer given  → Offer not given
14 Adda247 Publications For any detail, mail us at
Publications@adda247.com
Cracker Book for Bank (IBPS | SBI | RRB PO | Clerk) Mains Exams

26. If a person wants to buy T-shirts, what is minimum average price/t-


shirts he could get?
(a) Rs. 400 (b) Rs. 360 (c) Rs. 300
(d) Rs.600 (e) Rs. 500

27. If cost price of J₂ for shopkeeper is Rs. 1000. Find MRP of J₂ if he earns a
profit of 20% on selling a single piece.
(a) Rs. 1600 (b) Rs. 1800 (c) Rs. 1500
(d) Rs. 1700 (e) None of these
2
28. Find the cost price of S₂ if he earns a profit of 14 % on C.P.
7
(a) Rs. 1464 (b) Rs. 1098 (c) Rs. 1281
(d) Rs. 1379 (e) None of these

29. A boy buys 5 pieces of T₂, 4 pieces of J₁, & 1 piece of S₁ including offer.
Find the amount paid by him (in Rs.)
(a) 6000 (b) 7200 (c) 4800
(d) 3600 (e) 5400

30. If a person buys 4 pieces of T₂, 2 pieces of T₁ & 4 pieces of J₁ including


offer. Find overall discount% obtained by him.
(a) 40% (b) 44% (c) 42%
(d) 46% (e) 48%

Directions (31-35): Given below line graph shows the distance travelled
by car A in given time intervals and table shows ratio of average speed of
car A to average speed of car B in same time intervals. Both car started at
12:00 Noon.
90
80
70
60
50
40
30
12:00 – 1:00 1:00 – 2:00 2:00 – 3:00 3:00 – 4:00 4:00 – 5:00
15 Adda247 Publications For any detail, mail us at
Publications@adda247.com
Cracker Book for Bank (IBPS | SBI | RRB PO | Clerk) Mains Exams

Time Internal Ratio


12:00 – 1:00 6:5
1:00 – 2:00 14 : 11
2:00 – 3:00 6:7
3:00 – 4:00 4:3
4:00 – 5:00 4:7

31. What is average speed of car A during whole journey?


(a) 63 km/hr (b) 65 km/hr (c) 62 km/hr
(d) 70 km/hr (e) 68 km/hr

32. If both cars started from same point at same time, what is difference
between distance travelled by them till 3: 00 PM.
(a) 20 km (b) 15 km (c) 24 km
(d) 16 km (e) 12 km

33. Average speed of B between 2: 00 PM to 4: 00 PM is what % more or


less than average speed of A in same time interval?
1 2 4
(a) 7 % (b) 14 % (c) 28 %
7 7 7
5
(d) 35 % (e) None of these
7

34. If B have to cover a distance of 540 km till 9 : 00 PM, even after having a
rest of 45 minutes between 5 : 00 to 5 : 45 PM. What should be his
average speed for 4 hours between 5:00 PM to 9:00 PM.
1 4
(a) 60 km/hr (b) 59 km/hr (c) 58 km/hr
7 7
(d) 58 ¾ km/hr (e) 56 ⅔ km/hr

35. If A consumes 1liter petrol for every 15 km, and if A is 25% more
economic than B. What is the ratio of petrol consumed by them
till2:00PM?
(a) 91 : 92 (b) 93 : 97 (c) 104 : 105
(d) 106 : 107 (e) 103 : 106
16 Adda247 Publications For any detail, mail us at
Publications@adda247.com
Cracker Book for Bank (IBPS | SBI | RRB PO | Clerk) Mains Exams

Directions (36-40): The following line graph shows principal amount(in


10000) submitted by 5 persons in a scheme which offers simple interest
and table shows the rate of interest (ROI) per annum obtained by these
persons, time (in years) for which they invested their amount and third
column shows ratio of amount obtained after they withdraw their money to
that of interest obtained.
Some figures shown are missing. Calculate them according to question.

12
11
10
9
8
7
6
5
4
3
A B C D E

Persons ROI Time A:I


A — 5 8:3
B 16% — 41 : 16
C — 6 —
D 8% 5 —
E 15% — 29 : 9

36. What is the time period in months for which B invested his money?
(a) 60 (b) 48 (c) 64
(d) 72 (e) None of these

37. If the ratio of amount obtained by A to that of C is 4: 3. Find the ROI


obtained by C.
(a) 8% (b) 12% (c) 8.5%
(d) 10% (e) 12.5%
17 Adda247 Publications For any detail, mail us at
Publications@adda247.com
Cracker Book for Bank (IBPS | SBI | RRB PO | Clerk) Mains Exams

38. If a new person Vikas deposit half of the money invested by A, for 3
more years than C, calculate his ROI, if SI obtained by him is same as SI
obtained by D.
1 1 1
(a) 11 % (b) 9 % (c) 12 %
9 11 3
2
(d) 14 % (e) None of these
7

39. What is the ratio of time period for which E invested to that of B?
(a) 1 : 3 (b) 7 : 2 (c) 5 : 4
(d) 4 : 5 (e) 3 : 4

40. If after drawing his whole money, D invested 50% of this amount in CI
for 2 years at ROI/annum of 10% and remaining of the money he kept
with him. Calculate total money with him after 2 years.
(a) Rs. 169400 (b) Rs. 84700 (c) Rs. 154700
(d) Rs. 167400 (e) Can’t be determined

Directions (41-44): Bar-graph shows the percentage distribution of


distance covered in upstream and downstream by Rahul on different dates.

Upstream distance=6000 km Downstream distance=2000 km


32
28
24
20
16
12
8
4
0
1st Aug. 2nd Aug. 3rd Aug. 4th Aug. 5th Aug.

18 Adda247 Publications For any detail, mail us at


Publications@adda247.com
Cracker Book for Bank (IBPS | SBI | RRB PO | Clerk) Mains Exams

Table given below shows the speed of current in km/hr on different dates.

Date speed of current (in km/hr)


1st Aug. 10
2nd Aug. —
3rd Aug. 4
4th Aug. —
5th Aug. 3
1
41. If time taken by Rahul to swim upstream on 1st August. is 61 hr. more
3
than time taken by him to swim downstream on same date, then find
the speed of Rahul in still water?
(a) 16 km/hr (b)30 km/hr (c) 25 km/hr
(d) 20 km/hr (e) None of these

42. If speed of Rahul in still water on 2nd Aug. is 8 km/hr and speed of
Rahul in still water on 3rd Aug. is 25% more than his speed on 2nd Aug.
2
and time taken by him to travel upstream on 2 nd August is 166 % more
3
than time taken by him to travel downstream on same date, then find
the seven times of the speed of current on 2nd Aug.?
(a) 8 km/hr (b) 6 km/hr (c) 4 km/hr
(d)None of these (e) 2 km/hr

43. Speed of current on 4th Aug. is 50% of downstream speed of Rahul on


5th August, then time to cover 128 km upstream on 4th Aug. is ? [Given
that time of Rahul to cover upstream distance on 5th August is 80 hr and
assuming that speed of Rahul in still water is same on both days]
1 2
(a) 12 hr (b) 6 hr (c) 10 hr
3 3
(d) 21 hr (e)None of these

44. If ratio of speed of current on 2nd August to 4th August is 2 : 3 & speed of
Rahul on both days are same and time in upstream on 4th August. is 24
times the time taken in downstream on 2th August then find the speed
of Rahul in still water if speed of current on 3rd August is 50% of speed
of current on 2nd Aug.(approximately)?
(a) 21 km/h (b) 23km/h (c) None of these
(d) 25km/h (e) 20km/h
19 Adda247 Publications For any detail, mail us at
Publications@adda247.com
Cracker Book for Bank (IBPS | SBI | RRB PO | Clerk) Mains Exams

Directions (45-48): The following bar-graph shows the ratio of speed of


three different trains Rajdhani exp., Shatabadi exp. and Duranto exp. on five
different days of a week. And line graph shows the number of coaches
attached to Rajdhani express on different days of week. Length of each train
is the sum of the length of all coaches and length of engine.

Rajdhani Exp Shatabadi Exp Duranto Exp


7
6
5
4
3
2
1
0
Monday Tuesday Wednesday Friday Sunday

30

25

20

15

10
Monday Tuesday Wednesday Friday Sunday
Length of each coaches for every train = 15 m
And length of engine for every train = 20 m

20 Adda247 Publications For any detail, mail us at


Publications@adda247.com
Cracker Book for Bank (IBPS | SBI | RRB PO | Clerk) Mains Exams

45. On Tuesday, Rajdhani exp. crosses a tunnel of some length in 12.5 sec
while Duranto express having length 25% more than that of Rajdhani
exp. crosses the same tunnel on the same day in 11.9 sec. Then find the
time taken by Rajdhani exp. and Shatabadi exp. on the same day to
cross each other if they are running in opposite direction and the length
of Shatabadi exp. is 10 m less than that of Rajdhani exp.
(a) 8 sec (b) 9 sec (c) 10sec
(d) 11 sec (e) 12 sec

46. On Sunday, Duranto exp. having length 110 m less than that of Rajdhani
8
exp. crosses Rajdhani exp. travelling in opposite direction in 8 sec
9
then find the approximate time taken by Shatabadi exp. to cross
Duranto exp. on the same day when both are running in the same
direction and length of the Shatabadi exp. is 20% less than that of
Rajdhani exp.
(a) 79 sec (b) 81 sec (c) 75 sec
(d) 84 sec (e) 72 sec

47. On Friday, the speed of Rajdhani exp. was 90 km/hr and on the same
day, Shatabadi exp. started from station P and going to Q while Duranto
started from station Q and going to P. If speed of Shatabadi is increased
by 10% after first hour and 20% after second hour whereas the speed
of Duranto is increased by 20% after first hour and 25% after second
hour then they meet in 3 hours. Find the distance between P and Q.
(a) 947 km (b) 952 km (c) 955 km
(d) 957 km (e) 960 km

48. Duranto exp. having length 28% more than that of Rajdhani exp. on
Monday crosses a platform of same length in 12.8 sec. Then find the
difference between average speed of shatabadi and Rajdhani and the
speed of Duranto exp. on Monday.(in km/hr)
(a) 32.8 (b) 59.4 (c) 42.6
(d) 54.2 (e) 58.8
21 Adda247 Publications For any detail, mail us at
Publications@adda247.com
Cracker Book for Bank (IBPS | SBI | RRB PO | Clerk) Mains Exams

Directions (49-53): Pie chart given below shows length of six different
trains and table given below shows ratio between speed of six trains on
three different days. Study the data carefully and answer the following
questions.
Total length = 1600m

F, 8%
A, 16%

E, 22%
B, 18%

D, 12%

C, 24%

Train Speed on : Speed on : Speed on


Monday Tuesday Wednesday
A 2 : 3 : 2
B 3 : 4 : 5
C 4 : 6 : 5
D 4 : 4 : 7
E 6 : 9 : 5
F 4 : 5 : 3

49. On Wednesday, train ‘B’ crosses train ‘D’ coming from opposite
direction in 6 seconds. If speed of train ‘B’ on Monday is 97.2 km/hour
then in how much time train ‘F’ can cross train ‘D’ on Monday if train ‘D’
is coming from opposite direction and speed of train ‘F’ on Monday is
20 m/sec. (2 Marks)
(a) 6 seconds (b) 8 seconds (c) 10 seconds
(d) 12 seconds (e) 14 seconds
22 Adda247 Publications For any detail, mail us at
Publications@adda247.com
Cracker Book for Bank (IBPS | SBI | RRB PO | Clerk) Mains Exams

50. Train ‘C’ start from Delhi on Monday at 7:00 p.m and reach Kanpur on
next day at 3:00 pm. In return journey on Tuesday, train ‘C’ start from
Kanpur at 6 : 00 pm and reach Jaipur which is 180 km ahead of Delhi at
5 : 48 pm on Wednesday. Find the time taken by train ‘C’ to cross a pole
on Monday? (2 Marks)
(a) 8 seconds (b) 16 seconds (c) 19.2 seconds
(d) 14.4 seconds (e) 28.8 seconds

51. On Monday, train ‘A’ takes 2.5 hours more to cover 900 km distance
than train ‘C’. If train ‘A’ can cross a platform of length 128 in 12.8
seconds on Tuesday then find in how much time (in seconds) train ‘C’
can cross two poles 66 m apart from each other on Tuesday? (2 Marks)
(a) 12 seconds (b) 16 seconds (c) 20 seconds
(d) 24 seconds (e) 30 seconds
52. Ratio between speed of train ‘E’ to train ‘F” on Monday is 3 : 2. On
Tuesday train ‘E’ cross train ‘F’ running in same direction in 24 seconds
then find the time in which train ‘E’ can overtakes train ‘F’ on
Wednesday? (2 Marks)
(a) 48 seconds (b) 24 seconds (c) 12 seconds
(d) 36 seconds (e) 60 seconds

53. Ratio between time taken by train ‘B’ to train ‘D’ to cross a pole on
Monday is 1 : 1. The time taken by train ‘B’ to cross a pole on
Wednesday is what percent more/less than time taken by train ‘D’ to
cross a pole on Monday? (2 Marks)
(a) 30% (b) 40% (c) 50%
(d) 60% (e) 70%

Direction (54-58): Table given below shows profit percentage earned on


selling two different items X and Y and discount percentage offered by five
retailers on these items. Mark price of each article sold by each retailer is
same while cost price of article for each retailor may vary. Study the data
carefully & answer the following question

23 Adda247 Publications For any detail, mail us at


Publications@adda247.com
Cracker Book for Bank (IBPS | SBI | RRB PO | Clerk) Mains Exams

Items→ X Y
Retailor
Profit % Discount % Profit % Discount %

A 20% – – 15%
B – 26.5% 20% 32.5%
C 25% – 60% 24%
D 20% 34% – 37%
E 35% 46% – 28%

54. On article ‘X’, 28% discount is offered by ‘A’. If selling price of article ‘Y’
sold by ‘A’ is Rs 312 more than selling price of article ‘X’ sold by ‘A’ then
find profit percent earned by ‘A’ on selling article ‘Y’ given that average
of cost price of both article for ‘A’ is Rs 1520. (2 Marks)
(a) 18.5% (b) 22.5% (c) 27.5%
(d) 32.5% (e) 37.5%

55. Discount % on article ‘X’ offered by ‘C’ is 15% while profit % of article
1
‘Y’ sold by ‘E’ is 33 %. If difference between cost price of article ‘Y’ sold
3
by ‘B’ and ‘E’ together is Rs 216 more than cost price of article ‘X’ sold
by ‘C’ and ‘E’ together then find the cost price of article ‘Y’ sold by ‘C’? (2
Marks)
(a) Rs. 4940 (b) Rs. 3420 (c) Rs. 3800
(d) Rs. 4180 (e) Rs. 4560

56. Find the ratio between cost price of article ‘X’ for ‘B’ to cost price of
article ‘Y’ for ‘D’. If profit % of article ‘X’ sold by ‘B’ and profit % of
article ‘Y’ sold by ‘D’ is 68% and 20% respectively. (2 Marks)
(a) 2 : 3 (b) 5 : 6 (c) 1 : 2
(d) 5 : 8 (e) 5 : 7

57. Selling price of article ‘X’ sold by ‘E’ is same as cost price of article ‘Y’ for
‘E’. Find the profit % earned by ‘E’ on selling article ‘Y’? (1 Mark)
2
(a) 15% (b) 16 % (c) 25%
3
1 2
(d) 33 % (e) 41 %
3 3
24 Adda247 Publications For any detail, mail us at
Publications@adda247.com
Cracker Book for Bank (IBPS | SBI | RRB PO | Clerk) Mains Exams

58. Seller A marked article ‘X’ 100% above its cost price. If on selling both
article he earns total 25% profit, then find selling price of article ‘X’ sold
by ‘A’ is what percent less than cost price of article ‘Y’ for ‘A’. (2 Marks)
1 1
(a) 10% (b) 9 % (c) 8 %
11 3
(d) 20% (e) 15%

Directions (59-63): Line graph given below shows the distance between
Delhi to five different cities in kilometer and Table given below shows the
speed of five different cars in km/hr

3500

3000
Distance from Delhi (in km)

2500

2000

1500

1000

500

0
City A City B City C City D City E

Cars Speed (in kmph)


P 40
Q —
R 60
S —
T 75

NOTE: - Some data is missing you have to calculate according to question.

25 Adda247 Publications For any detail, mail us at


Publications@adda247.com
Cracker Book for Bank (IBPS | SBI | RRB PO | Clerk) Mains Exams

59. Time taken by car ‘P’ to travel from city ‘E’ to Delhi and then Delhi to
city ‘B’, is equal to the time taken by car ‘R’ to travel from Delhi to city
‘A’ and then city ‘A’ to city ‘B’. Find the distance between city ‘A’ and city
‘B’.
(a) 5650 km (b) 5750 km (c) 5450 km
(d) 5550 km (e) 5320 km

60. Find the approximate time car ‘T’ takes to reach city ‘E’ from city ‘A’ if
city ‘A’ and city ‘E’ is north and east direction of Delhi respectively.
(a) 24 hours (b) 27 hours (c) 20 hours
(d) 36 hours (e) 42 hours

61. Car Q and Car S start from Delhi for city B and city C respectively and
they reached in equal time. If Car Q and Car S starts from city B and city
D respectively at same time and move towards each other, then time
taken by car Q to cross car S is what percent of the time taken by car Q
to reach city B from Delhi. Distance between city B and city D is 1500
km.
(a) 25% (b) 20% (c) 30%
(d) 40% (e) 50%

62. A thief runs in a car S from Delhi to city E and after 6 hours of running,
a policeman started to catch him in a car R. Due to this, thief increases
the speed of his car by 100%. By this, the policeman is able to catch him
3
at th of the distance of city E from Delhi. Find the initial speed of car ‘S’.
5
(a) 15 km/hr (b) 27 km/hr (c) 20 km/hr
(d) 25 km/hr (e) 40 km/hr

63. Car P and Car Q start from Delhi for city A. Car Q first reaches at city A
and meets car P in between the way, 200 km from city ‘A’. Find after
how much time they will meet second time after first time meeting if
they continue their to and fro motion.
(a) 24 hours (b) 15 hours (c) 16 hours
(d) 25 hours (e) 20 hours

26 Adda247 Publications For any detail, mail us at


Publications@adda247.com
Cracker Book for Bank (IBPS | SBI | RRB PO | Clerk) Mains Exams

Direction (64-68): - Line chart given below shows time taken by five
different persons to complete a work ‘M’ alone. Ratio of efficiency of all five
persons remain same throughout any work. Study the data carefully and
answer the following questions.

70
60
50
40
30
20
10
0
Abhi Bhavya Neeraj Satish Veer

64. All five starts working together to complete work ‘X’. ‘Veer’ left after 8
days. Work done by ‘Bhavya’ is same as work done by ‘Neeraj’ while
‘Abhi’ and ‘Neeraj’ worked for same time. ‘Satish’ worked for ‘y’ days. If
‘Bhavya’, ‘Neeraj’ and ‘Satish’ together can complete work ‘X’ in 24 days
then find the value of ‘y’ if Bhavya worked for starting 10 days.
(a) 7 days (b) 9 days (c) 11 days
(d) 13 days (e) 15 days

65. Abhi and Neeraj together can complete work ‘Z’ in (A + 42) days while
Bhavya and Satish together can complete work ‘Z’ in (A + 15) days. All
start the work Z such that ratio between work done by Abhi, Bhavya
and Veer is 1 : 2 : 3, while ratio between days, Neeraj, Satish and Veer
worked is 2 : 2 : 1. Find how many days ‘Bhavya’ worked.
(a) 10 days (b) 15 days (c) 20 days
(d) 30 days (e) 40 days
27 Adda247 Publications For any detail, mail us at
Publications@adda247.com
Cracker Book for Bank (IBPS | SBI | RRB PO | Clerk) Mains Exams

66. All five persons started together to complete work ‘Y’. Veer worked for
starting 6 days and left the work. After 3 days more both Bhavya and
Satish left too. Remaining 40% work should be completed by Abhi and
Neeraj together but ‘Abhi’ left after ‘x’ days. Remaining work is
completed by ‘Neeraj’ in ‘z’ days. If ‘z – x = 3’, then number of days for
which ‘Neeraj’ worked is what percent more than number of days for
which ‘Abhi’ worked.
1 2
(a) 33 % (b) 50% (c) 66 %
3 3
(d) 75% (e) 100%

67. Abhi, Bhavya and Neeraj together starts to do work ‘M’. After 7 days
‘Neeraj’ left and after 3 days more ‘Abhi’ and ‘Bhavya’ left. Remaining
work is completed by Satish and Veer working alternatively in ‘y’ days.
If ‘y’ is integer then find ‘Veer’ worked for how many days?
(a) 3 days (b) 4 days (c) 5 days
(d) 6 days (e) Cannot be determined

68. Abhi, Bhavya and Satish starts working together to complete work ‘M’.
After 5 days, Bhavya and Satish replaced by Neeraj and Veer. After 5
more days Abhi left the work. After 1 more day Veer left too. Neeraj
worked for total ‘x’ days. In other case Abhi and Bhavya starts working
together to complete ‘M’. After 4 days both are replaced by Veer. Veer
worked for 5 days and replaced by Satish who worked for 8 days.
Remaining work is completed by Neeraj in ‘y’ days. Find (y − x)2 ?
(a) 25 (b) 36 (c) 49
(d) 64 (e) 81

28 Adda247 Publications For any detail, mail us at


Publications@adda247.com
Cracker Book for Bank (IBPS | SBI | RRB PO | Clerk) Mains Exams

Solutions

Solutions (1-4):
Total work = 5040 units (LCM of days taken by all)
5040
Efficiency of Neeraj = = 144 units/hour
35
5040
Efficiency of Aniket = = 168 units/hour
30
5040
Efficiency of Saurabh = = 112 units/hour
45
5040
Efficiency of Gopal = = 315 units/hour
16
5040
Efficiency of Veer = = 210 units/hour
24
5040
Efficiency of Sameer = = 280 units/hour
18
7
1. (a); New efficiency of Neeraj = 144 ×
8
= 126 units/hour
2
New efficiency of Gopal = 315 ×
3
= 210 units/hour
5040 1
Required time = = 11 hours
(126+112+210) 4
3
2. (c); New efficiency of Sameer = 280 ×
4
= 210 units/hour
ATQ –
(210+210 )(y) 2
=
168(y+1) 1
420y = 336y + 336
420y – 336y = 336
y = 4 hour
Total work = 420 × 4 + 168 × 5
= 2520 units
2520 1
Neeraj will complete alone in = = 17 hours
144 2

29 Adda247 Publications For any detail, mail us at


Publications@adda247.com
Cracker Book for Bank (IBPS | SBI | RRB PO | Clerk) Mains Exams

3. (d); Divyaraj work in one hour = 144 × 1.5


= 216 units
75
Manish work in one hour = 168 ×
100
= 126 units
Let Saurabh work for n hours and Divyaraj & Manish work for (n +
11.5) hours
ATQ –
N × 112 + (216 + 126)(n + 11.75)
= 5040
112n + 342n = 5040 – 4018.5
1021.5
n=
454
n = 2.25 hours
Total time = (2.25 + 2.25 + 11.75 )
1
= 16 hours
4

4. (d); Total work completed by Neeraj, Aniket, Veer and Sameer together
in one hour
= (144 + 168 + 210 + 280) = 802 units
In Second hour total work destroyed by Saurabh & Gopal together
= −(315 + 112) = − (427)
Total work in 2 hours = 802 – 427
= 375 units
24
Total work in 24 hours = 375 ×
2
= 4500 unit
On 25th hour remaining work by Neeraj, Aniket, Veer and Sameer
together
5040 −4500 270
= = hours
802 401
270
Required time = 24 hours
401

5. (a); Total marbels in bag Y = 20


7C 7×6 21
Required probability = 20 2 = =
C2 20×19 190

30 Adda247 Publications For any detail, mail us at


Publications@adda247.com
Cracker Book for Bank (IBPS | SBI | RRB PO | Clerk) Mains Exams

6. (d); Total marbels in bag X = 18


6C 6×5 5
Required probability = 18 2 = =
C2 18×17 51

7. (c); Total marbels in bag y = 20


2C × 7C 7
1 1
Required probability = 20C =
2 95

8. (e); Total marbels in bag X = 18


3C 4C
Required probability = 18 2 + 18 2
C2 C2
3 4×3 9 1
= + = =
9×17 18×17 153 17

9. (b); Total marbels in bag Y = 20


7C × 7C × 4C 49
1 1 1
Required probability = 20C
=
3 285

10. (b); Quantity of tank P and T together


(15+15)
= 1500 × = 450 ℓ
100
Efficiency of pipe B and D
12
1500×
Pipe B = 100
= 4.5 ℓ⁄m
40
20
1500×
Pipe D = 100
= 7.5 ℓ⁄m
40
Efficiency of pipe C and E
8
1500×
Pipe C = 100
= 4 ℓ⁄m
30
15
1500 
Pipe E  100  5l / m
45
According to question
= 12X + 9(X + 15) = 450
= 21X = 450 – 135
315
=X= = 15 minutes
21
Liter of water filled by pipe B and D together
= 15 (4.5 + 7.5) = 15 × 12 = 180ℓ
31 Adda247 Publications For any detail, mail us at
Publications@adda247.com
Cracker Book for Bank (IBPS | SBI | RRB PO | Clerk) Mains Exams

11. (c); Quantity of tank S


20
= 1500 × = 300ℓ
100
New efficiency of Pipe D
300 6
= × = 9 ℓ⁄m
40 5
Efficiency of pipe C and G
8 12
1500×100 1500×100
= × 125 +
30 30
= 5 + 6 = 11 l/ m
According to question = 9X + 11 × 15 = 300
= 9X = 300 – 165 = 9X = 135
135
X= = 15 minutes
9

12. (a); Given, total quantity of all seven tank


= 1500ℓ
Quantity of bigger tank K
120
= 1500 × = 1800ℓtr
100
Efficiency of pipe B, D and G together
12 20 12
1500× 1500× 1500×
100 100 100
= + +
40 40 30
= 4.5 + 7.5 + 6 = 18 ℓ/m
15
1500×
Efficiency of pipe A = 100
= 9 ℓ⁄m
25
Efficiency of Pipe F and C
18 8
1500× 1500×
100 100
= +
60 30
= 4.5 + 4 = 8.5 ℓ/m
According to question
Tank filled in three minutes alternatively
= (18 + 9 + 8.5) = 35.5 litre
In (50 × 3) minutes = 35.5 × 50 = 1775 litre
Next by (B + D + G) = 18 ℓtr/m
After 151 minutes remain quantity of bigger tank
= 1800 – (1775 + 18) = 7 ℓtr
7
Tank K, total filled in = 151 +
9
7
= 151 minutes
9

32 Adda247 Publications For any detail, mail us at


Publications@adda247.com
Cracker Book for Bank (IBPS | SBI | RRB PO | Clerk) Mains Exams

13. (d); Total quantity of tank M = (total quantity of tank S + U together)


20 18
= 1500 × + 1500 ×
100 100
= 300 + 270 = 570ℓ
Efficiency of pipe A, C and G together
15 8 12
1500×100 1500×100 1500×100
= + +
25 30 30
= 9 + 4 + 6 = 19 ℓ/m
(A + C + G) filled tank M together in
570
= = 30 minutes
19
But according to question —
17
tank 7 minutes more due to leak
19
leak empty quantity of water in 30 minutes
17
= (A + C + G) filled in 7 minutes
19
17 150
= 19 × 7 = 19 × = 150 ℓtr
19 19
150
Efficiency of leak = = 3 ℓ⁄m
30
Leak empty tank M in
570
= = 190 minutes
3
1
= 3 hr
6

14. (b); Given, total quantity of tank = 570 ℓ


Efficiency of pipe B, D and E together
12 20 8
1500× 1500× 1500× 175
100 100 100
= + + ×
40 40 30 100
= 4.5 + 7.5 + 7 = 19 ℓ/m
Leak at a height of 2/3 from the bottom of tank
2
= 570 × = 380 ℓtr
3
380 ℓ filled without leak
380
= = 20 minutes
19
Given, man notice after 32 minutes
Tank filled in 32 minutes = (20 + 12) minutes
1
= 380 + 12 × (19 – 19 × ) = 380 + 114 = 494
2
Remain = 570 – 494 = 76 ℓ

33 Adda247 Publications For any detail, mail us at


Publications@adda247.com
Cracker Book for Bank (IBPS | SBI | RRB PO | Clerk) Mains Exams

15. (b);
Q S P
75
32000 : 24000 : 24000 ×
100
16 : 12 : 9
Let total profit = 100X
12
Q extra profit share = 100X × = 12X
100
15
S extra profit share = 100X × = 15X
100
Remaining profit = 100X – (12X + 15X)
= 73X
73X
Share of P = (16+12+9) × 9 = 65700
657X
= = 65700
37
65700×37
X= = 3700
657
Total profit = 3700 × 100 = 370000 Rs.

16. (e); Ratio of investment by Q & R = 4 : 3


⇒ 4x : 3x
Ratio of time period = 5 : 3 ⇒ 5y : 3y
Ratio of profit share between Q & R
= 4x × 5y ∶ 3x × 3y = 20xy ∶ 9xy
= 20a ∶ 9a [say]
ATQ,
20a – 9a = 100%
10010
a= = 910
11
Now 35% profit = 29a = 29 × 910
65
65% profit = 29 × 910 ×
35
65 29×910×13
% profit = = 24505
2 2×7
R’s total profit share = 24505 + 9a
= 24505 + 8190 = 32695

34 Adda247 Publications For any detail, mail us at


Publications@adda247.com
Cracker Book for Bank (IBPS | SBI | RRB PO | Clerk) Mains Exams

17. (a); Lets S joined the business after X months


According to question
32000×36 3 36
= ⇒ x = = 4 months
24000×(36 –X) 2 9
18. (b); Profit share of R = 30500 – 24300 = 16200 Rs.
Lets R invested X Rs. for 12 months
ATQ -
81000×9 24300 81000×3 3
= = =
X×12 16200 4X 2
X = 40500 Rs.

19. (c); Q : R = 8 : 9
R:S=3:2
Q:R:S=8:9:6
Total profit = 37030
Q’s share
8
× 37030 = 12880
23
R’s share
9
× 37030 = 14490
23
S’s share
6
× 37030 = 9660
23
40
20. (c); Investment of Saurabh = 40,000 × = 16,000
100
30 75
Investment of Neeraj = 40,000 × × = 9,000
100 100
Ratio of profit share of Neeraj to Saurabh

8000 8000
Saurabh’s profit = × 101 = × 101 = 5050
59+101 160

35 Adda247 Publications For any detail, mail us at


Publications@adda247.com
Cracker Book for Bank (IBPS | SBI | RRB PO | Clerk) Mains Exams
30
21. (a); Investment of Rakesh = 40,000 × = 12,000
100
Investment of Aman
30 75 100
= 40,000 × × × = 6,000
100 100 150
Ratio of profit share of Aman to Rakesh
Aman : Rakesh
6000  8  8000  2  10, 000  2 : 12000(12  x)
84000 : 12000(12  x)
7 : (12  x)
ATQ,
7 7
= ⇒ x = 4 months
12−x 8
Rakesh worked for (12-x) = 12 -4
= 8 months

30
22. (e); Investment of Sandeep = 40000 × = 12000
100
30 75
Investment of Neeraj = 40000 × × = 9000
100 100
40
Investment of Satish = 40000 × + 4000 = 20000
100
Ratio of profit share of Sandeep, Neeraj and Satish

11500
Sandeep’s profit share = (12+6+5) × 12 = Rs 6000

23. (b); Investment of Aman


30 75 100
= 40,000 × × × = 6,000
100 100 150
40
Investment of Saurabh = 40,000 × = 16,000
100
Total amount they have after 2 years
120 120 80 80
= 6000 × × + 16000 × ×
100 100 100 100
= 8,640 + 10,240 = 18,880

36 Adda247 Publications For any detail, mail us at


Publications@adda247.com
Cracker Book for Bank (IBPS | SBI | RRB PO | Clerk) Mains Exams
50
24. (b); Total quantity of in vessel P = 240 × = 120 ml
100
Quantity of Mango juice in vessel P
93.75
= 192 × = 180 ml.
100
150
Quantity of water in vessel S = 160 ×
100
= 240 ml
Quantity of Orange juice in vessel S
= 240 – 40 = 200 ml
Ratio of (Milk : Water : Orange juice : Mango juice) in vessel P
= 240 : 120 : 144 : 180 = 20 : 10 : 12 : 15
Ratio of (milk : Water : Orange juice : Mango juice) in vessel S
= 160 : 240 : 200 : 120 = 4 : 6 : 5 : 3
Total orange juice in vessel T
12 5
= 114 × (20+10+12+15) + 180 × (4+6+5+3)
= 24 + 50 = 74 ml
74
Required % = (114+180) × 100
74 25
= × 100 = 25 %
294 147

2
25. (c); Quantity of milk in vessel R = 144 × = 96 ml
3
Quantity of Orange juice in vessel Q
3
= 160 × = 120 ml
4
Total solution in vessel Q
= (160 + 80 + 120 + 210) = 570 ml
Total solution in vessel R = 1086 – 570 = 516 ml
Let mango juice and total solution in vessel R be 8n and 43 n
respectively
ATQ—
96 + 144 + 192 + 7n = 43n
36n = 432
n = 12
Total mango juice in vessel R = 12 × 7 = 84 ml
210+84 294 49
Required ratio = = =
1086 1086 181

37 Adda247 Publications For any detail, mail us at


Publications@adda247.com
Cracker Book for Bank (IBPS | SBI | RRB PO | Clerk) Mains Exams

26. (c); Minimum average is obtained when he will buy 2T₂ t- shirts.
Price for 2 shirts = 600 × 2 = Rs. 1200
He will get 4 t-shirts in this price
1200
Price/t-shirt = = Rs. 300
4
NOTE: If he buys T₁ t-shirt, price will be
90
400 × = Rs. 360
100

27. (a); 20% profit means he is selling it in


20
1000 (1 + ) = Rs. 1200
100
This amount is obtained after 25% discount
4
Hence MRP = 1200 × = Rs. 1600
3

1
11 9 8
28. (c); SP of S₂ = 1647 [1– ] = 1647 [ ]
100 9

= 183 × 8 = Rs. 1464


ATQ,
2
14
7
CP [1 + ] = Rs. 1464
100
7
CP = 1464 × = Rs. 1281
8

29. (e); Price paid for 5 pieces of T₂ is actually price paid for
2 pieces of T₂ + 1 piece of T₂.
As he will get 2 – t-shirts (T₂) free after buying two = 2 × 600 + 600
= 1800
Similarly price for 4 pieces of J₁ = 2 × 1200
1 piece of S₁ = 1 × 1200 = Rs. 1200
Total = Rs. 1800 + Rs. 2400 + Rs. 1200 = Rs. 5400

30. (d); Actual price of 4 pieces of T₂ = 4 × 600 = 2400 Rs.


Actual price of 2 pieces of T₁ = 2 × 400 = 800 rs.
Actual price of 4 pieces of J₁ = 4 × 1200 = Rs 4800
Total actual price is Rs (2400 + 800 + 4800) = Rs 8000
Price paid for 4 pieces of T₂ = 2 × 600 = Rs. 1200
38 Adda247 Publications For any detail, mail us at
Publications@adda247.com
Cracker Book for Bank (IBPS | SBI | RRB PO | Clerk) Mains Exams

90
For 2 pieces of T₁ = 400 × ( ) × 2 = Rs. 720
100
For 4 pieces of J₁ = 2 × 1200 = Rs. 2400
Discount amount obtained
= (8000 – 4320) Rs. = 3680
3680
D% = × 100 = 46%
8000

Total distance
31. (c); Average speed =
Total time
60+70+60+80+40 310
= = = 62 km/hr
5 5

32. (b); Distance travelled by A till 3 PM is = 60 + 70 + 60 = 190 km


Now note that if A is travelling a distance of 60 km in 1 hour, then
its speed is also 60 km/hr.
Also, we know that ratio of distance travelled by two objects in
same time is same as ratio of their speed.
D S
∴ A= A
DB SB
Distance travelled by B between 12:00 to 1:00 PM
60 6
= ⇒ DB =50 km
DB 5
Similarly distance travelled by B between 1:00 PM – 2:00 PM
70 14
= ⇒ DB = 55 km
DB 11
Distance travelled between 2:00 – 3:00 PM
60 6
= = DB = 70 km
DB 7
Required difference = 190 – [50 + 55 + 70]
= 15 km

33. (a); Average speed of A between 2:00 to 4:00 pm


60+80
= = 70 km/hr
2
Average speed of B between 2:00 to 3:00 pm
7
= × 60=70 km/hr
6
Similarly between 3:00 to 4:00 pm
3
= 80 × = 60 km/hr
4

39 Adda247 Publications For any detail, mail us at


Publications@adda247.com
Cracker Book for Bank (IBPS | SBI | RRB PO | Clerk) Mains Exams

Average speed of B between 2:00 to 4:00 pm


60+70
= = 65 km/hr
2
70–65
Required answer = ( ) × 100
70
1 1
= × 100 = 7 %
14 7

34. (d); Distance covered by B till 5:00 PM is


5 11 7 3 7
(60 × ) + (70 × ) + (60 × ) + (80 × ) + (40 × )
6 14 6 4 4
= 50 + 55 + 70 + 60 + 70 = 305 km
Remaining distance = 540 – 305 = 235 km
235
Required average = = 58¾ km/hr
4

35. (c); A consumes 1 litre/15 km


100
B consume 15 ×
125
4
15 × = 1 litre/12 km
5
Distance travelled by A till 2:00 PM = 60 + 70 = 130 km
130 26
Petrol consumed = = litre
15 3
Distance travelled by B till 2:00 PM = 50 + 55 = 105 km
105 35
Petrol consumed = = litre
12 4
26 35
Required Ratio = ∶
3 4
⇒ 104: 105

36. (b); Let the amount obtained by B is 41x and S.I. obtained is 16x.
Therefore, principal is 41x – 16x = 25x = Rs75000
⇒ x = 3000/–
Therefore, SI obtained 16x = 16 × 3000 = 4800/-
Using formulae
PRT 48000×100
= 48000 ⇒ T =
100 16×75000
= 4 years = 48 months
40 Adda247 Publications For any detail, mail us at
Publications@adda247.com
Cracker Book for Bank (IBPS | SBI | RRB PO | Clerk) Mains Exams

37. (d); If we assume amount of A 8x and SI obtained 3x, then we can


calculate principal of A (5x) = Rs. 80000
x = 16000
and amount obtained by A = 16000 × 8 = 128000
3
∴ amount obtained by C = × 128000 = Rs. 96000
4
Let ROI obtained by C is m%/annum and from line graph, we know
that Principal submitted by him is Rs 60000.
Then,
60000(m)×6
= Rs. (96000 – 60000) = 36000
100
m = 10%

38. (a); Time for which Vikas invested is 6 + 3 = 9 years.


80000
Sum deposit by him = = Rs. 40000
2
100000×8×5
SI obtained by D = = Rs. 40000
100
40000×R×9
∴ ROI of Vikas = = 40000
100
100 1
R= = 11 %
9 9

39. (e); Let amount obtained by E is 29y & SI obtained is 9y


∴ 29y – 9y = 20y = 50000
[∵ 50000 is principal]
⇒ y = 2500
Interest obtained = 2500 × 9 = Rs. 22500
50000×15%×T
∴ = 22500
100
22500
T= = 3 years.
500×15
Similarly,
For B principal is Rs. 75000
(41–16) = 25 units = Rs. 75000
75000
⇒ 16 units (SI) = × 16 = Rs. 48000
25
∴ time for which B invested
75000×16×T
= 48000
100
48×100
T= = 4%
75×16
Required ratio = 3 : 4
41 Adda247 Publications For any detail, mail us at
Publications@adda247.com
Cracker Book for Bank (IBPS | SBI | RRB PO | Clerk) Mains Exams

40. (c); Amount obtained by D after drawing his whole money


100000×8×5
= 100000 + = Rs. 140000
100
Now he invested 50% of this = 70000
Amount obtained after two years from this
110 2
= 70000 × ( ) = Rs. 84700
100
Total amount with him = Rs. 84700 + Rs.70000
= Rs. 154700
41. (d); Upstream distance travelled on 1st Aug.
12
= × 6000 = 720km.
100
Downstream distance travelled on 1st Aug.
16
= × 3000 = 320 km
100
Let speed of Rahul in still water be x km/hr.
ATQ.
720 320 184
= = +
x−10 x+10 3
On solving
x = 20km/hr.
∴ speed of Rahul in still water = 20Km/hr.

42. (a); Speed of Rahul in still water on 3rd Aug.


125
= × 8 = 10Km/hr.
100
ATQ.
8 12
×600 8 ×2000
100 100
= ×
8−x 3 8+x
Solving we get
x = 8/7 km/hr.
∴ seven times Speed of Current = 8 Km/hr

43. (c); Let speed of Rahul in still water be x Km/hr.


32
100
×6000
∴ 80 = ⇒ x = 27Km/hr.
x−3
Downstream speed on 5 August =27 + 3 = 30 km/h
Speed of current on 4th Aug.
50
= (27+3) × = 15 Km/hr.
100
Time to cover 128 Km up stream on 4th Aug.
128 2
= = 10 hr.
27−15 3
42 Adda247 Publications For any detail, mail us at
Publications@adda247.com
Cracker Book for Bank (IBPS | SBI | RRB PO | Clerk) Mains Exams

44. (e); Let speed of current on 2nd Aug. & 4th Aug. be 2x km/hr & 3xkm/hr.
Let speed Rahul in still water be y km/hr.
ATQ.
If speed of current on 3 August is 4 km/h then
2x = 8
Therefore
3x = 12
Now,
12 28
100
×2000 100
×6000
24× = 2x ×
y+8 y−12
7y + 56 = 24y − 288
y = 20.22 km/h
Approximately 20km/h
45. (c); On Tuesday,
Length of Rajdhani exp → 24 × 15 + 20
= 380 m
Let the speed of Rajdhani exp on Tuesday be 8 x m/s
Speed of shatabadi exp = 7x m/s
Speed of duranto exp = 10 x m/s
Let the length of tunnel be y m
ATQ,
380+y
= 12.5
8x
⇒ 380 + y = 100x
⇒ 100x – y = 380 … (i)
125
And, length of Duranto exp = 380 ×
100
= 475 m
475+y
= 11.9
10x
⇒ 475 +y = 119x
⇒ 119x – y = 475 … (ii)
From (i) and (ii)
x = 5 m/s
380+370 750
Required time = = = 10 sec
15×5 75

43 Adda247 Publications For any detail, mail us at


Publications@adda247.com
Cracker Book for Bank (IBPS | SBI | RRB PO | Clerk) Mains Exams

46. (a); Length of Rajdhani on Sunday


= 29 × 15 + 20 = 455m
Length of Duranto exp = 345 m
Let the speed of Rajdhani, shatabadi and Duranto exp be 6x m/s, 5x
m/s and 4x m/s respectively.
ATQ,
455+345 80
= ⇒ x = 9 m/s
10x 9
80
Length of Shatabadi = 455× = 364 m
100
364+345
Required time = ≈ 79 sec
9

47. (d); Speed of Shatabadi exp on Friday


= 150 km/hr
Speed of Duranto exp on Friday
= 120 km/hr
Distance travelled by Shatabadi exp in 3 hours
= 150 + 150 × 1.10 + 150 × 1.1 × 1.2
= 513 km
Distance travelled by Duranto exp in 3 hours
= 120 + 120 × 1.2 + 120 × 1.2 × 1.25
= 444 km
Total distance between P and Q
= 513 + 444 = 957 km
48. (b); On Monday,
Length of Rajdhani exp. = 15 × 17 + 20
= 275 m
Length of Duranto exp. = 275 × 1.28
= 352 m
Let the speed of Rajdhani, shatabadi and Duranto
exp. on Monday be 3x m/s, 4x m/s, 5x m/s respectively.
ATQ,
352+352
= 12.8 ⇒ 704 = 64x ⇒ x = 11
5x
7x 3 18
Required difference = 5x − = ×11×
2 2 5
= 59.4 km/hr

44 Adda247 Publications For any detail, mail us at


Publications@adda247.com
Cracker Book for Bank (IBPS | SBI | RRB PO | Clerk) Mains Exams
18
49. (b); Length of train B = × 1600 = 288m
100
12
Length of Train D = × 1600 = 192m
100
8
Length of Train F = × 1600 = 128m
100
5
Speed of train B on Monday = 97.2 ×
18
= 27 m/sec
Speed of train ‘B’ on Wednesday
27
= × 5 = 45 m/sec
3
ATQ,
288 + 192 = (45 +y)×6
where y is the speed of train ‘D’ on Wednesday
⇒ y = 80 – 45 = 35 m/sec
35
Speed of train ‘D’ on Monday = ×4
7
= 20 m/sec
192+128
Time required to cross train F =
20+20
= 8 seconds

50. (e); Let speed of train ‘C’ on Monday, Tuesday and Wednesday be 4x, 6x
and 5x respectively.
Train ‘C’ travel 5 hours on Monday and 15 hours on Tuesday.
∴ Total distance = 5 × 4x + 15 × 6x = 110x
On the same day i.e, Tuesday, train ‘C’ start from Kanpur. It travels
6 hours on Tuesday and 17.8 hours on Wednesday.
∴ total distance travel = 6 × 6x + 17.8 × 5x
= 36x + 89x = 125x
ATQ, 125x = 110x + 180
⇒ 15x = 180 ⇒ x = 12
speed of train ‘C’ on Monday
40
= 12 × 4 = 48 km/hour = m/sec
3
24
Length of train ‘C’ = × 1600 = 384
100
384
Required time = × 3 = 28.8 sec
40

45 Adda247 Publications For any detail, mail us at


Publications@adda247.com
Cracker Book for Bank (IBPS | SBI | RRB PO | Clerk) Mains Exams

51. (a); Let, speed of train ‘A’ and train ‘C’ on Monday be ‘4x’ and ‘4y’
respectively
ATQ,
900 900
2.5 = −
4x 4y
1 1
2.5 = 225 [ − ]
x y
xy = 90 (y – x)
16
length of train ‘A’ = × 1600 = 256
100
256+128
speed of train ‘A’ on Tuesday =
12.8
384
= = 30 m/sec
12.8
⇒ Speed of train ‘A’ on Monday
30
= × 2 = 20 m/sec = 72 km/hr
3
⇒ 4x = 72 ⇒ x = 18
xy = 90(y – x)
y = 5(y – 18)
⇒ y = 22.5
Speed of train ‘C’ on Monday = 4y
= 4 × 22.5 = 90 km/hr
90
Speed of train ‘C’ on Tuesday = × 6
4
= 135 km/hr = 37.5 m/sec
24
Length of train ‘C’ = × 1600 = 384
100
384+66
Required time = = 12 seconds
37.5

22
52. (a); Length of train ‘E’ = × 1600 = 352
100
8
Length of train ‘F’ = × 1600 = 128
100
Let speed of train ‘E’ and train ‘F’ on Monday be 6x and 4y
respectively.
6x 3 x 1
⇒ = ⇒ =
4y 2 y 1
Let speed of train ‘E’ on Tuesday = 9x
So speed of train ‘F’ on Tuesday = 5y = 5x
ATQ,
46 Adda247 Publications For any detail, mail us at
Publications@adda247.com
Cracker Book for Bank (IBPS | SBI | RRB PO | Clerk) Mains Exams
352+128
9x – 5x = = 20 ⇒ 4x = 20
24
⇒x=5
Speed of train ‘E’ on Wednesday = 5 × 5
= 25 m/sec
Speed of train ‘F’ on Wednesday = 3 × 5
= 15 m/sec
352+128 480
Required time = =
25−15 10
= 48 seconds

53. (b); Let, speed of train ‘B’ on Monday, Tuesday & Wednesday be 3x, 4x
& 5x respectively.
And speed of train ‘D’ on Monday, Tuesday & Wednesday be 4y. 4y
& 7y respectively.
18
Length of train ‘B’ = × 1600 = 288
100
12
Length of train ‘D’ = × 1600 = 192
100
ATQ,
288
3x 1 3 4y 1
192 = ⇒ × =
1 2 3x 1
4y
y 1
⇒ = ⇒ x = 2y
x 2
288 57.6
Time taken by train ‘B’ on Wednesday to cross pole = =
5x x
192 96
Time taken by train ‘D’ on Monday to cross a pole = =
4y x
96 57.6
( − )×100 38.4
x x
Required % = 96 = × 100 = 40%
96
x

54. (c); Let, marked price of both articles be 600x


72
S.P. of article X = 600x × = 432x
100
85
S.P. of article Y = 600x × = 510x
100
ATQ,
510x − 432x = 312 ⇒ 78x = 312
⇒x=4

47 Adda247 Publications For any detail, mail us at


Publications@adda247.com
Cracker Book for Bank (IBPS | SBI | RRB PO | Clerk) Mains Exams
432×4
Cost price of article ′X ′ = × 100 = 1440
120
Cost price of article ‘Y’ = 1520 × 2 – 1440
= 3040 – 1440 = 1600
Profit % earned on selling article Y
510×4−1600 440
= × 100 = × 100 = 27.5%
1600 1600

55. (e); Let M.P. of each article sold by each seller be 800x
800x×67.5
Cost price of article Y sold by B =
120
= 450x
Cost price of article Y sold by E
800x×72
= × 3 = 432x
4×100
800x×85
Cost price of article X sold by C =
125
= 544x
800x×54
Cost price of article X sold by E =
135
= 320x
ATQ,
(450x + 432x) – 544x – 320x = 216
882x – 864x = 216
216
⇒x= = 12
18
M.P. of each article = 800 × 12 = 9600
9600×76
Cost price of article Y sold by C =
160
= Rs 4560

56. (b); Let M.P. of each article be 400x


400x×73.5
Cost price of article X sold by B = = 175x
168
400x×63
Cost price of article Y sold by D = = 210x
120
175x 5
Required ratio = =
210x 6

48 Adda247 Publications For any detail, mail us at


Publications@adda247.com
Cracker Book for Bank (IBPS | SBI | RRB PO | Clerk) Mains Exams

57. (d); Let M.P. of each article = 400x


400x×54
S.P. of article X sold by E = = 216x
100
Cost price of article Y sold by E = 216x
400x×72
Selling price of article Y sold by E =
100
= 288x
288x−216x
Profit % = × 100
216x
72x 1
= × 100 = 33 %
216x 3

58. (b); Let, M.P. of each article be 400x


400x
C.P. of article X sold by A = = 200x
2
120
S.P of article X sold by A = 200x ×
100
= 240x
85
SP of article Y sold of A = 400x ×
100
= 340x
Let, CP of article Y sold by A = y
ATQ,
125
(200x + y) × = 240x + 340x
100
580x
(200x + y) = × 4 ⇒ y = 264x
5
264x−240x 1
Required % = × 100 = 9 %
264x 11

59. (b); Distance travel by car P = 1500 + 3000


= 4500 km
4500
Total Time taken = = 112.5 hour
40
Time taken by car R from Delhi to City A
1000 50
= = hours
60 3
50 287.5
Time taken from city A to city B = 112.5– =
3 3
Distance from between City A to city B
287.5
= × 60 = 5750 km
3

49 Adda247 Publications For any detail, mail us at


Publications@adda247.com
Cracker Book for Bank (IBPS | SBI | RRB PO | Clerk) Mains Exams

60. (a);

Distance between city A and city E


= √10002 + 15002
= √1000000 + 2250000
= √3250000 = 500√13km
Approximate time taken by car ‘T’
500√13
= ≈ 24 hours
75

61. (c); Let speeds of car Q and car S be x and y respectively.


ATQ—
3000 2000 x 3
⇒ = ⇒ =
x y y 2
Let speed of car Q and car S be 3a and 2a respectively
Distance between city B and city D
= 1500 km
1500 300
Time taken to cross each other = =
5a a
3000 1000
Time taken by car Q to reach city B from Delhi = =
3a a
300×100
Required% = = 30%
1000

1500×3
62. (d); = 900 km
5
Time taken by car R to cover this distance
900
= = 15 hour
60
Let initial speed of car S = x km/hr
So, ATQ
6x + 15(2x) = 900
6x + 30x = 900
36x = 900 ⇒ x = 25 km/hr
50 Adda247 Publications For any detail, mail us at
Publications@adda247.com
Cracker Book for Bank (IBPS | SBI | RRB PO | Clerk) Mains Exams

63. (e); Distance between Delhi and city A = 1000 km


Distance covered by Car Q before first meeting = 1200 km
Distance covered by Car P before first meeting = 800 km
Speed of car P = 40 km/hr
800
⇒ Time for first meeting = = 20hr
40
1200
Speed of car Q = = 60km/hr
20
When car P reaches city ‘A’ distance covered by car ‘Q’
200
= × 60 = 300km
40
500 25
Time taken by car ‘Q’ to reach Delhi = = hr
60 3
25
Distance covered by car ‘P’ in hour
3
25 1000
= × 40 = km
3 3
Distance between car ‘Q’ and car ‘P’
1000 2000
= 1000 − =
3 3
2000
3 20
Time to meet = = hour
60+40 3
200 25 20
Total time = + + = 20 hours
40 3 3

64. (d); Total work = 24 × (4a + 2a + 4a) = 240a


Bhavya worked for10 days
⇒ Work done by ‘Bhavya’ = 40a
40a
⇒ Time taken by ‘Neeraj’ = = 20 days
2a
⇒ ‘Neeraj’ and ‘Abhi’ worked for 20 days
And ‘Veer’ worked for 8 days
20 × 3a + 10 × 4a + 20 × 2a + y × 4a + 8 × 6a = 240a
⇒ 118a + y × 4a = 240a
240a−188a 52a
⇒y= ⇒ y= = 13 days
4a 4a

65. (b); 5a × (A + 42) = 8a × (A + 15)


⇒ 5A + 210 = 8A + 120
⇒ A = 30
Total work = 5a (30 + 42) = 5a(72) = 360a

51 Adda247 Publications For any detail, mail us at


Publications@adda247.com
Cracker Book for Bank (IBPS | SBI | RRB PO | Clerk) Mains Exams

Let, Abhi, Bhavya and Veer worked for x, y and z days


ATQ,
3x : 4y : 6z = 1 : 2 : 3
Ratio between working days of Abhi, Bhavya and Veer
⇒x:y:z=2:3:3
Ratio between working days of Neeraj, Satish and Veer
=2:2:1
Let working days of Abhi, Bhavya, Neeraj, Satish and Veer be 2m,
3m, 6m and 6m and 3m days respectively
ATQ,
3a × 2m + 4a × 3m + 2a × 6m + 4a × 6m + 6a × 3m = 360a
⇒ 72am = 360a ⇒ m = 5
Bhavya worked for 5 × 3 = 15 days

66. (b); Veer, Bhavya and Satish worked for 6 days, 9 days and 9 days
respectively. Total 60% of work completed by them
⇒ 60% of work = 4a × 9 + 4a × 9 + 6a × 6
= 108a
108a
⇒ Total work = × 5= 180a
3
‘Abhi’ worked for (9 + x) days & ‘Neeraj’ worked for (9+ x + z) days
and completed 40% of work
⇒ 72a = 3a (9 + x) + 2a(9 + x+ z)
72 = 27 + 3x + 18 + 2x + 2z
27 = 5x +2z … (i)
And,
z–x=3 … (ii)
On solving (i) & (ii)
z = 6, x= 3
‘Abhi’ worked for (9 +3) = 12 days
‘Neeraj’ worked for (9 + x+ z) = 9 + 3 + 6
= 18
18−12
Required % = × 100
12
6
= × 100 = 50%
12

52 Adda247 Publications For any detail, mail us at


Publications@adda247.com
Cracker Book for Bank (IBPS | SBI | RRB PO | Clerk) Mains Exams

67. (b); There are two possibilities


First – Satish worked first
Second – Veer worked first
When satish worked first
remaining work
→ 120a – 10(3a+4a) – 7(2a) = 36a
Satish and Veer worked in 3 days = 30a
4th day satish’s worked = 4a
Veer’s worked for = 1/3 day
Now ‘y’ cannot be in fraction
2nd case-
When Veer worked first-
Veer and Satish worked for first 3 day
= 30a
Remaining work = 6/6= 1 day
So Veer worked for 4 days.

68. (d); In first case


ATQ,
10 5 x 5 6
+ + + + =1
40 30 60 30 20
x 53 x 7
⇒ + =1 ⇒ = ⇒ x=7
60 60 60 60
In second case
ATQ,
4 4 y 8 5 y 3
+ + + + =1 ⇒ + =1
40 30 60 30 20 60 4
y 1
⇒ = ⇒ y = 15
60 4
(y − x) = (15 − 7)2 = 82 = 64
2

53 Adda247 Publications For any detail, mail us at


Publications@adda247.com
Cracker Book for Bank (IBPS | SBI | RRB PO | Clerk) Mains Exams

1 Adda247 Publications For any detail, mail us at


Publications@adda247.com
Cracker Book for Bank (IBPS | SBI | RRB PO | Clerk) Mains Exams

Chapter
Data Sufficiency
15
BEST APPROACH TO SOLVE THE QUESTIONS

Data Sufficiency
These questions formerly asked in only MAINS Examination but now-a-
days these questions were seen in PRELIMINARY Examinations too. So,
student can’t deny the importance of DATA SUFFICIENY Questions.

In these questions some data are given in form of statements and one
question is asked. Student must choose that statement/statements which
is/are sufficient to solve the given question.

They consist of two types. In first type, two statements are given while in
second type, three statements are given, and student is expected to choose
the statement/statements among the given statements which is/are
required to solve the question. Five questions of each type are given below
to practice this topic.

Example1.
What is the length of train ‘A’?
A. Train A takes 8 second to cross a pole
B. Train A cross train B coming from opposite side in 8 second, speed of
train B is 25 m/sec and length of train B is half of train A.
C. Train A takes 10 sec to cross two pole 100 m apart.
(a) Only A and B together
(b) Only A and C together
(c) Any two of them
(d) Either B alone or A and C together
(e) All statements are required

2 Adda247 Publications For any detail, mail us at


Publications@adda247.com
Cracker Book for Bank (IBPS | SBI | RRB PO | Clerk) Mains Exams

Solution:

1. (c); Let length of train A = L


Speed of train A = x
L
From A = = 8
x
L+0.5L
From B = =8
x+25
L+100
From C = = 10
x
From any of these two-statement question be solved.

Example 2:
In how many days B and C together can complete work?
A. Per day efficiency of A, B and C is in the ratio 3 : 2 : 4
B. A and B together can complete the work in 7.2 days
C. Time taken by A, B and C alone to complete the work is in the ratio
4:6:3.
(a) Only A and B together
(b) Either A and B or B and C
(c) Any two of them
(d) Either B alone or A and C together
(e) All statements are required

Solution:
2. (b); From A per day efficiency = 3 : 2 : 4
⇒ ratio of time taken by A, B and C alone to complete work = 4 : 6 :
3 same as (C)
From B) time taken by A and B together = 7.2 days
Either B and A or B and C questions can be solved

Example3:
What is the base radius of the cone?
A. Total surface area of the cone is 462 cm²

3 Adda247 Publications For any detail, mail us at


Publications@adda247.com
Cracker Book for Bank (IBPS | SBI | RRB PO | Clerk) Mains Exams

1078√3
B. Volume of the given cone is 𝑐𝑚3 and height = 7√3 cm
3
C. Ratio of height and radius of the given cone is √3 : 1
(a) Only A and B together
(b) Only A and C together
(c) Any two of them
(d) Either B alone or A and C together
(e) All statements are required

Solution:
3. (d) From A) T.S.A = πrℓ + πr² = 462 cm2
1078√3 h
From B) V = = πr 2
3 3
And h = 7√3 cm
From C) h : r = √3 ∶ 1
Either B alone or A and C together, question can be solved.

Example 4:
Find the share of Satish in profit after one year among Satish, veer and
Yogesh
A. Investment of Veer, Satish and Yogesh is in the ratio 6: 8: 9 and share of
Yogesh in profit is Rs.900
B. Satish and Veer invested Rs.8000 and Rs.6000 for 9 months and one
year respectively. Difference b/w profit earned by Satish and Veer is
Zero.
C. Satish, Veer and Yogesh invested for 9 months, 1 year and 8 months
respectively and share of Veer in profit is 900.
(a) Either A and B or B and C
(b) Either A and B or A and C
(c) Any two of them
(d) Either B alone or A and C together
(e) Either A and C or B and C

4 Adda247 Publications For any detail, mail us at


Publications@adda247.com
Cracker Book for Bank (IBPS | SBI | RRB PO | Clerk) Mains Exams

Solution:
4. (e); Veer : Satish : Yogesh
From A and C) 6×12 : 8×9 : 9×8
72 : 72 : 72
1 : 1 : 1
Share of Veer = Share of Satish Rs 900
Satish : Veer
From (B and C) 8000×9 : 6000×12
1 : 1
Share of Veer share of Satish = Rs 900
Either A and C or B and C question can be solved.

Example 5:
What will be the probability of choosing two fresh eggs simultaneously
from tray A given that there are only two trays i.e. tray A and tray B?
A. Tray B contains 30 Eggs in which fresh and rotten eggs in the ratio 7 : 3.
Ratio of fresh eggs in Tray A and Tray B is 2 : 3
B. Tray A contains 20 Eggs in total out of which 14 are fresh and rest are
rotten.
C. Tray B contains 30 Eggs and Tray A contains 20 Eggs respectively. Ratio
of Rotten eggs and fresh eggs in Tray A is 7 : 3
(a) Either A or B
(b) Either B or C
(c) Any one of them
(d) A alone
(e) All statements are required

Solution:
5. (b); From A) Tray B = 30
Rotten eggs in Tray B = 9 eggs & fresh eggs in Tray B = 21 eggs
Tray A, Fresh eggs = 14 eggs
From B) Tray A = 20 eggs
Fresh eggs in Tray A = 14 Rotten eggs in Tray A = 6
From C)
Tray B = 30 eggs
Tray A = 20 Eggs
Rotten eggs in Tray A =6 eggs
Fresh eggs in Tray A = 14 eggs
From Either B or C question can be solved.
5 Adda247 Publications For any detail, mail us at
Publications@adda247.com
Cracker Book for Bank (IBPS | SBI | RRB PO | Clerk) Mains Exams

Practice Exercise Based on new Pattern

Directions (1-5): Given below in each question there two statements (I)
and (II). You have to determine, which statement is sufficient to give the
answer of question. Also there are five alternatives given, you have choose
one alternative as your answer of the questions:

1. What will perimeter of smaller rectangle?


I. Ratio between length of smaller and larger rectangle is 4 : 5 and
breadth of both rectangle is equal. Difference between Perimeter of
both rectangle is 8 cm.
II. Breadth of both rectangle is equal to side of square, whose area is
196 cm2 .
(a) Only statement I is sufficient
(b) Only statement II is sufficient
(c) Statement I and II both together sufficient
(d) Either statement I or Statement II alone sufficient
(e) Neither statement I or statement II sufficient

2. How many Students in college?


I. Ratio between girls to boy is 9 : 11.
II. Out of total girls in the college 20% are belongs to below eighteen
years age group. Total girls belongs to below eighteen years age
group are 9% of total students in college
(a) Only statement I is sufficient
(b) Only statement II is sufficient
(c) Statement I and II both together sufficient
(d) Either statement I or Statement II alone sufficient
(e) Neither statement I or statement II sufficient

3. What was profit shopkeeper made on article?


I. Shopkeeper sold article on 5% discount in the Rs. of 3800.
II. If shopkeeper sold article on marked price, he would made a profit
of 25%.
6 Adda247 Publications For any detail, mail us at
Publications@adda247.com
Cracker Book for Bank (IBPS | SBI | RRB PO | Clerk) Mains Exams

(a) Only statement I is sufficient


(b) Only statement II is sufficient
(c) Statement I and II both together sufficient
(d) Either statement I or Statement II alone sufficient
(e) either statement I or statement II sufficient

4. What is speed of boat?


I. Speed of boat in still water is two times more that speed of current.
II. Boat takes equal time to cover a distance downstream to 50% of
that distance upstream.
(a) Only statement I is sufficient
(b) Only statement II is sufficient
(c) Statement I and II both
(d) Either statement I or Statement II alone sufficient
(e) Neither statement I or statement II sufficient

5. Find the amount invested at the rate of 10%?


I. Total amount of Rs. 4500 invested in two different parts at the rate
of 20% p.a. and 10% p.a. for two years. Simple Interest obtained
from both parts are equal.
II. A man invested an amount in two schemes A and B in the ratio of 2 :
1 respectively. Scheme A offered simple interest at the rate of 10%
p.a. and Scheme B offered compound interest at the rate of 20% p.a.
and man got a total interest of Rs. 1260 after two years from both
scheme. Amount invested on SI is same as amount invested at the
rate of 10% in statement I?
(a) Only statement I is sufficient
(b) Only statement II is sufficient
(c) Statement I and II both
(d) Either statement I or Statement II alone sufficient
(e) Neither statement I or statement II sufficient

7 Adda247 Publications For any detail, mail us at


Publications@adda247.com
Cracker Book for Bank (IBPS | SBI | RRB PO | Clerk) Mains Exams

Directions (6-10): The following questions are accompanied by two


statements (I) and (II). You have to determine which statements(s) is/are
sufficient/necessary to answer the questions.
(a) Statement (I) alone is sufficient to answer the question but statement
(II) alone is not sufficient to answer the questions.
(b) Statement (II) alone is sufficient to answer the question but statement
(I) alone is not sufficient to answer the question.
(c) Both the statements taken together are necessary to answer the
questions, but neither of the statements alone is sufficient to answer
the question.
(d) Either statement (I) or statement (II) by itself is sufficient to answer
the question.
(e) Statements (I) and (II) taken together are not sufficient to answer the
question.

6. Ratio between length of two trains is 4 : 3. What will be difference


between lengths of both trains?
I. Speed of larger trains and smaller train is 72 km/hr and 90 km/hr
28
respectively. Both trains cross each other in 𝑠𝑒𝑐, when running
3
in opposite direction.
II. Speed of smaller train is 90 km/hr and it can cross a pole in 7.2 sec.

7. There are three men P, Q and R. Find the difference between time taken
by P & Q together to complete a task and time taken by Q & R together
to complete the same task?
I. ‘R’ takes twice as much time as ‘Q’ and thrice as much time as ‘P’
takes alone.
II. If they all three works together work will be completed in 4 days.

8. Satish sold an article to Ayush at 20% profit. If Ayush purchased article


from Satish in Rs. 1440, then find the profit percentage of Veer if
Satish bought this article form Veer?
I. Veer sold the article on Rs. 240 more than its cost price to Satish.
II. If Veer sold article to Ayush on same price as Satish sold to Ayush,
then he made overall profit of 50%.

8 Adda247 Publications For any detail, mail us at


Publications@adda247.com
Cracker Book for Bank (IBPS | SBI | RRB PO | Clerk) Mains Exams

9. If x : y = 11 : 9 and y : z = 3 : 4, then find (x + y) – 1.5z = ?


I. Average of all three is two more than average of x and y.
1 1
II. Sum of 9 % of x and 11 % of y is equal to (36)0.5.
11 9

10. Ratio between length and breadth of rectangle ‘X’ is 7 : 4. Find area of a
square ‘Y’?
I. Length of rectangle ‘X’ is two times of radius of circle, whose area is
616 cm2.
II. Perimeter of rectangle ‘X’ is 20 cm more than perimeter of square
‘Y’.

Directions (11-14): The following questions are accompanied by three


statements (A) or (I) and (B) or (II), You have to determine which
statement(s) is/are sufficient/necessary to answer the questions.

11. What is the minimum passing percentage in a test?


I. Raman scored 25% marks in the test and Sunil scored 288 marks
which is 128 more than that of Raman.
II. Raman scored 64 marks less than the minimum passing marks.
(a) Both I and II together are not sufficient
(b) Both I and II together are needed.
(c) Only I alone is sufficient.
(d) Only II alone is sufficient.
(e) Either I or II alone is sufficient

12. Whose body weight is second highest among the five boys Arun, Vinay,
Suraj, Raju and Pratap?
I. Average weight of Arun, Suraj and Vinay is 68 kg and average
weight of Raju and Pratap is 72 kg. Also Suraj is 78 kg. Raju is 68 kg
and Vinay is 46 kg.
II. Average weight of Arun, Suraj, Vinay and Raju is 68 kg and also
Suraj is 78 kg. Raju is 68 kg and Vinay is 46 kg. All of them have
different weights.
(a) Both I and II together are not sufficient
(b) Both I and II together are needed.
(c) Only I alone is sufficient.
(d) Only II alone is sufficient.
(e) Either I or II alone is sufficient
9 Adda247 Publications For any detail, mail us at
Publications@adda247.com
Cracker Book for Bank (IBPS | SBI | RRB PO | Clerk) Mains Exams

13. What is the population of the city A?


I. The ratio of the population of males and females in city A is 27 : 23
and the difference between their population is 100000.
II. The population of city A is 80% of that of city B. The difference
between populations of city A and city B is 312500.
(a) Both I and II together are not sufficient
(b) Both I and II together are needed.
(c) Only I alone is sufficient.
(d) Only II alone is sufficient.
(e) Either I or II alone is sufficient

14. How many students did participate in Singing?


I. The students who participated in dancing were 150% more than
that who participated in Singing.
II. 150 students participated in dancing.
(a) Both I and II together are not sufficient
(b) Both I and II together are needed.
(c) Only I alone is sufficient.
(d) Only II alone is sufficient.
(e) Either I or II alone is sufficient

Directions (15-19): The following questions are accompanied by two


statements A and B. You have to determine which statements(s) is/are
sufficient/necessary to answer the questions.
(a) Statement A alone is sufficient to answer the question but statement B
alone is not sufficient to answer the questions.
(b) Statement B alone is sufficient to answer the question but statement A
alone is not sufficient to answer the question.
(c) Both the statements taken together are necessary to answer the
questions, but neither of the statements alone is sufficient to answer
the question.
(d) Either statement A or statement B by itself is sufficient to answer the
question.
(e) Statements A and B taken together are not sufficient to answer the
question.
10 Adda247 Publications For any detail, mail us at
Publications@adda247.com
Cracker Book for Bank (IBPS | SBI | RRB PO | Clerk) Mains Exams

15. An equilateral triangle is inscribed in a circle. What will be the


difference between the area of circle and area of triangle?
(A) Radius of circle is given
(B) Sum of the perimeter of triangle and circle is given.

16. Is ‘n’ even or odd, if n is a natural number.


(A) 2n + 1 is divisible by 3
(B) 4n − 1 is divisible by 3

17. Five positive naturals numbers are given which may or may not be
equal the sum of these numbers.
(A) If numbers are arranged is ascending order then median is 18
while sum of first and last number and second and fourth number
is equal
(B) Median of these numbers is 18 while mode of these numbers is 12
and 24.

18. ‘X’ mark up an article 50% above its cost price. Find the cost price of
article.
(A) ‘X’ gave Rs 60 discount on mark price and earn 20% profit.
(B) If ‘X’ gave two successive discounts of 10% each then he will earn
Rs 43 as profit.

19. Amit tells truth 2 times out of x times while Ankush tells truth 5 times
out of ‘y’ times. Find the product of ‘x’ and ‘y’
(A) Probability that Amit and Ankush contradict with each other on
facts is 50%
(B) Probability that both tells truth is 20%

Directions (20-24): The following questions are accompanied by two


statements (A) and (B). You have to determine which statements(s) is/are
sufficient/necessary to answer the questions.

11 Adda247 Publications For any detail, mail us at


Publications@adda247.com
Cracker Book for Bank (IBPS | SBI | RRB PO | Clerk) Mains Exams

20. O is center, find area of shaded region

A. Length of AO is given.
B. AC is given in multiple of radius of semicircle.
(a) Only A
(b) Only B
(c) Either Only A or Only B
(d) A and B together
(e) A and B together are not sufficient

21. In a box three type of balls are there, Black, Red and White. If no. of
white balls is given then find out the probability of getting one white
ball.
A. Probability of getting one Red ball is given.
B. Probability of getting one black ball is given.
(a) Only A
(b) Only B
(c) Either Only A or Only B
(d) A and B together
(e) A and B together are not sufficient

22. What is the volume of the sphere?


A. Surface area of hemisphere is equal to the total surface area of the
cylinder having radius and height in ratio 3 : 4.
B. When we cut sphere into two hemi-sphere then total surface area
is equal to the area of a circle whose radius is 21 cm.
(a) Only A
(b) Only B
(c) Either Only A or Only B
(d) A and B together
(e) A and B together are not sufficient

12 Adda247 Publications For any detail, mail us at


Publications@adda247.com
Cracker Book for Bank (IBPS | SBI | RRB PO | Clerk) Mains Exams

23. PR is diameter of circle. Find, ∠QPO – ∠SRO = ?

A. ∠SPO = 40°
B. ∠PQR + ∠QRO = 120°
(a) Only A
(b) Only B
(c) Either Only A or Only B
(d) A and B together
(e) A and B together are not sufficient

24. A shopkeeper gets a loss of 70 Rs. when he sold an article at 20%


discount on M.P. Find cost price of Article.
A. % of mark up above cost price is equal to % discount given on M.P.
B. When no discount is given, article sold at profit of 350 Rs.
(a) Only A
(b) Only B
(c) Either Only A or Only B
(d) A and B together
(e) A and B together are not sufficient
Directions (25-29): The following questions are accompanied by three
statements (I), (II), and (III). You have to determine which statement(s)
is/are sufficient /necessary to answer the questions

25. What is distance between A and B ?


I. Two persons Amit and Abhi started simultaneously from A to B with
their speed in ratio 4 : 5.
13 Adda247 Publications For any detail, mail us at
Publications@adda247.com
Cracker Book for Bank (IBPS | SBI | RRB PO | Clerk) Mains Exams

II. Abhi reached reached Q one hour earlier than Amit.


III. Difference between speed of Amit and Abhi is 20 km/hr.
(a) Only I and II.
(b) Only II and III
(c) All I, II and III
(d) Cannot be answered even including all three statement
(e) None of these

26. What is the area of rectangle ?


I. If ratio of length and breadth of the rectangle is 3 : 2.
II. Circumference of a circle is 440 m and breadth of rectangle is 1/7 th
of diameter of circle.
III. If length is 50% more than breadth.
(a) Only III
(b) Only I and II or II and III.
(c) Only II
(d) All I, II and III
(e) None of these

27. How many students failed in class 11th ?


I. 400 Students passed in class 11th.
II. No. of students failed in class 11th is 20% of those failed in class 12th.
III. Ratio of student appeared to that of failed in class 11th is 5 : 3.
(a) Only I and III
(b) Only II
(c) Only I and II
(d) All I, II and III
(e) Cannot be answered even including all three statement

28. What is the rate of interest on some amount?


I. S.I. accrued in two years on same amount at same rate of interest is
Rs. 44000.
II. The amount after some years on S.I. is Rs. 154000.
III. Difference between the C.I. and S.I. earned in two years on the same
amount invested is Rs. 120.
14 Adda247 Publications For any detail, mail us at
Publications@adda247.com
Cracker Book for Bank (IBPS | SBI | RRB PO | Clerk) Mains Exams

(a) Only I and III


(b) Only III
(c) Only II and III
(d) Cannot be answered even including all statement
(e) None of these

29. What is the sum of two number?


I. The bigger no. is 6 more than the smaller no.
II. 40% of smaller no. is equal to 30% of bigger no.
III. The ratio b/w half of the bigger no. & one-third of smaller no. is 2 :
1.
(a) Only II & III
(b) Only I & II
(c) Any two of the three statement
(d) All statement is required
(e) I and II or I and III

Directions (30-33): In the following questions three statements either A, B


and C or I, II and III are given. You have to use your knowledge of
mathematics to answer which statement(s) is/are sufficient to answer the
question.
30. What is the cost price of wrist watch?
A. Shopkeeper gives 20% discount on a speaker.
B. The marked price of wrist watch is 25% more than that of speaker.
C. The shopkeeper earns a profit of 10% after selling the speaker.
(a) Any two of them are sufficient
(b) All the three statements are required
(c) Only C is sufficient.
(d) Even using all statements, answer cannot be found
(e) Only A is sufficient
31. What is the total quantity of milk in final mixture of milk and water
after adding some milk ?
A. 68 liters of initial mixture has ratio of milk and water as 11:6.
B. A certain amount of milk is added to this mixture to make the ratio
of water to milk as 6:13.
C. Selling the mixture at a certain rate, 35 % profit is obtained.
15 Adda247 Publications For any detail, mail us at
Publications@adda247.com
Cracker Book for Bank (IBPS | SBI | RRB PO | Clerk) Mains Exams

(a) Any two of them is sufficient


(b) A and B together are sufficient
(c) All the three statements are required
(d) Answer cannot be found even using all the three statements
(e) Only B and C are required

32. The speed of a train A and time taken by it to cross a tunnel is known.
Find length of tunnel.
A. Another train B is running in opposite direction to A with a speed
40% more than A.
B. Train B crosses a platform X and a pole in 24 sec and 8 sec
respectively before crossing the train A and tunnel.
C. The ratio of length of train A and platform X is 3 : 5.
(a) Only A and B are sufficient.
(b) Only B and C are sufficient
(c) Even using all the three statements answer cannot be found.
(d) All the three statements are required.
(e) Only B is sufficient

33. In how many ways 2 green balls can be chosen from box ‘A’ which
contains red, green and black balls?
A. Ratio of total number of balls in boxes A and B is 2 : 3 and box A
contains 5 green balls.
B. Total balls in box B is 18 while ratio of red and black balls in box A
is 3:4.
C. Box ‘B’ contains 5 red 7 black and 6 green balls only while ratio of
red and green balls in box ‘A’ is 3:5.
(a) Statement A and either B or C are sufficient
(b) Any two statements are sufficient
(c) All the three statements are required
(d) Statement B and either A or C are sufficient
(e) None of these

Directions (34-38): The following questions are accompanied by three


statements (I), (II), and (III). You have to determine which statements(s)
is/are sufficient/necessary to answer the questions.

16 Adda247 Publications For any detail, mail us at


Publications@adda247.com
Cracker Book for Bank (IBPS | SBI | RRB PO | Clerk) Mains Exams

34. Making a new t-shirt, consists of two parts first is cutting of cloth and
second is stitching the cloth. In how many days Nitin and Vishal
together can make 100 t-shirts.
I. Nitin can cut cloth for 20 t-shirts in a day and he is 20% slow as
that of Vishal in cutting clothes and 25% faster than Vishal for
stitching the cloth.
II. Time taken by Nitin alone to make 60 t-shirts is 9 days.
III. The ratio of days taken by Vishal to cut cloth for 50 t-shirts and to
stich 100 t-shirts is 4 : 25.
(a) Only I and III together or only II and III together
(b) All I, II and III together
(c) Any two of the three
(d) Only I and III together
(e) Either from I & II or I & III

35. Ratio of age of Aashish & Kullu is 4 : 5 and that of Nikhil and Yash is 7 :
8. What is their average age after 4 years.
I. Age difference of Nikhil and Aashish is 18 years.
II. Ratio of 12 years ago age of Aashish & Nikhil is 2 : 5 and Kullu &
Yash is 1 : 2.
III. 12 years later Aashish will be 150% of his present age.
(a) Any two of I, II and III are required.
(b) Only I and II are required
(c) Only II and III are required
(d) All I, II and III together are required
(e) II alone or from I and III together are required

36. Three athletes practice running on a circular track of 400 m. who runs
fastest among them?
I. When A starts running clockwise and B starts running
1
anticlockwise, they meet 2nd time at a distance of 57 meter from
7
6
starting point in clockwise direction after 22 seconds.
7
II. B runs 5 m/s faster than C. A is not twice or more fast than B and
neither B is twice or more fast than A or C.
III. When A, C and B, all runs in same direction, A and C only meet at
starting point, while ratio of speed of B to C is 3 : 2.

17 Adda247 Publications For any detail, mail us at


Publications@adda247.com
Cracker Book for Bank (IBPS | SBI | RRB PO | Clerk) Mains Exams

(a) Any one of them


(b) Only I and II together are sufficient
(c) Any two of the three together are sufficient
(d) None of the above
(e) All the three statements are not sufficient.

37. A man has 16 balls with him, in 3 colors i.e. Red, green and blue. Three
balls are drawn at random, what is the probability that all are blue.
1
I. The probability of drawing a blue ball is greater than and less than
8
5
. Also, the number of green balls he has is odd and number of red
16
balls he has is even.
3 1
II. Probability of drawing red ball is which is lesser than that of
8 16
green balls.
III. If a man lost one ball, probability of drawing a blue ball is ⅓.
(a) Only I and III or only II and III
(b) All I, II and III together
(c) Any one of them is sufficient
(d) Only I and III together sufficient
(e) None of the above

38. A man bought two bats and 6 identical balls, he sold all of them in a day,
calculate his overall profit %.
I. He sold one bat at a price of Rs. 600 and other at Rs. 420 and profit
is 20% on both bats. Each ball is sold at 12% profit.
II. Profit earned from 6 balls is Rs. 36 and profit earned from both bats
is Rs. 170.
III. Profit earned on each bat is 20% while cost price of each ball is Rs.
50.
(a) Any one of them
(b) Only I and II together are sufficient
(c) Any two of the three together are sufficient
(d) None of the above
(e) All three together are sufficient
Directions (39-42): The following questions are accompanied by three
statements A, B and C. You have to determine which statement(s) is/are
necessary/sufficient to answer the question.

18 Adda247 Publications For any detail, mail us at


Publications@adda247.com
Cracker Book for Bank (IBPS | SBI | RRB PO | Clerk) Mains Exams

39. Find the area of the shaded region?

A. Radius of bigger circle is given.


B. Diagonal of square ABCD is given.
C. Difference between area of bigger circle and area of smaller circle
is given.
(a) Either A or B alone is sufficient to answer the question
(b) Either B or C alone is sufficient to answer the question
(c) Either A or C alone is sufficient to answer the question
(d) Any of A, B or C alone is sufficient to answer the question
(e) Either only A or B and C together is sufficient to answer the
question

40. Find the surface area of cone?


A. Ratio between height and radius of cone is 8 : 7.
B. Radius of cone is half of the radius of hemisphere, which volume is
19404 cm³.
C. Radius of cone is 75% of radius of cylinder, which total surface area
of 2640 cm³ and ratio between height and radius of cylinder is 8 : 7.
(a) Either A and B or B and C are sufficient to answer the question
(b) Either A and B or A and C are sufficient to answer the question
(c) Either A and C or B and C are sufficient to answer the question
(d) A, B and C together are sufficient to answer the question
(e) Either only A or B and C together are sufficient to answer the
question
41. What will be 26 times of 26th term of Arithmetic progression (AP)?
A. If eight times of the 8th term of that A.P. is equal to 18th times the
18th term of that A.P.
19 Adda247 Publications For any detail, mail us at
Publications@adda247.com
Cracker Book for Bank (IBPS | SBI | RRB PO | Clerk) Mains Exams

B. If first term of AP is a prime number.


C. If difference between two terms in A.P. is an odd number.
(a) Only statement ‘A’ alone is sufficient to answer the question
(b) Only statement ‘B’ alone is sufficient to answer the question
(c) Only statement ‘C’ alone is sufficient to answer the question
(d) Any of two statements are sufficient to answer the question
(e) All three together are sufficient to answer the question

42. What is cost price of the article?


A. On selling the article at 119 Rs. shopkeeper earn profit % equal to
cost price of article.
2
B. If 14 % discount is given on the mark price, then its selling price
7
will be 102 Rs.
C. If article sold on profit percent half of the mark price, then profit is
41.65 Rs.
(a) Either A and B or B and C are sufficient to answer the question
(b) Either A and B or A and C are sufficient to answer the question
(c) Either A and C or B and C are sufficient to answer the question
(d) A, B and C together are sufficient to answer the question
(e) Either only A or B and C together are sufficient to answer the
question

43. A joker has three identical boxes having same number of balls but are
of different colors. In each box ‘x’ balls are red, 7 balls are blue and ‘y’
balls are green. Find (𝒙 + 𝒚)?
A. Probability of choosing one ball which is either red or blue from
𝟑
any one of the box, is .
𝟒
B. Number of ways to select two balls from any one of the box is 6.
C. Probability of choosing one ball which is either red or green from
𝟗
any one of the box is .
𝟏𝟔
(a) Only statement ‘A’ alone is sufficient to answer the question
(b) Only statement ‘B’ alone is sufficient to answer the question
(c) Only statement ‘C’ alone is sufficient to answer the question
(d) Any of two statements are sufficient to answer the question
(e) Both A and C are required together

20 Adda247 Publications For any detail, mail us at


Publications@adda247.com
Cracker Book for Bank (IBPS | SBI | RRB PO | Clerk) Mains Exams

44. Is (bc − ab) even integer or odd integer if a, b and c are integers?
A. a, b and c are in arithmetic progression.
B. Set {a,b,c} is co-prime and a,b and c are consecutive integers.
C. a, b and c all three are prime numbers.
(a) Either statement ‘A’ or ‘B’ alone is sufficient to answer the question
(b) Either statement ‘B’ or ‘C’ alone is sufficient to answer the question
(c) Either statement ‘C’ or ‘A’ alone is sufficient to answer the question
(d) Any of two statements are sufficient to answer the question
(e) Any one of them is sufficient to answer the question

45. Find the area of a rhombus?


A. The ratio of the diagonals of the rhombus is 3 : 4.
B. The perimeter of the rhombus is equal to that of a square whose
diagonal length is 20√2m.
C. The difference between the squares of the diagonals of rhombus is
448 m2.
(a) Statement ‘A’ and ‘B’ together are sufficient to answer the question
(b) Statement ‘B’ and ‘C’ together are sufficient to answer the question
(c) Statement ‘C’ and ‘A’ together are sufficient to answer the question
(d) Any of two statements are sufficient to answer the question
(e) Any one of them is sufficient to answer the question

46. What will be the sum of the ages of father and the son after five years?
A. Father’s present age is twice son’s present age
B. After ten years the ratio of father’s age to the son’s age will become
12 : 7.
C. Five years ago the difference between the father’s age and son’s age
was equal to the son’s present age.
(a) Statement ‘A’ and ‘B’ together are sufficient to answer the question
(b) Statement ‘B’ and ‘C’ together are sufficient to answer the question
(c) Statement ‘C’ and ‘A’ together are sufficient to answer the question
(d) Any of two statements are sufficient to answer the question
(e) Either ‘A’ and ‘B’ together or ‘B’ and ‘C’ together
47. In how much time Bhavya can row 64km in a stream to reach a
destination ‘A’ and return back?

21 Adda247 Publications For any detail, mail us at


Publications@adda247.com
Cracker Book for Bank (IBPS | SBI | RRB PO | Clerk) Mains Exams

A. Time taken by Bhavya to reach destination ‘A’ is twice than that of


to return back.
B. Ratio between speed of boat in still water to speed of boat in
downstream is 3 : 4
C. Upstream speed of boat is 4kmph less than downstream speed.
(a) Either statement ‘A’ and ‘B’ together or ‘B’ and ‘C’ together are
sufficient
(b) Either statement ‘A’ and ‘C’ together or ‘B’ and ‘C’ together are
sufficient
(c) Either statement ‘A’ and ‘B’ together or ‘A’ and ‘C’ together are
sufficient
(d) Any of two statements are sufficient to answer the question
(e) All three are required to solve the question

48. A, B and C together can complete work ‘X’ in 5 days. Find in how many
day ‘C’ alone can complete work ‘X’?
A. A and B together can complete work ‘X’ in 15 days if both worked
with half of their efficiencies.
B. B, who is 25% more efficient than ‘C’, takes 8 days more than ‘A’ to
complete work alone
C. ‘A’ is 25% less efficient than ‘B’.
(a) Either statement ‘A’ alone or ‘B’ alone is sufficient to answer the
question
(b) Either statement ‘B’ alone or ‘C’ alone is sufficient to answer the
question
(c) Either statement ‘C’ alone or ‘A’ alone is sufficient to answer the
question
(d) Any of two statements are sufficient to answer the question
(e) Any one of them is sufficient to answer the question

22 Adda247 Publications For any detail, mail us at


Publications@adda247.com
Cracker Book for Bank (IBPS | SBI | RRB PO | Clerk) Mains Exams

49. Find the sum of ‘x’ and ‘y’ ?


A. H.C.F and L.C.M of ‘x’ and ‘y’ is 21 and 3003 respectively.
B. Difference between x and y is 42
C. Both ‘x’ and ‘y’ are multiple of 21.
(a) Either statement ‘A’ and ‘B’ together or ‘B’ and ‘C’ together are
sufficient
(b) Either statement ‘A’ and ‘C’ together or ‘B’ and ‘C’ together are
sufficient
(c) Either statement ‘A’ and ‘B’ together or ‘A’ and ‘C’ together are
sufficient
(d) Any of two statements are sufficient to answer the question
(e) All three are required to solve the question

50. Find the area of shaded region if radius of each circle is same?

A. Area of circle is given.


B. Perimeter of triangle is given.
C. Area of triangle is approximately 310% more than area of a circle.
(a) Either statement ‘A’ or ‘B’ alone is sufficient to answer the question
(b) Either statement ‘B’ or ‘C’ alone is sufficient to answer the question
(c) Either statement ‘C’ or ‘A’ alone is sufficient to answer the question
(d) Any of two statements are sufficient to answer the question
(e) Any one of them is sufficient to answer the question
23 Adda247 Publications For any detail, mail us at
Publications@adda247.com
Cracker Book for Bank (IBPS | SBI | RRB PO | Clerk) Mains Exams

Solutions

1. (c); From I.
Lets length of larger rectangle and smaller rectangle be 5x and 4x
respectively.
2(5x+b) – 2(4x+b) = 8
Form II.
Breadth of rectangle = side of square
Side of square = 14 cm
From I & II we get
10x – 8x = 8
x = 4cm
perimeter of smaller rectangle
= 2 (4×4+14) = 60 cm
So, I and II both together sufficient to give answer

2. (e); From I,
Lets number of girls and boys be 9x and 11x respectively
From II,
Let total students 100x
Girls below eighteen years group
9
= 100x × = 9x
100
9𝑥
Given × 100 = 45x
20
Boys : girl = (100x–45x) : 45x = 11 : 9
From I and II we get same equation but we can’t get the answer.
So, Neither I nor II both sufficient to give answer of question.

3. (c); Form I,
Selling price of article = 3800 Rs.
3800
Marked price of article = × 100 = 4000
95
From II
4000
Cost price = × 100 = 3200
125

24 Adda247 Publications For any detail, mail us at


Publications@adda247.com
Cracker Book for Bank (IBPS | SBI | RRB PO | Clerk) Mains Exams

From I & II
Profit of shopkeeper = 3800 – 3200
= 600 Rs.

4. (e); From I,
Let speed of boat in
still water x km/hr and speed of current y km/hr
x = 3y …(1)
From II,
′𝑑′
Let bot cover ‘d’ distance downstream and distance in upstream
2
d
d 2 1 1
= ⇒ =
3y+y 3y−y 4y 4y
So, From I & II both not sufficient to answer of the question–

5. (d); From I,
Let amount invested on 20% is x Rs and on 10% is (4500–x) Rs.
𝑥 × 20 × 2 (4500– 𝑥 ) × 2 × 10
=
100 100
40x = 90000 – 20x
60x = 90000
x = 1500 Rs.
amount invested on 10% = (4500–1500) = 3000 Rs.
From II,
Lets man invested Rs 3x
Equivalent CI of two years on 20%
20×20
= 20 + 20 + = 44%
100
ATQ –
20 44
2𝑥 × + 𝑥× = 1260
100 100
108x = 126000
x = 1500 Rs.
Amount invested on 10% = 2 × 1500 = 3000 𝑅𝑠.
So, Either statement I alone or statement II alone sufficient is to
give answer of question
25 Adda247 Publications For any detail, mail us at
Publications@adda247.com
Cracker Book for Bank (IBPS | SBI | RRB PO | Clerk) Mains Exams

6. (d); Let length of two trains be 4x meter and 3x meter


From I,
5 (4𝑥+3𝑥)3
(90 + 72) × =
18 28
21x = 1260
x = 60 meters
Required difference = 60 × 4 − 60 × 3 = 60 meters
From II,
5 3𝑥
90 × =
18 7.2
3x = 180 meters
180
Length of larger train = 4x = × 4 = 240
3
Required difference = 60 meters
So, either I or II alone sufficient to give answer of question.

7. (c); From I and II together


Let P, Q and R takes 2x days, 3x days and 6x days respectively
So efficiency of P, Q and R is 3x, 2x and x unit/day
Total work = 4 × (3𝑥 + 2𝑥 + 𝑥 ) = 24x
So, we can determine required difference with I and II together

8. (d); Given, Cost price of article for Satish


1440
= × 100 = 1200 Rs.
120
From I,
Cost price of article for Veer = 1200 – 240
= 960 Rs.
240
Profit percentage of Veer = × 100 = 25%
960
From II,
100
Cost price of article for Veer = 1440 ×
150
= 960
Veer profit percentage
1200−960
= × 100 = 25%
960
So, either I or II alone sufficient to give answer of question.

26 Adda247 Publications For any detail, mail us at


Publications@adda247.com
Cracker Book for Bank (IBPS | SBI | RRB PO | Clerk) Mains Exams

9. (d); x : y : z = 11 : 9 : 12
Let x, y and z be 11a, 9a and 12a respectively
From I,
11a + 9a + 12a 11a + 9a
− =2
3 2
32a
− 10a = 2
3
𝑎=3
So,
(x + y) – 1.5z = (11 × 3 + 9 × 3)–1.5×3×12 = 6
From II,
1 1
11𝑎 × + 9𝑎 × = (62)0.5
11 9
2𝑎 = 6
𝑎=3
So, we can determine (x + y) – 1.5z from II also
So, Either statement I or Statement II alone sufficient

10. (c); Let length and breadth of rectangle be 7x and 4x respectively


From I,
Given, πr2 = 616
616×7
r2 =
22
r = 14 cm
length of rectangle = 14 × 2 = 28 𝑐𝑚
28
breadth of rectangle = × 4 = 16 𝑐𝑚
7
From I and II together,
2( l + b) – 4a = 20
2 (28 + 16) – 4a = 20
4a = 88 – 20
a = 17 cm
Area of Square = (17)2
= 289 cm2
So, Statement I and II both together sufficient

27 Adda247 Publications For any detail, mail us at


Publications@adda247.com
Cracker Book for Bank (IBPS | SBI | RRB PO | Clerk) Mains Exams

11. (b); If the max marks of exam = x


x
Raman =
4
x
⇒ = 288 − 128 = 160
4
x = 640
∴ Minimum passing marks
= 160 + 64 = 224
224
Required %= × 100 = 35%
640

12. (c); From I,


A + S+ V = 3 × 68 = 204 kg
R + P = 144 kg
A ⇒ 204 – 46 – 78 = 80 kg
P ⇒ 144 – 68 = 76 kg
S = 78 kg

13. (e); From I


27x − 23x = 100000
x = 25000
Population of city A = 50x
= 1250000
From II
Population of city B = x
4x
Population of city A =
5
4x
x− = 312500
5
x = 1562500
4
Population of city A = × 1562500
5
= 1250000

14. (b); From I and II


Students participating in dance = 150
Students who participate in singing
150×100
= = 60
250

28 Adda247 Publications For any detail, mail us at


Publications@adda247.com
Cracker Book for Bank (IBPS | SBI | RRB PO | Clerk) Mains Exams

15. (d); Form A → Let radius of circle = r


Then side of equilateral ∆ = √3𝑟
Area of equilateral triangle and circle can be find out and required
difference can also be find out easily.
From B → Let radius of circle =r
⇒ side of equilateral triangle = √3 r
Sum of peremeter of triangle and circle is given. By this value of ‘r’
can be find out and after this required difference can be find out
easily. Hence, either only A or only B is sufficient to answer the
question

16. (a); From A → 2𝑛 + 1 is divisible by 3


⇒ n =1, 3, 5, 7….
⇒ n = odd
From B) → 4𝑛 − 1 is divisible by 3
⇒ n =1, 2, 3…….
⇒ ‘n’ can be even as well as odd
Hence, Only A is sufficient to answer the question

17. (b); From A → Let numbers are a, b, c, d, e in ascending order


⇒ Median = 18 = c
And a+e=b+d
From B → let number are a, b, c, d, e is ascending order.
Median = 18 = c
Mode = 12 and 24
This means two number are 12 and two number are 24
⇒ Numbers are 12, 12, 18, 24, 24
Required sum 12 + 12 + 18 + 24 + 24 = 90
Hence, Only B is sufficient to answer the question.

18. (d); Let CP = 200x


⇒ MP = 300x
From A → SP = 300x – 60
120
200𝑥 × = 300𝑥 − 60
100
⇒ 60𝑥 = 60

29 Adda247 Publications For any detail, mail us at


Publications@adda247.com
Cracker Book for Bank (IBPS | SBI | RRB PO | Clerk) Mains Exams

⇒ CP = 200x = 200
From B → Two successive discount
10 × 10
= 10 + 10 − = 19%
100
81
S.P =200x + 43 = 300𝑥 ×
100
200x + 43 = 243x
x=1
CP = 200x = 200
Hence, either alone A or alone B is sufficient to answer the
question.

2
19. (b); Probability that Amit tells truth =
𝑥
𝑥−2
Probability that Amit tells lie =
𝑥
5
Probability that Ankush tells truth=
𝑦
𝑦−5
Probability that Ankush tells lie =
𝑦
2 𝑦−5 𝑥−2 5 1
From A→ × + × =
𝑥 𝑦 𝑥 𝑦 2
2 5 1
From B → × =
𝑥 𝑦 5
⇒ 𝑥𝑦 = 50
Hence only B is sufficient to answer the question.

20. (d); Given


AO + OC = OB
From A = AO → find
From B → AC = x (AO)
When ‘x’ multiple
From A & B together area of triangle and Area of semi circle find
out.
Required area
π(AO)2
= − √S(S – AO). (S – OC). (S – AC)
2
AO+OC+AC
S=
3
∴ A & B together sufficient to answer the question

30 Adda247 Publications For any detail, mail us at


Publications@adda247.com
Cracker Book for Bank (IBPS | SBI | RRB PO | Clerk) Mains Exams

21. (d); Given no. of white ball


Let → a
x
From A let probability →
y
Let no. of red ball → px, total balls → py
𝑠
From B → Let probability =
t
Let no. of black ball = qs, total balls = qt
From A & B
px + a + qs = qt = py
we know the values of x, y, s, t and a so we can find the value of p
and q
a a
So probability of white ball found = or
𝑞𝑡 py
∴ A & B together sufficient to answer the question

22. (b); Only B is sufficient to answer the question


When we cut sphere into hemisphere total surface area of two
hemisphere
Total surface area of two hemisphere
3πr² + 3πr² = π × 21 × 21
r = find out
So, volume of sphere can be find out.

23. (d); Given


∠PQR = ∠PSR = 90° {Angle of diameter}
A → ∠SRO = 180°– 90°(∠PSR) – 40°(∠ SPO)
∠SRO = 50°
B→ ∠PQR + ∠QRO = 120°
∠QRO = 120°– 90 = 30°
∠QPO = 60°
From B & A together, required difference can be find out.

24. (c); From A→ Discount % = 20% = Mark up%


If cost price is 100x then Markup price 120x and selling price is →
96x
So ATQ,
100x – 96x = 70

31 Adda247 Publications For any detail, mail us at


Publications@adda247.com
Cracker Book for Bank (IBPS | SBI | RRB PO | Clerk) Mains Exams
70
C. P = 100x = × 100x = 1750
4x
From B→ Let mark up price is → 100x
Then selling price is → 80x
ATQ,
100x – 80x = 350 + 70
20x = 420
100x = 2100
80x = 1680
C.P. → 1680 + 70 = 1750
So Either A or B alone required.
Solutions (25-29)

25. (c); From I, II & III


Let speed of Amit and Abhi be 4x and 5x km/hr respectively.
5x – 4x = 20
∴ x = 20 km/hr
Let distance be d km
d d
– =1
80 100
80×100
∴d= = 400 km
20

26. (b); From I and II


Let length and breadth be 3x and 2x
2πr = 440 [r → radius of circle]
r = 70 m
∴ breadth = 10 m
& length = 15 m
∴ Area = 10 × 15 = 150 m²
Statement I and III are same.

27. (a); From I


Passed = 400
From III
Let appeared & Passed student be 5x and 3x respectively
2x = 400 ⇒ x = 200
32 Adda247 Publications For any detail, mail us at
Publications@adda247.com
Cracker Book for Bank (IBPS | SBI | RRB PO | Clerk) Mains Exams

∴ failed = appeared – passed


= 1000 – 400
= 600

28. (a); From I


PRT(2)
= 44000
100
PR = 2200000
From II
PRT
P+ = 15400
100
From III
PR2
Difference =
1002
PR2
= 120
1002
From I and III R can be found.

29. (e); Let the smaller no. is x & bigger no. is y.


From I
y=x+6
From II,
40 30
×𝑥 = ×𝑦
100 100
From III,
𝑦
2 =2
𝑥 1
3
⇒ 3y = 4x
∴ from I and II or I and III

30. (d); From statement A,


Let MP of speaker = Rs. 100
∴ SP = 80 rupees
From B,
MP of wrist watch = 1.25 M.P of speaker
= Rs. 125 (From st. A)
From C,
33 Adda247 Publications For any detail, mail us at
Publications@adda247.com
Cracker Book for Bank (IBPS | SBI | RRB PO | Clerk) Mains Exams
100 800
C. P. of speaker = 80 × = (from st. A)
110 11
Here, there is no information about S.P. and Profit of watch. So,
answer cannot be found.

31. (b); From A,


11
Milk = 68 × = 44 ℓ
17
6
Water = 68 × = 24 ℓ
17
From B, let x ℓ milk is added to the mixture.
24 6
= ⇒x=8ℓ
44+x 13
∴ Total milk in final mixture = 44 + 8 = 52 ℓ

32. (d); From st. A,


Speed of train A = P m/sec (Given)
140 14P
Speed of train B = of P =
100 10
From st. B+A,
Length of platform
14P 14P
24 × – × 8
10 10
14P
= 16 × = 22.4 P
10
From C, Length of train A
3
= × 22.4P (with help of st. A and B together)
5
3
∴ Length of tunnel = Pt(given) – × 22.4P
5

33. (e); With help of statement A only, required number of ways can be
found out.

34. (e); From statements (I)


Nitin can cut cloth for 20 t-shirts in a day
5
∴ Vishal can cut cloth for 20 × = 25 t-shirts in a day
4
And if Vishal can stich 4x t-shirts in a day, Nitin will stich 5x t-
shirts.
It can’t be solved further.
34 Adda247 Publications For any detail, mail us at
Publications@adda247.com
Cracker Book for Bank (IBPS | SBI | RRB PO | Clerk) Mains Exams

From statement (II)


We only can obtain time taken by Nitin to build 100 t shirts which
is15 days
Nothing more can be obtained from (II) alone.
From statement (III)
Question can’t be solved from this statement alone.
So, by using (I) & (II)
Nitin will take 3 days to cut cloth for 60 t-shirts, Hence 6 days to
stich them.
Therefore, Nitin stiches 10 t-shirts per day
And Vishal stiches 8t-shirts per day. (From ratio obtained in (I))
Total time required for Nitin alone to make 100 t-shirts
100 100
= + = 15 days
20 10
Total time required for Vishal alone to make 100 t-shirts
100 100
= + = 4 + 12.5 = 16.5 days
25 8
Hence time can be calculated for both.

Using statement (I) and (III)


Days required by Vishal to cut cloth for 50
t-shirts is 2 days
∴ Days required by Vishal to stich 50
t-shirts in 12.5 days
Hence days required for both Nitin and Vishal can be calculate,
[first we will calculate, days required individually and then for both
of them]
Hence question can be solved through I & II or I & III.

35. (e); From Question,


Let age of Aashish and Kullu is 4x and 5x and that of Nikhil & Yash
is 7y and 8y.
From (I) statement
7y – 4x = 18 …eqn. (A)
It can’t be solved further.
From (II) statement

35 Adda247 Publications For any detail, mail us at


Publications@adda247.com
Cracker Book for Bank (IBPS | SBI | RRB PO | Clerk) Mains Exams
4𝑥–12 2
=
7𝑦–12 5
⇒ 20x – 60 = 14y – 24
⇒ 10x – 30 = 7y – 12
⇒ 10x – 7y = 18 …eq. (B)
5x–12 1
=
8y–12 2
⇒ 10x – 24 = 8y – 12
⇒ 5x – 12 = 4y – 6
5x – 4y = 6 …eqn. (C)
Solving eqn. (B) & eqn. (C)
We will get
x=y=6
Hence their age can be calculated. Also their average age after 4
years.
Therefore, statement (II) alone is efficient to solve question.
From (III) statement
4𝑥+12 3
= ⇒ x = 6, present age of Aashish is 24 years as that of Kullu
4𝑥 2
is 30 years.
Nothing more can be calculated
Now combining statement (III) & (I)
We will get x = y = 6.
Hence question can be solved.
Therefore, this question can be solved by statement (II) alone
or combining (I) and (III).

36. (b);

36 Adda247 Publications For any detail, mail us at


Publications@adda247.com
Cracker Book for Bank (IBPS | SBI | RRB PO | Clerk) Mains Exams

Let us draw a circular track of 400 meters with 4 point U, V, W and


X at anti-clockwise distance of 100m, 200 m and 300 meter.
From statement (i)
160
They meet second time in seconds that is somewhere between
7
U and X.
Therefore their sum of speed [as both were travelling in opposite
direction] is
800×7
= = 35 ms¯¹
160
As they meet between U and X.
There are two chances, either A have only covered a distance of 57
1 1
meter or total distance of (400 + 57 ) meter, hence, we can’t
7 7
determine who is more faster.
From statement (II)
It is clearly given that B is faster than C, but we can’t determine
who is faster between A and C.
From statement III
A and C only meet at starting points means either their speed is co-
prime of each other or one of their speed is integral multiple of
other and ratio of speed of B and C is given.
Now combining statement (i) and statement (ii)
It is given that B doesn’t run faster at twice or more speed. Hence
1
chances that A only travelled (400 + 57 ) meter distance, hence
7
faster than B, and it is given that B runs faster than C in statement
(ii).
Hence question can be answered from statement (i) and (ii)
We can’t answer question from (i) and (iii) or (ii) or (iii), as we
don’t know who is faster among A and C.

37. (c); From statement …(I)


2 5
Probability is greater than ⅛ i.e. and less than . Therefore, he
16 16
could either have 3 balls or 4 balls. Now sum of green balls and red
balls is odd. [Odd + Even = Odd]. 16 is even.
Therefore, blue balls must be odd. [Odd + Odd = Even].
Hence there are 3 blue balls.

37 Adda247 Publications For any detail, mail us at


Publications@adda247.com
Cracker Book for Bank (IBPS | SBI | RRB PO | Clerk) Mains Exams
3
(II) Probability of drawing red ball is
8
Therefore, he has 6 red balls.
3 1 7
Probability of drawing green ball = + =
8 16 16
He has 7 green balls.
∴ No. of blue balls he has = 16 – (6 + 7) = 3
Hence question can be answered.
(III) Let he have x balls. If one ball is lost, probability of drawing a
blue ball is

Which gives us x = 3
Hence question can be answered.
Question can be answered from either statement alone.

38. (c); From (I)


Total SP of bats = 1020
5
Therefore, CP of bats = 1020 × = Rs. 850
6
Profit % on ball = 12%
And more can’t be calculated.
From (II)
Profit on 1 ball = Rs.6
Profit on selling both bats = Rs. 170
From (III)
Cost price of each ball = Rs.50
Profit earned on each bat = 20%
From (I) and (II)
Total SP of bats = Rs. 1020
And CP of bats = Rs. 850
Total profit of 36 on balls is 12% of total CP of balls.

38 Adda247 Publications For any detail, mail us at


Publications@adda247.com
Cracker Book for Bank (IBPS | SBI | RRB PO | Clerk) Mains Exams
36
Hence profit on each ball is 12% which is = Rs. 6
6
Hence CP = Rs. 50/ ball
And question can be answered.
From (I) and (III)
Question can be answered, as we can calculate both CP and SP of
both balls & bat.
From (II) and (III)
Total profit earned is Rs. 206 [from (II)]
We can say that Rs. 170 is 20% of total CP of but, hence SP and CP
of both bats can be calculated.
CP of balls is given in statement (III), Hence question can be
answered from (II) and (III).
Hence, we can answer question from any two statements.

39. (d);

By joining the centers of all four circles, a square is formed which


contain 4 quadrant of circles which together makes a complete
circle.
Area of shaded region = Area of square(EFGH) - Area of bigger
circle – Area of smaller circle
Let radius of smaller circle and bigger circle be ‘r’ and ‘R’
respectively
Then relation between ‘r’ and ‘R’ is
𝑟 = 𝑅(√2 − 1)
A → Radius of bigger circle is given

39 Adda247 Publications For any detail, mail us at


Publications@adda247.com
Cracker Book for Bank (IBPS | SBI | RRB PO | Clerk) Mains Exams

By this side of square, radius of smaller circle can be find out. So,
area of shaded region can be find out
B → Diagonal of square is given
By this, side of square then radius of bigger circle and then radius
of smaller circle can be find out
After that shaded region can be find out
C → by using this we can find out the radius of the both circles and
side of square.
𝜋𝑅2 − 𝜋𝑟 2 = 𝑔𝑖𝑣𝑒𝑛
Relation between R and r is:-
𝑟 = 𝑅(√2 − 1)
Hence, Any of A, B or C alone is sufficient to answer the question

40. (b); From A,


Given 𝑟 ∶ ℎ = 7 ∶ 8
From B
Radius of hemisphere
Radius of cone =
2
2
Volume of hemisphere = 𝜋𝑟 3
3
2 3
𝜋𝑟 = 19404
3
19404 × 3 × 7
𝑟3 =
22 × 2
3
𝑟 = 9261
𝑟 = 21 cm

From A & B together —


21
Radius of cone = cm
2
10.5
Height of cone = × 8 = 12 cm
7
So from A and B we can determine the surface area of cone.
From A and C,
Given, radius of cone : height of cone = 7 : 8
Ratio between height and radius of cylinder = 8 : 7
2πr (r + h) = 2640 cm³
2π7x (8x + 7x) = 2640
40 Adda247 Publications For any detail, mail us at
Publications@adda247.com
Cracker Book for Bank (IBPS | SBI | RRB PO | Clerk) Mains Exams

x = 2 cm
75
Radius of cone = 7 × 2 × = 10.5 cm
100
10.5
Height of cone = × 8 = 12 cm
7
So, from A and B or from A and C, we can determine the surface
area of cone

41. (a); From A,


8𝑡8 = 18𝑡18
We have to find 26𝑡26
8(𝑎 + 7𝑑 ) = 18(𝑎 + 17𝑑)
8𝑎 + 56𝑑 = 18𝑎 + 306𝑑
10𝑎 + 250𝑑 = 0
(𝑎 + 25𝑑) = 0
So 26th term of = (𝑎 + 25𝑑 ) = 0
26 times = 26 × 0 = 0
From B,
No result determines
From C
No result determines
So, only statement A sufficient to give answer of question.

42. (e); From A:


Let cost price = profit percent = x
So,
𝑥×(100+𝑥)
= 119 Rs.
100
x² + 100x = 11900
x² + 100x – 11900 = 0
x = 70 Rs.
From B:
Selling price = 102 Rs.
102
M.P. ⇒ × 7 = 119 Rs.
6
From C:
If M.P. ⇒ 119 Rs.

41 Adda247 Publications For any detail, mail us at


Publications@adda247.com
Cracker Book for Bank (IBPS | SBI | RRB PO | Clerk) Mains Exams

Let cost price = x Rs.


From B and C together:
𝑥 119
× = 41.65 Rs.
100 2
x = 70 Rs.
Either only A or B and C together.

43. (c); From A,


ATQ,
𝑥+7 3
= ⇒ 3y – x = 7
𝑥+𝑦+7 4
From B,
As all boxes are identical and all colored balls are identical then
number of ways to choose balls is 6 ⇒ 𝑅𝑅, 𝐺𝐺, 𝐵𝐵, 𝑅𝐺, 𝑅𝐵, 𝐺𝐵
Where GG = Green | Green
By this value of x and y cannot be find out
From C
ATQ,
𝑥+𝑦 9
= ⇒x+y=9
𝑥+𝑦+7 16
So, only statement ‘C’ is sufficient to give answer of question.

44. (a); From A,


a, b and c are in A.P
Let common difference = ‘d’ and
⇒a = b−d
And c = b + d
(bc − ab) = b2 + bd − b2 + bd = 2bd
⇒ (bc − ab) is divisble by 2
From B,
Set {a, b, c} is coprime
⇒ all three have ′1′ as common multiple
And they are consecutive numbers
So , either b is even or a and c are both even.
In both cases-
b(c-a)= even

42 Adda247 Publications For any detail, mail us at


Publications@adda247.com
Cracker Book for Bank (IBPS | SBI | RRB PO | Clerk) Mains Exams

From C
All three are prime numbers but we can’t say whether in these
prime numbers 2 is taken or not. And if 2 is taken then b = 2 or not.
SO, in some cases (bc − ab) is divisble by 2 and in some cases
(bc − ab) is not divisble by 2. Correct conclusion cannot be inferred
from statement ‘C’ alone
So, either statement ‘A’ or ‘B’ alone is sufficient to give answer of
question.

45. (d); From A,


diagonal = 3x, 4x
From B,
4 a = 80, a = 20 ,𝑑12 + 𝑑22 = 1600
From C,
(4𝑥 )2 − (3𝑥 )2 = 448
x = 8, diagonal = 24, 32
1
Area = × 24 × 32 = 384 𝑚2
2
So, using any two of the three statement we can find area of
rhombus

46. (e); Let age of father = 𝑥


Age of son = 𝑦
From A,
𝑥 = 2𝑦
From B,
𝑥+10 12
=
𝑦+10 7
From C,
𝑥 − 𝑦 = 𝑦, 𝑥 = 2𝑦
So, Either A and B together or B and C together are sufficient to
answer the question

43 Adda247 Publications For any detail, mail us at


Publications@adda247.com
Cracker Book for Bank (IBPS | SBI | RRB PO | Clerk) Mains Exams

47. (b); Distance = 64 km


From A
Ratio between time to reach destination A and to return back = 2 :
1
1
Upstream to downstream Speed ratio = ∶ 1 ⇒ 1 : 2
2
From this we can find out the ratio between speed of boat in still
water to speed of stream
=3:1
From B
Ratio of speed of boat in still water to speed of boat in downstream
=3:4
From this we can find out the ratio between speed of boat in still
water to speed of stream
=3:1
From C
Difference between upstream speed to downstream speed = 4
km/hr
By using ‘A’ and ‘C’ together or ‘B’ and ‘C’ together, answer can be
find out.

48. (a); Let A, B and C can complete the work ‘X’ in a, b and c days
Respectively
1 1 1 1
⇒ + + =
a b c 5
From A
1 1 1
+ =
2a 2b 15
From B
4
b= c
5
and
b=a+8
From C
5
a=b
4
So, either statement ‘A’ alone or ‘B’ alone is sufficient to solve the
question.

44 Adda247 Publications For any detail, mail us at


Publications@adda247.com
Cracker Book for Bank (IBPS | SBI | RRB PO | Clerk) Mains Exams

49. (c); From A,


𝑥 × 𝑦 = 21 × 3003 = 21 × 21 × 11 × 13
L.C.M is 21 ⇒ Minimum value of ‘x’ or ‘y’ should be 21
From B,
(𝑥 − 𝑦) 𝑜𝑟 (𝑦 − 𝑥 ) = 42
From C
Both x and y are multiple of 21 ⇒ L.C.M of x and y is 21.
Hence, either statement ‘A’ and ‘B’ together or ‘A’ and ‘C’ together
are sufficient

50. (a); In the figure Triangle ABC is an equilateral triangle


And relation between radius or circle (r) and side of triangle (a) is
given below
𝑟 1
=
𝑎 2(1+√3)
From A,
Area is given ⇒ radius is given ⇒ side of triangle can be find out ⇒
Area of shaded region can be find out
From B,
Perimeter of triangle is given ⇒ radius of circle can be find out ⇒
Area of shaded region can be find out
From C
It is a basic condition of this figure and anything cannot be
concluded from this statement
Hence, either statement ‘A’ or ‘B’ alone is sufficient.

45 Adda247 Publications For any detail, mail us at


Publications@adda247.com
Cracker Book for Bank (IBPS | SBI | RRB PO | Clerk) Mains Exams

1 Adda247 Publications For any detail, mail us at


Publications@adda247.com
Cracker Book for Bank (IBPS | SBI | RRB PO | Clerk) Mains Exams

Chapter

16 Caselets

BEST APPROACH TO SOLVE THE QUESTIONS

A Caselet is another way of providing data, more often, explicitly. Unlike


various graphical methods of data representation, a caselet requires to be
solved completely before arriving at the answers to the questions that
follow it. Usually, in a caselet, data is provided through statements that
imply the logic or relation of various components of data. Some caselets are
based on Set Theory and concepts of Set theory are applied to them. All
other caselets are based on basic mathematical understanding.
A sample example of a caselet is as follows:

‘PO’ exam in Lucknow is 60% more than candidates who are appearing
for same exam in Agra. Number of candidates appearing for ‘PO’ exam in
Allahabad is 48% of total candidates appearing for ‘PO’ exam. Ratio
between total candidates appearing for ‘PO’ exam and ‘Clerk’ exam is 5 : 9.
Difference between candidates appearing for ‘PO’ exam in Lucknow and
Allahabad is 400. Ratio between candidates appearing for ‘PO’ exam to
‘Clerk’ exam in Agra and Lucknow is 5 : 12 and 8 ; 9 respectively.

Let total candidates appeared for ‘PO’ exam and ‘Clerk’ exam is 500x and
900x

Number of candidates appearing for ‘PO’ exam in Allahabad


48
= × 500x = 240x
100

Remaining candidates appeared for ‘PO’ exam in Agra and Lucknow


together = 500x – 240x = 260x
Let total candidates appeared for ‘PO’ exam in Agra = y
Then total candidates appeared for ‘PO’ exam in Lucknow = 1.6y

ATQ,
y + 1.6y = 260x ⇒ y = 100x
2 Adda247 Publications For any detail, mail us at
Publications@adda247.com
Cracker Book for Bank (IBPS | SBI | RRB PO | Clerk) Mains Exams
12
total candidates appeared for ‘Clerk’ exam in Agra = 100x × = 240x
5
9
total candidates appeared for ‘Clerk’ exam in Lucknow = 160x × = 180x
8
total candidates appeared for ‘Clerk’ exam in Allahabad
= 900x – 240 – 180x = 480x

EXAM AGRA LUCKNOW ALLAHABAD


PO 100x 160x 240x
CLERK 240x 180x 480x

Now ATQ
240x – 160x = 400
400
⇒𝑥 = =5
80

EXAM AGRA LUCKNOW ALLAHABAD


PO 500 800 1200
CLERK 1200 900 2400

Practice Exercise Based on new Pattern

Directions (1-5): Study the given passage carefully and answer the
questions.

Rahul, Sandy and Sati invested in ratio 2 : 3 : 4. After 4 months Sandy


added Rs. 1500 more in his investment and Rahul withdrew Rs. 800 from
his investment. After six months more Sati invested half of the investment
done by Rahul in first four months and Sandy invested 50% more than the
investment done by Sati in first 10 months. Rahul invested same as
investment done by Sandy in first four months. Ratio of profit of Sati to total
profit at the end of year is given as 125 : 376.

1. Profit of Sandy is approximately what percent of total profit?


(a) 64% (b) 48% (c) 72%
(d) 68% (e) 42%

3 Adda247 Publications For any detail, mail us at


Publications@adda247.com
Cracker Book for Bank (IBPS | SBI | RRB PO | Clerk) Mains Exams

2. What is the difference between profit share of Rahul and Sandy if total
profit is Rs.37,600?
(a) 12,000 (b) 16,400 (c) 18,500
(d) 22,900 (e) 20,000

3. Veer have 250% more than initial investment of Sati for a year. Find
total interest earned by him if he invested his amount in a scheme
which offers 20% p.a. for 2 years?
(a) Rs. 1400 (b) Rs. 1500 (c) Rs. 1540
(d) Rs.1600 (e) Rs.1640

4. What is the average of profit share of Sandy and Sati out of total profit
of Rs. 37,600?
(a) 18,220 (b) 18,250 (c) 16,420
(d) None of these (e) 12,490

5. If initial investment of Bhavya is one-third of initial investment of


Rahul, Sandy and Sati together then find the difference between initial
investment of Bhavya and Rahul.
(a) Rs.1000 (b) None of these (c) Rs.750
(d) Rs.500 (e) Rs.250

Directions (6-10): There are 2000people live in a village. Out of 2000


people some people are watching ‘3 idiots’ ‘Ravan’ and ‘PK’ on Sunday and
some are not watching any movie. Number of people did not watch any
movie is equal to number people watching ‘PK’ only. Number of people who
are watching ‘Ravan’ is 720. Ratio between number of people watching ‘3
Idiots’ and ‘Ravan’ only to number of people watching ‘Ravan’ and ‘PK’ only
is 2 : 3. Number of people watching ‘3 Idiots ’ and ‘PK’ both is half of number
of people watching only ‘PK’. Number of people watching ‘3 idiots’ only is
50% more than number of people watching ‘Ravan’ only. Number of people
watching all the three movies is 4% of the total number of people in the
village. Number of people watching ‘Ravan’ only is same as number of
people watching ‘Ravan’ and ‘PK’ only.

4 Adda247 Publications For any detail, mail us at


Publications@adda247.com
Cracker Book for Bank (IBPS | SBI | RRB PO | Clerk) Mains Exams

6. Number of people who did not watch any movie are what percent less
than number of people who watching only Ravan, only PK and only 3
Idiots together? (approx.)
(a) 55% (b) 50% (c) 65%
(d) 62% (e) 70%

7. Find the ratio between people who watching only Ravan and only PK
together to people who watching only Ravan & 3 idiots and only PK & 3
idiots together ?
(a) 16 : 5 (b) 16 : 7 (c) 16 : 13
(d) 16 : 11 (e) 76 : 43

8. Out of total people watching 3 idiots only, ratio between male to female
is 4 : 1 and that of people watching Ravan only is 5 : 3. Find total
number of males , who watching 3 idiots and Ravan only ?
(a) 436 (b) 428 (c) 440
(d) 438 (e) 442

9. People watching only Ravan & PK are what percent more than people
watching only Raven & 3 idiots?
(a) 50 % (b) 45% (c) 55%
(d) 40% (e) 60%

10. People who are watching only 3 Idiots, only PK and only 3 Idiots and PK
both is how much more than people who are watching Ravan?
(a) 252 (b) 192 (c) 672
(d) 488 (e) 272

Directions (11-15): Given below is the data about students appeared in


two exams i.e., A and B in six different years i.e., 2011 to 2016. Total
students appeared in both exam in 2016 is 8000 while in 2013 it is 5800.
Average number of students appeared in exam B in 2011 and 2013 is 3100
and is in the ratio 18 : 13. Students appeared in exam A in year 2015 is
1
33 % more than students appeared in exam B in same year. Total students
3
appeared in 2016 is 25% more than total students appeared in 2011.

5 Adda247 Publications For any detail, mail us at


Publications@adda247.com
Cracker Book for Bank (IBPS | SBI | RRB PO | Clerk) Mains Exams
26
Students appeared in exam A in 2016 is 62 % more than students
27
appeared in exam B is 2015. Ratio of total students appeared in 2016 &
2014 is 16 : 13. Total number of students appeared in exam A in all six
years is 21,100. Students appeared in exam B in 2011 is same as student
appeared in exam A in 2015.Students appeared in exam A in 2012 is 700
more than that of students appeared in same exam in 2014. Students
appeared in exam B in 2014 is 1200 less than that of in same exam in 2012.

11. In which year total students appeared in both exam is 3rd highest?
(a) 2012 (b) 2014 (c) 2016
(d) 2011 (e) Other than the given options

12. What is the respective ratio between students appeared in exam A in


year 2011, 2012 and 2014 together to the students appeared in exam B
in year 2013, 2014 and 2016 together?
(a) 95 : 97 (b) 99 : 97 (c) 98 : 97
(d) 99 : 95 (e) None of these

13. Find the difference between average number of students appeared in


exam A and average of students appeared in exam B in starting four
years?
(a) 250 (b) 225 (c) 215
(d) 200 (e) None of these

14. Students appeared in exam A in 2013 is how much less than students
appeared in exam B in 2012?
(a) 1400 (b) 1000 (c) 1100
(d) 1200 (e) 1300

15. Total number of students appeared in both exam in 2012 is what


percent more than total number of students appeared in both exam in
2011?
(a) 25.25% (b) 28.25% (c) 31.25%
(d) 34.25% (e) 37.25%
6 Adda247 Publications For any detail, mail us at
Publications@adda247.com
Cracker Book for Bank (IBPS | SBI | RRB PO | Clerk) Mains Exams

Directions (16-20): There are 1000 students in a college. Out of 1000


students some appeared in exams ‘X’, ‘Y’ and ‘Z’ while some not. Number of
student not appeared in any exam is equal to number of students appeared
in exam ‘Z’ only. Number of students appeared in exam ‘Y’ is 360. Ratio of
number of students appeared in exam ‘X’ and ‘Y’ only to number of students
appeared in exam ‘Y’ and ‘Z’ only is 2 : 3. Number of student appeared in
exam ‘X’ and ‘Z’ both is half of number of students appeared in only exam
‘Z’. Number of students appeared in exam ‘X’ only is 50% more than
number of students appeared in ‘Y’ only. Number of students appeared in
all the three exam is 4% of the total number of students in the college.
Number of students appeared in ‘Y’ exam only is same as number of
students appeared in ‘Y’ and ‘Z’ only.

16. How many students appeared in at least two exams?


(a) 240 (b) 260 (c) 300
(d) 360 (e) 500

17. How many students appeared in two exams only?


(a) 280 (b) 220 (c) 340
(d) 300 (e) 260

18. How many students appeared in at most two exams?


(a) 240 (b) 260 (c) 300
(d) 500 (e) 960

19. How many students not appeared in exam Y?


(a) 440 (b) 360 (c) 540
(d) 640 (e) None of these

20. How many students appeared in exam X or in exam Z?


(a) 240 (b) 360 (c) 500
(d) 680 (e) 760

Directions (21-25): Number of songs recorded by Sonu Nigam in 2008 is


300. And number of songs recorded by Sandeep and Arijit Singh in 2010 are
in ratio 5 : 4. Total number of songs recorded in 2009 is 250% more than
songs recorded by Arijit Singh in 2009. Total songs recorded by Sandeep in
7 Adda247 Publications For any detail, mail us at
Publications@adda247.com
Cracker Book for Bank (IBPS | SBI | RRB PO | Clerk) Mains Exams
4
all three years is 650. Average of songs recorded in 2010 is th of songs
3
recorded by Arijit Singh in 2008. Songs recorded by Arijit Singh in 2009 is
2th
of songs recorded by Sonu Nigam in 2008. Total songs recorded in 2008
5
is 6 times of songs recorded by Arijit Singh in 2009 and songs recorded by
Sandeep in 2008 is 25% less than songs recorded by Arijit Singh in same
year. Ratio of songs recorded by Sonu Nigam and Sandeep in 2009 is 2 : 3.

21. Number of songs recorded by Arijit Singh in all years is what percent
more/less than total songs recorded in 2008 ?
2 7 2
(a) 12 % (b) 17 % (c) 21 %
4 9 3
2
(d) 27 % (e) None of these
5

22. What is the difference of average of total number of songs recorded by


Sandeep in 2008 and 2009 and average number of songs recorded by
Sonu Nigam and Arijit Singh in 2010?
(a) 155 (b) 165 (c) None of these
(d) 150 (e) 145

23. What is the ratio of total number of songs recorded in 2010 to total
number songs recorded in 2011, if total number of songs recorded in
2011 is 50% more than total number of songs recorded in 2009 ?
(a) 32 : 17 (b) None of these (c) 21 : 19
(d) 33 : 19 (e) 32 : 21

24. Average number of songs recorded by Sonu Nigam is approximately


what percent more/less than average number of songs recorded in
2009?
(a) 104% (b) None of these (c) 124%
(d) 114% (e) 110%

25. Average number of songs recorded by all in all three years and is how
much more than average number of songs recorded by Sandeep and
Arijit Singh in 2009?
(a) None of these (b) 525 (c) 650
(d) 550 (e) 450
8 Adda247 Publications For any detail, mail us at
Publications@adda247.com
Cracker Book for Bank (IBPS | SBI | RRB PO | Clerk) Mains Exams

Directions (26-30): Study the following passage & answer the


questions that follows.
There are three schools in a town named A, B & C. In each school students
knows only Hindi, Only English and both language.
Total number of student in school A is 1600. The no. of student who know
only Hindi in school C is 40% of the total students in school A. And total
students who know only single language in school A are equal to student
who knows both language in same School and ratio of student who know
only Hindi to only English in school A is 2 : 3. Student who know both
language in all three schools is 50% more than student who knows only
English in school B. Student who knows only Hindi in all schools is equal to
the total students in school A except student who knows only English. Ratio
of students who know only English in school B to C is 1 : 4 and average of
student who know only English in all the school is 460. Student who know
both language in school C is half of student who know only Hindi in school
3
A. Total student in school B is th of total student from all the school who
4
knows only Hindi.

26. Total student in school A who know only one language is what percent
more/less than total student in school B who knows both language?
(a) 50% (b) 60% (c) 70%
(d) None of these (e) 45%

27. What is ratio of total student who knows only English in school B and C
together to total student in school B ?
(a) 3 : 2 (b) 7 : 5 (c) None of these
(d) 5 : 3 (e) 15 : 14

28. What is sum of average of student who knows only English in all the
3th
school and of student who knows only single language in school B
4
and C together.
(a) 1735 (b) 1335 (c) 1220
(d) 1525 (e) 1785
9 Adda247 Publications For any detail, mail us at
Publications@adda247.com
Cracker Book for Bank (IBPS | SBI | RRB PO | Clerk) Mains Exams

29. 40% of total student in school B is what percent more/less than half of
student who know only Hindi in school A ?
(a) 140% (b) 110% (c) 90%
(d) None of these (e) 130%

30. What is difference of average of total student in all three schools and
average of student who knows both language in school B and C ?
(a) 920 (b) 840 (c) 890
(d) 990 (e) None of theses

Directions (31-35): Data regarding investment of three different persons


in three different schemes is given below. Study the data carefully and
answer the following questions.

→ Out of total amount invested by ‘Rahul’ in all the three schemes, 25% is
invested in scheme ‘X’. Remaining amount is invested in scheme ‘Y’ and
‘Z’ equally.

→ ‘Veer’s’ investment in scheme ‘X’ is 37.5% less than that of ‘Anurag’s’ in


same scheme. Total amount invested by ‘Rahul’, ‘Veer’ and ‘Anurag’ in
all the three schemes is in the ratio of 4 : 5 : 6. Ratio between amount
invested by ‘Veer’ in scheme ‘Y’ to in ‘Z’ is 3 : 2. Amount invested by
‘Veer’ in scheme ‘Y’ is 80% more than that in scheme ‘X’.

→ Amount invested by Anurag in scheme ‘Y’ and ‘Z’ together is Rs. 20,000
more than the amount invested by ‘Rahul’ in both the same schemes
together. Amount invested by Anurag in scheme ‘Y’ is 200% more than
that in scheme ‘Z’.

31. Rate of interest on scheme ‘X’, ‘Y’ and ‘Z” is 10%, 20% and 30% p.a. at
Simple interest. Find total interest earned by Rahul after 2 years.
(a) Rs. 44,000 (b) Rs. 41,000 (c) Rs. 34,000
(d) Rs. 30,000 (e) Rs. 24,000
10 Adda247 Publications For any detail, mail us at
Publications@adda247.com
Cracker Book for Bank (IBPS | SBI | RRB PO | Clerk) Mains Exams

32. Veer and Anurag both invested in scheme ‘X’. After 8 months Veer
withdraw his total amount while Anurag withdraw his total amount
after 12 months. If total profit of both is Rs. 7,650 then find the profit
share of Veer.
(a) Rs. 2,250 (b) Rs. 3,150 (c) Rs. 4050
(d) Rs. 4950 (e) Rs. 6750

33. Total amount invested in scheme ‘Y’ by all three together is what
percent more than total amount invested by all three in scheme ‘Z’ all
together?
(a) 50.25% (b) 56.75% (c) 62.75%
(d) 68.75% (e) 72.25%

34. Find the ratio between amount invested by Veer on scheme ‘X’ to
amount invested by Rahul in scheme ‘Z’?
(a) 4 : 3 (b) 1 : 1 (c) 2 : 3
(d) 5 : 8 (e) 5 : 6

35. Scheme ‘Z’ offers 20% p.a. at Compound interest. Find interest earned
by Veer is what percent more than interest earned by Anurag in that
scheme?
(a) 75% (b) 50% (c) 100%
(d) 150% (e) 200%

Direction (36-40): Data given below about five vessels M, N, O, P & Q


contains mixture of different liquid read the data carefully and answer the
questions.

Vessel M contains mixture of milk and water, vessel N contains mixture of


water and orange juice, vessel O contains mixture of orange juice and
Vodka, vessel P contains mixture of Vodka and apple juice and vessel Q
contains mixture of apple juice and Scotch.

Mixture of milk and water in vessel M in the ratio of 7 : 2 and quantity of


water in vessel N equal to quantity of milk in vessel M. Mixture of orange
juice and vodka into the vessel O in the ratio of 5 : 3 and total quantity of
mixture in vessel O is 30 liter more than total quantity of mixture in vessel
N.
11 Adda247 Publications For any detail, mail us at
Publications@adda247.com
Cracker Book for Bank (IBPS | SBI | RRB PO | Clerk) Mains Exams
2
Total quantity of mixture in vessel Q in 66 % of total quantity of mixture in
3
vessel M. Total quantity of mixture in vessel O is 50% more than total
quantity of mixture in vessel P.

Ratio of orange juice in vessel N and O is 7 : 15, while ratio of vodka in


vessel O and P is 3 : 2. Total quantity of Scotch in vessel Q is 30 liter and
total quantity of water in vessel M is 40 liter. Quantity of orange juice in
vessel N is 70 liters.

36. If 72 𝑙 of mixture from vessel M and 36 mixture taken out from vessel N
and mixed in another vessel G, then what will be ratio of milk, `water
and orange juice in vessel G?
(a) 15 : 5 : 8 (b) 14 : 10 : 3 (c) 14 : 7 : 3
(d) 15 : 8 : 3 (e) 14 : 10: 7

37. 36 liter of mixture from vessel Q taken out and mixed in a vessel C,
which contains 21 liter scotch, what quantity of apple juice should
mixed in vessel C to obtain ratio of apple juice and scotch 2 : 1:
(a) 36 (b) 33 (c) 38
(d) 37 (e) 32

38. If vessel N and O mixed together in a big vessel Z and 22.5 liter of
mixture taken out from vessel Z, find quantity of Vodka in 22.5 liter of
mixture?
(a) 4.5 liter (b) 9 liter (c) 2.25 liter
(d) 1.5 liter (e) 3.6 liter

39. If 64 liter of mixture from vessel O, 48 liter of mixture from P and 60


liter of mixture from vessel Q taken out and mixed together in vessel A,
find the ratio of orange juice, vodka, apple juice and scotch in vessel A?
(a) 40 : 42 : 75 : 25 (b) 40 : 25 : 75 : 52 (c) 40 : 52 : 75 : 15
(d) 30 : 52 : 75 : 15 (e) 40 : 42 : 75 : 15
12 Adda247 Publications For any detail, mail us at
Publications@adda247.com
Cracker Book for Bank (IBPS | SBI | RRB PO | Clerk) Mains Exams

Directions (40-42): Given below is the data about players participated for
state level and National level championship in six different years i.e., 2001
to 2006. Total players participated for both Championship in 2006 is 6000
while in 2003 is 4800. Total number of players participated for national
level championship in 2001 and 2003 is 3600 and is in ratio of 11 : 7. Player
participated for state championship in 2005 is 25% more than player
participated for national Championship in same year. Total players
participated in 2006 is 20% more than total players participated in 2001.
1
Players participated for state Championship in 2006 is 62 % more than
2
players participated for national championship in 2005. Ratio of total
players participated in 2006 to 2004 is 15 : 13. Total number of players
participated for state championship in all six years is 20000. Players
participated for national championship in 2001 is same as players
participated for state championship in 2005. Players participated for state
championship in 2002 is 800 more than that of players participated for
same championship in 2004. Players participated for National level
championship in 2004 is 1200 less than that of for same Championship in
2002.?

40. What is the respective ratio between players participated for state level
championship in year 2001, 2002 and 2004 together to the players
participated for national level championship in year 2003, 2004 and
2005 together?
(a) 1145:493 (b) 1154:441 (c) 1154:439
(d) 439:1154 (e) 1105:439

41. Find the difference between average number of players participated for
state level championship and average of players participated for
national level championship for starting four years?
(a) 1900 (b) 1920 (c) 1820
(d) 1780 (e) 1940

42. Players participated for state level championship in 2003 is how much
more than players participated for national level championship in
2002?
(a) 930 (b) 940 (c) 950
(d) 960 (e) 970
13 Adda247 Publications For any detail, mail us at
Publications@adda247.com
Cracker Book for Bank (IBPS | SBI | RRB PO | Clerk) Mains Exams

Direction (43-47): Given below statistical data of top five wheat


producer states of India in the three successive years. Read the data
carefully and answer the question given below.
Year 2015– Uttar Pradesh and Punjab together produced 15860 thousand
ton, while Punjab production was 1180 ton less than that of Uttar Pradesh.
Madhya Pradesh produced 25% less than Uttar Pradesh, Haryana produced
690 thousand ton less than that of Madhya Pradesh. Production of
Rajasthan was 70% of total production of Haryana
Year 2016– Production of Punjab decreased by 20% compare to previous
year, while production of Haryana increased by 800 thousand ton compare
1
to previous year. Madhya Pradesh produced 33 % more than previous
3
year, while Rajasthan produced 2390 thousand ton more wheat than
previous year. All five states produced total 36612 thousand ton wheat in
2016.
Year 2017– Uttar Pradesh and Haryana together produced 16520
thousand ton, while Uttar Pradesh production was 2840 ton more than that
of Haryana. Punjab produced 628 thousand ton more wheat than previous
year and Madhya Pradesh produced 360 thousand ton more wheat than
previous year. All five states produced total 38200 thousand ton wheat in
2017.

43. Total wheat produced by Uttar Pradesh is how much more than total
wheat produced by Madhya Pradesh in the all three given years?
(a) 3550 thousand ton (b) 3750 thousand ton
(c) 3650 thousand ton (d) 3250 thousand ton
(e) 3150 thousand ton

44. Total wheat produced by Madhya Pradesh in the year 2016 & 2017
together is what percent more than total wheat produced by Haryana
in the year 2015 & 2016 together?
38 38 38
(a) 40 % (b) 44 % (c) 36 %
61 61 61
38 38
(d) 42 % (e) 48 %
61 61

14 Adda247 Publications For any detail, mail us at


Publications@adda247.com
Cracker Book for Bank (IBPS | SBI | RRB PO | Clerk) Mains Exams

45. Find the difference between average quantity of wheat produced by


Uttar Pradesh, Madhya Pradesh and Haryana in the year 2015 and
average quantity of wheat produced by Madhya Pradesh, Haryana and
Rajasthan in the year 2017?
(a) 360 thousand ton (b) 400 thousand ton (c) 420 thousand ton
(d) 450 thousand ton (e) 470 thousand ton

46. Find the ratio between total wheat produced by Rajasthan & Punjab
together in the year 2017 to total wheat produced by Haryana &
Rajasthan together in the year 2016?
(a) 160 : 161 (b) 150 : 161 (c) 170 : 171
(d) 160 : 169 (e) 160 : 171

47. Find the total wheat produced by Rajasthan and Uttar Pradesh in the
year 2016 and 2017 together?
(a) 30700 thousand ton (b) 30800 thousand ton
(c) 31700 thousand ton (d) 32400 thousand ton
(e) 33400 thousand ton

Direction (48–51): Given below data is about the number of


candidates who got final selection into UPSC exam from five different
states (Uttar Pradesh, Kerala, Maharashtra, Madhya Pradesh and
Bihar) in three different successive years. Read the data carefully and
answer the questions:
2012– Total number of Candidates selected from Uttar Pradesh are 20%
more than total number of Candidates selected from Kerala and total
number of selected Candidates from these two states are 440. Average
number of Candidates selected from Uttar Pradesh, Kerala and Maharashtra
is 220 and total number of Candidates selected from Madhya Pradesh is 70
less than that of from Maharashtra. Total number of Candidates selected
from these five states in the year 2012 is 930.
2013– Total number of Candidates selected from Uttar Pradesh is 33
1
% more than that of total number of Candidates selected from same state
3
in previous year, while total number of Candidates selected from Kerala is
30 less than average number of Candidates selected from Kerala &
Maharashtra in the year 2012. Total number of Candidates selected from
15 Adda247 Publications For any detail, mail us at
Publications@adda247.com
Cracker Book for Bank (IBPS | SBI | RRB PO | Clerk) Mains Exams

Maharashtra is 55 more than that of total number of Candidates selected


from same state in previous year and total number of candidates selected
from Madhya Pradesh is equal to total number of candidates selected from
Kerala. Total number of Candidates selected from these five states in the
year 2013 is 1130.
2014– Total number of candidates selected from Uttar Pradesh and
Maharashtra is 40 more and 25 less than total number of candidates
selected from Uttar Pradesh and Maharashtra in previous year respectively.
Average number of candidates selected from Uttar Pradesh, Kerala and
Maharashtra is 280, while average number of candidates selected from
Madhya Pradesh & Bihar is 195. Total number of selected candidates from
Bihar is 20 more than that of Madhya Pradesh.

48. Total number of candidates selected from Kerala & Maharashtra


together in the year 2014 is what percent more/less than total number
of candidates selected from Uttar Pradesh in the year 2012 & 2014
together?
(a) 10% (b) 20% (c) 15%
(d) 25% (e) 5%

49. If total number of candidates selected from Karnataka in the year 2014
is 50% more than total number of candidates selected from Madhya
Pradesh in the year 2013, then find average number of candidates
selected from Karnataka, Uttar Pradesh, Madhya Pradesh & Bihar in the
year 2014?
(a) 225 (b) 255 (c) 205
(d) 215 (e) 200

50. Find the ratio between total number of candidates selected from Kerala
to total number of selected candidates from Bihar in the given three
years?
(a) 63 : 50 (b) 71 : 50 (c) 61 : 50
(d) 67 : 50 (e) 69 : 50
16 Adda247 Publications For any detail, mail us at
Publications@adda247.com
Cracker Book for Bank (IBPS | SBI | RRB PO | Clerk) Mains Exams

51. Total number of candidates selected from Maharashtra, Madhya


Pradesh and Bihar together in the year 2013 is what percent more/less
than total number of candidates selected from Madhya Pradesh in the
year 2012 & 2013 together?
10 10 10
(a) 92 % (b) 90 % (c) 94 %
11 11 11
10 10
(d) 91 % (e) 95 %
11 11

Directions (52-56): Neeraj have some toys which are in the form of
different structures. These are cylindrical, conical, spherical. Other than
solid conical structure, all two are of both types i.e., hollow as well as solid.

→ Volume of a conical toy is three times of the volume of a solid


cylindrical toy while radius of a solid spherical toy is half than that the
radius of a conical toy. Outer radius of hollow cylindrical toys is same as
radius of solid spherical toy while average of outer radius and inner
radius of hollow cylindrical toys is equal to radius of solid cylindrical
toy. Height of cylindrical, conical and hollow cylindrical toys is same i.e,
14c.m
→ Number of solid spherical toys is 20% of total number of toys Neeraj
have. Number of hollow spherical toys is 150% more than number of
conical toys. Ratio between number of solid cylindrical toys to number
of conical toys is 3 : 2. Total number of hollow cylindrical toys is 40% of
total number of toys Neeraj have and also ‘20’ more than the total
number of solid spherical toys Neeraj have.
→ Volume of a hollow spherical toy is 33,957 cm² whose inner radius is
half of its outer radius. Volume of a hollow spherical toy is 5.25 time of
volume of conical toy.

52. Find the total space taken by all solid spherical toys? (in cm³)
(a) 97020 (b) 48510 (c) 72765
(d) 14553 (e) 24255

53. Find the number of conical toys Neeraj have?


(a) 40 (b) 20 (c) 15
(d) 12 (e) 8

17 Adda247 Publications For any detail, mail us at


Publications@adda247.com
Cracker Book for Bank (IBPS | SBI | RRB PO | Clerk) Mains Exams

54. Find the curved surface area of one hollow cylindrical toy? (in cm²)
(a) 616 (b) 1232 (c) 924
(d) 462 (e) 1386

55. Find the ratio between outer radius of hollow spherical toy to radius of
solid cylindrical toy?
(a) 4 : 1 (b) 3 : 2 (c) 3 : 1
(d) 4 : 3 (e) 2 : 1

56. Volume of one hollow cylindrical toy is how much more then volume of
one cylindrical toy?(in cm3 )
(a) 4312 (b) 3234 (c) 2696
(d) 2156 (e) 1078

Direction (57-61): - Data about speed and length of six different trains is
given below. Study the data carefully and answer the following question.

Speed of six trains A, B, C, D, E and F are a, b, c, d, e and f respectively while


time taken by six trains to cross a pole is 𝑃𝐴 , 𝑃𝑏 , 𝑃𝑐 , 𝑃𝑑 , 𝑃𝑒 and 𝑃𝑓 respectively.
Length of six trains A, B, C, D, E and F are 𝑙𝑎 , 𝑙𝑏 , 𝑙𝑐 , 𝑙𝑑 , 𝑙𝑒 and 𝑙𝑓 respectively

→ Train C can cross a pole in double time in which train E can cross a pole.
Time taken by train D to cross a platform of 180 m length is twice than
that time taken by same train to cross a pole. Train F can cross a
platform double of its length in 36 seconds. Speed of train ‘E’ is 20%
more than speed of train ‘C’. Train ‘A’ can cross a platform of 100 m
length in same time in which train ‘F’ can cross a pole.

→ Train ‘B’ can cross two poles 60 m apart from each other in 24 seconds.
Train ‘B’ can cross train ‘C’ in 10 seconds if they move towards each
other. Time taken by train ‘D’ to cross a platform of 300 m length is
same time in which train ‘C’ can cross a pole.

Given that
a:b→5:3 ⇒d:f→4:3
𝑃𝐴 ∶ 𝑃𝑏 → 2 ∶ 5 ⇒ 𝑙𝑑 ∶ 𝑙𝑒 → 1 ∶ 2
18 Adda247 Publications For any detail, mail us at
Publications@adda247.com
Cracker Book for Bank (IBPS | SBI | RRB PO | Clerk) Mains Exams

57. Train ‘C’ and Train ‘D’ enter in a tunnel from opposite sides at same
time. Find the length of tunnel if both trains completely take exit from
the tunnel at same time?
(a) 900 m (b) 1200 m (c) 1500 m
(d) 1800 m (e) 2100 m

58. Find the time in which train ‘B’ can cross a platform of same as length
as of train ‘F’?
(a) 50 seconds (b) 52 seconds (c) 54 seconds
(d) 56 seconds (e) 58 seconds

59. Train ‘A’ starts from station Rewari and move towards Gurgaon. After 4
hours Train ‘F’ starts from station Rewari and move towards Gurgaon.
After how much time Train ‘F’ will overtake train ‘A’ if length of both
trains is taken as negligible?
(a) 5 hours after start of Train ‘F’
(b) 5 hours after start of train ‘A’
(c) 9 hours after start of train ‘F’
(d) 9 hours after start of train ‘A’
(e) Both (a) and (d)

60. Time taken by train ‘A’ to cross a platform of length 250 m length is
what percent less than time taken by train ‘D’ to overtake train ‘F’ when
they move in same direction?
(a) 37.5% (b) 62.5% (c) 50%
2 2
(d) 166 % (e) 266 %
3 3

61. Train ‘C’ starts from Gurgaon station while train ‘D’ starts from Delhi
station at same time. Both starts to move towards each other. After how
much time trains will completely cross each other if distance between
Delhi and Gurgaon is 30km?
(a) 208 seconds (b) 218 seconds (c) 223 seconds
(d) 228 seconds (e) 238 seconds

19 Adda247 Publications For any detail, mail us at


Publications@adda247.com
Cracker Book for Bank (IBPS | SBI | RRB PO | Clerk) Mains Exams

Directions (62-66) Satish, Abhi and Bhavya are three persons who each
invested some amount in three different schemes (P, Q and R). Data tells
about amount invested and time of investment by them. Study the data
carefully & answer the following questions.

→ Amount invested by Satish in scheme ‘P’ is half than that amount


invested by Bhavya in same scheme. Abhi invested Rs 80,000 in scheme
‘P’ and time for which he invested in scheme ‘P’ is 5 months more than
time for which Satish invested in same scheme. Ratio of amount
invested by Satish and Bhavya in Scheme ‘R’ is same as ratio of amount
invested by Satish and Bhavya in scheme ‘P’. Abhi invested 20,000 less
in scheme ‘R’ than that of amount invested by Bhavya in scheme ‘R’.

→ Time of investment of Abhi and Bhavya is same in scheme ‘Q’. Amount


invested by Abhi in scheme ‘Q’ is 96% of amount invested by Satish in
scheme ‘Q’. Bhavya invested Rs 1,00,000 more than Abhi in scheme ‘Q’.
Ratio of amount invested by Satish in scheme ‘P’ to scheme ‘Q’ is 3 : 5.
Ratio of amount invested by Bhavya in scheme ‘P’ to scheme ‘Q’ is 15 :
22.

→ Out of total profit earned from scheme ‘Q’ Bhavya got 50%. Ratio of
profit share of Satish and Abhi is 3 : 4 in scheme ‘R’ while time of
investment of Satish and Abhi is scheme ‘R’ is 4 : 3. Bhavya invested for
10 month in scheme ‘R’.

Satish got 25% of total profit both in scheme ‘R’ as well as in scheme ‘P’.
Time of investment of Satish in scheme ‘P’ is 2 months less that of in
scheme ‘R’. Satish invested for 16 months in scheme ‘Q’

62. Abhi and Bhavya both invested same amount they invested in scheme
‘Q’ in two different schemes i.e, S1 and S2 respectively. S1 and S2 offers
20% p.a at C.I and 25% p.a at S.I respectively. Find the difference
between interest earned by both after 2 years?
(a) 36,800 (b) 41,900 (c) 47,000
(d) 52,100 (e) 57,200
20 Adda247 Publications For any detail, mail us at
Publications@adda247.com
Cracker Book for Bank (IBPS | SBI | RRB PO | Clerk) Mains Exams

63. If Bhavya earned Rs. 28,980 profits from scheme ‘R’ which is 125%
more than profit earned by him from scheme ‘P’, then find total profit
earned by Satish from scheme ‘P’ and ‘R’ together?
(a) 33,948 (b) 26,588 (c) 30,268
(d) 22,908 (e) 37,628

64. Find interest earned by Satish if scheme ‘Q’ offers S.I. at the rate of 15%
p.a?
(a) Rs26500 (b) Rs29000 (c) Rs28000
(d) Rs25000 (e) Rs27500

65. Amount invested by Bhavya in scheme ‘R’ is what percent more than
amount invested by Satish in scheme ‘P’?
(a) 140% (b) 100% (c) 144%
(d) 50% (e) 20%

66. If Abhi doubles his investment in scheme ‘P’ than before, then find what
percent of decrement is seen in Bhavya’s profit percentage?
4
(a) 20% (b) 40% (c) 28 %
7
6 1
(d) 42 % (e) 57 %
7 7

21 Adda247 Publications For any detail, mail us at


Publications@adda247.com
Cracker Book for Bank (IBPS | SBI | RRB PO | Clerk) Mains Exams

Solutions
Solutions (1-5)
Let investment of Rahul, Sandy and Sati be 2x, 3x and 4x respectively.
Ratio of profit

Rahul : Sandy : Sati


2x × 4 : 3x × 4 : 4x × 10
+(2x – 800)×6 +(3x+1500)× 6 +(5x × 2)
+(5x–800)× 2 (9x +1500) × 2
30x – 6400 : 48x + 12000 : 50x

ATQ,
50x 125
= ⇒ x = 250
128x+5600 376
Ratio of profit share of Rahul, Sandy and Sati is
1100 ∶ 24000 ∶ 12500 → 11 ∶ 240 ∶ 125

240
1. (a); Required percentage = × 100
376
= 63.829% ≃ 64%

2. (d); Required difference


240−11
= × 37600 = 22,900
376

3. (c); Investment of Veer


350
= 4 × 250 × = 3500
100
Interest earned by Veer
20 2
= 3500 [1 + ] − 3500 = 1540
100

240+125 37600
4. (b); Required average = × =18,250
2 376

2x+3x+4x
5. (e); Investment of Bhavya = = 3x
3
= 3 × 250 = Rs.750
Required profit = 750 - 2× 250 = 750 – 500 = 250
22 Adda247 Publications For any detail, mail us at
Publications@adda247.com
Cracker Book for Bank (IBPS | SBI | RRB PO | Clerk) Mains Exams

Solutions (6–10):

Total people = 2000


Let total people watching only ‘PK’ = a
Total number of people watching ‘Ravan’ = 720
Ratio between number of people watching only ‘3 Idiiots’ and ‘Ravan’ to
number of people watching only ‘Ravan’ and ‘PK’ = 2 : 3

𝑎
Number of people watching ‘3 Idiot’ and ‘PK’ =
2
Number of people watching all three movies
4
= 2000 × = 80
100

Number of people watching ‘Ravan’ only = number of people watching


‘Ravan’ and ‘PK’ = 3x
2
Number of people = watching ‘3 Idiots’ and ‘Ravan’ only = × 3x = 2x
3

Number of people watching ‘3 Idiots’ only


150 3
= Number of people watching ‘Ravan’ only × = 3𝑥 × = 4.5x
100 2

Now, 3x + 3x + 2x + 80 = 720
640
8x = 720 – 80 ⇒ x = ⇒ x = 80
8

23 Adda247 Publications For any detail, mail us at


Publications@adda247.com
Cracker Book for Bank (IBPS | SBI | RRB PO | Clerk) Mains Exams

Now,
a
12.5x + a + + a = 2000 - 80
2
5a
= (1920 – 1000) ⇒ a = 368
2

6. (d); People watching only Ravan, only PK and only 3 Idiots


= 240 + 360 + 368 = 1000
968 −368
Required percentage = × 100 = 62%
968

240+368
7. (e); Required ratio = = 76 : 43
160+184

8. (d); Total number of males , who watching 3 idiots and Ravan only
4 5
= 360 × + 240 = 438
5 8

240 −160 80
9. (a); Required percentage = × 100 = × 100 = 50%
160 160

10. (b); Required difference


= 360 + 184 + 368 – 240 – 160 – 80 – 240
= 912 – 720 = 192

Solutions (11-15):

Total students appeared in 2016 = 8000


Total students appeared in 2013 = 5800
Total students appeared in exam B is 2011 & 2013 = 6200

24 Adda247 Publications For any detail, mail us at


Publications@adda247.com
Cracker Book for Bank (IBPS | SBI | RRB PO | Clerk) Mains Exams
6200
Total students appeared in exam B in 2011 = × 18 = 3600
31
6200
Total students appeared in exam B in 2013 = × 13 = 2600
31
Total students appeared in exam A in 2013 = 5800 – 2600 = 3200
8000
Total students appeared 2011 = × 100 = 6400
125
Total students appeared in exam A in 2011 = 6400 – 3600 = 2800
8000
Total students appeared in 2014 = × 13= 6500
16
Students appeared in exam B in 2011
= Students appeared in exam A in 2015 = 3600
3600
Students appeared in exam B in 2015 = × 3 = 2700
4
1700
Students appear in exam A in 2016 = [1 + ] × 2700 = 4400
2700
Students appear in exam B in 2016 = 8000 – 4400 = 3600
Let, student appeared in exam A in 2014 = x
student appeared in exam A in 2012 = x + 700
⇒ x + x + 700 + 2800 + 3200 + 3600 + 4400 = 21,100
2x = 6400 ⇒ x = 3200
Students appeared in exam A in 2014 = 3200
Students appeared in exam A in 2012 = 3200 + 700 = 3900
Students appeared in exam B in 2014 = 6500 – 3200 = 3300
Students appeared in exam B in 2012 = 3300 + 1200 = 4500

A B Total
2011 2800 3600 6400
2012 3900 4500 8400
2013 3200 2600 5800
2014 3200 3300 6500
2015 3600 2700 6300
2016 4400 3600 8000
Total 21,100 20,300

11. (b); According to table its in 2014.

2800 + 3200 + 3900 9900 99


12. (d); Required ratio = = =
2600 + 3300 + 3600 9500 95

25 Adda247 Publications For any detail, mail us at


Publications@adda247.com
Cracker Book for Bank (IBPS | SBI | RRB PO | Clerk) Mains Exams

13. (b); Average students appeared in exam A in starting four years


2800+3900+3200+3200
= = 3275
4
Average students appeared in exam B in starting four years
3600+4500+2600+3300
= = 3500
4
Required difference = 225.

14. (e); Required difference = 4500 – 3200 = 1300

8400 –6400 2000


15. (c); Required % = × 100 = × 100 = 31.25%
6400 6400

Solutions (16-20):

Total students = 1000


Let, students appear in exam Z only = a
Total students appeared in exam Y = 360
Ratio of number of students appeared in exam X and Y only to students
appeared in exam Y and Z only = 2 : 3
Students appeared in exam X and Z both = a/2
4
Number of students appeared in all three exams = × 1000 = 40
100
Number of students appeared in Y exam only
= No. of students appeared in Y and Z only = 3x
2
Number of students appeared in exam X and Y only = × 3x = 2x
3

Now, 2x + 3x + 3x + 40 = 360 ⇒ x = 40
a
and, 12.5x + a + + a = 1000
2

26 Adda247 Publications For any detail, mail us at


Publications@adda247.com
Cracker Book for Bank (IBPS | SBI | RRB PO | Clerk) Mains Exams
5a
= 500 ⇒ a = 200
2

16 (c); Students appeared in atleast two exams = 80 + 60 + 40 + 120 = 300

17. (e); Students appeared in two exams only = 80 + 60 + 120 = 260

18. (e); Students appeared in atmost two exams


= 180 + 120 + 200 + 60 + 80 + 120 + 200 = 960

19. (d); Student not appeared in exam Y = 1000 – 360 = 640

20. (d); Students appeared in exam X or in exam Z


= 180 + 60 + 40 + 80 + 200 + 120 = 680

Solutions (21-25):

No. of songs recorded by Sonu Nigam in 2008 = 300


2
No. of songs recorded by Arijit Singh in 2009 = × 300 = 120
5
Total number of songs recorded in 2008 = 6 × 120 = 720
Let number of songs recorded by Arijit Singh in 2008 be 𝑥.
75
∴𝑥+ 𝑥 = (720– 300)
100
∴ 𝑥 = 240
∴ No. of songs recorded by Arijit Singh and Sandeep in 2008 are 240 & 180
respectively

27 Adda247 Publications For any detail, mail us at


Publications@adda247.com
Cracker Book for Bank (IBPS | SBI | RRB PO | Clerk) Mains Exams
350
Total no. of songs recorded in 2009 = × 120 = 420
100
2
No. of songs recorded by Sonu Nigam in 2009 = × (420– 120) = 120
5
∴ No. of songs recorded by Sandeep in 2009 = 180
4
Total no. of songs recorded in 2010 = 3 × × 240 = 960
3
No. of songs recorded by Sandeep in 2010 = 650 – 180 – 180 = 290
290
No. of songs recorded by Arijit Singh in 2010 = × 4 = 232.
5
No. of songs recorded by Sonu Nigam in 2010
= 960 – 290 – 232 = 438

Years
2008 2009 2010
Singers
Sonu Nigam 300 120 438
Arijit Singh 240 120 232
Sandeep 180 180 290
Total 720 420 960

720–(240+120+232) 7
21. (b); Required percentage = × 100 = 17 %
720 9

438+2321 180+180
22. (a); Required difference = ( )–( ) = 335 – 180 = 155
2 2

960
23. (e); Required ratio = 150 = 32 : 21
420 ×
100

24. (a); Average number of songs recorded by Sonu Nigam


300+120+438 858
= = = 286
3 3
420
Average number of songs recorded in 2009. = = 140
3
286–140
Required percentage = × 100 ≃ 104%
140

25. (d); Required difference


720+420+960 120+180
=( )–( ) = 700 – 150 = 550
3 2

28 Adda247 Publications For any detail, mail us at


Publications@adda247.com
Cracker Book for Bank (IBPS | SBI | RRB PO | Clerk) Mains Exams

Solutions (26-30):

Total student in school A = 1600


40
Student who knows only Hindi in C = × 1600 = 640
100
Let student who knows only Hindi & only English in school A be 2𝑥 and 3𝑥
respectively.
∴ Total student who knows both language in school A = 5𝑥

ATQ,
10𝑥 = 1600 ⇒ 𝑥 = 160
∴ Student who knows only Hindi in A = 320
& only English in A = 480
Both language in A = 800
Student who knows only Hindi in B = 800 + 320 – 640 – 320 = 160
Total student who knows Only English = 3 × 460 = 1380
1
Student who knows only English in B = (1380– 480) = 180
5
Student who knows only English in C = 900 – 180 = 720
320
Student who knows both language in C = = 160
2
2
Total student in school B = 1120 × = 840
4
∴ Student who knows both language in B = 840 – 160 – 180 = 500

School Only Hindi Only English Both Hindi and English Total
A 320 480 800 1600
B 160 180 500 840
C 640 720 160 1520
Total 1120 1380 1460

(320+480)–500 300
26. (b); Required percentage = × 100 = × 100 = 60%
500 500

180+720 900
27. (e); Required ratio = = = 15 : 14
840 840

1380 3
28. (a); Required sum = + [160 + 180 + 640 + 720]
3 4
= 460 + 1275 = 1735
29 Adda247 Publications For any detail, mail us at
Publications@adda247.com
Cracker Book for Bank (IBPS | SBI | RRB PO | Clerk) Mains Exams
40 1
×840– ×320 336–160
100 2
29. (b); Required percentage = 1 × 100 = × 100 = 110%
×320 160
2

1600+840+1520 500+160
30. (d); Required difference= ( )–( ) = 1320 – 330 = 990
3 2

Solutions (31-35):

Total amount invested by ‘Rahul’, ‘Veer’ and ‘Anurag’ is in the ratio 4 : 5 : 6.


Let total amount invested by Rahul, Veer and Anurag in all the three
schemes be 16x, 20x and 24x
25
Amount invested by Rahul in scheme ‘X” = × 16𝑥 = 4𝑥
100
16𝑥−4𝑥
Amount invested by Rahul in scheme ‘Y’ or ‘Z’ = = 6𝑥
2
Let Amount invested by Veer in scheme Y and Z be 3y and 2y respectively
3𝑦 5𝑦
Then amount invested by Veer in scheme X is = × 100 =
180 3
5𝑦
Ratio between amount invested by Veer in scheme ‘X’, ‘Y’ and ‘Z’ = ∶ 3𝑦 ∶
3
2𝑦 → 5 ∶ 9 ∶ 6
Amount invested by Veer in scheme ‘X’, ‘Y’ and ‘Z’ is 5x, 9x and 6x
respectively.
5𝑥
Amount invested by Anurag in scheme ‘X’ = × 8 = 8𝑥
5
Amount invested by Anurag in scheme ‘Y’ and ‘Z’ together = 24𝑥–8𝑥 = 16𝑥
Amount invested by Rahul in scheme ‘Y’ and ‘Z’ together = 12𝑥

ATQ,
16𝑥 − 12𝑥 = 20,000 ⇒ 𝑥 = 5,000
Total amount invested by Anurag in scheme ‘Y’ and ‘Z’ together = 80,000
Let amount invested by Anurag in scheme ‘Z’ = ‘a’
Amount invested by Anurag in scheme ‘Y’ = 3𝑎
⇒ 𝑎 + 3𝑎 = 80,000 ⇒ 𝑎 = 20,000

X Y Z
Rahul 20,000 30,000 30,000
Veer 25,000 45,000 30,000
Anurag 40,000 60,000 20,000

30 Adda247 Publications For any detail, mail us at


Publications@adda247.com
Cracker Book for Bank (IBPS | SBI | RRB PO | Clerk) Mains Exams

31. (c); Required interest


20,000×10×2 30,000×20×2 30,000×30×2
= + +
100 100 100
= 4,000 + 12,000 + 18,000 = Rs. 34,000

32. (a); Ratio between profit share of Veer and Anurag


Veer : Anurag
→ 25,000 × 8 ∶ 40,000 × 12 → 5 ∶ 12
5
Profit share of Veer = × 7650 = Rs 2,250
17

33. (d); Total amount invested in scheme ‘Y’


= 30,000 + 45,000 + 60,000 = 1,35,000
Total amount invested in scheme ‘Z’
= 30,000 + 30,000 + 20,000 = 80,000
1,35,000−80,000 55,000
Required % = × 100 = × 100 = 68.75%
80,000 80,000

25,000 5
34. (e); Required Ratio = =
30,000 6

20 2
35. (b); Interest earned by Veer = 30,000 × [1 + ] − 30,000 = 13,200
100
20 2
Interest earned by Anurag = 20,000 × [1 + ] − 20,000 = 8,800
100
13,200−8,800
Required % = × 100 = 50%
8,800

Solutions (36 – 40)

Vessel M
Water ⇒ 40 liters
40
So milk in M ⇒ × 7 = 140 liters
2
Total quantity in vessel M = 180 liters
Total quantity quantity in vessel Q
180×2
= = 120 liters
3
Scotch in vessel Q = 30 liters
Apple juice in vessel Q = 90 liters

31 Adda247 Publications For any detail, mail us at


Publications@adda247.com
Cracker Book for Bank (IBPS | SBI | RRB PO | Clerk) Mains Exams

In vessel N
Total quantity = 70 + 140 = 210 liter
Water = 140 liters
Orange juice = 70 liters

In Vessel O
Total quantity = 210 + 40 = 240 liters
Orange juice in vessel O : Orange juice in vessel N = 7 : 15
70
Orange juice in vessel O = × 15 = 150 liters
7
Vodoka in vessel = 240 – 150 = 90 liters

In Vessel P
2
Total quantity = 66 % of total quantity of vessel P
3
2
= × 240 = 160
3
2
Vodka = of vodka in vessel O
3
2
90 × = 60 liters
3
Apple juice = 160 – 60 = 100 liters

M N O P Q
Milk 140 — — — —
Water 40 140 — — —
Orange juice — 70 150 — —
Vodka — — 90 60 —
Apple Juice — — — 100 90
Scotch — — — — 30

36. (b); In vessel G—


7
Milk in 72 liter of mixture = 72 × = 56 liter
9
2
Water in 72 liter of mixture = 72 × = 16 liter
9
2
Water in 36 liter of mixture =36 × =24 ltr.
3
1
Orange juice in 36 liter of mixture = 36 × = 12 liter
3
Ratio of milk : water : Orange juice in vessel G = 56 : (16 + 24) : 12 =
14 : 10 : 3
32 Adda247 Publications For any detail, mail us at
Publications@adda247.com
Cracker Book for Bank (IBPS | SBI | RRB PO | Clerk) Mains Exams
3
37. (b); Apple juice in vessel C = 36 × = 27 liter
4
1
Scotch in vessel C = 36 × + 21 = 9 + 21 = 30 liter
4
27+𝑥 2
Let 𝑥 liter of apple juice mixed = =
30 1
𝑥 = 60 – 27 ⇒ 𝑥 = 33 liter

38. (a); Ratio of water, Orange juice and Vodka vessel Z


= 140 : (70 + 150) : 90 = 14 : 22 : 9
Vodka in 22.5 liter of mixture
9 9
= 22.5 × (14+22+9) = 22.5 × = 4.5 liter
45

39. (e); Ratio of orange juice, Vodka, apple juice & scotch in vessel A
5 3 3 5 3 1
=[(64 × ) : (64 × + 48 × ) : (48 × + 60 × ) ∶ 60 × ]
8 8 8 8 4 4
= [ 40 : (24 + 18) : (30 + 45) : 15) = 40 : 42 : 75 : 15

Solutions (40-42):

Total players participated in 2006 = 6000


Total players participated in 2003 = 4800
Total players participated for national level Championship in 2001 & 2003
= 3600
Total players participated for National level championship in 2001
11
= 3600 × = 2200
18
Total players participated for national Level championship in 2003
7
= 3600 × = 1400
18
Total player participated for state level championship in 2003
= 4800 – 1400 = 3400
Total player participated in 2001
100
= 6000 × = 5000
120
Total player participated in 2001 for state level championship
= 5000 – 2200 = 2800
Total player participated in 2004
6000
= × 13 = 5200
15

33 Adda247 Publications For any detail, mail us at


Publications@adda247.com
Cracker Book for Bank (IBPS | SBI | RRB PO | Clerk) Mains Exams

Players participated for national championship in 2001


= Players participated for state championship in 2005 = 2200
2200
Player participated for national championship in 2005 = × 4 = 1760
5
Player participated for state level championship in 2006
5 13
= [1 + ] × 1760 = × 1760 = 220 × 13 = 2860
8 8
Player participated for national championship in 2006 = 6000–2860 = 3140
Let players participated for state level championship in 2004 = X
Players participated for state level championship in 2002 = X + 800
⇒ X + X + 800 + 3400 + 2800 + 2200 + 2860 =20000
2X = 20000 – 12060
7940
X= ⇒ X = 3970
2
Player participated for state championship level in 2004 = 3970
Player participated for state level championship in 2002= 3970+800 = 4770
Players participated for national level championship in 2004 = 5200 – 3970
= 1230
Players participated for national level championship in 2002 = 1230 + 1200
= 2430

state level National Level


Years Total
championship Championship
2001 2800 2200 5000
2002 4770 2430 7200
2003 3400 1400 4800
2004 3970 1230 5200
2005 2200 1760 3960
2006 2860 3140 6000

2800+4770+3970 11540
40. (c); Required ratio = = = 1154 : 439
1400+1230+1760 4390

41. (b); Required difference


2800+4770+3400+3970 2200+2430+1400+1230
=( )–( )= 3735 – 1815 = 1920
4 4

42. (e); Required difference = 3400 – 2430 = 970

34 Adda247 Publications For any detail, mail us at


Publications@adda247.com
Cracker Book for Bank (IBPS | SBI | RRB PO | Clerk) Mains Exams

Solutions (43-47):

15860−1180
2015 → Production of wheat in Punjab = = 7340 thousands ton
2
Production of wheat in Uttar Pradesh = 734 + 1180 = 8520 thousands ton
75
Production of wheat in Madhya Pradesh= 8520 × = 6390 thousands ton
100
Production of wheat in Haryana = 6390 – 690 = 5700 thousands ton
70
Production of wheat in Rajasthan = 5700 × = 3990 thousands ton
100

80
2016 → Production of wheat in Punjab = 7340 × = 5872 thousands ton
100
Production of wheat in Haryana = 5700 + 800 = 6500 thousands ton
4
Production of wheat in Madya Pradesh= 6390 × = 8520 thousands ton
3
Production of wheat in Rajasthan = 3990 + 2390 = 6380 thousands ton
Production of wheat in Uttar Pradesh
= 36612 – (5872 + 6500 + 8520 + 6380) = 9340 thousands ton

16520−2840
2017 → Production of wheat in Haryana = = 6840 thousands ton
2
Production of wheat in uttar Pradesh = 6840 + 2840 = 9680 thousands ton
Production of Wheat in Punjab = 5872 + 628 = 6500 thousands ton
Production of wheat in Madhya Pradesh= 8520 + 360 = 8880 thousands ton
Production of wheat in Rajasthan
= 38200 – (6840 – 9680 + 6500 + 8880) = 6300 thousands ton

Production of wheat (thousands ton)


States
2015 2016 2017
Uttar Pradesh 8520 9340 9680
Punjab 7340 5872 6500
Madhya Pradesh 6390 8520 8880
Haryana 5700 6500 6840
Rajasthan 3990 6380 6300

35 Adda247 Publications For any detail, mail us at


Publications@adda247.com
Cracker Book for Bank (IBPS | SBI | RRB PO | Clerk) Mains Exams

43. (b); Total wheat produced by Uttar Pradesh in all three years
= 8520 + 9340 + 9680
= 27540 thousands ton
Total wheat produced by Madhya Pradesh in all three years
= 6390 + 8520 + 8880
= 23790 thousands ton
Required difference = 27540 – 23790
= 3750 thousands ton

44. (d); Total wheat produced by Madhya Pradesh in year 2016 & 2017
together
= 8520 + 8880 = 17400 thousands ton
Total wheat produced by Haryana in the year 2015 & 2016
together = 5700 + 6500
= 12200 thousands ton
17400 –12200 5200 38
Required% = × 100 = × 100 = 42 %
12200 12200 61

45. (e); Average quantity of wheat produced by Uttar Pradesh, Madhya


Pradesh and Haryana in the year 2015
8520+6390+5700 20610
= = = 6870 thousands ton
3 3
Average quantity of wheat produced by Madhya Pradesh, Haryana
and Rajasthan in the year 2017
6840+6300+8880 22020
= = = 7340 thousands ton
3 3
Required difference = 7340 – 6870 = 470 thousands ton

6300+6500 12800
46. (a); Required ratio = = = 160 : 161
6500+6380 12880

47. (c); Total Wheat produced by Rajasthan and Uttar Pradesh in the year
2016 & 2017 together = (9340 + 6380) + (9680 + 6300)
= 31700 thousands ton
36 Adda247 Publications For any detail, mail us at
Publications@adda247.com
Cracker Book for Bank (IBPS | SBI | RRB PO | Clerk) Mains Exams

Solutions (48-51):

2012 →
Let total number of candidates selected from Kerala be 5𝑥 and from Uttar
Pradesh be 6𝑥

ATQ—
5𝑥 + 6𝑥 = 440 ⇒ 𝑥 = 40
Total number of selected candidate from Uttar Pradesh = 240
Total number of selected candidate from Kerala = 200
Total number of candidate selected from Maharashtra
= 220 × 3 – (240 + 200) = 660 – 440 = 220
Total number of candidates selected from Madhya Pradesh = 220 – 70= 150
Total number of candidates selected from Bihar
= 930 – (240 + 200 + 220 + 150) = 930 – 810 = 120

2013 →
4
Total number of candidates selected from Uttar Pradesh = 240 × = 320
3
Total number of candidates selected from Kerala
(200+220)
= – 30 = 210 – 30 = 180
2
Total number of candidates selected from Maharashtra = 220 + 55 = 275
Total number of candidates selected from Madhya Pradesh = 180
Total number of candidates selected from Bihar
= 1130 – (320 + 180 + 275 + 180) = 1130 – (955) = 175

2014 →
Total number of candidates selected from Uttar Pradesh = 320 + 40 = 360
Total number of candidate selected from Maharashtra = 275 – 25 = 250
Total number of candidate selected from Kerala
= 280 × 3 – (360 + 250) = 840 – 610 = 230
Total number of candidate selected from Madhya Pradesh and Bihar = 195
× 2 = 390
390–20
Total number of candidate selected from Madhya Pradesh = = 185
2
Total number of candidate selected from Bihar = 185 + 20 = 205

37 Adda247 Publications For any detail, mail us at


Publications@adda247.com
Cracker Book for Bank (IBPS | SBI | RRB PO | Clerk) Mains Exams

48. (b); Total number of candidates selected from Kerala & Maharashtra
together in 2014
= (250 + 230) = 480
Total number of candidates selected from Uttar Pradesh in 2012 &
2014 together
= 240 + 360 = 600
600 –480
Required percentage = ×100 =20%
600

49. (b); Total number of candidate selected from Karnataka in the year
2014
150
= 180 × = 270
100
270+360+185+205 1020
Required average = = = 255
4 4

(200+180+230) 610
50. (c); Required ratio = (120+175+205) = = 61 : 50
500

51. (b); Total number of candidates selected from Maharashtra, Madhya


Pradesh and Bihar together in 2013 = (275 + 180 + 175) =630
Total number of candidates selected from Madhya Pradesh in the
year 2012 & 2013 together = (150 + 180) = 330
630 –330 300 1000 10
Required percentage = × 100 = × 100 = = 90 %
330 330 11 11

Solution (52-56):

Volume of hollow spherical toy = 33,957𝑐𝑚2


Let Outer radius of hollow spherical toy = R
𝑅
Inner radius of hollow spherical toy =
2

38 Adda247 Publications For any detail, mail us at


Publications@adda247.com
Cracker Book for Bank (IBPS | SBI | RRB PO | Clerk) Mains Exams

ATQ,
4 𝑅 3
𝜋 (𝑅3 − ( ) ) = 33,957
3 2
3
⇒ R = 9261 ⇒ R = 21
Outer radius of hollow spherical toy = 21cm
Inner radius of hollow spherical toy = 10.5 cm
Volume of conical toy
33957 14
= = 6468 = 𝜋(𝑟𝑎𝑑𝑖𝑢𝑠 𝑜𝑓 𝑐𝑜𝑛𝑒)2 ×
5.25 3
⇒ Radius of cone = 21 cm
6468
Volume of solid cylindrical toy = = 2156
3
= π (radius of cylinder)² × (height of cylinder)
⇒ Radius of Cylinder = 7cm
21
Radius of Solid Spherical toy = = 10.5𝑐𝑚
2
Outer Radius of hollow cylindrical toy = 10.5𝑐𝑚
Inner radius of hollow cylindrical toy = 7 × 2 − 10.5 = 3.5 cm

Let total number of toys = 100x


ATQ,
Number of solid spherical toys = 20x
Number of hollow cylindrical toys = 40x

ATQ,
40𝑥 − 20𝑥 = 20 ⇒ 𝑥 = 1
Let Number pf conical toys = 2𝑦
Number of hollow spherical toys = 5𝑦
Number of solid cylindrical toys = 3𝑦

ATQ,
2𝑦 + 5𝑦 + 3𝑦 = 100 − 40 − 20 = 40 ⇒ 𝑦 = 4

39 Adda247 Publications For any detail, mail us at


Publications@adda247.com
Cracker Book for Bank (IBPS | SBI | RRB PO | Clerk) Mains Exams

Toy Number of Toys


Conical 8
Solid Cylindrical 12
Solid Spherical 20
Hollow Cylindrical 40
Hollow Spherical 20

52. (a); Space taken by one solid spherical toy


= Volume of one solid spherical toy
4
= 𝜋(10.5)3 = 4851𝑐𝑚3
3
Total space taken by solid spherical toys
= 20 × 4851 = 97020𝑐𝑚2

53. (e); Number of conical toys Neeraj have = 8

54. (b); Curved surface area of one hollow cylindrical toy


= 2𝜋 × (3.5 + 10.5) × 14 = 1232 cm2
21 3
55. (c); Required Ratio = =
7 1

56. (d); Volume of one hollow cylindrical toy


= 𝜋 × 14 × (10.52 − 3.52 ) = 4312 cm3
Required difference
= 4312 − 2156 = 2156𝑐𝑚3

Solution (57-61): Time taken by train ‘D’ to cross a platform of 180 m


length is twice than that time taken by same train to cross a pole.
ℓ +180 2ℓ
⇒ d = d ⇒ ℓd = 180m
d d
But, ℓd ∶ ℓe = 1 ∶ 2 ⇒ ℓe = 360m
Train ‘F’ can cross a platform double of its length in 36 seconds
ℓ +2ℓ ℓ
⇒ f f = 36 ⇒ f = 12 seconds
f f
⇒ Train ‘F’ can cross a pole in 12 seconds
Train ‘A’ can cross a platform of 100m length in same time in which train ‘F’
can cross a pole i.e, 12 seconds
ℓ +100
⇒ A = 12
a

40 Adda247 Publications For any detail, mail us at


Publications@adda247.com
Cracker Book for Bank (IBPS | SBI | RRB PO | Clerk) Mains Exams

Train ‘B’ can cross two poles 60 m apart from each other in 24 seconds
ℓ +60
⇒ 24 = B
b
But a : b → 5 : 3
And, PA : PB → 2 ∶ 5
ℓa
a 2 ℓa b ℓa 2 2
ℓb ⇒ = × ⇒ = ⇒ ℓa = ℓb
5 a ℓb ℓb 3 3
b

And,
2(ℓA +100) (ℓb +60) 5 3ℓa
= ⇒ 2(ℓa + 100) = ( + 60)
a b 3 2
15 3
6ℓa + 600 = ℓa + 300 ⇒ 300 = ℓa
2 2
⇒ ℓa = 200m

And, ℓb = 300m
200+100 300
a= = = 25 m/sec
12 12
25
b = × 3 = 15 m/sec
5
Train ‘C’ can cross a pole in double time in which train ‘E’ can cross a pole.
ℓ 2ℓ
⇒ C= e
c e
But,
e = 1.2c
ℓ 2ℓ 5
⇒ c = e ⇒ ℓc = ℓe
c 1.2c 3

But ℓe = 360 m
⇒ ℓc = 600m
Train ‘B’ can cross train ‘C’ in 10 seconds if they move in opposite direction
ℓ +ℓ 300+600
⇒ b c=b+c⇒ = 15 + c ⇒ 90 – 15 = C ⇒ c = 75 m/sec
10 10
Time train by train ‘D’ to cross a platform of 300m is same time in which
train ‘C’ can cross a pole.
ℓd +300 ℓ 180+300 600 480
⇒ = c⇒ = ⇒d= = 60 m/sec
d c d 75 8
d:f→4:3
60
⇒ f = × 3 = 45 m/sec
4
ℓf
= 12 ⇒ ℓf = 540 m
f

41 Adda247 Publications For any detail, mail us at


Publications@adda247.com
Cracker Book for Bank (IBPS | SBI | RRB PO | Clerk) Mains Exams

But
e = 1.2c ⇒ e = 1.2 × 75 = 90 m/sec

Train Speed (in m/sec) Length (in m)


A 25 200
B 15 300
C 75 600
D 60 180
E 90 360
F 45 540
57. (c); Let length of tunnel is ‘x’ meter
Time taken by train ‘C’ to cross tunnel is same as time taken by
train ‘D’ to cross tunnel
180+𝑥 600+𝑥
= ⇒ 900 + 5𝑥 = 2400 + 4𝑥
60 75
𝑥 = 1500 𝑚
300+540 840
58. (d); Required time = = = 56 seconds
15 15

59. (e); Speed of train ‘A’ = 25m/sec = 90km /hr


Distance covered by train ‘A’ in 4 hours
= 90 × 4 = 360 km
Let Train ‘F’ will overtake train ‘A’ in ‘x’ time
360×5
Then, 𝑥 = (45−25)×18 =5 hours
Train ‘F’ meet train ‘A’ after 5 hours start of train ‘F’ or after 9
hours start of train ‘A’

60. (b); Time taken by train ‘A’ to cross a platform of 250m length
200+250
= = 18
25
Time taken by train ‘D’ to overtake train ‘F’
180+540
= = 48sec
60−45
48−18
Required % = × 100 = 62.5%
48

30,000+600+180
61. (d); Required time = = 228 seconds
75+60
42 Adda247 Publications For any detail, mail us at
Publications@adda247.com
Cracker Book for Bank (IBPS | SBI | RRB PO | Clerk) Mains Exams

Solutions (62-66):
Ratio of amount invested by Satish and Bhavya in scheme ‘P’ and in ‘R’ is
same that is 1 : 2
Let Bhavya and Satish invested 2z and z in scheme ‘R’
⇒ Amount invested by Abhi in scheme ‘R’ is (2z – 20,000)
Ratio of profit share of Satish and Abhi in scheme ‘R’ is 3 : 4 while time of
investment of Satish and Abhi in scheme ‘R’ is 4 : 3.
3 𝑧×4
⇒ = ⇒ 18z – 1,80,000 = 16z
4 (2𝑧−20,000)×3
z = 90,000

Amount invested Amount invested Amount invested by


Scheme
by Satish by Abhi Bhavya
R 90,000 1,60,000 1,80,000

In scheme R,
Bhavya invested for 10 months while Satish got 25% of profit out of total
profit
If ratio between Satish’s and Abhi’s profit share is 3 : 4, then Bhavya’s profit
3
share is × 100 − 7 = 5
25
⇒ Ratio of profit share of Satish, Abhi and Bhavya is 3 : 4 : 5.
Let Satish invested for ‘y’ month
1,80,000×10 5
= ⇒ y = 12 month
90,000×𝑦 3
12
Abhi invested for = × 3 = 9 month
4
Time of investment of Satish in scheme ‘P’ is 2 months less than that of in
scheme ‘Q’
⇒ Satish invested for 10 months in scheme ‘P’.
Time of investment of Abi in scheme ‘P’ is 5 months more than time for
Satish invested
⇒ Abhi invested for 15 months in scheme ‘P’
Let amount invested by Satish in scheme ‘Q’ is = 5b
5𝑏
⇒ Amount invested by Satish in scheme ‘P’ is = × 3 = 3𝑏
5
96
And, Amount invested by Abhi in scheme ‘Q’ is = 5𝑏 × = 4.8𝑏
100
Amount invested by Bhavya in scheme ‘Q’ is = 4.8b + 1,00,000

43 Adda247 Publications For any detail, mail us at


Publications@adda247.com
Cracker Book for Bank (IBPS | SBI | RRB PO | Clerk) Mains Exams

Amount invested by Bhavya in scheme ‘P’ is = 3b × 2 = 6b

ATQ,
6𝑏 15
= ⇒ 132b = 72b + 15,00, 000
4.8𝑏 + 1,00,000 22
b= 25,000

Amount Invested
Scheme Satish Abhi Bhavya
P 75,000 80,000 1,50,000
Q 1,25,000 1,20,000 2,20,000
R 90,000 1,60,000 1,80,000

Let, Bhavya invested for ‘y’ month in scheme ‘P’.

ATQ,
1 75,000×10
= ⇒ y = 7 months
4 75,000×10+80,000×15+1,50,000×𝑦
Time of investment of Abhi and Bhavya is same in scheme ‘Q’ and ratio
between profit sharing of Bhavya to total profit in scheme ‘Q’ is 1 : 2.
Let, Abhi and Bhavya invested for ‘5a’ months and Satish invested for ‘x’
month.

ATQ,
1 2,20,000×5𝑎
=
1 1,25,000×16+1,20,000×5𝑎
⇒ 1,25,000 × 16 = 1,00,000 × 5a ⇒ 5a= 20 months
So Bhavya and Veer invested in scheme ‘Q’ for 20 months

44 Adda247 Publications For any detail, mail us at


Publications@adda247.com
Cracker Book for Bank (IBPS | SBI | RRB PO | Clerk) Mains Exams
120 120
62. (e); Interest earned by Abhi = 1,20,000 × × − 1,20,000 = 52,800
100 100
Interest earned by Bhavya
25
= 2,20,000 × 2 × = 1,10,000
100
Required difference = 1,10,000 –52,800 = 57,200

63. (b); Ratio of profit sharing between Satish, Abhi and Bhavya in scheme
‘P’ and scheme ‘R’ is 5 : 8 : 7 and 3 : 4 : 5 respectively.
28,980
Profit earned by Satish from scheme ‘R’ = × 3 = 17,388
5
28980
Profit earned by Bhavya from scheme ‘P’ = × 100 = 12,880
225
12,880
Profit earned by Satish from scheme ‘P’ = × 5 = 9,200
7
Total profit earned by Satish from scheme ‘P’ and ‘R’ together
= 17,388 + 9,200 = 26,588
20
64. (d); Time of investment of Satish = × 4 = 16 months
5
15 16
Interest earned by Satish = 1,25,000 × × = 25,000
100 12

1,80,000−75,000 105
65. (a); Required % = × 100 = × 100 = 140%
75,000 75

66 (c); Ratio of profit sharing between Satihs, Abhi and Bhavya in scheme
‘P’ is 5 : 8 : 7
Let total profit = 20
7
Bhavya’s profit = × 100
20
= 35%
When Abhi doubles his investment then ratio of profit sharing
between Satish, Abhi and Bhavya in scheme ‘P’ is 5 : 16 : 7
Let total profit = 28
7
Bhavya’s profit = × 100 = 25%
28
35−25 4
% Decrement in profit % ×100 = 28 %
35 7

45 Adda247 Publications For any detail, mail us at


Publications@adda247.com

Вам также может понравиться